You are on page 1of 395

M.K.

GUPTA
CA EDUCATION
9811429230 / 9212011367
WWW.MKGEDUCATION.COM

INCOME TAX
(Volume – 1)
 COMPUTATION OF TOTAL INCOME AND TAX LIABILITY 17-91
 TAXABILITY OF GIFT 92-104
 ADVANCE PAYMENT OF TAX 105-119
 RESIDENTIAL STATUS & SCOPE OF TOTAL INCOME 120-163
 INCOME UNDER THE HEAD HOUSE PROPERTY 164-234
 DEDUCTION FROM GROSS TOTAL INCOME 235-282
 AGRICULTURAL INCOME 283-299
 CLUBBING OF INCOME 300-317
 INCOME UNDER THE HEAD OTHER SOURCES 318-335
 DEDUCTION OF TAX AT SOURCE 336-361
 SET OFF AND CARRY FORWARD OF LOSSES 362-381
 PROVISIONS FOR FILING OF RETURN OF INCOME 382-392
 MISCELLANEOUS TOPICS 393-395

Including
EXAMINATION QUESTIONS

44th Edition
CA (INTER)
MAY-2019/NOV-2019 Author
P.Y. 2018-19 This Book is the result of combined efforts of
A.Y. 2019-20 Chartered Accountants/ company executives /
F.A. – 2018 other professionals / feedback of our thousands of students

FOR ONLINE PURCHASE OF OUR BOOK PLEASE SEE DETAILS GIVEN ON


OUR WEBSITE :WWW.MKGEDUCATION.COM

`700
2

PAPER – 4: TAXATION
(One paper ─ Three hours –100 Marks)
Objective: To develop an understanding of the provisions of income-tax law and goods and services tax law
and to acquire the ability to apply such knowledge to make computations and address basic application
oriented issues.
SECTION A: INCOME TAX LAW (60 MARKS)
Contents:
1. Basic Concepts
(i) Income-tax law: An introduction
(ii) Important definitions in the Income-tax Act, 1961
(iii) Concept of previous year and assessment year
(iv) Basis of Charge and Rates of Tax
2. Residential status and scope of total income
(i) Residential status
(ii) Scope of total income
3. Incomes which do not form part of total income (other than charitable trusts and institutions,
political parties and electoral trusts)
(i) Incomes not included in total income
(ii) Tax holiday for newly established units in Special Economic Zones
4. Heads of income and the provisions governing computation of income under different heads
(i) Salaries
(ii) Income from house property
(iii) Profits and gains of business or profession
(iv) Capital gains
(v) Income from other sources
5. Income of other persons included in assessee's total income
(i) Clubbing of income: An introduction
(ii) Transfer of income without transfer of assets
(iii) Income arising from revocable transfer of assets
(iv) Clubbing of income of income arising to spouse, minor child and son’s wife in certain cases
(v) Conversion of self-acquired property into property of HUF
6. Aggregation of income; Set-off, or carry forward and set-off of losses
(i) Aggregation of income
(ii) Concept of set-off and carry forward and set-off of losses
(iii) Provisions governing set-off and carry forward and set-off of losses under different heads of income
(iv) Order of set-off of losses
7. Deductions from gross total income
(i) General provisions
(ii) Deductions in respect of certain payments
(iii) Specific deductions in respect of certain income
(iv) Deductions in respect of other income
(v) Other deductions
8. Computation of total income and tax liability of individuals
(i) Income to be considered while computing total income of individuals
(ii) Procedure for computation of total income and tax liability of individuals
9. Advance tax, tax deduction at source and introduction to tax collection at source
(i) Introduction
(ii) Direct Payment
(iii) Provisions concerning deduction of tax at source
(iv) Advance payment of tax
3

(v) Interest for defaults in payment of advance tax and deferment of advance tax
(vi) Tax collection at source – Basic concept
(vii) Tax deduction and collection account number
10. Provisions for filing return of income and self-assessment
(i) Return of Income
(ii) Compulsory filing of return of income
(iii) Fee and Interest for default in furnishing return of income
(iv) Return of loss
(v) Provisions relating to belated return, revised return etc.
(vi) Permanent account number
(vii) Persons authorized to verify return of income
(viii) Self-assessment
SECTION B – INDIRECT TAXES (40 MARKS)
Contents:
1. Concept of indirect taxes
(i) Concept and features of indirect taxes
(ii) Principal indirect taxes
2. Goods and Services Tax (GST) Laws
(i) GST Laws: An introduction including Constitutional aspects
(ii) Levy and collection of CGST and IGST
a) Application of CGST/IGST law
b) Concept of supply including composite and mixed supplies
c) Charge of tax
d) Exemption from tax
e) Composition levy
(iii) Basic concepts of time and value of supply
(iv) Input tax credit
(v) Computation of GST liability
(vi) Registration
(vii) Tax invoice; Credit and Debit Notes; Electronic way bill
(viii) Returns
(ix) Payment of tax including reverse charge
Note – If any new legislation(s) is enacted in place of an existing legislation(s), the syllabus will accordingly
include the corresponding provisions of such new legislation(s) in place of the existing legislation(s) with
effect from the date to be notified by the Institute. Similarly, if any existing legislation ceases to have effect,
the syllabus will accordingly exclude such legislation with effect from the date to be notified by the Institute.
Students shall not be examined with reference to any particular State GST Law.
Consequential/corresponding amendments made in the provisions of the Income-tax law and Goods and
Services Tax laws covered in the syllabus of this paper which arise out of the amendments made in the
provisions not covered in the syllabus will not form part of the syllabus. Further, the specific
inclusions/exclusions in the various topics covered in the syllabus will be effected every year by way of
Study Guidelines.
4

WEIGHTAGE
Intermediate Course Paper 4: Taxation (100 Marks)
Section A: Income-tax Law (60 Marks)
I. (5%-10%)
1. Basic Concepts
(i) Income-tax law: An introduction
(ii) Important definitions in the Income-tax Act, 1961
(iii) Concept of previous year and assessment year
(iv) Basis of Charge and Rates of Tax

II. (10%-15%)
2. Residential status and scope of total income
(i) Residential status
(ii) Scope of total income

III. (25%-30%)
3. Incomes which do not form part of total income (other than charitable trusts and institutions,
political parties and electoral trusts)
(i) Incomes not included in total income
(ii) Tax holiday for newly established units in Special Economic Zones
4. Heads of income and the provisions governing computation of income under different heads
(i) Salaries
(ii) Income from house property
(iii) Profits and gains of business or profession
(iv) Capital gains
(v) Income from other sources

IV (15%-20%)
5. Income of other persons included in assessee's total income
(i) Clubbing of income: An introduction
(ii) Transfer of income without transfer of assets
(iii) Income arising from revocable transfer of assets
(iv) Clubbing of income of income arising to spouse, minor child and son’s wife in certain cases
(v) Conversion of self-acquired property into property of HUF
6. Aggregation of income; Set-off, or carry forward and set-off of losses
(i) Aggregation of income
(ii) Concept of set-off and carry forward and set-off of losses
(iii) Provisions governing set-off and carry forward and set-off of losses under different heads of income
(iv) Order of set-off of losses
7. Deductions from gross total income
(i) General provisions
(ii) Deductions in respect of certain payments
(iii) Specific deductions in respect of certain income
(iv) Deductions in respect of other incomes
(v) Other deductions

V. (20%-25%)
8. Computation of total income and tax liability of individuals
(i) Income to be considered while computing total income of individuals
(ii) Procedure for computation of total income and tax liability of individuals
5

VI. (10%-15%)

9. Advance tax, tax deduction at source


(i) Introduction
(ii) Direct Payment
(iii) Provisions concerning deduction of tax at source
(iv) Advance payment of tax
(v) Interest for defaults in payment of advance tax and deferment of advance tax
10 Introduction to tax collection at source
(i) Tax collection at source – Basic concept
(ii) Tax deduction and collection account number
11. Provisions for filing return of income and self-assessment
(i) Return of Income
(ii) Compulsory filing of return of income
(iii) Fee and Interest for default in furnishing return of income
(iv) Return of loss
(v) Provisions relating to belated return, revised return etc.
(vi) Permanent account number
(vii) Persons authorized to verify return of income
(viii) Self-assessment

Section B: Indirect Taxes (40 Marks)


I (30%-50% )
1. Levy and collection of CGST and IGST
(a) Application of CGST/IGST law
(b) Concept of supply including composite and mixed supplies
(c) Charge of tax (including reverse charge)
(d) Exemption from tax
(e) Composition levy
2. Basic concepts of time and value of supply
3. Input tax credit

II (20%-30%)
1. Computation of GST liability

III (25%-40%)
1. Registration
2. Tax invoice; Credit and Debit Notes; Electronic way bill
3. Returns
4. Payment of tax

IV (0 - 5%)
1. Concept of indirect taxes - Concept and features of indirect taxes; Principal indirect taxes
2. GST Laws: An introduction including Constitutional aspects
6

ETI AGARWAL
ALL INDIA TOPPER OF CA-IPC (NOV-13)
ROLL NO. - 366539

MARKS IN TAXATION:89%
(HIGHEST MARKS IN TAXATION ALL OVER INDIA)
(AGGREGATE MARKS 79.71%)

(FEEDBACK)
A man for whom teaching is neither a business nor a profession, rather a passion for doing good,
great and unique in the field of teaching is none other than MK Gupta Sir.

Sir"s unmatchable style of teaching coupled with his patience and calmness in dealing with students
is simply excellent.

The structure of learning pattern, regular mock tests, motivational cash prizes and student friendly
study material covering practical illustrations, past year questions and bare act.. all contributed to
making this journey easy and building up the confidence needed for IPCC.

Moreover, the vast knowledge and experience of the faculty assisted in making the concepts crystal
clear and handling each n every doubt of students.

The administration and management stands second to none.

MK GUPTA classes is a place which can change the word impossible 2 I M POSSIBLE. It made
me a better person both personally n professionally.

I think 4 success 4 elements are necessary-desire, dedication, direction and discipline...and all the 4
i got from Sir..

THANK YOU so much Sir..

In the end i would just like to say MK GUPTA SIR NOT ONLY MAKES CA. HE MAKES
HUMANS!!

ETI AGARWAL
7

AKSHAY JAIN
ALL INDIA TOPPER OF CA-IPC (NOV-13)
ROLL NO.- 368162

MARKS IN TAXATION : 87%


(SECOND HIGHEST MARKS IN TAXATION ALL OVER INDIA)
(AGGREGATE MARKS 79.71%)

(FEEDBACK)
Experience of those four months with M.K. GUPTA SIR was out of the world.

As a teacher, M.K. GUPTA SIR is just like a sea of knowledge & you get each and everything from
very beginning to end from him.

Sir is really a nice person. He is very motivational and his words of motivation can influence
anybody to work hard & make their parents proud.

M.K. GUPTA CA EDUCATION is the only place where the provisions of tax laws are combined
with the practical knowledge. Study material provided is excellent and it contains numerous
problems covering all aspects and such type of problems are not available anywhere. Sir is not
giving any home work rather home work is done in the class itself and students are invited to solve
the problem before the entire class.

Be honest towards your studies & Sir will show you the way of success. The way, Sir is making
students ready for the professional world is praiseworthy. Exposure given by sir to face interview of
Big four CA Firms is excellent.

The test Series conducted by the Sir in all the subjects of IPC is very nice Scheme to score such
good marks and exam are conducted in the similar manner as it is conducted by ICAI.

I would like to express my gratitude to Sir because it was only his efforts that helped me reach this
position.

Sir its your Success.

A Message to all : -

“COME & HAVE A TIME THAT YOU WILL CHERISH THROUGHOUT YOUR LIFE”.

AKSHAY JAIN
8

VIJENDER AGGARWAL
ALL INDIA TOPPER OF CA-IPCC (NOV-10)
ROLL NO. - 174639

MARKS IN TAXATION:92%
(HIGHEST MARKS IN TAXATION ALL OVER INDIA)
(AGGREGATE MARKS 83.71%)

(FEEDBACK)
A person who possesses such vast knowledge in the field of taxation, that we people can only dream
of, is none other than M. K. Gupta Sir.

He possesses the rare ability to teach this procedural subject with utmost ease, enabling his students
to grasp all the provisions without any confusion.

The quality of study material provided is such that a good study of it helped me score 92 marks. The
variety and complexity of practical problems covered in the books are not available anywhere else.

One can find many places where taxation is being taught but it is hardly possible to find a better
place where tax laws are combined with their practical applicability to ensure that all concepts are
crystal clear.

Sir is extremely generous. Money-making doesn’t appear to be his priority and it is clearly reflected
in his classes, where the infrastructure and administration stands second to none and students are
awarded handsome cash-prizes not only in classes but also in tests, which are regularly conducted.

Thanking Sir for all what he has done would be an insult since it was only his efforts that helped me
reach this position. Sir, its your success. The relationship between us started in CPT only and
continued in IPCC and I hope it will continue forever.

VIJENDER AGGARWAL
9

PRACHI JAIN
ALL INDIA TOPPER OF CA-PCC (MAY-10)
ROLL NO. - 66312

MARKS IN TAXATION:88%
(HIGHEST MARKS IN TAXATION ALL OVER INDIA)
(AGGREGATE MARKS 77.67%)

(FEEDBACK)

M. K. Gupta Sir is an outstanding teacher. He is not only a good teacher but a good person by heart.
His way of teaching is excellent. There are many provisions in tax but Sir repeats every provision
atleast two times. This helps in understanding those provisions easily.

His books are very good. Everything from theory to PRACTICAL ILLUSTRATION,
EXAMINATION QUESTIONS and BARE ACT is covered in his books.

Sir’s staff and management is also very good. Everything is handled in a systematic manner and on
time. Overall it was a good experience.

Thanks Sir !! :-

PRACHI JAIN
10

RESULTS OF NOV – 2014 EXAMINATION


(CA-Intermediate (IPC))
NO OTHER TEACHER OF TAXATION IN INDIA HAS BETTER RESULT THAN OURS

WE SINCERELY THANK OUR STUDENTS FOR THEIR HARD WORK

PRASHANT YADAV ( 354233 ) SECURES 92 MARKS IN TAXATION


(SECOND HIGHEST IN INDIA)
MARKS SECURED BY SOME OF THE STUDENTS IS AS UNDER:

Sl. No   Name of the Students  Marks in Taxation   Roll No.  


1  PRASHANT YADAV  92  354233 
2  MOHIT SHARMA  89  353392 
3  MANISHA BHAMBRI  89  456626  
4  ANISH SHRESHTA   88  344028 
5  KAPIL KHANNA  85  341539 
6  JITENDRA  85  337780 
7  PUNEET WASAN   84  368537 
8  ISHA MALIK  84  339842 
9  RUPAL GARG  84  393844 
10  AKANSHA GOEL   84  336693 
11  PRASIT SHARMA  84  344702 
12  MANSI BAJAJ  83  354329 
13  RASHI GUPTA  83  337864 
14  HARSH AGARWAL  83  491097 
15  RAHUL ARORA  83  337403 
16  SEJAL MEHTA   83  353096 
19  ANNU SETHI  83  353491 
17  ASHISH GUPTA  82  353575 
18  RAGHAV GUPTA   82  491122 
20  SHREYA MALIK   82  340228 
21  PRABHAW AGARWALLA  82  369428 
22  PRABHAT RANJAN  81  347926 
23  ANKIT KHEMKA   81  338055 
24  ARTI SRIVASTAVA  80  347859 
25  SHIVANGI GUPTA  80  337956 
OPINION OF OUR STUDENTS

1. PRASHANT YADAV (Roll No.354233) 92 Marks


M.K. Gupta Sir is an outstanding teacher. He possesses very vast knowledge of taxation. Sir repeats every
concept atleast three times which makes all concepts crystal clear. Study material provided is very good, it
covers everything from illustration to examination problem and from theory to Bare Act. Staff and
infrastructure facilities of MKG Classes is incomparable. Thank you Sir for your love and support.

2. MOHIT SHARMA (Roll No.353392) 89 Marks


A brilliant personality in my life who has motivated the student to a good path. He is very different from
others. Sir concentrates not only on the Marks but also on the overall development of the student.
I am truly glad that I studied from Sir. He taught me how to compete in life. Every student get very good
marks with a little effort, if he is a student of M.K. Gupta CA Education.
11

3. MANISHA BHAMBRI (Roll No.456626) 89 Marks


M.K. Gupta Sir is the best teacher I have ever met. His study material being the best helped me a lot in my
exams. He is the most sincere teacher who never waste a single moment and gives his best towards his
profession. He teaches not only the theoretical portion but the practical approach too. He teaches us how to
be a good human being and how to live life happily. Thank you Sir for your support every time I needed.

4. ANISH SHRESTHA (Roll No.344028) 88 Marks


M.K. Gupta Sir is a very excellent teacher. The way he is dedicated towards teaching make us to be
dedicated towards our study. Every concept and every doubt of taxation whichever do I had, he has make
clear. The best thing about Sir is, he use to revise the concept more than 3 times which makes student very
easy for preparing their exam.
You will have all the sufficient material for study and lots of questions with answers for practice a
systematically designed materials.
Thank a lot to Sir for being so much helpful and lot of love.

5. KAPIL KHANNA (Roll No.341539) 85 Marks


Sir ‘M.K. Gupta’ is the best teacher for Taxation. I feel fortunate to be his student, the amount of knowledge
he imparts is fantastic and uncomparable. He is a person who burns himself up like a candle to light the path
of his dearest students to the road of success. I wish Sir teaches all the subjects of IPCC, since he is simply
the best. Thank you for everything Sir. It you and only you who can guide students like us to reach the
zenith.

6. JITENDRA (Roll No.337780) 85 Marks


Before joining CA, I was so much scary about the “Taxation” but after joining M.K. Gupta CA Education
for taking taxation class my scary converted into my strength now. This is just because of Sir’s knowledge
& teaching style with practicality. Study material provided by Sir is also awesome for study.

7. PUNEET WASAN (Roll No.368537) 84 Marks


M.K. Gupta Sir has a vast knowledge in the subject. The topics taken in the class are very well planned.
I found the book really very good. Infact, I practiced all the previous attempts questions of each chapter and
every small question was covered in the chapter. I recommend the students to be thorough with book and
one will score undoubtedly high marks in tax. All the best!! Thank you so much Sir.

8. ISHA MALIK (Roll No.339842) 84 Marks


I do not have words to express my greatfulness for M.K. Gupta Sir. He really possesses vast knowledge and
rich experience in taxation. Study material provided by Sir is also very good which covers everything for
getting through the exam. There is no doubt that due to excellent coaching given by Sir, I have been able to
secure good marks. I pray to God for his long, happy and prospective life. I wish him to continue give
coaching to the prospective students for a longer period. I appeal to all the students who qualify CPT to take
coaching from Gupta Sir for getting sure success. Thank you Sir.

9. RUPAL GARG(Roll No.393844) 84 Marks


M.K. Gupta Sir is, as I believe, the best teacher for Taxation. He is so knowledgeable that I was totally awe
inspired by him. Every day in the class was exciting as he explains everything with real examples and full
depth. The books are superb with lots of practical questions. Thank you Sir.

10. AKANSHA GOEL (Roll No.336693) 84 Marks


It was a great experience studying from M.K. Gupta Sir. He has a vast pool of the knowledge of the subject.
The book is a comprehensive one too.
12

11. PRASIT SHARMA (Roll No.344702) 84 Marks


Taking about the coaching, the teaching style of M.K. Gupta Sir is too much excellent. He has good dealing
with student in every situation. If anyone asked about the taxes coaching, I prefer M.K. Gupta Sir because he
is the one & only best teacher in Taxation.

12. MANSI BAJAJ (Roll No.354329) 83 Marks


Sir teaches so well and clarifies all our queries. He makes us understand the whole concept very clearly. He
is an amazing teacher and the best teacher in the field of Taxation.

13. RASHI GUPTA (Roll No.337864) 83 Marks


M.K. Gupta Sir is a very friendly and helping teacher. He always answered my queries well. His coaching
classes are very knowledgeable and books are also very good.

14. HARSH AGARWAL (Roll No.491097) 83 Marks


M.K. Gupta Sir is a good teacher. He teaches all aspects of Taxation whether it is practical knowledge or
theoretical knowledge. He teaches every point for 2-3 times and it gets learn in class only. His practical
knowledge about the subject is very good.

15. RAHUL ARORA (Roll No.337403) 83 Marks


M.K. Gupta Sir is a great mentor. Sir has excellent knowledge about the subject. He makes every concept
crystal clear. Every concept is explained atleast twice in the class. He connect every topic with practical life.
Study material is excellent. Bare Act is covered in the study material. Three months experience with M.K.
Gupta Sir is memorable moments of my life. Thank you Sir, for your guidance and encouragement.

16. SEJAL MEHTA (Roll No.353096) 83 Marks


Coaching for Taxation was an enriching experience in terms of the conceptual clarity which I gained on
each and every topic. Learning tax became so easy with the simplified notes provided.
Also, the kind of knowledge that Sir shares with the students is very commendable and useful in
understanding the practical aspects of Taxation. Attending the coaching is worth the time spent.

17. ANU SETHI (Roll No.353491) 83 Marks


I have never seen teacher like M.K. Gupta Sir. His way to teaching, knowledge and experience is awesome
i.e. brilliant. Overall regards for such marks is only M.K. Gupta Sir.

18. ASHISH GUPTA (Roll No.353575) 82 Marks


M.K. Gupta Sir is a very good teacher and he has a very vast knowledge of taxation. He gives his best to
every student in a class. The atmosphere of the class when he was teaching in a class is very awesome. I am
giving all my credit to M.K. Gupta Sir for securing marks in Taxation.

19. RAGHAV GUPTA (Roll No.491122) 82 Marks


M.K. Gupta Sir is an outstanding teacher. He possesses a very vast knowledge about the subject. His way of
teaching is fabulous. Every concept is explained with help of an example. Study material is all exhaustive
that he provides. Also, queries are taken up promptly. Thank you Sir for your guidance.

20. SHREYA MALIK (Roll No.340228) 82 Marks


M.K. Gupta Sir is the best teacher I have ever come across. His level of knowledge is tremendous. The way
he teaches, with so much patience and willingness, keeps every student motivated. The marks I have scored
in tax is all because of him. Thank you so much Sir. I am a student of video class and I have never met Sir in
person. I would be grateful if I would be given a chance to meet him in person.
13

21. PRABHAW KUMAR AGARWALLA (Roll No.369428) 82 Marks


Teaching was excellent and queries handled were excellent. Teaching methodology was really excellent and
helped a lot to me.

22. PRABHAT RANJAN (Roll No.347926) 81 Marks


M.K. Gupta Sir has a very deep knowledge about the subject and his practical approach towards the subject.
Sir repeats every provision atleast twice. This helps in understanding those provision easily.
The books notes and all the management is done very properly and in a smooth manner. All in all the best
way to study tax.

23. ANKIT KHEMKA (Roll No.338055) 81 Marks


M.K. Gupta Sir is excellent teacher of Tax. He repeats the provision two to three times and doubts are also
taken by the faculty. His books are also very good. Bare Act is covered in his books for more understanding
about the Act. Sir also provide regular test and prize also given by him motivates the student to work hard.
Environment provided by M.K. Gupta Classes is also very good to study.

24. ARTI SRIVASTAVA (Roll No.347859) 80 Marks


Sir’s unmatchable style of teaching. Regular mock test, also help in to achieve good marks in Taxation. Sir’s
books contain illustration. Past year question also help to achiever to good marks. Sir’s build confidence in
every student to achieve success in life. Thank you, so much Sir.

25. SHIVANGI GUPTA (Roll No.337956) 80 Marks


M.K. Gupta Sir is an amazing teacher. The tax subject is all about provisions so many sections but Sir makes
it simpler for us out of all the subjects, I found Taxation to be the most interesting one.
Sir’s study material and notes are sufficient. Study material covers all the past year exam questions, practice
questions with solutions. His practical experiences help our understanding level to reach new heights. Thank
you Sir for everything.
14

FEEDBACK ON FACEBOOK FROM STUDENTS WHO


APPEARED IN MAY 2017- CA INTERMEDIATE (IPC)
TAXATION PAPER HELD ON 09TH MAY 2017
M.K. Gupta: 9TH MAY 2017:06:00 PM
My students have given feedback to me regarding todays Tax Paper that entire paper was from our book. I
would like our students to comment.
Vedansh Agarwal Yess sir Mstt hua h exam
Çhïrãg Jāîñ ꞏ sir you're outstanding and lovable teacher .Your book is totally sufficient to score 80 above
marks in your subject
M.K. Gupta Thank u everyone n love u all
Çhïrãg Jāîñ we also love you.... sir....
Pulkit Sharma Yes sir thanks alot even if we forgot sth your comments in class has made us remember that
answer or that point .Thanks for being there.
Neelamber Budhiraja Sir yeh toh always Hona hi Hota hai exam aapki book se bahar aa hi nhi skta
Pandit Rahul Sharma Sir ab to bta he do Ca ka exam paper app he bnate ho
Idrees Saifi Very easy paper And ap ke lie
M.K. Gupta Love u all Dear sweet students
Maneet Jain Yes sir !!!! That ques of VAT and CST was like exactly from book ....
I missed that illustration first...But while doing second revision....I attempted that ques and I was doing it
wrongly....But thanks to your book sir ! Attempted for 100 marks sir !
Neha Sahni Yesss sir every ques is from ur book
Muskan Mittal true sir
Riya Kharbanda True sir
Hunny Jain Yes sir
Sharma Prashant Yes sir vat and service tax were same sums from your book and practice manual also
Jaiwish Rajpal Thnq sir... For being our teacher
Shivam Minocha Yes sir easy paper tha
Pooja Garg Sir paper was so easy
Hemant Sahu Paper is like to easy..Thanku sir
Tanvi Jain yes !! todays paper was from ur book sir...because of you I can attempt full paper before time..oll
credit goes to you sir!!
Yash Singhal Paper was easy and from your book sir
Râjâñ Kûshwähä Dear sir , we are thankful to you ,... Attempted 100 marks
Mohit Saxena thanks sir every question is from book
Sâñdéèp Trîpãthī Sir really 100% questions covered from your book..Nd i did 100 marks paper thanku
sir
YoGesh It was awesome sir Mje Agye Paper dekr
M.K. Gupta Love u all. Thank u so much
Laxmi Dutt Joshi Yes sir u have heard ryt !!!! the paper was from ur book ,,nd u , u are
awesome nd btw wese bhi teacher kbhi galat nhi hoty ,,vo hamesha sahi hoty h ,and hume
unki baat manni chahiye bcz vo hamre bhale k liye he kehtey h Laxmi Dutt Joshi or aj ka paper dekh kr
bahut achaa lgaa Spicy suraj Ritesh Raj all questions are from ur book ....thnks for being our teacher....we
always love u nd respect u...
Aman Tiwari Sir paper was super se bhi uppprrrrr
Ankee Mittal All ppr from book.... All que covered in class....
Thnkew sir...... ❤ ❤ ❤
Maansi Jain thankyou so much Sir ❤
Ankur Goyal All paper from your book sir ,love u sir ji ❤
Ankur Rai Very true sir.. Service tax ki kayi questions copy paste the
15

RAvi Gupta Luv u sir and thanks for giving such kind of preparations for exams #100% paper cover from
the best teacher
Ankur Rai Thanks a ton sir and love you too sir
Govinda Sharma Yes sir thank you sir
Sanyam Jain Yes sir thanks alot even if we forgot sth your comments in class has made us remember that
answer or that point. Thanks for being there.
Dixit Kalra Yes sir all concepts were also discussed in class, & from our book
Fca Manish Mittal Easy paper Sir
Mohit Gupta Yes sir.. paper was so easy thanku so much
Amrit Pal Singh thankuh sir...
Anirudh Goyal Sir thanku so much .....100% paper from ur book....
Ankit Gola Sir ji... superb paper... so damn easy Tanuj Goyal dont frgt nov16
Âñüj Kùmär Too much easy.. ☺ ☺ ☺ Maja aa gya sir
Nikhil Arya Sir paper full aapki book mae cover tha sir lg hi ni rha tha ki hum exm de rhe hn sir asa lg rha
tha ki ghr pe aapki book ke questions kr rhe hn ..sir really uh r god of taxation..thnku so mch
sir.. ;) :). :
Abhay Rawat #shaandaar
M.K. Gupta Thank u everyone. Love u all
Muskan Mittal We luv u toooo
Avish Singhal Totally Paper Was From Your Book Thanks Sir
Priyanka Manav Thankuu so much sir. Ppr was damn easy...
Ravi Kumar Thanks sir
Deepanshu Deol Tod kr diya sir ji
Ritik Singhal Bhaut easy tha mja aa gya sbse acha tax ka hi gya h tysm sir
Bhavya Arora All paper from book sir ji .....
Ghanshyam Yadav That's diference between Great teacher and you are always my Fevrate teacher
Rachit Satija Thank You so much Sir! ❤ R ka Naam le kr exam shuru kiya tha!
Anand Kumar All paper from sir's book Thanku sir You're so diff from all teachers
Himanshu Bansal mast osm.. paper.. sir..
Kajal Jain Thank you soo much for giving us deep knowledge about each and every topic
Garima Singla Really sir !!! It was awesome Thankuu so much sir!!
Vicky Jindal thank u sir
Prakash Virat even examples was also from your book sir
Anshu Prakash Maza aa gya question dekh kr sir.. U r the best teacher.. Mujhe to charo exam me tax hi sbse
jyada easy lga.. :-)
Aditya Aggarwal Paper seemed to be so easy...thank you sir
Kapish Jain it was good
Shrishty Sharma yessss sir today's entire paper was from ur book....
Tanisha Mishra You're the best sir.. obviously from your book only
Ikshika Garg The best coaching classes taken.
Ayush Goyal Thank u so much sir, paper was very easy
Vishu Aggarwal Thnku sir
Sumit Tripathi sir paper was easy.. bcz of your classes...study
Shubham Goyal Class me pdhey bhi they
Sumit Tripathi padhe to the be.
Lakshay Bansal Chah GE guru
Gourav Bajaj Thanku sir....
Sachin Aggarwal Very easy paper sirji. Halwa tha. Bas khana tha.
Deepak Kumar Prasad very easy paper sir
Gaurav Garg Yes, everything in d paper is from ur book sirjii.....
Vikas Zha Yes sir maximum questions were from your book. I didn't got tym to study pm and I have gone
through your book only and it made my day
16

Antriksh Garg My exam was awesome only because of u sir, every question is from book. You have prepare
for giving our best in examination by conducting, regular test. THANKS A LOT , SIR
Rishabh Verma Damn easy paper all from your book thanks sir ji
SäçHíñ Sharma easy
Amit Jain All paper from sir's book
Jatin Garg it was an easy paper... we just did it good because of your guidance.... very lucky that we got to
study from you.... had a nice time in your class... and got so much from you.... thanku again sir
Chhavi Arora Yess..paper was easy nd all questions were from mkg study material....
Ankit Ankit great sir
Tarun Bansal Good sir
Nitin Kumar Pal ꞏMaja aa Gaya sir aisa paper karke, pura paper mock test Jaisa tha Jo apne liye tha . Your
mock test helps a lot
Samkit Jain Best paper and best book sir
Shubham Shukla Yes Sir, whole paper from your book and i thank god that he gave me chance study with
you
Nikhil Singla You are Great Sir!... Lovvv u sir gggg
Sanidhay Salgotra Yes Sir, paper was really easy or maybe felt easy because the way you teach is
amazing.
Anand Kumar Yes sir
Sanyam Jain Yes sir thanks alot even if we forgot sth your comments in class has made us remember that
answer or that point .Thanks for being there.
Mohd Aăřįf Whole paper from ur book sir . Best book Best teacher M.k gupta sir
Keshav Mangla Yeees sir paper was easy nd our coverage in classroom is more than enough & had a great
experience in ur classes sir
Karan Rawal Thank u sir....your teachings helped a lot... ☺
Computation of Total Income And Tax Liability 17

COMPUTATION OF TOTAL INCOME


AND TAX LIABILITY
BASIS OF CHARGE AND RATES OF TAXES
SECTIONS PARTICULARS
15 to 17 Salary
22 to 27 House Property
28 to 44DB Business/Profession
45 to 55A Capital Gains
56 to 59 Other Sources
288A Rounding off of Total Income
288B Rounding off of Tax
87A Rebate from Income Tax
2(24)(ix) Meaning of Casual Income
115BB Rate of tax on Casual Income
112 Long Term Capital Gains
111A Short Term Capital Gains when STT has been paid
2(31) Meaning of Person
10(1) Agricultural Income
10(34) Dividend Income from Domestic Companies
10(35) Income from UTI or Mutual Funds

Question 1: Write a note on Computation of Total Income.


Answer: Computation of Total Income
If the income is taxable, it will be further divided into five different categories of income which are called
heads of income i.e. if the income is received from the employer, it will be considered to be income under
the head salary; if the income is in connection with letting out of house property, income is taxable under the
head house property; if the income is from any business or profession, it is taxable under the head profits
and gains of business/profession; if any capital asset (gold, land, house etc) has been transferred, income is
taxable under the head capital gains; if there is any other income like interest or winnings from a lottery etc,
it is covered under the head other sources.
Income shall be computed under each head i.e. expenses incurred shall be deducted from the gross receipt as
per the provisions of the relevant head.
Income computed under each head shall be added up to compute the gross total income.
Certain concessions are allowed from the gross total income which are called deduction from gross total
income under section 80C to 80U.
After permitting the deductions, remaining income is called total income.
Computation of total income can be shown mathematically in the manner given below:
Total Income of an assessee shall be computed in the following steps:
Compute the income of the assessee under all the five heads, permitting exemption/deductions of each head.
`
(i) Income from Salaries (Section 15 to 17) ………..
(ii) Income from House Property (Section 22 to 27) ………..
(iii) Profits and gains of Business or Profession (Section 28 to 44DB) ………..
(iv) Capital Gains (Section 45 to 55A) ………..
(v) Income from Other Sources (Section 56 to 59) ………..
Gross Total Income ………..
_______
Deductions from gross total income [Section 80C to 80U] ………..
_______
Total Income ………..
Computation of Total Income And Tax Liability 18

_______
Total Income shall be rounded off u/s 288A in the multiples of 10 and for this purpose, any paisa shall be
ignored and if the last digit is 5 or more, it will be rounded off to the higher multiple otherwise it will be
rounded off to the lower multiple.
Example
(i) `6,28,456 shall be rounded off as 6,28,460
(ii) `6,28,455 shall be rounded off as 6,28,460
(iii) `6,28,454 shall be rounded off as 6,28,450
(iv) `6,28,455.99 shall be rounded off as 6,28,460
(v) `6,28,454.99 shall be rounded off as 6,28,450

Question 2: Write a note on Computation of Tax Liability of individual.


Answer: Computation of Tax Liability
Tax liability of an individual shall be computed at the slab rates given in the relevant Finance Act i.e.
Finance Act, 2018 and the rates are as given below:
 Resident individual of the age of 60 years or more at any time upto the end of relevant previous year
but less than eighty years (senior citizen)
If total income is upto `3,00,000 NIL
On next `2,00,000 5%
On next `5,00,000 20%
On Balance amount 30%
“Senior Citizen” means an individual resident in India who is of the age of 60 years or more at any
time during the relevant previous year. If date of birth is 1st April, 1959, 60 year will be completed on
31-03-19 and assessee shall be senior citizen in previous year 2018-19.
 Resident individual of the age of 80 years or more at any time upto the end of relevant previous year
(Very senior citizen)
If total income is upto `5,00,000 NIL
On next `5,00,000 20%
On Balance amount 30%
“Very Senior Citizen” means an individual resident in India who is of the age of 80 years or more at
any time during the relevant previous year. If date of birth is 1st April, 1939, 80 year will be completed
on 31-03-19 and assessee shall be very senior citizen in previous year 2018-19.
 Any other Individual
Income shall be taxable at the slab rates given below:
If total Income upto `2,50,000 NIL
On next `2,50,000 5%
On next `5,00,000 20%
On Balance amount 30%
Example
(i) Mr. X has total income of `6,00,000
(ii) Mr. X has total income of `8,00,000
(iii) Mr. X has total income of `10,00,000
(iv) Mr. X has total income of `12,00,000
(v) Mr. X has total income of `20,00,000
Solution:
(i) Total income 6,00,000
On first `2,50,000 Nil
On next `2,50,000 @ 5% 12,500
On balance `1,00,000 @ 20% 20,000
Tax before health and education cess 32,500
Add: health & education cess @ 4% 1,300
Tax Liability 33,800
(ii) Total income 8,00,000
Computation of Total Income And Tax Liability 19

On first `2,50,000 Nil


On next `2,50,000 @ 5% 12,500
On balance `3,00,000 @ 20% 60,000
Tax before health and education cess 72,500
Add: health & education cess @ 4% 2,900
Tax Liability 75,400
(iii) Total income 10,00,000
On first `2,50,000 Nil
On next `2,50,000 @ 5% 12,500
On balance `5,00,000 @ 20% 1,00,000
Tax before health and education cess 1,12,500
Add: health & education cess @ 4% 4,500
Tax Liability 1,17,000

(iv) Total income 12,00,000


On first `2,50,000 Nil
On next `2,50,000 @ 5% 12,500
On balance `5,00,000 @ 20% 1,00,000
On balance `2,00,000 @ 30% 60,000
Tax before health and education cess 1,72,500
Add: health & education cess @ 4% 6,900
Tax Liability 1,79,400

(v) Total income 20,00,000


On first `2,50,000 Nil
On next `2,50,000 @ 5% 12,500
On balance `5,00,000 @ 20% 1,00,000
On balance `10,00,000 @ 30% 3,00,000
Tax before health and education cess 4,12,500
Add: health & education cess @ 4% 16,500
Tax Liability 4,29,000

Question 3: Explain Health and Education Cess


Answer: Health and Education Cess
If any tax is charged for any specific purpose, it is called Cess. Health and Education Cess shall be charged
@ 4% on the amount of income tax.
Rounding off of Tax Section 288B
Any amount payable, and the amount of refund due, shall be rounded off in the multiples of `10 in the
similar manner as in case of total income under section 288A.
Question 4: Explain Previous Year and Assessment Year
Answer: Every person has to pay tax on Income of a particular financial year and such year is called
previous year. Further computation of income and tax liability is computed in the subsequent year and it is
called assessment year, eg. if income is to be computed for financial year 2018-19, it will be called previous
year and subsequent year i.e. 2019-20 shall be called assessment year. The term previous year is defined u/s
3 and assessment year is defined u/s 2(9).
Question 5: Explain Budget / Finance Bill / Finance Act
Answer: Budget / Finance Bill / Finance Act
Every year budget is presented in general on the last date of February and all the amendments are given in
general in the budget e.g. Budget presented in 2018 shall be called budget 2018 and subsequently it will be
called Finance Bill 2018 and after it has been passed by the parliament and signed by the President, it will be
called Finance Act 2018 and its provisions shall be applicable from previous year 2018-19/ assessment year
2019-20.
Computation of Total Income And Tax Liability 20

For the students appearing in May/Nov-2019, previous year shall be 2018-19 and assessment year shall be
2019-20.

Illustration 1:
Mr. X has income as given below:
Income under the head Salary 4,00,000
Income under the head House Property 5,00,000
Income under the head Business/Profession 6,30,253
Deductions allowed under section 80C to 80U are `1,10,000.
Compute the income the tax liability for previous year 2018-19.
Solution:
Computation of Total Income of Mr. X
Previous Year 2018-19, Assessment Year 2019-20
`
Income under the head Salary 4,00,000.00
Income under the head House Property 5,00,000.00
Income under the Business/Profession 6,30,253.00
Gross Total Income 15,30,253.00
Less: Deduction u/s 80C to 80U (1,10,000.00)
Total Income 14,20,253.00
Rounded off u/s 288A 14,20,250.00
Computation of Tax Liability
Total Income 14,20,250.00
First 2,50,000 - Nil
On next 2,50,000 x 5% - 12,500.00
On next 5,00,000 x 20% - 1,00,000.00
On balance 4,20,250 x 30% - 1,26,075.00
2,38,575.00
Add: HEC @ 4% 9,543.00
Tax Liability 2,48,118.00
Rounded off u/s 288B 2,48,120.00
(b) Presume he has completed age of 60 years as on 31.03.2019.
Solution:
Computation of Total Income of Mr. X
Previous Year 2018-19, Assessment Year 2019-20
`
Income under the head Salary 4,00,000.00
Income under the head House Property 5,00,000.00
Income under the Business/Profession 6,30,253.00
Gross Total Income 15,30,253.00
Less: Deduction u/s 80C to 80U (1,10,000.00)
Total Income 14,20,253.00
Rounded off u/s 288A 14,20,250.00
Computation of Tax Liability
Total Income 14,20,250.00
First 3,00,000 - Nil
On next 2,00,000 x 5% - 10,000.00
On next 5,00,000 x 20% - 1,00,000.00
On balance 4,20,250 x 30% - 1,26,075.00
2,36,075.00
Add: HEC @ 4% 9,443.00
Tax Liability 2,45,518.00
Rounded off u/s 288B 2,45,520.00
Computation of Total Income And Tax Liability 21

(c) Presume he has completed age of 80 years as on 31.03.2019.


Solution:
Computation of Total Income of Mr. X
Previous Year 2018-19, Assessment Year 2019-20
`
Income under the head Salary 4,00,000.00
Income under the head House Property 5,00,000.00
Income under the Business/Profession 6,30,253.00
Gross Total Income 15,30,253.00
Less: Deduction u/s 80C to 80U (1,10,000.00)
Total Income 14,20,253.00
Rounded off u/s 288A 14,20,250.00
Computation of Tax Liability
Total Income 14,20,250.00
First 5,00,000 - Nil
On next 5,00,000 x 20% - 1,00,000.00
On balance 4,20,250 x 30% - 1,26,075.00
2,26,075.00
Add: HEC @ 4% 9,043.00
Tax Liability 2,35,118.00
Rounded off u/s 288B 2,35,120.00

Illustration 2:
Compute tax liability in the following cases for the assessment year 2019-20.
(i) Mr. X (resident) has total income of `15,00,000
(ii) Mr. X (non-resident) has total income of `15,00,000
(iii) Mrs. X (resident) has total income of `15,00,000
(iv) Mrs. X (non-resident) has total income of `15,00,000
(v) Mr. X (resident), aged 60 years has total income of `15,00,000
(vi) Mrs. X (resident), aged 60 years has total income of `15,00,000
(vii) Mr. X (non-resident), aged 60 years has total income of `15,00,000
(viii) Mrs. X (non-resident), aged 60 years has total income of `15,00,000
(ix) Mr. X (resident), aged 80 years has total income of `15,00,000
(x) Mrs. X (resident), aged 80 years has total income of `15,00,000
(xi) Mr. X (non-resident), aged 80 years has total income of `15,00,000
(xii) Mrs. X (non-resident), aged 80 years has total income of `15,00,000
Solution: `
(i) Computation of Tax Liability
Total Income 15,00,000.00
First 2,50,000 - Nil
On next 2,50,000 x 5% - 12,500.00
On next 5,00,000 x 20% - 1,00,000.00
On balance 5,00,000 x 30%- 1,50,000.00
Tax before health and education cess 2,62,500.00
Add: HEC @ 4% 10,500.00
Tax Liability 2,73,000.00

(ii) Computation of Tax Liability


Total Income 15,00,000.00
First 2,50,000 - Nil
On next 2,50,000 x 5% - 12,500.00
On next 5,00,000 x 20% - 1,00,000.00
On balance 5,00,000 x 30%- 1,50,000.00
Computation of Total Income And Tax Liability 22

Tax before health and education cess 2,62,500.00


Add: HEC @ 4% 10,500.00
Tax Liability 2,73,000.00
(iii) Computation of Tax Liability
Total Income 15,00,000.00
First 2,50,000 - Nil
On next 2,50,000 x 5% - 12,500.00
On next 5,00,000 x 20% - 1,00,000.00
On balance 5,00,000 x 30%- 1,50,000.00
Tax before health and education cess 2,62,500.00
Add: HEC @ 4% 10,500.00
Tax Liability 2,73,000.00
(iv) Computation of Tax Liability
Total Income 15,00,000.00
First 2,50,000 - Nil
On next 2,50,000 x 5% - 12,500.00
On next 5,00,000 x 20% - 1,00,000.00
On balance 5,00,000 x 30%- 1,50,000.00
Tax before health and education cess 2,62,500.00
Add: HEC @ 4% 10,500.00
Tax Liability 2,73,000.00
(v) Computation of Tax Liability
Total Income 15,00,000.00
First 3,00,000 - Nil
On next 2,00,000 x 5% - 10,000.00
On next 5,00,000 x 20% - 1,00,000.00
On balance 5,00,000 x 30%- 1,50,000.00
Tax before health and education cess 2,60,000.00
Add: HEC @ 4% 10,400.00
Tax Liability 2,70,400.00

(vi) Computation of Tax Liability


Total Income 15,00,000.00
First 3,00,000 - Nil
On next 2,00,000 x 5% - 10,000.00
On next 5,00,000 x 20% - 1,00,000.00
On balance 5,00,000 x 30%- 1,50,000.00
Tax before health and education cess 2,60,000.00
Add: HEC @ 4% 10,400.00
Tax Liability 2,70,400.00

(vii) Computation of Tax Liability


Total Income 15,00,000.00
First 2,50,000 - Nil
On next 2,50,000 x 5% - 12,500.00
On next 5,00,000 x 20% - 1,00,000.00
On balance 5,00,000 x 30%- 1,50,000.00
Tax before health and education cess 2,62,500.00
Add: HEC @ 4% 10,500.00
Tax Liability 2,73,000.00

(viii) Computation of Tax Liability


Total Income 15,00,000.00
Computation of Total Income And Tax Liability 23

First 2,50,000 - Nil


On next 2,50,000 x 5% - 12,500.00
On next 5,00,000 x 20% - 1,00,000.00
On balance 5,00,000 x 30%- 1,50,000.00
Tax before health and education cess 2,62,500.00
Add: HEC @ 4% 10,500.00
Tax Liability 2,73,000.00

(ix) Computation of Tax Liability


Total Income 15,00,000.00
First 5,00,000 - Nil
On next 5,00,000 x 20% - 1,00,000.00
On balance 5,00,000 x 30%- 1,50,000.00
Tax before health and education cess 2,50,000.00
Add: HEC @ 4% 10,000.00
Tax Liability 2,60,000.00

(x) Computation of Tax Liability


Total Income 15,00,000.00
First 5,00,000 - Nil
On next 5,00,000 x 20% - 1,00,000.00
On balance 5,00,000 x 30%- 1,50,000.00
Tax before health and education cess 2,50,000.00
Add: HEC @ 4% 10,000.00
Tax Liability 2,60,000.00

(xi) Computation of Tax Liability


Total Income 15,00,000.00
First 2,50,000 - Nil
On next 2,50,000 x 5% - 12,500.00
On next 5,00,000 x 20% - 1,00,000.00
On balance 5,00,000 x 30%- 1,50,000.00
Tax before health and education cess 2,62,500.00
Add: HEC @ 4% 10,500.00
Tax Liability 2,73,000.00

(xii) Computation of Tax Liability


Total Income 15,00,000.00
First 2,50,000 - Nil
On next 2,50,000 x 5% - 12,500.00
On next 5,00,000 x 20% - 1,00,000.00
On balance 5,00,000 x 30%- 1,50,000.00
Tax before health and education cess 2,62,500.00
Add: HEC @ 4% 10,500.00
Tax Liability 2,73,000.00

Question 6: Explain surcharge in case of individual.


Answer: Surcharge shall be applicable
- @ 10% provided total income is exceeding ` 50 lakhs but it is upto ` 1 crore (i.e.100 lakhs) .
- @ 15% provided total income is exceeding ` 1 crore.
Health & education cess shall be charged on the total of tax plus surcharge.
e.g. Mr. X has total income of `110,00,000, in this case his tax liability shall be
Total Income 110,00,000.00
Computation of Total Income And Tax Liability 24

Tax at slab rate 31,12,500.00


Add: Surcharge @ 15% 4,66,875.00
Tax before health and education cess 35,79,375.00
Add: HEC @ 4% 1,43,175.00
Tax Liability 37,22,550.00

If Mr. X has total income of `100,00,000, his tax liability shall be


Total Income 100,00,000.00
Tax at slab rate 28,12,500.00
Add: Surcharge @ 10% 2,81,250.00
Tax before health and education cess 30,93,750.00
Add: HEC @ 4% 1,23,750.00
Tax Liability 32,17,500.00

If Mr. X has total income of `50,00,000, his tax liability shall be


Total Income 50,00,000.00
Tax at slab rate 13,12,500.00
Add: HEC @ 4% 52,500.00
Tax Liability 13,65,000.00

Marginal Relief
If there is marginal increase in income over `50 lakhs/ `100 lakhs, surcharge is applicable on entire amount
of income tax and as a result increase in tax is more than the increase in income. In order to remove this
defect, assessee shall be allowed relief to the extent increase in tax is more than the increase in income and it
is called marginal relief and it can be shown in the manner given below:
e.g. If Mr. X has total income of `51,00,000, his tax liability shall be computed in the manner given below:
Total Income 51,00,000.00
Tax on `51,00,000 at slab rate 13,42,500.00
Add: Surcharge @ 10% 1,34,250.00
Tax before marginal relief 14,76,750.00
Less: Marginal Relief (64,250.00)
Working Note:
Tax + surcharge @10% on income of `51,00,000 14,76,750
Tax on income of `50,00,000 (13,12,500)
Increase in tax 1,64,250
Increase in income 1,00,000
Marginal Relief (1,64,250 – 1,00,000) 64,250
Tax after marginal relief 14,12,500.00
Add: HEC @ 4% 56,500.00
Tax Liability 14,69,000.00
e.g. If Mr. X has total income of `102,00,000, his tax liability shall be computed in the manner given below:
Total Income 102,00,000.00
Tax on `102,00,000 at slab rate 28,72,500.00
Add: Surcharge @ 15% 4,30,875.00
Tax before marginal relief 33,03,375.00
Less: Marginal Relief (9,625.00)
Working Note:
Tax + surcharge @15% on income of `102,00,000 33,03,375
Tax + surcharge @10% on income of `100,00,000 (30,93,750)
Increase in tax 2,09,625
Increase in income 2,00,000
Marginal Relief (2,09,625 – 2,00,000) 9,625
Tax after marginal relief 32,93,750.00
Computation of Total Income And Tax Liability 25

Add: HEC @ 4% 1,31,750.00


Tax Liability 34,25,500.00

e.g. If Mr. X has total income of `101,80,000, his tax liability shall be computed in the manner given below:
Total Income 101,80,000.00
Tax on `101,80,000 at slab rate 28,66,500.00
Add: Surcharge @ 15% 4,29,975.00
Tax before marginal relief 32,96,475.00
Less: Marginal Relief (22,725.00)
Working Note:
Tax + surcharge @15% on income of `101,80,000 32,96,475
Tax + surcharge @10% on income of `100,00,000 (30,93,750)
Increase in tax 2,02,725
Increase in income 1,80,000
Marginal Relief (2,02,725 – 1,80,000) 22,725
Tax after marginal relief 32,73,750.00
Add: HEC @ 4% 1,30,950.00
Tax Liability 34,04,700.00
A person having total income from `50 lakhs to `100 lakhs shall be eligible for marginal relief upto total
income of `51,95,890 and afterwards he will not be eligible for marginal relief when slab of `2,50,000 is
applicable. (for slab rate of `3,00,000, such income shall be `51,95,520 and for slab of `5,00,000, such
income shall be `51,94,030)
A person having total income above `100 lakhs shall be eligible for marginal relief upto total income of
`102,14,690 and afterwards he will not be eligible for marginal relief when slab of `2,50,000 is
applicable. (for slab rate of `3,00,000, such income shall be `102,14,500 and for slab of `5,00,000, such
income shall be `102,13,740)
Illustration 3:
Compute tax liability in the following cases for the assessment year 2019-20.
(i) Mr. X (resident) has total income of `50,05,000
(ii) Mr. X (non-resident) has total income of `52,00,000
(iii) Mrs. X (resident) has total income of `101,00,000
(iv) Mrs. X (non-resident) has total income of `85,00,000
(v) Mr. X (resident), aged 60 years has total income of `106,00,000
(vi) Mrs. X (resident), aged 60 years has total income of `57,00,000
(vii) Mr. X (non-resident), aged 60 years has total income of `108,00,000
(viii) Mrs. X (non-resident), aged 60 years has total income of `101,50,000
(ix) Mr. X (resident), aged 80 years has total income of `54,25,000
(x) Mrs. X (resident), aged 80 years has total income of `102,00,000
(xi) Mr. X (non-resident), aged 80 years has total income of `55,22,380
(xii) Mrs. X (non-resident), aged 80 years has total income of `45,00,000

Solution: `
(i) Computation of Tax Liability
Total Income 50,05,000
Tax on `50,05,000 at slab rate 13,14,000
Add: Surcharge @ 10% 1,31,400
Tax before marginal relief 14,45,400
Less: Marginal Relief (1,27,900)
Working Note:
Tax + surcharge on income of `50,05,000 14,45,400
Tax on income of `50,00,000 (13,12,500)
Increase in tax 1,32,900
Computation of Total Income And Tax Liability 26

Increase in income 5,000


Marginal Relief (1,32,900 – 5,000) 1,27,900
Tax after marginal relief 13,17,500
Add: HEC @ 4% 52,700
Tax Liability 13,70,200

(ii) Computation of Tax Liability


Total Income 52,00,000.00
Tax on `52,00,000 at slab rate 13,72,500.00
Add: Surcharge @ 10% 1,37,250.00
Tax before health & education cess 15,09,750.00
Add: HEC @ 4% 60,390.00
Tax Liability 15,70,140.00

(iii) Computation of Tax Liability


Total Income 101,00,000.00
Tax on `101,00,000 at slab rate 28,42,500.00
Add: Surcharge @ 15% 4,26,375.00
Tax before marginal relief 32,68,875.00
Less: Marginal Relief (75,125.00)
Working Note:
Tax + surcharge @15% on income of `101,00,000 32,68,875
Tax + surcharge @10% on income of `100,00,000 (30,93,750)
Increase in tax 1,75,125
Increase in income 1,00,000
Marginal Relief (1,75,125 – 1,00,000) 75,125
Tax after marginal relief 31,93,750.00
Add: HEC @ 4% 1,27,750.00
Tax Liability 33,21,500.00

(iv) Computation of Tax Liability


Total Income 85,00,000.00
Tax on `85,00,000 at slab rate 23,62,500.00
Add: Surcharge @ 10% 2,36,250.00
Tax before health & education cess 25,98,750.00
Add: HEC @ 4% 1,03,950.00
Tax Liability 27,02,700.00

(v) Computation of Tax Liability


Total Income 106,00,000
Tax on `106,00,000 at slab rate 29,90,000
Add: Surcharge @ 15% 4,48,500
Tax before health & education cess 34,38,500.00
Add: HEC @ 4% 1,37,540.00
Tax Liability 35,76,040.00

(vi) Computation of Tax Liability


Total Income 57,00,000
Tax on `57,00,000 at slab rate 15,20,000
Add: Surcharge @ 10% 1,52,000
Tax before health & education cess 16,72,000.00
Add: HEC @ 4% 66,880.00
Tax Liability 17,38,880.00
Computation of Total Income And Tax Liability 27

(vii) Computation of Tax Liability


Total Income 108,00,000.00
Tax on `108,00,000 at slab rate 30,52,500.00
Add: Surcharge @ 15% 4,57,875.00
Tax before health & education cess 35,10,375.00
Add: HEC @ 4% 1,40,415.00
Tax Liability 36,50,790.00

(viii) Computation of Tax Liability


Total Income 101,50,000.00
Tax on `101,50,000 at slab rate 28,57,500.00
Add: Surcharge @ 15% 4,28,625.00
Tax before marginal relief 32,86,125.00
Less: Marginal Relief (42,375.00)
Working Note:
Tax + surcharge @15% on income of `101,50,000 32,86,125
Tax + surcharge @10% on income of `100,00,000 (30,93,750)
Increase in tax 1,92,375
Increase in income 1,50,000
Marginal Relief (1,92,375 – 1,50,000) 42,375
Tax after marginal relief 32,43,750.00
Add: HEC @ 4% 1,29,750.00
Tax Liability 33,73,500.00

(ix) Computation of Tax Liability


Total Income 54,25,000.00
Tax on `54,25,000 at slab rate 14,27,500.00
Add: Surcharge @ 10% 1,42,750.00
Tax before health & education cess 15,70,250.00
Add: HEC @ 4% 62,810.00
Tax Liability 16,33,060.00
(x) Computation of Tax Liability
Total Income 102,00,000
Tax on `102,00,000 at slab rate 28,60,000
Add: Surcharge @ 15% 4,29,000
Tax before marginal relief 32,89,000
Less: Marginal Relief (9,000)
Working Note:
Tax + surcharge @15% on income of `102,00,000 32,89,000
Tax + surcharge @10% on income of `100,00,000 (30,80,000)
Increase in tax 2,09,000
Increase in income 2,00,000
Marginal Relief (2,09,000 – 2,00,000) 9,000
Tax after marginal relief 32,80,000
Add: HEC @ 4% 1,31,200
Tax Liability 34,11,200

(xi) Computation of Tax Liability


Total Income 55,22,380.00
Tax on `55,22,380 at slab rate 14,69,214.00
Add: Surcharge @ 10% 1,46,921.40
Tax before health & education cess 16,16,135.40
Computation of Total Income And Tax Liability 28

Add: HEC @ 4% 64,645.42


Tax Liability 16,80,780.82
Rounded off u/s 288B 16,80,780.00

(xii) Computation of Tax Liability


Total Income 45,00,000.00
Tax on `45,00,000 at slab rate 11,62,500.00
Add: Surcharge Nil
Tax before health & education cess 11,62,500.00
Add: HEC @ 4% 46,500.00
Tax Liability 12,09,000.00

Question 7: Write a note on Rebate under section 87A.


Answer: Rebate in case of Resident Individual Section 87A
 Rebate i.e. concession from income tax shall be allowed only to RESIDENT INDIVIDUAL (not to
non-resident individual or any other person).
 Rebate shall be allowed only if total income is not exceeding `3,50,000
 Rebate shall be allowed upto `2,500.
 Health & education cess shall be applied only after permitting rebate under section 87A.
 Rebate shall be allowed even from tax on LTCG or STCG under section 111A or Casual Income.
 No Rebate shall be allowed from LTCG u/s 112A.
E.g.
Mr. X has total income of `3,50,000, his tax liability shall be
If in the above case total income is `3,50,000, tax liability shall be
Computation of Tax Liability
Total Income 3,50,000.00
Tax on `3,50,000 at slab rate 5,000.00
Less: Rebate u/s 87A (2,500.00)
Tax before health & education cess 2,500.00
Add: HEC @ 4% 100.00
Tax Liability 2,600.00

If in the above case total income is `3,46,000, tax liability shall be


Computation of Tax Liability
Total Income 3,46,000
Tax on `3,46,000 at slab rate 4,800
Less: Rebate u/s 87A (2,500)
Tax before health & education cess 2,300
Add: HEC @ 4% 92
Tax Liability 2,392
Rounded off u/s 288B 2,390

If in the above case total income is `3,51,000, tax liability shall be


Computation of Tax Liability
Total Income 3,51,000.00
Tax on `3,51,000 at slab rate 5,050.00
Less: Rebate u/s 87A Nil
Tax before health & education cess 5,050.00
Add: HEC @ 4% 202.00
Tax Liability 5,252.00
Rounded off u/s 288B 5,250.00
Computation of Total Income And Tax Liability 29

Illustration 4:
Mr. X has gross total income `4,10,000 and deduction allowed under section 80C to 80U are `60,000.
Compute his tax liability Previous year 2018-19, assessment year 2019-20.
Solution:
Computation of Tax Liability
Gross Total Income 4,10,000
Less: Deduction u/s 80C to 80U (60,000)
Total Income 3,50,000
Tax on `3,50,000 at slab rate 5,000
Less: Rebate u/s 87A (2,500)
Tax before health & education cess 2,500
Add: HEC @ 4% 100
Tax Liability 2,600
(b) Presume he is a resident and is aged 62 years.
Solution:
Computation of Tax Liability
Gross Total Income 4,10,000
Less: Deduction u/s 80C to 80U (60,000)
Total Income 3,50,000
Tax on `3,50,000 at slab rate 2,500
Less: Rebate u/s 87A (2,500)
Tax before health & education cess Nil
Tax Liability Nil
(c) Presume he is a non-resident and is aged 62 years.
Solution:
Computation of Tax Liability
Gross Total Income 4,10,000
Less: Deduction u/s 80C to 80U (60,000)
Total Income 3,50,000
Tax on `3,50,000 at slab rate 5,000
Less: Rebate u/s 87A Nil
Tax before health & education cess 5,000
Add: HEC @ 4% 200
Tax Liability 5,200
(d) Presume he is a non-resident and is aged 82 years.
Solution:
Computation of Tax Liability
Gross Total Income 4,10,000
Less: Deduction u/s 80C to 80U (60,000)
Total Income 3,50,000
Tax on `3,50,000 at slab rate 5,000
Less: Rebate u/s 87A Nil
Tax before health & education cess 5,000
Add: HEC @ 4% 200
Tax Liability 5,200

Illustration 5:
Compute tax liability in the following cases for the assessment year 2019-20.
(i) Mr. X (resident) has total income of `3,20,000
(ii) Mr. X (non-resident) has total income of `3,50,000
(iii) Mrs. X (resident) has total income of `3,45,000
(iv) Mrs. X (non-resident) has total income of `3,20,000
(v) Mr. X (resident), aged 60 years has total income of `3,40,000
Computation of Total Income And Tax Liability 30

(vi) Mrs. X (resident), aged 60 years has total income of `3,40,000


(vii) Mr. X (non-resident), aged 60 years has total income of `3,40,000
(viii) Mrs. X (non-resident), aged 60 years has total income of `3,40,000
(ix) Mr. X (resident), aged 80 years has total income of `6,00,000
(x) Mrs. X (resident), aged 80 years has total income of `6,00,000
(xi) Mr. X (non-resident), aged 80 years has total income of `3,50,000
(xii) Mrs. X (non-resident), aged 80 years has total income of `3,40,000
Solution: `
(i) Computation of Tax Liability
Total Income 3,20,000
Tax on `3,20,000 at slab rate 3,500
Less: Rebate u/s 87A (2,500)
Tax before health & education cess 1,000
Add: HEC @ 4% 40
Tax Liability 1,040

(ii) Computation of Tax Liability


Total Income 3,50,000
Tax on `3,50,000 at slab rate 5,000
Add: HEC @ 4% 200
Tax Liability 5,200
Note: Rebate under section 87A is not allowed to non-resident.

(iii) Computation of Tax Liability


Total Income 3,45,000
Tax on `3,45,000 at slab rate 4,750
Less: Rebate u/s 87A (2,500)
Tax before health & education cess 2,250
Add: HEC @ 4% 90
Tax Liability 2,340

(iv) Computation of Tax Liability


Total Income 3,20,000
Tax on `3,20,000 at slab rate 3,500
Add: HEC @ 4% 140
Tax Liability 3,640
Note: Rebate under section 87A is not allowed to non-resident.

(v) Computation of Tax Liability


Total Income 3,40,000
Tax on `3,40,000 at slab rate 2,000
Less: Rebate u/s 87A (2,000)
Tax Liability Nil

(vi) Computation of Tax Liability


Total Income 3,40,000
Tax on `3,40,000 at slab rate 2,000
Less: Rebate u/s 87A (2,000)
Tax Liability Nil

(vii) Computation of Tax Liability


Total Income 3,40,000
Tax on `3,40,000 at slab rate 4,500
Computation of Total Income And Tax Liability 31

Add: HEC @ 4% 180


Tax Liability 4,680
Note: Rebate under section 87A is not allowed to non-resident.

(viii) Computation of Tax Liability


Total Income 3,40,000
Tax on `3,40,000 at slab rate 4,500
Add: HEC @ 4% 180
Tax Liability 4,680
Note: Rebate under section 87A is not allowed to non-resident.

(ix) Computation of Tax Liability


Total Income 6,00,000
Tax on `6,00,000 at slab rate 20,000
Add: HEC @ 4% 800
Tax Liability 20,800

(x) Computation of Tax Liability


Total Income 6,00,000
Tax on `6,00,000 at slab rate 20,000
Add: HEC @ 4% 800
Tax Liability 20,800

(xi) Computation of Tax Liability


Total Income 3,50,000
Tax on `3,50,000 at slab rate 5,000
Add: HEC @ 4% 200
Tax Liability 5,200
Note: Rebate under section 87A is not allowed to non-resident.

(xii) Computation of Tax Liability


Total Income 3,40,000
Tax on `3,40,000 at slab rate 4,500
Add: HEC @ 4% 180
Tax Liability 4,680
Note: Rebate under section 87A is not allowed to non-resident.

Illustration 6:
Compute tax liability in the following cases for the assessment year 2019-20.
(i) Mr. X (resident) has total income of `3,55,000
(ii) Mr. X (non-resident) has total income of `4,20,000
(iii) Mrs. X (resident) has total income of `3,08,000
(iv) Mrs. X (non-resident) has total income of `12,00,000
(v) Mr. X (resident), aged 60 years has total income of `22,00,000
(vi) Mrs. X (resident), aged 60 years has total income of `105,00,000
(vii) Mr. X (non-resident), aged 60 years has total income of `70,00,000
(viii) Mrs. X (non-resident), aged 60 years has total income of `3,00,000
(ix) Mr. X (resident), aged 80 years has total income of `3,99,000
(x) Mrs. X (resident), aged 80 years has total income of `103,00,000
(xi) Mr. X (non-resident), aged 80 years has total income of `12,00,000
(xii) Mrs. X (non-resident), aged 80 years has total income of `9,00,000
Computation of Total Income And Tax Liability 32

Solution: `
(i) Computation of Tax Liability
Total Income 3,55,000.00
Tax on `3,55,000 at slab rate 5,250.00
Add: HEC @ 4% 210.00
Tax Liability 5,460.00

(ii) Computation of Tax Liability


Total Income 4,20,000
Tax on `4,20,000 at slab rate 8,500
Add: HEC @ 4% 340
Tax Liability 8,840

(iii) Computation of Tax Liability


Total Income 3,08,000
Tax on `3,08,000 at slab rate 2,900
Less: Rebate u/s 87A (2,500)
Tax before health & education cess 400
Add: HEC @ 4% 16
Tax Liability 416
Rounded off u/s 288B 420

(iv) Computation of Tax Liability


Total Income 12,00,000
Tax on `12,00,000 at slab rate 1,72,500
Add: HEC @ 4% 6,900
Tax Liability 1,79,400

(v) Computation of Tax Liability


Total Income 22,00,000
Tax on `22,00,000 at slab rate 4,70,000
Add: HEC @ 4% 18,800
Tax Liability 4,88,800

(vi) Computation of Tax Liability


Total Income 105,00,000
Tax on `105,00,000 at slab rate 29,60,000
Add: Surcharge @ 15% 4,44,000
Tax before health & education cess 34,04,000
Add: HEC @ 4% 1,36,160
Tax Liability 35,40,160

(vii) Computation of Tax Liability


Total Income 70,00,000
Tax on `70,00,000 at slab rate 19,12,500
Add: Surcharge @ 10% 1,91,250
Tax before health & education cess 21,03,750
Add: HEC @ 4% 84,150
Tax Liability 21,87,900

(viii) Computation of Tax Liability


Total Income 3,00,000
Tax on `3,00,000 at slab rate 2,500
Computation of Total Income And Tax Liability 33

Add: HEC @ 4% 100


Tax Liability 2,600
Note: Rebate under section 87A is not allowed for non-resident.

(ix) Computation of Tax Liability


Total Income 3,99,000
Tax on `3,99,000 at slab rate Nil
Tax Liability Nil

(x) Computation of Tax Liability


Total Income 103,00,000
Tax on `103,00,000 at slab rate 28,90,000
Add: Surcharge @ 15% 4,33,500
Tax before health & education cess 33,23,500
Add: HEC @ 4% 1,32,940
Tax Liability 34,56,440

(xi) Computation of Tax Liability


Total Income 12,00,000
Tax on `12,00,000 at slab rate 1,72,500
Add: HEC @ 4% 6,900
Tax Liability 1,79,400

(xii) Computation of Tax Liability


Total Income 9,00,000
Tax on `9,00,000 at slab rate 92,500
Add: HEC @ 4% 3,700
Tax Liability 96,200

Question 8: Explain taxability of Casual Income.


Answer: As per section 115BB, casual income shall be taxable @ 30%.
As per section 2(24)(ix), casual income means any winnings from lotteries, crossword puzzles, races
including horse races, card games and other games of any sort or from gambling or betting of any form or
nature whatsoever.
Lottery includes winnings from prizes awarded to any person by draw of lots or by chance or in any other
manner whatsoever, under any scheme or arrangement by whatever name called.
Card game and other game of any sort includes any game show, an entertainment programme on television
or electronic mode, in which people compete to win prizes or any other similar game.
Casual income shall be taxable under the head Other Sources and it will be included in the gross total
income and also total income but while computing tax liability, casual income shall be separated from total
income and shall be taxable @ 30%.

If assessee has incurred any expenditure in connection with earning of casual income, such expenditure shall
not be allowed to be deducted, eg. Mr. X purchased lottery tickets of `10,000 and he had a winning of `
1,00,000, in this case expenditure of `10,000 shall not be allowed to be deducted and income of `1,00,000
shall be taxable @ 30%.

As per section 58(4), deduction under section 80C to 80U shall not be allowed from casual income however
as per section 87A, rebate shall be allowed.

Illustration 7:
Mr. X has income under the head Salary `5,00,000 and casual income `3,00,000 and deduction under
section 80C to 80U `2,00,000, in this case his tax liability shall be
Computation of Total Income And Tax Liability 34

Computation of Total Income of Mr. X


Previous Year 2018-19, Assessment Year 2019-20
`
Income under the head Salary 5,00,000.00
Income under the Other Sources (Casual income) 3,00,000.00
Gross Total Income 8,00,000.00
Less: Deduction u/s 80C to 80U (2,00,000.00)
Total Income 6,00,000.00
Computation of Tax Liability
Tax on casual income `3,00,000 @ 30% 90,000.00
Tax on normal income `3,00,000 at slab rate 2,500.00
Tax before health & education cess 92,500.00
Add: HEC @ 4% 3,700.00
Tax Liability 96,200.00

(b) If in the above case deduction allowed under section 80C to 80U is `6,00,000, tax liability shall be
Computation of Total Income of Mr. X
Previous Year 2018-19, Assessment Year 2019-20
`
Income under the head Salary 5,00,000.00
Income under the Other Sources (Casual income) 3,00,000.00
Gross Total Income 8,00,000.00
Less: Deduction u/s 80C to 80U (5,00,000.00)
Total Income 3,00,000.00
Computation of Tax Liability
Tax on casual income `3,00,000 @ 30% 90,000.00
Less: Rebate u/s 87A (2,500.00)
Tax before health & education cess 87,500.00
Add: HEC @ 4% 3,500.00
Tax Liability 91,000.00

Illustration 8: Mr. X has casual income of `102,00,000 and deduction allowed under section 80C to 80U
are `5,00,000, in this case his tax liability shall be
Computation of Total Income of Mr. X
Previous Year 2018-19, Assessment Year 2019-20
`
Income under the Other Sources (Casual income) 102,00,000.00
Gross Total Income 102,00,000.00
Less: Deduction u/s 80C to 80U Nil
Total Income 102,00,000.00
Computation of Tax Liability
Tax on casual income `102,00,000 @ 30% 30,60,000.00
Add: Surcharge @ 15% 4,59,000.00
Tax before marginal relief 35,19,000.00
Less: Marginal Relief (19,000.00)
Working Note:
Tax + surcharge on income of `102,00,000 35,19,000
Tax + surcharge on income of `100,00,000 (33,00,000)
Increase in tax 2,19,000
Increase in income 2,00,000
Marginal Relief (2,19,000 – 2,00,000) 19,000
Tax after marginal relief 35,00,000.00
Add: HEC @ 4% 1,40,000.00
Computation of Total Income And Tax Liability 35

Tax Liability 36,40,000.00

Illustration 9: Mr. X has income under the head Salary `70,000 and casual income `2,50,000 and deduction
u/s 80C to 80U `40,000. Compute his tax liability assessment year 2019-20.
Solution:
Computation of Total Income of Mr. X
Previous Year 2018-19, Assessment Year 2019-20
`
Income under the head Salary 70,000.00
Income under the Other Sources (Casual income) 2,50,000.00
Gross Total Income 3,20,000.00
Less: Deduction u/s 80C to 80U (40,000.00)
Total Income 2,80,000.00
Computation of Tax Liability
Tax on casual income `2,50,000 @ 30% 75,000.00
Tax on normal income `30,000 at slab rate Nil
Less: Rebate u/s 87A (2,500.00)
Tax before health & education cess 72,500.00
Add: HEC @ 4% 2,900.00
Tax Liability 75,400.00
(b) Presume he is non-resident.
Solution:
Computation of Total Income of Mr. X
Previous Year 2018-19, Assessment Year 2019-20
`
Income under the head Salary 70,000.00
Income under the Other Sources (Casual income) 2,50,000.00
Gross Total Income 3,20,000.00
Less: Deduction u/s 80C to 80U (40,000.00)
Total Income 2,80,000.00
Computation of Tax Liability
Tax on casual income `2,50,000 @ 30% 75,000.00
Tax on normal income `30,000 at slab rate Nil
Less: Rebate u/s 87A Nil
Tax before health & education cess 75,000.00
Add: HEC @ 4% 3,000.00
Tax Liability 78,000.00

A person having Total Income from `50 lakhs to `100 lakhs shall be eligible for marginal relief upto total
income of `52,23,880 in case of casual income.
A person having Total Income above `100 lakhs shall be eligible for marginal relief upto total income of
`102,29,000 in case of casual income.

Illustration 10: Mr. X has casual income ` 100 lakh, in this case his Tax Liability for P.Y. 2018-19
A.Y. 2019-20 shall be-
Computation of Tax Liability
Tax on casual income `100,00,000 @ 30% 30,00,000.00
Add: Surcharge @ 10% 3,00,000.00
Tax before health & education cess 33,00,000.00
Add: HEC @ 4% 1,32,000.00
Tax Liability 34,32,000.00
Computation of Total Income And Tax Liability 36

(b) Mr. X has casual income ` 101 lakh, in this case his Tax Liability for P.Y. 2018-19 A.Y. 2019-20
shall be-
Computation of Tax Liability
Tax on casual income `101,00,000 @ 30% 30,30,000.00
Add: Surcharge @ 15% 4,54,500.00
Tax before marginal relief 34,84,500.00
Less: Marginal Relief (84,500.00)
Working Note:
Tax + surcharge on income of `101,00,000 34,84,500
Tax + surcharge on income of `100,00,000 (33,00,000)
Increase in tax 1,84,500
Increase in income 1,00,000
Marginal Relief (1,84,500 – 1,00,000) 84,500
Tax after marginal relief 34,00,000.00
Add: HEC @ 4% 1,36,000.00
Tax Liability 35,36,000.00

(c) Mr. X has casual income ` 102 lakh, in this case his Tax Liability for P.Y. 2018-19 A.Y. 2019-20
shall be-
Computation of Tax Liability
Tax on casual income `102,00,000 @ 30% 30,60,000.00
Add: Surcharge @ 15% 4,59,000.00
Tax before marginal relief 35,19,000.00
Less: Marginal Relief (19,000.00)
Working Note:
Tax + surcharge on income of `102,00,000 35,19,000
Tax + surcharge on income of `100,00,000 (33,00,000)
Increase in tax 2,19,000
Increase in income 2,00,000
Marginal Relief (2,19,000 – 2,00,000) 19,000
Tax after marginal relief 35,00,000.00
Add: HEC @ 4% 1,40,000.00
Tax Liability 36,40,000.00

(d) Mr. X has casual income `103 lakh, in this case his Tax Liability for P.Y. 2018-19 A.Y. 2019-20
shall be-
Computation of Tax Liability
Tax on casual income `103,00,000 @ 30% 30,90,000.00
Add: Surcharge @ 15% 4,63,500.00
Tax before health & education cess 35,53,500.00
Add: HEC @ 4% 1,42,140.00
Tax Liability 36,95,640.00

Question 9: Explain taxability of Capital Gains.


Answer: If any capital asset has been transferred like land, building, gold etc. profit shall be called capital
gains and if the asset has been transferred within a period of three years, capital gains shall be short term and
shall be taxable at the normal rate and if asset is sold after 3 years, it will be long term capital gain and as
per section 112, it shall be taxable @ 20% and also deductions under section 80C to 80U i.e. Chapter VI-A,
shall not be allowed from long term capital gains.
In case of listed shares or units of equity oriented mutual fund etc., period of three years shall be taken as
one year.
If any person has transferred listed equity shares or listed units of equity oriented mutual funds or listed units
of a business trust and has paid securities transaction tax, in such cases long term capital gain shall be
Computation of Total Income And Tax Liability 37

taxable @ 10% u/s 112A but only amount in excess of `1,00,000 and short term capital gains shall be
covered under section 111A and shall be taxable @ 15% and deductions under section 80C to 80U i.e.
Chapter VI-A, shall not be allowed from such short term capital gains.
Equity oriented mutual fund means such mutual funds in which more than 65% of the total proceeds have
been invested in the equity shares of the domestic company.
Rebate u/s 87A shall be allowed from tax on LTCG or STCG 111A. (No Rebate u/s 87A from LTCG 112A)
Special provision for resident individual
In case of a resident individual if total income excluding long term capital gains and short term capital gain
covered under section 111A and casual income is below the amount which is exempt from income tax
(i.e.2,50,000/3,00,000/5,00,000), in such cases deficiency in the exemption shall be allowed from long term
capital gains or short term capital gain under section 111A as the case may be. Such benefit is not allowed to
a non-resident.

Illustration 11:
Mr. X has incomes as given below:
 Income under the head salary 35,000
 Income under the head house property 45,000
 Income under the head business/profession 30,000
 Long term capital gains 1,10,000
 Long term capital gains u/s 112A 5,00,000
 Short term capital gains 25,000
 Short term capital gains u/s 111A 7,00,000
 Casual Income (winnings of lottery) 55,000
 Other income 3,000
Deductions allowed under section 80C to 80U 2,00,000
Compute his tax liability for the assessment year 2019-20.
Solution: `
Computation of Total Income
 Income under the head salary 35,000
 Income under the head house property 45,000
 Income under the head business/profession 30,000
 Long term capital gains 1,10,000
 Long term capital gains u/s 112A (5,00,000-1,00,000) 4,00,000
 Short term capital gains 25,000
 Short term capital gains u/s 111A 7,00,000
 Casual Income (winnings of lottery) 55,000
 Other income 3,000
Gross Total Income 14,03,000
Less: Deduction u/s 80C to 80U (1,38,000)
Total Income 12,65,000

Computation of Tax Liability


Tax on Long term capital gains (`1,10,000 – 1,10,000)@ 20% u/s 112 Nil
Tax on Short term capital gains (`7,00,000 – 1,40,000)@ 15% u/s 111A 84,000
Tax on Long term capital gains 4,00,000 @ 10% u/s 112A 40,000
Tax on Casual Income `55,000 @ 30% u/s 115BB 16,500
Tax on normal income Nil
Tax before health & education cess 1,40,500
Add: HEC @ 4% 5,620
Tax Liability 1,46,120
Computation of Total Income And Tax Liability 38

(b) Presume he is non-resident.


Solution: `
Computation of Total Income
 Income under the head salary 35,000
 Income under the head house property 45,000
 Income under the head business/profession 30,000
 Long term capital gains 1,10,000
 Long term capital gains u/s 112A (5,00,000-1,00,000) 4,00,000
 Short term capital gains 25,000
 Short term capital gains u/s 111A 7,00,000
 Casual Income (winnings of lottery) 55,000
 Other income 3,000
Gross Total Income 14,03,000
Less: Deduction u/s 80C to 80U (1,38,000)
Total Income 12,65,000
Computation of Tax Liability
Tax on Long term capital gains `1,10,000 @ 20% u/s 112 22,000
Tax on Short term capital gains `7,00,000 @ 15% u/s 111A 1,05,000
Tax on Long term capital gains `4,00,000 @ 10% u/s 112A 40,000
Tax on Casual Income `55,000 @ 30% u/s 115BB 16,500
Tax on normal income Nil
Tax before health & education cess 1,83,500
Add: HEC @ 4% 7,340
Tax Liability 1,90,840

(c) Presume he is resident and is aged 62 years.


Solution: `
Computation of Total Income
 Income under the head salary 35,000
 Income under the head house property 45,000
 Income under the head business/profession 30,000
 Long term capital gains 1,10,000
 Long term capital gains u/s 112A (5,00,000-1,00,000) 4,00,000
 Short term capital gains 25,000
 Short term capital gains u/s 111A 7,00,000
 Casual Income (winnings of lottery) 55,000
 Other income 3,000
Gross Total Income 14,03,000
Less: Deduction u/s 80C to 80U (1,38,000)
Total Income 12,65,000
Computation of Tax Liability
Tax on Long term capital gains (`1,10,000 – 1,10,000)@ 20% u/s 112 Nil
Tax on Short term capital gains (`7,00,000 – 1,90,000)@ 15% u/s 111A 76,500
Tax on Long term capital gains `4,00,000 @ 10% u/s 112A 40,000
Tax on Casual Income `55,000 @ 30% u/s 115BB 16,500
Tax on normal income Nil
Tax before health & education cess 1,33,000
Add: HEC @ 4% 5,320
Tax Liability 1,38,320
Computation of Total Income And Tax Liability 39

(d) Presume he is resident and is aged 82 years.


Solution: `
Computation of Total Income
 Income under the head salary 35,000
 Income under the head house property 45,000
 Income under the head business/profession 30,000
 Long term capital gains 1,10,000
 Long term capital gains u/s 112A (5,00,000-1,00,000) 4,00,000
 Short term capital gains 25,000
 Short term capital gains u/s 111A 7,00,000
 Casual Income (winnings of lottery) 55,000
 Other income 3,000
Gross Total Income 14,03,000
Less: Deduction u/s 80C to 80U (1,38,000)
Total Income 12,65,000
Computation of Tax Liability
Tax on Long term capital gains (`1,10,000 – 1,10,000)@ 20% u/s 112 Nil
Tax on Short term capital gains (`7,00,000 – 3,90,000)@ 15% u/s 111A 46,500
Tax on Long term capital gains `4,00,000 @ 10% u/s 112A 40,000
Tax on Casual Income `55,000 @ 30% u/s 115BB 16,500
Tax on normal income Nil
Tax before health & education cess 1,03,000
Add: HEC @ 4% 4,120
Tax Liability 1,07,120

Illustration 12: Compute tax liability for the assessment year 2019-20 in the following situations:
(i) Mr. X is resident in India and has income under the head house property `40,000 and income under the
head salary `30,000 and long term capital gains `4,80,000.
(ii) Presume in the above situation the assessee is Mrs. X.
(iii) Presume in the above situation the assessee is Mrs. X and she is aged about 70 years.
(iv) Presume in the above situation the assessee is Mr. X and he is aged about 70 years.
(v) Presume in the above situation the assessee is Mrs. X and she is aged about 85 years.
(vi) Presume in the above situation the assessee is Mr. X and he is aged about 85 years.
(vii) Presume in all the above situations, the assessee is non-resident in India.
Solution: `
(i)
Computation of Total Income
Income under the head Salary 30,000
Income under the head House Property 40,000
Income under the head Capital Gains (LTCG) 4,80,000
Gross Total Income 5,50,000
Less: Deduction u/s 80C to 80U Nil
Total Income 5,50,000
Computation of Tax Liability
Tax on LTCG `3,00,000 (4,80,000 – 1,80,000) @ 20% u/s 112 60,000
Tax on `70,000 at slab rate Nil
Tax before health & education cess 60,000
Add: HEC @ 4% 2,400
Tax Liability 62,400

(ii)
Total Income 5,50,000
Computation of Total Income And Tax Liability 40

Computation of Tax Liability


Tax on LTCG `3,00,000 (4,80,000 – 1,80,000) @ 20% u/s 112 60,000
Tax on `70,000 at slab rate Nil
Tax before health & education cess 60,000
Add: HEC @ 4% 2,400
Tax Liability 62,400

(iii)
Total Income 5,50,000
Computation of Tax Liability
Tax on LTCG `2,50,000 (4,80,000 – 2,30,000) @ 20% u/s 112 50,000
Tax on `70,000 at slab rate Nil
Tax before health & education cess 50,000
Add: HEC @ 4% 2,000
Tax Liability 52,000

(iv)
Total Income 5,50,000
Computation of Tax Liability
Tax on LTCG `2,50,000 (4,80,000 – 2,30,000) @ 20% u/s 112 50,000
Tax on `70,000 at slab rate Nil
Tax before health & education cess 50,000
Add: HEC @ 4% 2,000
Tax Liability 52,000

(v)
Total Income 5,50,000
Computation of Tax Liability
Tax on LTCG `50,000 (4,80,000 – 4,30,000) @ 20% u/s 112 10,000
Tax on `70,000 at slab rate Nil
Tax before health & education cess 10,000
Add: HEC @ 4% 400
Tax Liability 10,400

(vi)
Total Income 5,50,000
Computation of Tax Liability
Tax on LTCG `50,000 (4,80,000 – 4,30,000) @ 20% u/s 112 10,000
Tax on `70,000 at slab rate Nil
Tax before health & education cess 10,000
Add: HEC @ 4% 400
Tax Liability 10,400

(vii)
In situation (i)
Total Income 5,50,000
Computation of Tax Liability
Tax on LTCG `4,80,000 @ 20% u/s 112 96,000
Tax on `70,000 at slab rate Nil
Tax before health & education cess 96,000
Add: HEC @ 4% 3,840
Tax Liability 99,840
Computation of Total Income And Tax Liability 41

In situation (ii)
Total Income 5,50,000
Computation of Tax Liability
Tax on LTCG `4,80,000 @ 20% u/s 112 96,000
Tax on `70,000 at slab rate Nil
Tax before health & education cess 96,000
Add: HEC @ 4% 3,840
Tax Liability 99,840
In situation (iii)
Total Income 5,50,000
Computation of Tax Liability
Tax on LTCG `4,80,000 @ 20% u/s 112 96,000
Tax on `70,000 at slab rate Nil
Tax before health & education cess 96,000
Add: HEC @ 4% 3,840
Tax Liability 99,840
In situation (iv)
Total Income 5,50,000
Computation of Tax Liability
Tax on LTCG `4,80,000 @ 20% u/s 112 96,000
Tax on `70,000 at slab rate Nil
Tax before health & education cess 96,000
Add: HEC @ 4% 3,840
Tax Liability 99,840
In situation (v)
Total Income 5,50,000
Computation of Tax Liability
Tax on LTCG `4,80,000 @ 20% u/s 112 96,000
Tax on `70,000 at slab rate Nil
Tax before health & education cess 96,000
Add: HEC @ 4% 3,840
Tax Liability 99,840
In situation (vi)
Total Income 5,50,000
Computation of Tax Liability
Tax on LTCG `4,80,000 @ 20% u/s 112 96,000
Tax on `70,000 at slab rate Nil
Tax before health & education cess 96,000
Add: HEC @ 4% 3,840
Tax Liability 99,840

Illustration 13: Compute tax liability in the following cases


(i) Mr. X a resident has long term capital gains `3,50,000.
(ii) Mr. X a resident has casual income `3,50,000.
(iii) Mr. X a resident has short term capital gains u/s 111A `3,50,000.
(iv) Mr. X a non-resident has long term capital gains `3,50,000.
(v) Mr. X a non-resident has casual income `3,50,000.
(vi) Mr. X a non-resident has short term capital gains u/s 111A `3,50,000.
(vii) Mr. X a non-resident aged 61 years has long term capital gains `3,50,000.
(viii) Mr. X a non-resident aged 61 years has casual income `3,50,000.
(ix) Mr. X a non-resident aged 61 years has short term capital gains u/s 111A `3,50,000.
Computation of Total Income And Tax Liability 42

Solution: `
(i)
Income under the head capital gains (LTCG) 3,50,000
Total Income 3,50,000
Computation of Tax Liability
Tax on Long term capital gains `1,00,000 (`3,50,000 – `2,50,000)@ 20% u/s 112 20,000
Less: Rebate u/s 87A (2,500)
Tax before health & education cess 17,500
Add: HEC @ 4% 700
Tax Liability 18,200

(ii)
Income under the head other sources (casual income) 3,50,000
Total Income 3,50,000
Computation of Tax Liability
Tax on casual income `3,50,000 @ 30% u/s 115BB 1,05,000
Less: Rebate u/s 87A (2,500)
Tax before health & education cess 1,02,500
Add: HEC @ 4% 4,100
Tax Liability 1,06,600

(iii)
Income under the head capital gains (STCG u/s 111A) 3,50,000
Total Income 3,50,000
Computation of Tax Liability
Tax on Short term capital gains `1,00,000 (`3,50,000 – `2,50,000)@ 15% u/s 111A 15,000
Less: Rebate u/s 87A (2,500)
Tax before health & education cess 12,500
Add: HEC @ 4% 500
Tax Liability 13,000

(iv)
Income under the head capital gains (LTCG) 3,50,000
Total Income 3,50,000

Computation of Tax Liability


Tax on Long term capital gains `3,50,000 @ 20% u/s 112 70,000
Add: HEC @ 4% 2,800
Tax Liability 72,800

(v)
Income under the head other sources (casual income) 3,50,000
Total Income 3,50,000
Computation of Tax Liability
Tax on casual income `3,50,000 @ 30% u/s 115BB 1,05,000
Add: HEC @ 4% 4,200
Tax Liability 1,09,200

(vi)
Income under the head capital gains (STCG u/s 111A) 3,50,000
Total Income 3,50,000
Computation of Tax Liability
Tax on Short term capital gains `3,50,000 @ 15% u/s 111A 52,500
Computation of Total Income And Tax Liability 43

Add: HEC @ 4% 2,100


Tax Liability 54,600

(vii)
Income under the head capital gains (LTCG) 3,50,000
Total Income 3,50,000
Computation of Tax Liability
Tax on Long term capital gains `3,50,000 @ 20% u/s 112 70,000
Add: HEC @ 4% 2,800
Tax Liability 72,800

(viii)
Income under the head other sources (casual income) 3,50,000
Total Income 3,50,000
Computation of Tax Liability
Tax on casual income `3,50,000 @ 30% u/s 115BB 1,05,000
Add: HEC @ 4% 4,200
Tax Liability 1,09,200

(ix)
Income under the head capital gains (STCG u/s 111A) 3,50,000
Total Income 3,50,000
Computation of Tax Liability
Tax on Short term capital gains `3,50,000 @ 15% u/s 111A 52,500
Add: HEC @ 4% 2,100
Tax Liability 54,600

Illustration 14: Compute tax liability for the assessment year 2019-20 in the following situations:
(i) Mr. X is resident in India and his incomes are as follows:
(a) Income under the head Salary `1,20,000
(b) Income under the head House Property `60,000
(c) Long term capital gains `2,20,000
(d) Short term capital gain under section 111A `1,10,000
(e) Casual Income `90,000
(f) Deduction under section 80C to 80U `2,00,000 .
(ii) Presume in the above situation the assessee is Mrs. X.
(iii) Presume in the above situation the assessee is Mrs. X and she is aged about 70 years.
(iv) Presume in the above situation the assessee is Mr. X and he is aged about 70 years.
(v) Presume in the above situation the assessee is Mrs. X and she is aged about 83 years.
(vi) Presume in the above situation the assessee is Mr. X and he is aged about 83 years.
(vii) Presume in the above situation the assessee is Mr. X and he is aged about 70 years and he is non-
resident.
(viii) Presume in the above situation the assessee is Mr. X and he is aged about 83 years old and he is non-
resident.
Solution:
(i) ` `
Computation of Total Income
Income under the head Salary 1,20,000
Income under the head House Property 60,000
Income under the head Capital Gains
Long term capital gains 2,20,000
Short term capital gains u/s 111A 1,10,000 3,30,000
Income under the head Other Sources (Casual Income) 90,000
Computation of Total Income And Tax Liability 44

Gross Total Income 6,00,000


Less: Deduction u/s 80C to 80U (1,80,000)
Total Income 4,20,000
Computation of Tax Liability
Tax on LTCG (2,20,000 – 2,20,000) @ 20% u/s 112 Nil
Tax on STCG `80,000 (`1,10,000 – 30,000) @ 15% u/s 111A 12,000
Tax on Casual income `90,000 @ 30% u/s 115BB 27,000
Tax on normal income at slab rate Nil
Tax before health & education cess 39,000
Add: HEC @ 4% 1,560
Tax Liability 40,560
(ii)
Total Income 4,20,000
Computation of Tax Liability
Tax on LTCG (2,20,000 – 2,20,000) @ 20% u/s 112 Nil
Tax on STCG `80,000 (`1,10,000 – 30,000) @ 15% u/s 111A 12,000
Tax on Casual income `90,000 @ 30% u/s 115BB 27,000
Tax on normal income at slab rate Nil
Tax before health & education cess 39,000
Add: HEC @ 4% 1,560
Tax Liability 40,560

(iii)
Total Income 4,20,000
Computation of Tax Liability
Tax on LTCG (2,20,000 – 2,20,000) @ 20% u/s 112 Nil
Tax on STCG `30,000 (1,10,000 – 80,000) @ 15% u/s 111A 4,500
Tax on Casual Income `90,000 @ 30% u/s 115BB 27,000
Tax on normal income at slab rate Nil
Tax before health & education cess 31,500
Add: HEC @ 4% 1,260
Tax Liability 32,760

(iv)
Total Income 4,20,000
Computation of Tax Liability
Tax on LTCG (2,20,000 – 2,20,000) @ 20% u/s 112 Nil
Tax on STCG `30,000 (1,10,000 – 80,000) @ 15% u/s 111A 4,500
Tax on Casual Income `90,000 @ 30% u/s 115BB 27,000
Tax on normal income at slab rate Nil
Tax before health & education cess 31,500
Add: HEC @ 4% 1,260
Tax Liability 32,760

(v)
Total Income 4,20,000
Computation of Tax Liability
Tax on LTCG (2,20,000 – 2,20,000) @ 20% u/s 112 Nil
Tax on STCG (1,10,000 – 1,10,000) @ 15% u/s 111A Nil
Tax on Casual Income `90,000 @ 30% u/s 115BB 27,000
Tax on normal income at slab rate Nil
Tax before health & education cess 27,000
Add: HEC @ 4% 1,080
Computation of Total Income And Tax Liability 45

Tax Liability 28,080

(vi)
Total Income 4,20,000
Computation of Tax Liability
Tax on LTCG (2,20,000 – 2,20,000) @ 20% u/s 112 Nil
Tax on STCG (1,10,000 – 1,10,000) @ 15% u/s 111A Nil
Tax on Casual Income `90,000 @ 30% u/s 115BB 27,000
Tax on normal income at slab rate Nil
Tax before health & education cess 27,000
Add: HEC @ 4% 1,080
Tax Liability 28,080

(vii)
Total Income 4,20,000
Computation of Tax Liability
Tax on LTCG `2,20,000 @ 20% u/s 112 44,000
Tax on STCG `1,10,000 @ 15% u/s 111A 16,500
Tax on Casual income `90,000 @ 30% u/s 115BB 27,000
Tax on normal income at slab rate Nil
Tax before health & education cess 87,500
Add: HEC @ 4% 3,500
Tax Liability 91,000

(viii)
Total Income 4,20,000
Computation of Tax Liability
Tax on LTCG `2,20,000 @ 20% u/s 112 44,000
Tax on STCG `1,10,000 @ 15% u/s 111A 16,500
Tax on Casual income `90,000 @ 30% u/s 115BB 27,000
Tax on normal income at slab rate Nil
Tax before health & education cess 87,500
Add: HEC @ 4% 3,500
Tax Liability 91,000

Illustration 15:
Mr. X has long term capital gain `30 lakh and normal income `72 lakh, in this case his tax liability shall be
Total income 102,00,000.00
LTCG `30,00,000 x 20% 6,00,000.00
Normal income at slab rate 19,72,500.00
Tax before surcharge 25,72,500.00
Add: Surcharge @ 15% 3,85,875.00
Tax before health & education cess 29,58,375.00
Add: HEC @ 4% 1,18,335.00
Tax Liability 30,76,710.00

Illustration 16:
Mr. X has long term capital gain `31 lakh and normal income `70 lakh, in this case his tax liability shall be
Total income 101,00,000.00
LTCG `31,00,000 x 20% 6,20,000.00
Normal income at slab rate 19,12,500.00
Tax before surcharge 25,32,500.00
Add: Surcharge @ 15% 3,79,875.00
Computation of Total Income And Tax Liability 46

Tax before marginal relief 29,12,375.00


Less: Marginal relief (48,625.00)
Tax on `101 lakhs 29,12,375
Tax on `100 lakhs
(`100 lakhs can be normal income `70 lakhs + LTCG `30 lakhs or
normal income `69 lakhs and LTCG `31 lakhs . It is not given in
the Act what combination should be taken. Hence it is a question
of law and any of the combination can be taken and it will be
correct)
If first combination is taken, income tax shall be
Normal income `70 lakhs 19,12,500
LTCG `30 lakhs 6,00,000
Total 25,12,500
Add: Surcharge @ 10% 2,51,250
Total 27,63,750
Increase in tax (29,12,375 – 27,63,750) 1,48,625
Marginal relief (1,48,625 – 1,00,000) 48,625
Tax before health & education cess 28,63,750.00
Add: HEC @ 4% 1,14,550.00
Tax Liability 29,78,300.00

Second option: Normal income `69 lakhs and LTCG `31 lakhs
Total income 101,00,000.00
LTCG `31,00,000 x 20% 6,20,000.00
Normal income at slab rate 19,12,500.00
Tax before surcharge 25,32,500.00
Add: Surcharge @ 15% 3,79,875.00
Tax before marginal relief 29,12,375.00
Less: Marginal relief (59,625.00)
Tax on `101 lakhs 29,12,375
Tax on `100 lakhs
(`100 lakhs can be normal income `70 lakhs + LTCG `30 lakhs or
normal income `69 lakhs and LTCG `31 lakhs . It is not given in
the Act what combination should be taken. Hence it is a question
of law and any of the combination can be taken and it will be
correct)
If second combination is taken, income tax shall be
Normal income `69 lakhs 18,82,500
LTCG `31 lakhs 6,20,000
Total 25,02,500
Add: Surcharge @ 10% 2,50,250
Total 27,52,750
Increase in tax (29,12,375 – 27,52,750) 1,59,625
Marginal relief (1,59,625 – 1,00,000) 59,625
Tax before health & education cess 28,52,750.00
Add: HEC @ 4% 1,14,110.00
Tax Liability 29,66,860.00

Question 10: Write a note on taxability of income of Partnership Firm/Limited Liability Partnership
Firm.
Answer: Partnership firm/LLP
Long term capital gains are taxable @ 20%, STCG u/s 111A shall be taxable @ 15% , LTCG u/s 112A
shall be taxable in excess of 1,00,000 @ 10% and casual income @ 30% and other incomes are also taxable
Computation of Total Income And Tax Liability 47

@ 30%.
Surcharge shall be applicable @ 12% provided total income is exceeding ` 1 crore.
Marginal Relief
Marginal relief shall be allowed if income has exceeded `100 lakhs.
Health & education cess is applicable @ 4%

Deductions under section 80C to 80U shall be allowed in the normal manner.

Partnership firm is regulated through Partnership Act,1932 and Limited Liability Partnership firm is
regulated through Limited Liability Partnership Act, 2008.

Question 11: Write a note on taxability of income of domestic company.


Answer: Domestic Company
Long term capital gains are taxable @ 20%, STCG u/s 111A shall be taxable @ 15% , LTCG u/s 112A
shall be taxable in excess of 1,00,000 @ 10% and casual income @ 30% and other incomes are also taxable
@ 30%.

Surcharge shall be applicable


- @ 7% provided total income is exceeding `100 lakhs but it is upto `1000 lakhs
- @ 12% provided total income is exceeding `1000 lakhs.
Marginal relief shall be allowed if income has exceeded `100 lakhs / 1000 lakhs
Health & education cess is applicable @ 4%
Deductions under section 80C to 80U shall be allowed in the normal manner.
(If total turnover or gross receipts in P.Y. 2016-17 does not exceed 250 crores, tax rate shall be 25% instead
of 30%)
Example
Compute the tax liability of X Ltd., a domestic company, assuming that the total income of X Ltd. is
`1,01,00,000 and the total income does not include any income in the nature of capital gains.
Answer
Total income 1,01,00,000
Tax on @ 30% 30,30,000
Add: Surcharge @ 7% 2,12,100
Tax before marginal relief 32,42,100
Less: Marginal Relief (1,42,100)
Working Note:
Tax + surcharge on income of `101,00,000 32,42,100
Tax on income of `100,00,000 (30,00,000)
Increase in tax 2,42,100
Increase in income 1,00,000
Marginal Relief (2,42,100 – 1,00,000) 1,42,100
Tax after marginal relief 31,00,000
Add: HEC @ 4% 1,24,000
Tax Liability 32,24,000
Example
Compute the tax liability of X Ltd., a domestic company, assuming that the total income of X Ltd. is
`10,01,00,000 and the total income does not include any income in the nature of capital gains.
Answer
Total income 10,01,00,000
Tax on @ 30% 300,30,000
Add: Surcharge @ 12% 36,03,600
Tax before marginal relief 336,33,600
Less: Marginal Relief (14,33,600)
Working Note:
Computation of Total Income And Tax Liability 48

Tax + surcharge @ 12% on income of `10,01,00,000 336,33,600


Tax + surcharge @ 7% on income of `1000,00,000 (321,00,000)
Increase in tax 15,33,600
Increase in income 1,00,000
Marginal Relief (15,33,600 – 1,00,000) 14,33,600
Tax after marginal relief 322,00,000
Add: HEC @ 4% 12,88,000
Tax Liability 334,88,000
Question 12: Write a note on taxability of income of Foreign company.
Answer: Foreign Company
Long term capital gains are taxable @ 20%, STCG u/s 111A shall be taxable @ 15%, LTCG u/s 112A
shall be taxable in excess of 1,00,000 @ 10% and casual income @ 30% and other incomes are taxable @
40%.
Surcharge shall be applicable
- @ 2% provided total income is exceeding `100 lakhs but it is upto `1000 lakhs.
- @ 5% provided total income is exceeding `1000 lakhs

Marginal relief shall be allowed if income has exceeded `100 lakhs / 1000 lakhs
Health & education cess is applicable @ 4%
Deductions under section 80C to 80U shall be allowed in the normal manner.

Question 13: Explain meaning of domestic company.


Answer: Meaning of domestic company
As per section 2(22A), Domestic Company means an Indian company. A foreign company shall also be
considered to be domestic company if it has complied with the following three conditions:
1) The share-register of shareholders is maintained at its principal place of business in India throughout the
year.
2) The AGM of the company is held in India.
3) The dividends are payable only in India.
If any foreign company has complied with all the above conditions, it will be considered to be domestic
company otherwise it will be considered to be foreign company.
Illustration 17 (A): Compute tax liability of ABC Ltd. a domestic company in the following situations:
(i) The company has income under the head Business/Profession `50,000.
(ii) The company has income under the head Business/Profession `1,00,000.
(iii) The company has income under the head Business/Profession `500,00,000.
(iv) The company has income under the head Business/Profession `100,00,000.
(v) The company has long term capital gains of `50,000.
(vi) The company has long term capital gains of `200,00,000.
(vii) The company has long term capital gains of `5,00,000.
(viii) The company has long term capital gains of `10,20,000.
(ix) The company has income under the head Business/Profession `11 crore.
Solution: `
(i) Computation of Tax Liability
Income under the head Business/Profession 50,000
Total Income 50,000
Tax on `50,000 @ 30% 15,000
Add: HEC @ 4% 600
Tax Liability 15,600

(ii) Computation of Tax Liability


Income under the head Business/Profession 1,00,000
Total Income 1,00,000
Tax on `1,00,000 @ 30% 30,000
Computation of Total Income And Tax Liability 49

Add: HEC @ 4% 1,200


Tax Liability 31,200

(iii) Computation of Tax Liability


Income under the head Business/Profession 500,00,000
Total Income 500,00,000
Tax on `500,00,000 @ 30% 150,00,000
Add: Surcharge @ 7% 10,50,000
Add: HEC @ 4% 6,42,000
Tax Liability 166,92,000

(iv) Computation of Tax Liability


Income under the head Business/Profession 100,00,000
Total Income 100,00,000
Tax on `100,00,000 @ 30% 30,00,000
Add: HEC @ 4% 1,20,000
Tax Liability 31,20,000

(v) Computation of Tax Liability


Income under the head Capital Gains (long term capital gains) 50,000
Total Income 50,000
Tax on `50,000 @ 20% 10,000
Add: HEC @ 4% 400
Tax Liability 10,400

(vi) Computation of Tax Liability


Income under the head Capital Gains (long term capital gains) 200,00,000
Total Income 200,00,000
Tax on `200,00,000 @ 20% 40,00,000
Add: Surcharge @ 7% 2,80,000
Add: HEC @ 4% 1,71,200
Tax Liability 44,51,200

(vii) Computation of Tax Liability


Income under the head Capital Gains (long term capital gains) 5,00,000
Total Income 5,00,000
Tax on `5,00,000 @ 20% 1,00,000
Add: HEC @ 4% 4,000
Tax Liability 1,04,000

(viii) Computation of Tax Liability


Income under the head Capital Gains (long term capital gains) 10,20,000
Total Income 10,20,000
Tax on `10,20,000 @ 20% 2,04,000
Add: HEC @ 4% 8,160
Tax Liability 2,12,160

(ix) Computation of Tax Liability


Income under the head Business/profession 11,00,00,000
Total Income 11,00,00,000
Tax on `11,00,00,000 @ 30% 330,00,000
Add: Surcharge @ 12% 39,60,000
Tax before health & education cess 369,60,000
Computation of Total Income And Tax Liability 50

Add: HEC @ 4% 14,78,400


Tax Liability 384,38,400

Illustration 17(B): Presume in all the above situations the assessee is a partnership firm .
Solution: `
(i) Computation of Tax Liability
Income under the head Business/Profession 50,000
Total Income 50,000
Tax on `50,000 @ 30% 15,000
Add: HEC @ 4% 600
Tax Liability 15,600

(ii) Computation of Tax Liability


Income under the head Business/Profession 1,00,000
Total Income 1,00,000
Tax on `1,00,000 @ 30% 30,000
Add: HEC @ 4% 1,200
Tax Liability 31,200

(iii) Computation of Tax Liability


Income under the head Business/Profession 500,00,000
Total Income 500,00,000
Tax on `500,00,000 @ 30% 150,00,000
Add: Surcharge @ 12% 18,00,000
Tax before health & education cess 168,00,000
Add: HEC @ 4% 6,72,000
Tax Liability 174,72,000

(iv) Computation of Tax Liability


Income under the head Business/Profession 100,00,000
Total Income 100,00,000
Tax on `100,00,000 @ 30% 30,00,000
Add: HEC @ 4% 1,20,000
Tax Liability 31,20,000

(v) Computation of Tax Liability


Income under the head Capital Gains (long term capital gains) 50,000
Total Income 50,000
Tax on `50,000 @ 20% 10,000
Add: HEC @ 4% 400
Tax Liability 10,400

(vi) Computation of Tax Liability


Income under the head Capital Gains (long term capital gains) 200,00,000
Total Income 200,00,000
Tax on `200,00,000 @ 20% 40,00,000
Add: Surcharge @ 12% 4,80,000
Tax before health & education cess 44,80,000
Add: HEC @ 4% 1,79,200
Tax Liability 46,59,200

(vii) Computation of Tax Liability


Income under the head Capital Gains (long term capital gains) 5,00,000
Computation of Total Income And Tax Liability 51

Total Income 5,00,000


Tax on `5,00,000 @ 20% 1,00,000
Add: HEC @ 4% 4,000
Tax Liability 1,04,000

(viii) Computation of Tax Liability


Income under the head Capital Gains (long term capital gains) 10,20,000
Total Income 10,20,000
Tax on `10,20,000 @ 20% 2,04,000
Add: HEC @ 4% 8,160
Tax Liability 2,12,160

(ix) Computation of Tax Liability


Income under the head Business/profession 11,00,00,000
Total Income 11,00,00,000
Tax on `11,00,00,000 @ 30% 330,00,000
Add: Surcharge @ 12% 39,60,000
Tax before health & education cess 369,60,000
Add: HEC @ 4% 14,78,400
Tax Liability 384,38,400

Illustration 17(C): Presume in all the above situations the assessee is a foreign company.
Solution: `
(i) Computation of Tax Liability
Income under the head Business/Profession 50,000
Total Income 50,000
Tax on `50,000 @ 40% 20,000
Add: HEC @ 4% 800
Tax Liability 20,800

(ii) Computation of Tax Liability


Income under the head Business/Profession 1,00,000
Total Income 1,00,000
Tax on `1,00,000 @ 40% 40,000
Add: HEC @ 4% 1,600
Tax Liability 41,600

(iii) Computation of Tax Liability


Income under the head Business/Profession 500,00,000
Total Income 500,00,000
Tax on `500,00,000 @ 40% 200,00,000
Add: Surcharge @ 2% 4,00,000
Add: HEC @ 4% 8,16,000
Tax Liability 212,16,000

(iv) Computation of Tax Liability


Income under the head Business/Profession 100,00,000
Total Income 100,00,000
Tax on `100,00,000 @ 40% 40,00,000
Add: HEC @ 4% 1,60,000
Tax Liability 41,60,000
Computation of Total Income And Tax Liability 52

(v) Computation of Tax Liability


Income under the head Capital Gains (long term capital gains) 50,000
Total Income 50,000
Tax on `50,000 @ 20% 10,000
Add: HEC @ 4% 400
Tax Liability 10,400

(vi) Computation of Tax Liability


Income under the head Capital Gains (long term capital gains) 200,00,000
Total Income 200,00,000
Tax on `200,00,000 @ 20% 40,00,000
Add: Surcharge @ 2% 80,000
Add: HEC @ 4% 1,63,200
Tax Liability 42,43,200

(vii) Computation of Tax Liability


Income under the head Capital Gains (long term capital gains) 5,00,000
Total Income 5,00,000
Tax on `5,00,000 @ 20% 1,00,000
Add: HEC @ 4% 4,000
Tax Liability 1,04,000

(viii) Computation of Tax Liability


Income under the head Capital Gains (long term capital gains) 10,20,000
Total Income 10,20,000
Tax on `10,20,000 @ 20% 2,04,000
Add: HEC @ 4% 8,160
Tax Liability 2,12,160

(ix) Computation of Tax Liability


Income under the head Business/profession 11,00,00,000
Total Income 11,00,00,000
Tax on `11,00,00,000 @ 40% 440,00,000
Add: Surcharge @ 5% 22,00,000
Tax before health & education cess 462,00,000
Add: HEC @ 4% 18,48,000
Tax Liability 480,48,000

Illustration 18: X (HUF) has incomes as given below:


1. Income under the head Business/Profession `5,00,000
2. Income under the head House Property `3,00,000
3. Long term capital gains `4,00,000
4. Short term capital gains under section 111A `3,00,000
5. Casual Income `2,00,000
6. Deductions allowed under section 80C to 80U `35,000
Compute tax liability of HUF for the assessment year 2019-20.

Solution: ` `
Computation of Total Income
Income under the head Business/Profession 5,00,000.00
Income under the head House Property 3,00,000.00
Income under the head Capital Gains
Long term capital gains 4,00,000
Computation of Total Income And Tax Liability 53

Short term capital gains u/s 111A 3,00,000 7,00,000.00


Income under the head Other Sources (Casual Income) 2,00,000.00
Gross Total Income 17,00,000.00
Less: Deduction u/s 80C to 80U (35,000.00)
Total Income 16,65,000.00
Computation of Tax Liability
Tax on LTCG `4,00,000 @ 20% u/s 112 80,000.00
Tax on STCG `3,00,000 @ 15% u/s 111A 45,000.00
Tax on Casual income `2,00,000 @ 30% u/s 115BB 60,000.00
Tax on `7,65,000 at slab rate 65,500.00
Tax before health & education cess 2,50,500.00
Add: HEC @ 4% 10,020.00
Tax Liability 2,60,520.00

Question 14: Write a note on Computation of Tax Liability of HUF.


Answer: Tax liability of Hindu undivided family
Hindu undivided family means any family which is Hindu by religion and its senior most male member is
called karta and karta is responsible for control and management of HUF. Parental property / business etc
received by karta shall be considered to be common property and taxability shall be as given below:
Normal income of Hindu undivided family shall be computed at the normal slab rate as given below:
Income shall be taxable at the slab rates given below:
If total Income upto `2,50,000 NIL
On next `2,50,000 5%
On next `5,00,000 20%
On Balance amount 30%
Slab rate of senior citizen is not applicable to HUF even if age of Karta or its member is exceeding 60 years
or 80 years.
Surcharge shall be applicable
@ 10% if total income has exceeded `50 lakhs but upto `100 lakhs.
@ 15% if total income has exceeded `100 lakhs.
All other provisions shall be similar to individual but rebate under section 87A is not allowed. Tax rates for
LTCG /LTCG 112A/ STCG u/s 111A and casual income are the same for all the persons.
If normal income of resident HUF is less than the exemption limit, the difference of the amount shall be
allowed to be deducted from long term capital gain and if long term capital gains are not sufficient, it will be
allowed to be adjusted from short term capital gains under section 111A but it will not be allowed to be
adjusted from casual income.
(What is HUF is given in the Hindu Law and it is not covered in the syllabus)
Example
XY HUF has income under the head business/profession `20 lakhs and its Karta Mr. X has individual
income `12 lakhs, in this case tax liability of HUF and that of Karta shall be
Tax liability of HUF `20 lakhs at slab rate 4,12,500
Add: HEC @ 4% 16,500
Tax Liability 4,29,000
Tax Liability of Karta `12 lakhs at slab rate 1,72,500
Add: HEC @ 4% 6,900
Tax Liability 1,79,400

Question 15: Write a note on Computation of Tax Liability of Body of Individuals/Association of


Persons.
Answer: Tax liability of BOI/AOP
Body of individual means a group of individuals which is neither a company nor a partnership firm. If it is
registered in some other Act, it will be called incorporated BOI. E.g. cooperative society or Trust etc. If such
a group includes persons other than individual also, it will be called AOP.
Computation of Total Income And Tax Liability 54

In general normal income shall be taxable at normal slab rate but rate may change as per provisions of
section 167B. (NOT COVERED IN SYLLABUS)
Surcharge shall be applicable
@ 10% if total income has exceeded `50 lakhs but upto `100 lakhs.
@ 15% if total income has exceeded `100 lakhs.
Deductions under section 80C to 80U shall be allowed in the normal manner.

Question 16: Write a note on Computation of Tax Liability of Local Authority.


Answer: Tax liability of local authority
In order to maintain any town or city, there is always some authority responsible and such authority is called
local authority e.g. MCD in Delhi. Such authority is allowed to collect house tax with regard to every type of
house property and also some other tax are collected by such authority. In general income of such authority
is exempt from income tax under section 10(20) but if such authority is doing any business, its income is
taxable just like a partnership firm. Deductions under section 80C to 80U shall be allowed in the normal
manner.

Question 17: Explain meaning of Person Section 2(31).


Answer: Meaning of Person Section 2(31)
“Person” includes—
(i) an individual,
(ii) a Hindu undivided family,
(iii) a company,
(iv) a firm,
(v) an association of persons or a body of individuals, whether incorporated or not,
(vi) a local authority,
(vii) every artificial juridical person, not covered above and income is taxable as slab rate (juridical means
legal) e.g. ICAI or Delhi University etc. Surcharge shall be applicable @ 10% if total income has exceeded
`50 lakhs but upto `100 lakhs and @ 15% if total income has exceeded `100 lakhs. Deductions under
section 80C to 80U shall be allowed in the normal manner.

Question 18 [V. Imp.]: Discuss Partial Integration of Agricultural Income?


Or
Discuss Indirect Taxing of Agricultural Income?
Or
Under the Constitution, the power to levy a tax on agricultural income vests in the States. However,
Parliament has also levied a tax on such income. Explain how this has been achieved?
Answer:
Agricultural Income Section 10(1)
Under section 10(1), any agricultural income in India is fully exempt from income tax but if the
agricultural income is from outside India, it is chargeable to tax. (As per entry no. 82 of Union List, Central
Government has the power to levy income tax on income except agricultural income and power to levy tax
on agricultural income has been given to the State Government vide entry no. 46 of State List)

Indirect taxing of agricultural income or partial integration of agricultural income (Under the
constitution, the power to levy a tax on agricultural income vests in the states. However, parliament
has also levied a tax on such income. Explain how this has been achieved?)
If any person has agricultural income as well as non-agricultural income, his tax liability shall be computed
in the manner given below:
1. Compute tax on the total of agricultural income and non- agricultural income considering it to be total
income of the assessee.
2. Compute tax on exemption limit (`2,50,000 / 3,00,000 / 5,00,000) and agricultural income considering
Computation of Total Income And Tax Liability 55

it to be total income.
3. Deduct tax computed under Step 2 from Step 1 and apply surcharge if any and allow rebate if any and
health & education cess.
4. Long term capital gain, casual income and short term capital gain u/s 111A shall not be taken into
consideration for the purpose of partial integration
5. If Agricultural income is upto `5,000, or non-agricultural income is upto the limit not chargeable to tax
(`2,50,000/3,00,000/5,00,000), partial integration is not applicable.
6. Partial integration is not applicable in case of a partnership firm or a company.

Illustration 19: Compute tax liability in the following cases:


(i) Mr. X has income under the head business/profession ` 5,00,000 and agricultural Income ` 2,50,000.
(ii) Mr. Y has income under the head business/profession ` 10,00,000 and agricultural Income ` 2,50,000
(iii) Mr. Z has income under the head business/profession ` 10,00,000 and agricultural Income ` 2,50,000.
Deductions allowed under section 80C to 80U ` 1,00,000.
(iv) Mr. A has income under the head business/profession ` 5,00,000 and agricultural Income ` 10,00,000.
(v) Mr. A has income under the head business/profession ` 2,00,000 and agricultural Income ` 10,00,000.
(vi) Mr. A has LTCG ` 10,00,000 and agricultural Income ` 10,00,000.
(vii) Mr. B, a Non - Resident has LTCG ` 10,00,000 and agricultural Income ` 10,00,000.
(viii) Mr. A has Casual Income ` 10,00,000 and agricultural Income ` 10,00,000.

Solution: `
(i) Computation of Total Income
Income under the head Business/Profession 5,00,000
Gross Total Income 5,00,000
Less: Deduction u/s 80C Nil
Total Income 5,00,000
Agricultural Income 2,50,000
Computation of Tax Liability
Normal income 5,00,000
Step 1. Tax on (5,00,000 + 2,50,000) at slab rate 62,500.00
Step 2. Tax on (`2,50,000 + 2,50,000) at slab rates (12,500.00)
Step 3. Deduct Tax at Step 2 from Tax at Step 1 50,000.00
Tax before health & education cess 50,000.00
Add: HEC @ 4% 2,000.00
Tax Liability 52,000.00

(ii) Computation of Total Income


Income under the head Business/Profession 10,00,000
Gross Total Income 10,00,000
Less: Deduction u/s 80C Nil
Total Income 10,00,000
Agricultural Income 2,50,000
Computation of Tax Liability
Normal income 10,00,000
Step 1. Tax on (10,00,000 + 2,50,000) at slab rate 1,87,500.00
Step 2. Tax on (`2,50,000 + 2,50,000) at slab rates (12,500.00)
Step 3. Deduct Tax at Step 2 from Tax at Step 1 1,75,000.00
Tax before health & education cess 1,75,000.00
Add: HEC @ 4% 7,000.00
Tax Liability 1,82,000.00

(iii) Computation of Total Income


Income under the head Business/Profession 10,00,000
Computation of Total Income And Tax Liability 56

Gross Total Income 10,00,000


Less: Deduction u/s 80C (1,00,000)
Total Income 9,00,000
Agricultural Income 2,50,000
Computation of Tax Liability
Normal income 9,00,000
Step 1. Tax on (9,00,000 + 2,50,000) at slab rate 1,57,500.00
Step 2. Tax on (`2,50,000 + 2,50,000) at slab rates (12,500.00)
Step 3. Deduct Tax at Step 2 from Tax at Step 1 1,45,000.00
Tax before health & education cess 1,45,000.00
Add: HEC @ 4% 5,800.00
Tax Liability 1,50,800.00

(iv) Computation of Total Income


Income under the head Business/Profession 5,00,000
Gross Total Income 5,00,000
Less: Deduction u/s 80C Nil
Total Income 5,00,000
Agricultural Income 10,00,000
Computation of Tax Liability
Normal income 5,00,000
Step 1. Tax on (5,00,000 + 10,00,000) at slab rate 2,62,500.00
Step 2. Tax on (`2,50,000 + 10,00,000) at slab rates (1,87,500.00)
Step 3. Deduct Tax at Step 2 from Tax at Step 1 75,000.00
Tax before health & education cess 75,000.00
Add: HEC @ 4% 3,000.00
Tax Liability 78,000.00

(v) There will be no partial Integration as normal income is less than the exemption limit and Tax Liability
is Nil.

(vi) There will be no partial Integration as normal income is Nil


Computation of Total Income
Income under the head Capital Gains (LTCG) 10,00,000
Gross Total Income 10,00,000
Less: Deduction u/s 80C Nil
Total Income 10,00,000
Computation of Tax Liability
Tax on LTCG ` 7,50,000 (`10,00,000- ` 2,50,000) @ 20% u/s 112 1,50,000
Add: HEC @ 4% 6,000
Tax Liability 1,56,000

(vii) There will be no partial Integration as normal income is Nil


Computation of Total Income
Income under the head Capital Gains (LTCG) 10,00,000
Gross Total Income 10,00,000
Less: Deduction u/s 80C Nil
Total Income 10,00,000
Computation of Tax Liability
Tax on LTCG `10,00,000 @ 20% u/s 112 2,00,000
Add: HEC @ 4% 8,000
Tax Liability 2,08,000
Computation of Total Income And Tax Liability 57

(viii) There will be no partial Integration as normal income is Nil


Computation of Total Income
Income under the head Other Sources (Casual Income) 10,00,000
Gross Total Income 10,00,000
Less: Deduction u/s 80C Nil
Total Income 10,00,000

Computation of Tax Liability


Tax on Casual Income `10,00,000 @ 30% 3,00,000
Add: HEC @ 4% 12,000
Tax Liability 3,12,000

Illustration 20: (a) Mr. X, aged 68 years, has income under the head House Property `3,25,000, agricultural
income of `1,00,000, Long term capital gain amounting to `45,000 and casual income `35,000. He is
eligible for deduction under section 80C `20,000.
Compute tax liability of Mr. X for assessment year 2019-20.
Solution: `
Computation of Total Income
Income under the head House Property 3,25,000
Income under the head Capital Gains (Long term capital gain) 45,000
Income under the head Other Sources (Casual Income) 35,000
Gross Total Income 4,05,000
Less: Deduction u/s 80C (20,000)
Total Income 3,85,000
Agricultural Income 1,00,000
Computation of Tax Liability
Tax on casual income `35,000 @ 30% u/s 115BB 10,500.00
Tax on long term capital gain `45,000 @ 20% u/s 112 9,000.00
Normal income 3,05,000
Step 1. Tax on (3,05,000 + 1,00,000) at slab rate 5,250.00
Step 2. Tax on (`3,00,000 + 1,00,000) at slab rates (5,000.00)
Step 3. Deduct Tax at Step 2 from Tax at Step 1 250.00
Tax before health & education cess 19,750.00
Add: HEC @ 4% 790.00
Tax Liability 20,540.00

(b) Presume in the above question, Mr. X is Non Resident.


Solution: `
Computation of Total Income
Income under the head House Property 3,25,000
Income under the head Capital Gains (LTCG) 45,000
Income under the head Other Sources (Casual Income) 35,000
Gross Total Income 4,05,000
Less: Deduction u/s 80C (20,000)
Total Income 3,85,000
Agricultural Income 1,00,000
Computation of Tax Liability
Tax on casual income `35,000 @ 30% u/s 115BB 10,500.00
Tax on long term capital gain `45,000 @ 20% u/s 112 9,000.00
Normal income 3,05,000
Step 1. Tax on (3,05,000 + 1,00,000) 7,750.00
Step 2. Tax on (`2,50,000 + 1,00,000) at slab rates (5,000.00)
Step 3. Deduct Tax at Step 2 from Tax at Step 1 2,750.00
Computation of Total Income And Tax Liability 58

Tax before health & education cess 22,250.00


Add: HEC @ 4% 890.00
Tax Liability 23,140.00

(c) Presume in the above question, Mr. X is resident and do not have any income from house property.
Solution: `
There will be no partial integration.
Computation of Total Income
Income under the head Capital Gains (LTCG) 45,000
Income under the head Other Sources (casual income) 35,000
Gross Total Income 80,000
Less: Deduction u/s 80C to 80U NIL
Total Income 80,000
Agricultural Income 1,00,000
Computation of Tax Liability
Tax on casual income `35,000 @ 30% u/s 115BB 10,500
Tax on long term capital gain (`45,000- 45,000) Nil
Normal income Nil
Tax on normal income Nil
Less: Rebate u/s 87A (2,500)
Tax before health & education cess 8,000
Add: HEC @ 4% 320
Tax Liability 8,320

Illustration 21: Mrs. X has income asunder –


Income under the head Salary 1,55,000
Income under the head Capital Gains
Long term capital gain 27,000
Short term capital gain 1,09,000
Income under the head Other Sources (lottery) 7,000
Agricultural income 1,05,000
Deduction allowed under section 80C to 80U 7,000
Compute her tax liability for the assessment year 2019-20 in two situations –
(i) She is resident
(ii) She is non-resident.
Solution: `
(i) She is resident
Computation of Total Income
Income under the head Salary 1,55,000
Income under the head Capital Gains
Long term capital gain 27,000
Short term capital gain 1,09,000
Income under the head Other Sources (lottery) 7,000
Gross Total Income 2,98,000
Less: Deduction u/s 80C to 80U (7,000)
Total Income 2,91,000
Agricultural income 1,05,000
Computation of Tax Liability
Tax on casual income `7,000 @ 30% u/s 115BB 2,100.00
Tax on long term capital gain `27,000 @ 20% u/s 112 5,400.00
Normal income `2,57,000
Step 1. Tax on (2,57,000 + 1,05,000) 5,600.00
Step 2. Tax on (`2,50,000 + 1,05,000) at slab rates (5,250.00)
Computation of Total Income And Tax Liability 59

Step 3. Deduct Tax at Step 2 from tax at Step 1 350.00


Less: Rebate u/s 87A (2,500.00)
Tax before health & education cess 5,350.00
Add: HEC @ 4% 214.00
Tax Liability 5,564.00
Rounded off u/s 288B 5,560.00

(ii) She is non-resident


Computation of Total Income
Income under the head Salary 1,55,000
Income under the head Capital Gains
Long term capital gain 27,000
Short term capital gain 1,09,000
Income under the head Other Sources (Casual Income) 7,000
Gross Total Income 2,98,000
Less: Deduction u/s 80C to 80U (7,000)
Total Income 2,91,000
Agricultural income 1,05,000
Computation of Tax Liability
Tax on casual income `7,000 @ 30% u/s 115BB 2,100.00
Tax on long term capital gain `27,000 @ 20% u/s 112 5,400.00
Normal income `2,57,000
Step 1. Tax on (2,57,000 + 1,05,000) 5,600.00
Step 2. Tax on (`2,50,000 + 1,05,000) at slab rates (5,250.00)
Step 3. Deduct Tax at Step 2 from tax at Step 1 350.00
Tax before health & education cess 7,850.00
Add: HEC @ 4% 314.00
Tax Liability 8,164.00
Rounded off u/s 288B 8,160.00
Note: Rebate under section 87A is not allowed to non-resident.

Illustration 22: Mr. X has agricultural income of `4,900 and non-agricultural income of `2,65,000.
Compute his tax liability for the assessment year 2019-20.
Solution: `
Agricultural Income 4,900
Non Agricultural income 2,65,000
In this case, Agricultural income is upto `5000/-, thereby, partial integration shall not be applicable.
Computation of Tax Liability
Tax on `2,65,000 at slab rate 750
Less: Rebate u/s 87A (750)
Tax Liability Nil

Illustration 23: Mr. X has agricultural income of `5,00,000 and non-agricultural income of `2,50,000.
Compute his tax liability for the assessment year 2019-20.
Solution:
His tax liability shall be nil, since his non-agricultural income is `2,50,000 and partial integration is not
applicable.

Illustration 24: Mr. X (aged 70 years) has agricultural income of `3,80,000 and non-agricultural income of
`3,00,000.
Compute his tax liability for the assessment year 2019-20.
Solution:
His tax liability shall be nil, since his non-agricultural income is `3,00,000 and partial integration is not
Computation of Total Income And Tax Liability 60

applicable.

Illustration 25: Mrs. X has agricultural income of `1,00,000, income under the head salary amounting to `
3,05,000, long term capital gain of `10,00,000 and casual income of `1,00,000 (winnings of a game show on
TV).
Compute her tax liability for the assessment year 2019-20. Her date of birth is 01.04.1959.

Solution: `
Computation of Total Income
Income under the head Salary 3,05,000.00
Income under the head Capital Gains (LTCG) 10,00,000.00
Income under the head Other Sources (Casual Income) 1,00,000.00
Gross Total Income 14,05,000.00
Less: Deduction u/s 80C to 80U Nil
Total Income 14,05,000.00
Agricultural Income 1,00,000.00
Computation of Tax Liability
Tax on casual income `1,00,000 @ 30% u/s 115BB 30,000.00
Tax on long term capital gain of `10,00,000 @ 20% u/s 112 2,00,000.00
Normal income `3,05,000
Step 1. Tax on (3,05,000 + 1,00,000) 5,250.00
Step 2. Tax on (`3,00,000 + 1,00,000) at slab rates (5,000.00)
Step 3. Deduct Tax at Step 2 from Tax at Step 1 250.00
Tax before health & education cess 2,30,250.00
Add: HEC @ 4% 9,210.00
Tax Liability 2,39,460.00
Note: 1. Casual income shall include all the activities as per section 2(24)(ix).

Illustration 26: Mr. X has income from business `102,00,000 and agricultural income `10,00,000.
Compute his tax liability
Solution:
Total income 102,00,000
Step 1. Tax on (agricultural + non agricultural income)
i.e. Tax on ` 112,00,000/- at slab rates 31,72,500
Step 2. Tax on (`2,50,000 + agricultural income) at slab rates (1,87,500)
Step 3. Deduct Tax at Step 2 from Tax at Step 1 29,85,000
Add: Surcharge @ 15% 4,47,750
Tax + surcharge before HEC 34,32,750
Marginal relief (15,250)
Working Note:
Tax + surcharge on income `102,00,000 34,32,750
Tax + surcharge if total income was `100 lakhs
100,00,000 + 10,00,000 = 110,00,000 31,12,500
2,50,000 + 10,00,000 = 12,50,000 (1,87,500)
Tax 29,25,000
Tax + surcharge @10% on `100 lakhs (32,17,500)
Increase in tax 2,15,250
Increase in income 102,00,000 – 100,00,000 = (2,00,000)
Marginal relief (2,15,250 – 2,00,000) 15,250
Tax before health & education cess 34,17,500
Add: HEC @ 4% 1,36,700
Tax Liability 35,54,200
Computation of Total Income And Tax Liability 61

Question 19: Write a note on taxability of dividends.


Answer: Taxability of Dividends
Dividend income from the domestic company shall be exempt from tax in the hands of the shareholder as
per section 10(34). (However dividends from a foreign company shall continue to be taxed in the hands of
the shareholder.)
As per section 115O / 115P / 115Q the domestic company has to pay additional income tax @ 15% +
surcharge @ 12% + HEC @ 4%.
The effective rate shall be 15%
Add: Surcharge @ 12% 1.8%
Total 16.8%
Add: HEC @ 4% 0.672%
Total 17.472%
Example
ABC Ltd. is a domestic company and has total income `80,00,000. It has declared the dividends of
`10,00,000 and one of the shareholders Mr. X gets dividends of `25,000. In this case, tax liability of the
company and Mr. X shall be: `
Total Income 80,00,000.00
Income tax @ 30% 24,00,000.00
Add: HEC @ 4% 96,000.00
Tax Liability 24,96,000.00
Amount of Dividend 10,00,000.00
Additional Income Tax
(10,00,000 / 82.528% x 17.472%) 2,11,709.97
Rounded off u/s 288B 2,11,710.00
Tax liability of Mr. X shall be nil.
The company has to pay surcharge on additional income tax in every case even if total income is less than
`100,00,000
Alternative Calculation:
Amount of Dividend 10,00,000.00
Additional Income Tax
(10,00,000 / 85% x 15%) 1,76,470.59
Add: Surcharge @ 12% 21,176.47
Tax plus Surcharge 1,97,647.06
Add: health & education Cess @ 4% 7,905.88
Total Amount of AIT 2,05,552.94
Rounded off u/s 288B 2,05,550.00
Illustration 27(A): ABC Ltd., a domestic company has current profits of `150 lakhs and the company has
distributed dividends of `55 lakhs. Mr. X, one of the shareholder has received dividend of `7,00,000.
Compute income tax liability of the company and that of shareholder.
Calculate additional income tax payable by the company also.
Solution:
Tax liability and additional tax liability of the company shall be as given below: `
Profit before tax 150,00,000.00
Income tax on `150,00,000 @ 30% 45,00,000.00
Add: Surcharge @ 7% 3,15,000.00
Add: HEC @ 4% 1,92,600.00
Income tax liability 50,07,600.00
Dividend 55,00,000.00
Additional Income Tax
(55,00,000 / 82.528% x 17.472%) 11,64,404.81
Rounded off u/s 288B 11,64,400.00
Tax liability of the shareholder shall be nil.
Computation of Total Income And Tax Liability 62

Illustration 27(B): Presume in the above case the company is the foreign company.
Solution:
Tax liability and additional tax liability of the company shall be as given below: `
Profit before tax 150,00,000.00
Income tax on `150,00,000 @ 40% 60,00,000.00
Add: Surcharge @ 2% 1,20,000.00
Add: HEC @ 4% 2,44,800.00
Income tax liability 63,64,800.00
Additional income tax of the foreign company shall be nil.
Tax liability of the shareholder shall be as given below:
Dividend from foreign company 7,00,000.00
Tax on `7,00,000 at slab rate 52,500.00
Add: HEC @ 4% 2,100.00
Tax Liability 54,600.00

Dividend in excess of ` 10 lakh


As per section 115BBDA, Dividend received by All Assessee except a domestic company or a fund or
institution or trust or any university, exceeding `10 lakh shall be taxable @ 10%. No further deduction or
expenditure shall be allowed from such income.
E.g. Mr. X has dividend of ` 15 lakh from a domestic company and he has deduction under section 80C to
80U ` 1,00,000, in this case his tax liability shall be
Dividend Income 15,00,000
Less: Exempt u/s 10(34) 10,00,000
Income under the head Other Sources 5,00,000
Gross Total Income 5,00,000
Less: Deduction u/s 80C to 80U Nil
Total Income 5,00,000
Computation of Tax Liability
Tax on ` 5,00,000 @ 10% u/s 115BBDA 50,000
Add: HEC @ 4% 2000
Tax Liability 52,000

Question 20: Write a note on Interest or dividend income from UTI or Mutual funds.
Answer: Interest or dividend income from UTI or Mutual funds Section 10(35)
If any person has received any interest or dividend from the UTI or Mutual Fund notified under section
10(23D), such income is exempt from income tax.
If UTI or Mutual Funds have distributed any interest or dividend, as per section 115R, 115S, 115T UTI or
Mutual Funds, have to pay additional income tax (Corporate dividend tax) at the rate of 25% plus surcharge
@ 12% plus HEC @ 4%, if the amount is distributed to individual or Hindu Undivided Family.
The effective rate shall be 25%
Add: Surcharge @ 12% 3%
Total 28%
Add: HEC @ 4% 1.12%
Total 29.12%
If amount is distributed to any other person, rate shall be 30% plus surcharge @ 12% plus HEC @ 4%.
The effective rate shall be 30%
Add: Surcharge @ 12% 3.6%
Total 33.6%
Add: HEC @ 4% 1.344%
Total 34.944%
e.g. If a mutual fund has paid dividend of `50 lakhs out of which dividends paid to individual or HUF are
`20 lakhs, additional income tax payable shall be
Amount of Dividend 50,00,000.00
Computation of Total Income And Tax Liability 63

Amount Paid to Individual & HUF 20,00,000.00


Balance Paid to other than Individual & HUF 30,00,000.00
Additional Income Tax
(20,00,000 / 70.88% x 29.12%) 8,21,670.43
(30,00,000 / 65.056% x 34.944%) 16,11,411.71
Total 24,33,082.14
Rounded off u/s 288B 24,33,080.00
If any such dividend or interest has been distributed by equity oriented mutual fund, rate of AIT shall be
10%.
Illustration 28: Mr. X has incomes as given below: `
 Received dividend from a domestic company 2,00,000
 Received interest in connection with deposit given to a domestic company 3,00,000
 Received dividend from foreign company 1,50,000
 Received interest from a foreign company 2,50,000
 Received dividend from UTI 3,00,000
 Received interest from UTI 2,00,000
 Received dividend from mutual fund 1,10,000
 Received interest from mutual fund 3,00,000
 Agricultural Income 7,00,000
Compute his total income and tax liability for A.Y. 2019-20.
Solution:
Computation of Total Income `
Received dividend from a domestic company (Exempt u/s 10(34)) Nil
Received interest in connection with deposit given to a domestic company 3,00,000
Received dividend from foreign company 1,50,000
Received interest from a foreign company 2,50,000
Received dividend from UTI (Exempt u/s 10(35)) Nil
Received interest from UTI (Exempt u/s 10(35)) Nil
Received dividend from mutual fund (Exempt u/s 10(35)) Nil
Received interest from mutual fund (Exempt u/s 10(35)) Nil
Gross Total Income 7,00,000
Less: Deduction u/s 80C to 80U Nil
Total Income 7,00,000
Agricultural Income 7,00,000
Computation of Tax Liability
Step 1. Tax on (7,00,000 + 7,00,000) 2,32,500
Step 2. Tax on (`2,50,000 + 7,00,000) at slab rates (1,02,500)
Step 3. Deduct Tax at Step 2 from Tax at Step 1 1,30,000
Tax before health & education cess 1,30,000
Add: HEC @ 4% 5,200
Tax Liability 1,35,200
Computation of Total Income And Tax Liability 64

PRACTICE PROBLEMS
TOTAL PROBLEMS 17
Problem 1.
Compute tax liability in the following cases for the assessment year 2019-20.
(i) Mr. X (resident) has total income of `22,00,000
(ii) Mr. X (non-resident) has total income of `22,00,000
(iii) Mrs. X (resident) has total income of `22,00,000
(iv) Mrs. X (non-resident) has total income of `22,00,000
(v) Mr. X (resident), aged 60 years has total income of `22,00,000
(vi) Mrs. X (resident), aged 60 years has total income of `22,00,000
(vii) Mr. X (non-resident), aged 60 years has total income of `22,00,000
(viii) Mrs. X (non-resident), aged 60 years has total income of `22,00,000
(ix) Mr. X (resident), aged 80 years has total income of `22,00,000
(x) Mrs. X (resident), aged 80 years has total income of `22,00,000
(xi) Mr. X (non-resident), aged 80 years has total income of `22,00,000
(xii) Mrs. X (non-resident), aged 80 years has total income of `22,00,000
Answer = (i) Tax Liability: `4,91,400; (ii) `4,91,400; (iii) `4,91,400; (iv) `4,91,400; (v) `4,88,800; (vi)
`4,88,800; (vii) `4,91,400; (viii) `4,91,400; (ix) `4,78,400; (x) `4,78,400; (xi) `4,91,400; (xii) `4,91,400

Problem 2.
Compute tax liability in the following cases for the assessment year 2019-20.
(i) Mr. X (resident) has total income of `100,05,000
(ii) Mr. X (non-resident) has total income of `102,00,000
(iii) Mrs. X (resident) has total income of `90,00,000
(iv) Mrs. X (non-resident) has total income of `4,98,000
(v) Mr. X (resident), aged 60 years has total income of `4,05,000
(vi) Mrs. X (resident), aged 60 years has total income of `102,05,000
(vii) Mr. X (non-resident), aged 60 years has total income of `25,00,000
(viii) Mrs. X (non-resident), aged 60 years has total income of `4,50,000
(ix) Mr. X (resident), aged 80 years has total income of `3,80,000
(x) Mrs. X (resident), aged 80 years has total income of `110,00,000
(xi) Mr. X (non-resident), aged 80 years has total income of `99,99,000
(xii) Mrs. X (non-resident), aged 80 years has total income of `12,00,000
Answer = (i) Tax Liability: `32,22,700; (ii) `34,25,500; (iii) `28,74,300; (iv) `12,900; (v) `5,460; (vi)
`34,27,840; (vii) `5,85,000; (viii) `10,400; (ix) Nil ; (x) `37,07,600; (xi) `32,17,160; (xii) `1,79,400

Problem 3.
Compute tax liability in the following cases for the assessment year 2019-20.
(i) Mr. X (resident) has total income of `50,20,000
(ii) Mr. X (non-resident) has total income of `53,00,000
(iii) Mrs. X (resident), aged 60 years has total income of `51,00,000
(iv) Mr. X (resident), aged 80 years has total income of `54,00,000
(v) Mr. X (non-resident), aged 80 years has total income of `51,22,000
(vi) Mrs. X (non-resident), has total income of `50,80,000
Answer = (i) Tax Liability: `13,85,800; (ii) `16,04,460; (iii) `14,66,400; (iv) `16,24,480; (v) `14,91,880;
(vi) `14,48,200

Problem 4.
Compute tax liability in the following cases for the assessment year 2019-20.
(i) Mr. X (resident) has total income of `3,30,000
(ii) Mr. X (non-resident) has total income of `3,30,000
(iii) Mrs. X (resident) has total income of `3,30,000
Computation of Total Income And Tax Liability 65

(iv) Mrs. X (non-resident) has total income of `3,30,000


(v) Mr. X (resident), aged 60 years has total income of `3,30,000
(vi) Mrs. X (resident), aged 60 years has total income of `3,30,000
(vii) Mr. X (non-resident), aged 60 years has total income of `3,30,000
(viii) Mrs. X (non-resident), aged 60 years has total income of `3,30,000
(ix) Mr. X (resident), aged 80 years has total income of `3,30,000
(x) Mrs. X (resident), aged 80 years has total income of `3,30,000
(xi) Mr. X (non-resident), aged 80 years has total income of `3,30,000
(xii) Mrs. X (non-resident), aged 80 years has total income of `3,30,000
Answer = (i) Tax Liability: `1,560; (ii) `4,160; (iii) `1,560; (iv) `4,160; (v) Nil; (vi) Nil; (vii) `4,160; (viii)
`4,160; (ix) Nil; (x) Nil; (xi) 4,160; (xii) `4,160

Problem 5.
Mr. X has income asunder: `
 Income under the head salary 2,40,000
 Income under the head house property 1,55,000
 Income under the head business/profession 3,30,000
 Long term capital gains 1,20,000
 Short term capital gains 35,000
 Casual income (winnings of lottery) 65,000
Deductions allowed under section 80C to 80U 45,000
Compute his tax liability for the assessment year 2019-20.
Answer = Tax Liability: `1,02,960

(b) Presume the assessee is Mrs. X.


Answer = Tax Liability: `1,02,960

(c) Presume the assessee is Mr. X, aged 65 years.


Answer = Tax Liability: `1,00,360

(d) Presume the assessee is Mrs. X (non-resident).


Answer = Tax Liability: `1,02,960

(e) Presume the assessee is Mr. X (non-resident) aged 65 years.


Answer = Tax Liability: `1,02,960

(f) Presume the assessee is Mr. X, aged 85 years.


Answer = Tax Liability: `89,960

(g) Presume the assessee is Mr. X (non-resident) aged 85 years.


Answer = Tax Liability: `1,02,960

Problem 6.
Compute tax liability for the assessment year 2019-20 in the following situations:
(i) Mr. X is resident in India and has income under the head house property `50,000 and income under the
head salary `30,000 and long term capital gains `8,00,000.
(ii) Presume in the above situation the assessee is Mrs. X.
(iii) Presume in the above situation the assessee is Mrs. X and she is aged about 70 years.
(iv) Presume in the above situation the assessee is Mr. X and he is aged about 70 years.
(v) Presume in the above situation the assessee is Mrs. X and she is aged about 87 years.
(vi) Presume in the above situation the assessee is Mr. X and he is aged about 87 years.
(vii) Presume in all the above situations, the assessee is non-resident in India.
Answer = (i) `1,31,040; (ii) `1,31,040; (iii) `1,20,640; (iv) `1,20,640; (v) `79,040; (vi) `79,040; (vii)
Computation of Total Income And Tax Liability 66

Situation (i): `1,66,400; Situation (ii): `1,66,400; Situation (iii): `1,66,400; Situation (iv): `1,66,400; (v):
`1,66,400; Situation (vi): `1,66,400

Problem 7.
Compute tax liability for the assessment year 2019-20 in the following situations:
(i) Mr. X is resident in India and his incomes are as follows:
(a) Income under the head Salary `90,000
(b) Income under the head House Property `60,000
(c) Long term capital gains `2,30,000
(d) Short term capital gain under section 111A `2,40,000
(e) Casual Income `70,000
(f) Deduction under section 80C to 80U `2,00,000 .
(ii) Presume in the above situation the assessee is Mrs. X.
(iii) Presume in the above situation the assessee is Mrs. X and she is aged about 70 years.
(iv) Presume in the above situation the assessee is Mr. X and he is aged about 70 years.
(v) Presume in the above situation the assessee is Mr. X and he is aged about 70 years old and he is non-
resident.
(vi) Presume in the above situation the assessee is Mrs. X and she is aged about 82 years.
(vii) Presume in the above situation the assessee is Mr. X and he is aged about 82 years.
(viii) Presume in the above situation the assessee is Mr. X and he is aged about 82 years old and he is non-
resident.
Answer = (i) `56,160; (ii) `56,160; (iii) `48,360; (iv) `48,360; (v) `1,07,120; (vi) `21,840; (vii) `21,840;
(viii) `1,07,120

Problem 8
Compute tax liability in the following situations:
(i) Mrs. X has income under the head House Property `8,42,324.
(ii) Mr. X has income under the head Business Profession `14,42,336.
(iii) Mr. X aged 66 years has long term capital gains `11,35,335.
(iv) Mr. X has long term capital gains of `13,35,334.90.
(v) Mrs. X has short term capital gains under section 111A `10,20,335.
(vi) Mrs. X, non-resident, has long term capital gains `5,40,337.
(vii) Mr. X, non-resident, aged about 66 years has winning of a lottery `7,20,000.
(viii) Mr. X aged 86 years has long term capital gains `15,65,385.
(ix) Mr. X, non-resident, aged about 90 years has winning of a lottery `10,20,000.
Answer = (i) `84,200; (ii) `2,55,010; (iii) `1,73,750; (iv) `2,25,750; (v) `1,20,170; (vi) `1,12,390; (vii)
`2,24,640; (viii) `2,21,600; (ix) `3,18,240

Problem 9.
Compute tax liability of ABC Ltd. a domestic company in the following situations for assessment year
2019-20:
(i) The company has income under the head Business/Profession `70,000.
(ii) The company has income under the head Business/Profession `150,00,000.
(iii) The company has income under the head Business/Profession `6,00,000.
(iv) The company has income under the head Business/Profession `10,30,000.
(v) The company has long term capital gains of `700,00,000.
(vi) The company has long term capital gains of `1,50,000.
(vii) The company has long term capital gains of `6,00,000.
(viii) The company has long term capital gains of `10,30,000.
(ix) The company has casual income `400,00,000.
Answer = (i) Tax Liability: `21,840; (ii) `50,07,600; (iii) `1,87,200; (iv) `3,21,360; (v) `155,79,200; (vi)
`31,200; (vii) `1,24,800; (viii) `2,14,240; (ix) `133,53,600
Computation of Total Income And Tax Liability 67

(b) Presume all the above situations the company is a foreign company.
Answer = (i) Tax Liability: `29,120; (ii) `63,64,800; (iii) `2,49,600; (iv) `4,28,480; (v) `148,51,200; (vi)
`31,200; (vii) `1,24,800; (viii) `2,14,240; (ix) `127,29,600

Problem 10.
ABC (HUF) has incomes as given below:
1. Income under the head Business/Profession `6,00,000
2. Income under the head House Property `4,00,000
3. Long term capital gains `4,50,000
4. Short term capital gains under section 111A `3,50,000
5. Casual Income `3,50,000
6. Deductions allowed under section 80C to 80U `1,25,000
Compute tax liability of HUF for the assessment year 2019-20.
Answer = Tax Liability: `3,48,400

Problem 11.
Mr. X has income under the head salary `3,00,000 and income under the head house property `16,00,000
and long term capital gain `1,00,000 and agricultural income `4,00,000. Deductions allowed under section
80C to 80U `60,000.
Compute his income and tax liability for Assessment Year 2019-20.
Answer = Total Income: `19,40,000; Tax Liability: `4,80,480

(b) Presume assessee is Mrs. X and is aged 64 years.


Answer = Total Income: `19,40,000; Tax Liability: `4,70,080

Problem 12.
Mr. X has income under the head house property `3,00,000 and long term capital gain `5,00,000 and
agricultural income `3,00,000. Deductions under section 80C to 80U `1,00,000.
Compute his income and tax liability for Assessment Year 2019-20.
Answer = Total Income: `7,00,000; Tax Liability: `93,600

(b) Presume Mr. X is aged 82 years


Answer = Total Income: `7,00,000; Tax Liability: `41,600

Problem 13.
Mrs. X has casual income `5,00,000 and short term capital gain under section 111A `7,00,000 and
agricultural income `3,00,000.
Compute her tax liability for Assessment Year 2019-20.
Answer = Tax Liability: `2,26,200

(b) Presume she is non-resident and is aged 63 years.


Answer = Tax Liability: `2,65,200

Problem 14.
Mr. X has agricultural income `10,00,000 and income from business `12,00,000 and casual income
`5,00,000 and he has completed the age of 80 years on 31.03.2019. Compute his tax liability Assessment
Year 2019-20.
Answer = Tax Liability: `3,74,400

(b) Presume he has completed 80 years on 01.04.2019.


Answer = Tax Liability: `4,36,800
Computation of Total Income And Tax Liability 68

Problem 15.
Mrs. X has income as given below: `
Income under the head Salary 3,00,000
Income under the head House Property 1,00,000
Short Term Capital Gain 50,000
Short Term Capital Gain111A 2,00,000
Long Term Capital Gain 1,50,000
Casual Income 70,000
Deduction u/s 80C to 80U 1,10,000
Agricultural Income 5,00,000
Compute her Tax Liability for the A.Y.2019-20
Answer = Tax Liability: `1,02,960

(b) Presume in above she is aged 81 years.


Answer = Tax Liability: `51,480

(c) Presume in (a) above she is Non Resident and deduction u/s 80C-80U is `3,00,000.
Answer = Tax Liability: `84,240

Problem 16.
Mrs. X has income under the head house property `2,00,000 and long term capital gain `10,00,000 and
agricultural income `7,00,000. Deduction under section 80C to 80U `60,000.
Compute her income and tax liability for Assessment Year 2019-20.
Answer = Total Income: `11,40,000; Tax Liability: `1,85,120

(b) Presume Mrs. X is aged 79 years and income under the head house property is `10,00,000
Answer = Total Income: `19,40,000; Tax Liability: `4,07,680

Problem 17.
ABC Ltd., a domestic company has total income of `500,00,000 and company has distributed dividend of
`65,00,000 and one of the shareholder Mr. X has received dividend of `5,00,000. Compute tax liability and
additional tax liability of the company and also that of the shareholder.
Answer = Income tax liability `166,92,000 and additional income tax liability `13,76,110. Tax liability of
shareholder is Nil.

(b) Presume that ABC Ltd. is a foreign company.


Answer = Income tax liability `212,16,000 and additional income tax liability is nil. Tax liability of
shareholder `13,000.
Computation of Total Income And Tax Liability 69

SOLUTIONS
TO
PRACTICE PROBLEMS
Solution 1: `
(i) Computation of Tax Liability
Total Income 22,00,000
Tax on `22,00,000 at slab rate 4,72,500
Add: HEC @ 4% 18,900
Tax Liability 4,91,400
(ii) Computation of Tax Liability
Total Income 22,00,000
Tax on `22,00,000 at slab rate 4,72,500
Add: HEC @ 4% 18,900
Tax Liability 4,91,400
(iii) Computation of Tax Liability
Total Income 22,00,000
Tax on `22,00,000 at slab rate 4,72,500
Add: HEC @ 4% 18,900
Tax Liability 4,91,400
(iv) Computation of Tax Liability
Total Income 22,00,000
Tax on `22,00,000 at slab rate 4,72,500
Add: HEC @ 4% 18,900
Tax Liability 4,91,400
(v) Computation of Tax Liability
Total Income 22,00,000
Tax on `22,00,000 at slab rate 4,70,000
Add: HEC @ 4% 18,800
Tax Liability 4,88,800
(vi) Computation of Tax Liability
Total Income 22,00,000
Tax on `22,00,000 at slab rate 4,70,000
Add: HEC @ 4% 18,800
Tax Liability 4,88,800
(vii) Computation of Tax Liability
Total Income 22,00,000
Tax on `22,00,000 at slab rate 4,72,500
Add: HEC @ 4% 18,900
Tax Liability 4,91,400
(viii) Computation of Tax Liability
Total Income 22,00,000
Tax on `22,00,000 at slab rate 4,72,500
Add: HEC @ 4% 18,900
Tax Liability 4,91,400
(ix) Computation of Tax Liability
Total Income 22,00,000
Tax on `22,00,000 at slab rate 4,60,000
Computation of Total Income And Tax Liability 70

Add: HEC @ 4% 18,400


Tax Liability 4,78,400
(x) Computation of Tax Liability
Total Income 22,00,000
Tax on `22,00,000 at slab rate 4,60,000
Add: HEC @ 4% 18,400
Tax Liability 4,78,400
(xi) Computation of Tax Liability
Total Income 22,00,000
Tax on `22,00,000 at slab rate 4,72,500
Add: HEC @ 4% 18,900
Tax Liability 4,91,400
(xii) Computation of Tax Liability
Total Income 22,00,000
Tax on `22,00,000 at slab rate 4,72,500
Add: HEC @ 4% 18,900
Tax Liability 4,91,400

Solution 2: `
(i) Computation of Tax Liability
Total Income 3,30,000
Tax on `3,30,000 at slab rate 4,000
Less: Rebate u/s 87A (2,500)
Tax before health & education cess 1,500
Add: HEC @ 4% 60
Tax Liability 1,560
(ii) Computation of Tax Liability
Total Income 3,30,000
Tax on `3,30,000 at slab rate 4,000
Add: HEC @ 4% 160
Tax Liability 4,160
Note: Rebate under section 87A is not allowed to non-resident.
(iii) Computation of Tax Liability
Total Income 3,30,000
Tax on `3,30,000 at slab rate 4,000
Less: Rebate u/s 87A (2,500)
Tax before health & education cess 1,500
Add: HEC @ 4% 60
Tax Liability 1,560
(iv) Computation of Tax Liability
Total Income 3,30,000
Tax on `3,30,000 at slab rate 4,000
Add: HEC @ 4% 160
Tax Liability 4,160
Note: Rebate under section 87A is not allowed to non-resident.
(v) Computation of Tax Liability
Total Income 3,30,000
Tax on `3,30,000 at slab rate 1,500
Less: Rebate u/s 87A (1,500)
Tax Liability Nil
(vi) Computation of Tax Liability
Total Income 3,30,000
Tax on `3,30,000 at slab rate 1,500
Computation of Total Income And Tax Liability 71

Less: Rebate u/s 87A (1,500)


Tax Liability Nil
(vii) Computation of Tax Liability
Total Income 3,30,000
Tax on `3,30,000 at slab rate 4,000
Add: HEC @ 4% 160
Tax Liability 4,160
Note: Rebate under section 87A is not allowed to non-resident.
(viii) Computation of Tax Liability
Total Income 3,30,000
Tax on `3,30,000 at slab rate 4,000
Add: HEC @ 4% 160
Tax Liability 4,160
Note: Rebate under section 87A is not allowed to non-resident.
(ix) Computation of Tax Liability
Total Income 3,30,000
Tax Liability Nil
(x) Computation of Tax Liability
Total Income 3,30,000
Tax Liability Nil
(xi) Computation of Tax Liability
Total Income 3,30,000
Tax on `3,30,000 at slab rate 4,000
Add: HEC @ 4% 160
Tax Liability 4,160
Note: Rebate under section 87A is not allowed to non-resident.
(xii) Computation of Tax Liability
Total Income 3,30,000
Tax on `3,30,000 at slab rate 4,000
Add: HEC @ 4% 160
Tax Liability 4,160
Note: Rebate under section 87A is not allowed to non-resident.

Solution 3: `
(i) Computation of Tax Liability
Total Income 50,20,000
Tax on `50,20,000 at slab rate 13,18,500
Add: Surcharge @ 10% 1,31,850
Tax before marginal relief 14,50,350
Less: Marginal Relief (1,17,850)
Working Note:
Tax + surcharge on income of `50,20,000 14,50,350
Tax on income of `50,00,000 (13,12,500)
Increase in tax 1,37,850
Increase in income 20,000
Marginal Relief (1,37,850 –20,000) 1,17,850
Tax after marginal relief 13,32,500
Add: HEC @ 4% 53,300
Tax Liability 13,85,800
(ii) Computation of Tax Liability
Total Income 53,00,000.00
Tax on `53,00,000 at slab rate 14,02,500.00
Add: Surcharge @ 10% 1,40,250.00
Computation of Total Income And Tax Liability 72

Tax before health & education cess 15,42,750.00


Add: HEC @ 4% 61,710.00
Tax Liability 16,04,460.00
(iii) Computation of Tax Liability
Total Income 51,00,000
Tax on `51,00,000 at slab rate 13,40,000
Add: Surcharge @ 10% 1,34,000
Tax before marginal relief 14,74,000
Less: Marginal Relief (64,000)
Working Note:
Tax + surcharge on income of `51,00,000 14,74,000
Tax on income of `50,00,000 (13,10,000)
Increase in tax 1,64,000
Increase in income 1,00,000
Marginal Relief (1,64,000 –1,00,000) 64,000
Tax after marginal relief 14,10,000
Add: HEC @ 4% 56,400
Tax Liability 14,66,400
(iv) Computation of Tax Liability
Total Income 54,00,000.00
Tax on `54,00,000 at slab rate 14,20,000.00
Add: Surcharge @ 10% 1,42,000.00
Tax before health & education cess 15,62,000.00
Add: HEC @ 4% 62,480.00
Tax Liability 16,24,480.00
(v) Computation of Tax Liability
Total Income 51,22,000.00
Tax on `51,22,000 at slab rate 13,49,100.00
Add: Surcharge @ 10% 1,34,910.00
Tax before marginal relief 14,84,010.00
Less: Marginal Relief (49,510)
Working Note:
Tax + surcharge on income of `51,22,000 14,84,010
Tax on income of `50,00,000 (13,12,500)
Increase in tax 1,71,510
Increase in income 1,22,000
Marginal Relief (1,71,510 –1,22,000) 49,510
Tax after marginal relief 14,34,500 .00
Add: HEC @ 4% 57,380.00
Tax Liability 14,91,880.00
(vi) Computation of Tax Liability
Total Income 50,80,000.00
Tax on `50,80,000 at slab rate 13,36,500.00
Add: Surcharge @ 10% 1,33,650.00
Tax before marginal relief 14,70,150.00
Less: Marginal Relief (77,650)
Working Note:
Tax + surcharge on income of `50,80,000 14,70,150
Tax on income of `50,00,000 (13,12,500)
Increase in tax 1,57,650
Increase in income 80,000
Marginal Relief (1,57,650 –80,000) 77,650
Tax after marginal relief 13,92,500 .00
Computation of Total Income And Tax Liability 73

Add: HEC @ 4% 55,700.00


Tax Liability 14,48,200.00

Solution 4: `
(i) Computation of Tax Liability
Total Income 100,05,000.00
Tax on `100,05,000 at slab rate 28,14,000.00
Add: Surcharge @ 15% 4,22,100.00
Tax before marginal relief 32,36,100.00
Less: Marginal Relief (1,37,350.00)
Working Note:
Tax + surcharge on income of `100,05,000 32,36,100
Tax + surcharge on income of `100,00,000 (30,93,750)
Increase in tax 1,42,350
Increase in income 5,000
Marginal Relief (1,42,350– 5,000) 1,37,350
Tax after marginal relief 30,98,750.00
Add: HEC @ 4% 1,23,950.00
Tax Liability 32,22,700.00
(ii) Computation of Tax Liability
Total Income 102,00,000.00
Tax on `102,00,000 at slab rate 28,72,500.00
Add: Surcharge @ 15% 4,30,875.00
Tax before marginal relief 33,03,375.00
Less: Marginal Relief (9,625.00)
Working Note:
Tax + surcharge on income of `102,00,000 33,03,375
Tax on income of `100,00,000 (30,93,750)
Increase in tax 2,09,625
Increase in income 2,00,000
Marginal Relief (2,09,625 – 2,00,000) 9,625
Tax after marginal relief 32,93,750.00
Add: HEC @ 4% 1,31,750.00
Tax Liability 34,25,500.00
(iii) Computation of Tax Liability
Total Income 90,00,000.00
Tax on `90,00,000 at slab rate 25,12,500.00
Add: Surcharge @ 10% 2,51,250.00
Tax before health & education cess 27,63,750.00
Add: HEC @ 4% 1,10,550.00
Tax Liability 28,74,300.00
(iv) Computation of Tax Liability
Total Income 4,98,000.00
Tax on `4,98,000 at slab rate 12,400.00
Add: HEC @ 4% 496.00
Tax Liability 12,896.00
Rounded off u/s 288B 12,900.00
(v) Computation of Tax Liability
Total Income 4,05,000.00
Tax on `4,05,000 at slab rate 5,250.00
Add: HEC @ 4% 210.00
Tax Liability 5,460.00
Computation of Total Income And Tax Liability 74

(vi) Computation of Tax Liability


Total Income 102,05,000.00
Tax on `102,05,000 at slab rate 28,71,500.00
Add: Surcharge @ 15% 4,30,725.00
Tax before marginal relief 33,02,225.00
Less: Marginal Relief (6,225.00)
Working Note:
Tax + surcharge on income of `102,05,000 33,02,225
Tax on income of `100,00,000 (30,91,000)
Increase in tax 2,11,225
Increase in income 2,05,000
Marginal Relief (2,11,225 – 2,05,000) 6,225
Tax after marginal relief 32,96,000.00
Add: HEC @ 4% 1,31,840.00
Tax Liability 34,27,840.00
(vii) Computation of Tax Liability
Total Income 25,00,000.00
Tax on `25,00,000 at slab rate 5,62,500.00
Add: HEC @ 4% 22,500.00
Tax Liability 5,85,000.00
(viii) Computation of Tax Liability
Total Income 4,50,000.00
Tax on `4,50,000 at slab rate 10,000.00
Add: HEC @ 4% 400.00
Tax Liability 10,400.00
(ix) Computation of Tax Liability
Total Income 3,80,000.00
Tax on `3,80,000 at slab rate Nil
Tax Liability Nil
(x) Computation of Tax Liability
Total Income 110,00,000.00
Tax on `110,00,000 at slab rate 31,00,000.00
Add: Surcharge @ 15% 4,65,000.00
Tax before health & education cess 35,65,000.00
Add: HEC @ 4% 1,42,600.00
Tax Liability 37,07,600.00
(xi) Computation of Tax Liability
Total Income 99,99,000.00
Tax on `99,99,000 at slab rate 28,12,200.00
Add: Surcharge @ 10% 2,81,220.00
Tax before health & education cess 30,93,420.00
Add: HEC @ 4% 1,23,736.80
Tax Liability 32,17,156.80
Rounded off u/s 288B 32,17,160.00
(xii) Computation of Tax Liability
Total Income 12,00,000.00
Tax on `12,00,000 at slab rate 1,72,500.00
Add: HEC @ 4% 6,900.00
Tax Liability 1,79,400.00

Solution 5: ` `
Computation of Total Income
Income under the head Salary 2,40,000
Computation of Total Income And Tax Liability 75

Income under the head House Property 1,55,000


Income under the head Business/Profession 3,30,000
Income under the head Capital gains
Long term capital gains 1,20,000
Short term capital gains 35,000 1,55,000
Income under the head Other Sources (Casual Income) 65,000
Gross Total Income 9,45,000
Less: Deductions u/s 80C to 80U (45,000)
Total Income 9,00,000
Computation of Tax Liability
Tax on Long term capital gains `1,20,000 @ 20% u/s 112 24,000
Tax on Casual Income `65,000 @ 30% u/s 115BB 19,500
Tax on Normal income `7,15,000 at slab rate 55,500
Tax before health & education cess 99,000
Add: HEC @ 4% 3,960
Tax Liability 1,02,960

Solution 5(b): `
Total Income 9,00,000
Computation of Tax Liability
Tax on Long term capital gains `1,20,000 @ 20% u/s 112 24,000
Tax on Casual Income `65,000 @ 30% u/s 115BB 19,500
Tax on Normal income `7,15,000 at slab rate 55,500
Tax before health & education cess 99,000
Add: HEC @ 4% 3,960
Tax Liability 1,02,960

Solution 5(c): `
Total Income 9,00,000
Computation of Tax Liability
Tax on Long term capital gains `1,20,000 @ 20% u/s 112 24,000
Tax on Casual Income `65,000 @ 30% u/s 115BB 19,500
Tax on Normal income `7,15,000 at slab rate 53,000
Tax before health & education cess 96,500
Add: HEC @ 4% 3,860
Tax Liability 1,00,360

Solution 5(d): `
Total Income 9,00,000
Computation of Tax Liability
Tax on Long term capital gains `1,20,000 @ 20% u/s 112 24,000
Tax on Casual Income `65,000 @ 30% u/s 115BB 19,500
Tax on Normal income `7,15,000 at slab rate 55,500
Tax before health & education cess 99,000
Add: HEC @ 4% 3,960
Tax Liability 1,02,960

Solution 5(e): `
Total Income 9,00,000
Computation of Tax Liability
Tax on Long term capital gains `1,20,000 @ 20% u/s 112 24,000
Tax on Casual Income `65,000 @ 30% u/s 115BB 19,500
Tax on Normal income `7,15,000 at slab rate 55,500
Computation of Total Income And Tax Liability 76

Tax before health & education cess 99,000


Add: HEC @ 4% 3,960
Tax Liability 1,02,960

Solution 5(f): `
Total Income 9,00,000
Computation of Tax Liability
Tax on Long term capital gains `1,20,000 @ 20% u/s 112 24,000
Tax on Casual Income `65,000 @ 30% u/s 115BB 19,500
Tax on Normal income `7,15,000 at slab rate 43,000
Tax before health & education cess 86,500
Add: HEC @ 4% 3,460
Tax Liability 89,960

Solution 5(g): `
Total Income 9,00,000
Computation of Tax Liability
Tax on Long term capital gains `1,20,000 @ 20% u/s 112 24,000
Tax on Casual Income `65,000 @ 30% u/s 115BB 19,500
Tax on Normal income `7,15,000 at slab rate 55,500
Tax before health & education cess 99,000
Add: HEC @ 4% 3,960
Tax Liability 1,02,960

Solution 6: `
(i)
Computation of Total Income
Income under the head Salary 30,000
Income under the head House Property 50,000
Income under the head Capital Gains (LTCG) 8,00,000
Gross Total Income 8,80,000
Less: Deduction u/s 80C to 80U Nil
Total Income 8,80,000
Computation of Tax Liability
Tax on LTCG `6,30,000 (8,00,000 – 1,70,000) @ 20% u/s 112 1,26,000
Tax on `80,000 at slab rate Nil
Tax before health & education cess 1,26,000
Add: HEC @ 4% 5,040
Tax Liability 1,31,040
(ii)
Total Income 8,80,000
Computation of Tax Liability
Tax on LTCG `6,30,000 (8,00,000 – 1,70,000) @ 20% u/s 112 1,26,000
Tax on `80,000 at slab rate Nil
Tax before health & education cess 1,26,000
Add: HEC @ 4% 5,040
Tax Liability 1,31,040
(iii)
Total Income 8,80,000
Computation of Tax Liability
Tax on LTCG `5,80,000 (8,00,000 – 2,20,000) @ 20% u/s 112 1,16,000
Tax on `80,000 at slab rate Nil
Tax before health & education cess 1,16,000
Computation of Total Income And Tax Liability 77

Add: HEC @ 4% 4,640


Tax Liability 1,20,640
(iv)
Total Income 8,80,000
Computation of Tax Liability
Tax on LTCG `5,80,000 (8,00,000 – 2,20,000) @ 20% u/s 112 1,16,000
Tax on `80,000 at slab rate Nil
Tax before health & education cess 1,16,000
Add: HEC @ 4% 4,640
Tax Liability 1,20,640
(v)
Total Income 8,80,000
Computation of Tax Liability
Tax on LTCG `3,80,000 (8,00,000 – 4,20,000) @ 20% u/s 112 76,000
Tax on `80,000 at slab rate Nil
Tax before health & education cess 76,000
Add: HEC @ 4% 3,040
Tax Liability 79,040
(vi)
Total Income 8,80,000
Computation of Tax Liability
Tax on LTCG `3,80,000 (8,00,000 – 4,20,000) @ 20% u/s 112 76,000
Tax on `80,000 at slab rate Nil
Tax before health & education cess 76,000
Add: HEC @ 4% 3,040
Tax Liability 79,040
(vii)In situation (i)
Total Income 8,80,000
Computation of Tax Liability
Tax on LTCG `8,00,000 @ 20% u/s 112 1,60,000
Tax on `80,000 at slab rate Nil
Tax before health & education cess 1,60,000
Add: HEC @ 4% 6,400
Tax Liability 1,66,400
In situation (ii)
Total Income 8,80,000
Computation of Tax Liability
Tax on LTCG `8,00,000 @ 20% u/s 112 1,60,000
Tax on `80,000 at slab rate Nil
Tax before health & education cess 1,60,000
Add: HEC @ 4% 6,400
Tax Liability 1,66,400
In situation (iii)
Total Income 8,80,000
Computation of Tax Liability
Tax on LTCG `8,00,000 @ 20% u/s 112 1,60,000
Tax on `80,000 at slab rate Nil
Tax before health & education cess 1,60,000
Add: HEC @ 4% 6,400
Tax Liability 1,66,400
In situation (iv)
Total Income 8,80,000
Computation of Total Income And Tax Liability 78

Computation of Tax Liability


Tax on LTCG `8,00,000 @ 20% u/s 112 1,60,000
Tax on `80,000 at slab rate Nil
Tax before health & education cess 1,60,000
Add: HEC @ 4% 6,400
Tax Liability 1,66,400
In situation (v)
Total Income 8,80,000
Computation of Tax Liability
Tax on LTCG `8,00,000 @ 20% u/s 112 1,60,000
Tax on `80,000 at slab rate Nil
Tax before health & education cess 1,60,000
Add: HEC @ 4% 6,400
Tax Liability 1,66,400
In situation (vi)
Total Income 8,80,000
Computation of Tax Liability
Tax on LTCG `8,00,000 @ 20% u/s 112 1,60,000
Tax on `80,000 at slab rate Nil
Tax before health & education cess 1,60,000
Add: HEC @ 4% 6,400
Tax Liability 1,66,400

Solution 7:
(i)
Computation of Total Income
Income under the head Salary 90,000
Income under the head House Property 60,000
Income under the head Capital Gains
Long term capital gains 2,30,000
Short term capital gains u/s 111A 2,40,000 4,70,000
Income under the head Other Sources (Casual Income) 70,000
Gross Total Income 6,90,000
Less: Deduction u/s 80C to 80U (1,50,000)
Total Income 5,40,000
Computation of Tax Liability
Tax on LTCG (2,30,000 – 2,30,000) @ 20% u/s 112 Nil
Tax on STCG `2,20,000 (`2,40,000 – 20,000) @ 15% u/s 111A 33,000
Tax on Casual Income `70,000 @ 30% u/s 115BB 21,000
Tax on normal income at slab rate Nil
Tax before health & education cess 54,000
Add: HEC @ 4% 2,160
Tax Liability 56,160
(ii)
Total Income 5,40,000
Computation of Tax Liability
Tax on LTCG (2,30,000 – 2,30,000) @ 20% u/s 112 Nil
Tax on STCG `2,20,000 (`2,40,000 – 20,000) @ 15% u/s 111A 33,000
Tax on Casual Income `70,000 @ 30% u/s 115BB 21,000
Tax on normal income at slab rate Nil
Tax before health & education cess 54,000
Add: HEC @ 4% 2,160
Tax Liability 56,160
Computation of Total Income And Tax Liability 79

(iii)
Total Income 5,40,000
Computation of Tax Liability
Tax on LTCG (2,30,000 – 2,30,000) @ 20% u/s 112 Nil
Tax on STCG `1,70,000 (2,40,000 – 70,000) @ 15% u/s 111A 25,500
Tax on Casual Income `70,000 @ 30% u/s 115BB 21,000
Tax on normal income at slab rate Nil
Tax before health & education cess 46,500
Add: HEC @ 4% 1,860
Tax Liability 48,360
(iv)
Total Income 5,40,000
Computation of Tax Liability
Tax on LTCG (2,30,000 – 2,30,000) @ 20% u/s 112 Nil
Tax on STCG `1,70,000 (2,40,000 – 70,000) @ 15% u/s 111A 25,500
Tax on Casual Income `70,000 @ 30% u/s 115BB 21,000
Tax on normal income at slab rate Nil
Tax before health & education cess 46,500
Add: HEC @ 4% 1,860
Tax Liability 48,360
(v)
Total Income 5,40,000
Computation of Tax Liability
Tax on LTCG `2,30,000 @ 20% u/s 112 46,000
Tax on STCG `2,40,000 @ 15% u/s 111A 36,000
Tax on Casual Income `70,000 @ 30% u/s 115BB 21,000
Tax on normal income at slab rate Nil
Tax before health & education cess 1,03,000
Add: HEC @ 4% 4,120
Tax Liability 1,07,120
(vi)
Total Income 5,40,000
Computation of Tax Liability
Tax on LTCG (2,30,000 – 2,30,000) @ 20% u/s 112 Nil
Tax on STCG (2,40,000 – 2,40,000) @ 15% u/s 111A Nil
Tax on Casual Income `70,000 @ 30% u/s 115BB 21,000
Tax on normal income at slab rate Nil
Tax before health & education cess 21,000
Add: HEC @ 4% 840
Tax Liability 21,840
(vii)
Total Income 5,40,000
Computation of Tax Liability
Tax on LTCG (2,30,000 – 2,30,000) @ 20% u/s 112 Nil
Tax on STCG (2,40,000 – 2,40,000) @ 15% u/s 111A Nil
Tax on Casual Income `70,000 @ 30% u/s 115BB 21,000
Tax on normal income at slab rate Nil
Tax before health & education cess 21,000
Add: HEC @ 4% 840
Tax Liability 21,840
(viii)
Total Income 5,40,000
Computation of Total Income And Tax Liability 80

Computation of Tax Liability


Tax on LTCG `2,30,000 @ 20% u/s 112 46,000
Tax on STCG `2,40,000 @ 15% u/s 111A 36,000
Tax on Casual Income `70,000 @ 30% u/s 115BB 21,000
Tax on normal income at slab rate Nil
Tax before health & education cess 1,03,000
Add: HEC @ 4% 4,120
Tax Liability 1,07,120

Solution 8:
(i) `
Income under the head House Property 8,42,324.00
Gross Total Income 8,42,324.00
Less: Deduction u/s 80C to 80U Nil
Total Income 8,42,324.00
Rounded off u/s 288A 8,42,320.00
Computation of Tax Liability
Tax on `8,42,320 slab rate 80,964.00
Add: HEC @ 4% 3,238.56
Tax Liability 84,202.56
Rounded off u/s 288B 84,200.00
(ii)
Income under the head Business/Profession 14,42,336.00
Gross Total Income 14,42,336.00
Less: Deduction u/s 80C to 80U Nil
Total Income 14,42,336.00
Rounded off u/s 288A 14,42,340.00
Computation of Tax Liability
Tax on `14,42,340 slab rate 2,45,202.00
Add: HEC @ 4% 9,808.08
Tax Liability 2,55,010.08
Rounded off u/s 288B 2,55,010.00
(iii)
Income under the head Capital Gains (LTCG) 11,35,335.00
Gross Total Income 11,35,335.00
Less: Deduction u/s 80C to 80U Nil
Total Income 11,35,335.00
Rounded off u/s 288A 11,35,340.00
Computation of Tax Liability
Tax on `8,35,340 (11,35,340 – 3,00,000) @ 20% u/s 112 1,67,068.00
Add: HEC @ 4% 6,682.72
Tax Liability 1,73,750.72
Rounded off u/s 288B 1,73,750.00
(iv)
Income under the head Capital Gains (LTCG) 13,35,334.90
Gross Total Income 13,35,334.90
Less: Deduction u/s 80C to 80U Nil
Total Income 13,35,334.90
Rounded off u/s 288A 13,35,330.00
Computation of Tax Liability
Tax on `10,85,330 (13,35,330 – 2,50,000) @ 20% u/s 112 2,17,066.00
Add: HEC @ 4% 8,682.64
Tax Liability 2,25,748.64
Computation of Total Income And Tax Liability 81

Rounded off u/s 288B 2,25,750.00


(v)
Income under the head Capital Gains (STCG) 10,20,335.00
Gross Total Income 10,20,335.00
Less: Deduction u/s 80C to 80U Nil
Total Income 10,20,335.00
Rounded off u/s 288A 10,20,340.00
Computation of Tax Liability
Tax on `7,70,340 (10,20,340 – 2,50,000) @ 15% u/s 111A 1,15,551.00
Add: HEC @ 4% 4,622.04
Tax Liability 1,20,173.04
Rounded off u/s 288B 1,20,170.00
(vi)
Income under the head Capital Gains (LTCG) 5,40,337.00
Gross Total Income 5,40,337.00
Less: Deduction u/s 80C to 80U Nil
Total Income 5,40,337.00
Rounded off u/s 288A 5,40,340.00
Computation of Tax Liability
Tax on `5,40,340 @ 20% u/s 112 1,08,068.00
Add: HEC @ 4% 4,322.72
Tax Liability 1,12,390.72
Rounded off u/s 288B 1,12,390.00
(vii)
Income under the head Other Sources (winning from lottery) 7,20,000.00
Gross Total Income 7,20,000.00
Less: Deduction u/s 80C to 80U Nil
Total Income 7,20,000.00
Computation of Tax Liability
Tax on `7,20,000 @ 30% u/s 115BB 2,16,000.00
Add: HEC @ 4% 8,640.00
Tax Liability 2,24,640.00
(viii)
Income under the head Capital Gains (LTCG) 15,65,385.00
Gross Total Income 15,65,385.00
Less: Deduction u/s 80C to 80U Nil
Total Income 15,65,385.00
Rounded off u/s 288A 15,65,390.00
Computation of Tax Liability
Tax on `10,65,390 (15,65,390 – 5,00,000) @ 20% u/s 112 2,13,078.00
Add: HEC @ 4% 8,523.12
Tax Liability 2,21,601.12
Rounded off u/s 288B 2,21,600.00
(ix)
Income under the head Other Sources (winning from lottery) 10,20,000.00
Gross Total Income 10,20,000.00
Less: Deduction u/s 80C to 80U Nil
Total Income 10,20,000.00
Computation of Tax Liability
Tax on `10,20,000 @ 30% u/s 115BB 3,06,000.00
Add: HEC @ 4% 12,240.00
Tax Liability 3,18,240.00
Computation of Total Income And Tax Liability 82

Solution 9: `
(i) Computation of Tax Liability
Income under the head Business/Profession 70,000
Total Income 70,000
Tax on `70,000 @ 30% 21,000
Add: HEC @ 4% 840
Tax Liability 21,840
(ii) Computation of Tax Liability
Income under the head Business/Profession 150,00,000
Total Income 150,00,000
Tax on `150,00,000 @ 30% 45,00,000
Add: Surcharge @ 7% 3,15,000
Add: HEC @ 4% 1,92,600
Tax Liability 50,07,600
(iii) Computation of Tax Liability
Income under the head Business/Profession 6,00,000
Total Income 6,00,000
Tax on `6,00,000 @ 30% 1,80,000
Add: HEC @ 4% 7,200
Tax Liability 1,87,200
(iv) Computation of Tax Liability
Income under the head Business/Profession 10,30,000
Total Income 10,30,000
Tax on `10,30,000 @ 30% 3,09,000
Add: HEC @ 4% 12,360
Tax Liability 3,21,360
(v) Computation of Tax Liability
Income under the head Capital Gains (long term capital gains) 700,00,000
Total Income 700,00,000
Tax on `700,00,000 @ 20% 140,00,000
Add: Surcharge @ 7% 9,80,000
Add: HEC @ 4% 5,99,200
Tax Liability 155,79,200
(vi) Computation of Tax Liability
Income under the head Capital Gains (long term capital gains) 1,50,000
Total Income 1,50,000
Tax on `1,50,000 @ 20% 30,000
Add: HEC @ 4% 1,200
Tax Liability 31,200
(vii) Computation of Tax Liability
Income under the head Capital Gains (long term capital gains) 6,00,000
Total Income 6,00,000
Tax on `6,00,000 @ 20% 1,20,000
Add: HEC @ 4% 4,800
Tax Liability 1,24,800
(viii) Computation of Tax Liability
Income under the head Capital Gains (long term capital gains) 10,30,000
Total Income 10,30,000
Tax on `10,30,000 @ 20% 2,06,000
Add: HEC @ 4% 8,240
Tax Liability 2,14,240
(ix) Computation of Tax Liability
Income under the head Other Sources (Casual Income) 400,00,000
Computation of Total Income And Tax Liability 83

Total Income 400,00,000


Tax on `400,00,000 @ 30% 120,00,000
Add: Surcharge @ 7% 8,40,000
Add: HEC @ 4% 5,13,600
Tax Liability 133,53,600

Solution 9(b): `
(i) Computation of Tax Liability
Income under the head Business/Profession 70,000
Total Income 70,000
Tax on `70,000 @ 40% 28,000
Add: HEC @ 4% 1,120
Tax Liability 29,120
(ii) Computation of Tax Liability
Income under the head Business/Profession 150,00,000
Total Income 150,00,000
Tax on `150,00,000 @ 40% 60,00,000
Add: Surcharge @ 2% 1,20,000
Add: HEC @ 4% 2,44,800
Tax Liability 63,64,800
(iii) Computation of Tax Liability
Income under the head Business/Profession 6,00,000
Total Income 6,00,000
Tax on `6,00,000 @ 40% 2,40,000
Add: HEC @ 4% 9,600
Tax Liability 2,49,600
(iv) Computation of Tax Liability
Income under the head Business/Profession 10,30,000
Total Income 10,30,000
Tax on `10,30,000 @ 40% 4,12,000
Add: HEC @ 4% 16,480
Tax Liability 4,28,480
(v) Computation of Tax Liability
Income under the head Capital Gains (long term capital gains) 700,00,000
Total Income 700,00,000
Tax on `700,00,000 @ 20% 140,00,000
Add: Surcharge @ 2% 2,80,000
Add: HEC @ 4% 5,71,200
Tax Liability 148,51,200
(vi) Computation of Tax Liability
Income under the head Capital Gains (long term capital gains) 1,50,000
Total Income 1,50,000
Tax on `1,50,000 @ 20% 30,000
Add: HEC @ 4% 1,200
Tax Liability 31,200
(vii) Computation of Tax Liability
Income under the head Capital Gains (long term capital gains) 6,00,000
Total Income 6,00,000
Tax on `6,00,000 @ 20% 1,20,000
Add: HEC @ 4% 4,800
Tax Liability 1,24,800
(viii) Computation of Tax Liability
Income under the head Capital Gains (long term capital gains) 10,30,000
Computation of Total Income And Tax Liability 84

Total Income 10,30,000


Tax on `10,30,000 @ 20% 2,06,000
Add: HEC @ 4% 8,240
Tax Liability 2,14,240
(ix) Computation of Tax Liability
Income under the head Other Sources (Casual Income) 400,00,000
Total Income 400,00,000
Tax on `400,00,000 @ 30% 120,00,000
Add: Surcharge @ 2% 2,40,000
Add: HEC @ 4% 4,89,600
Tax Liability 127,29,600

Solution 10: `
Computation of Total Income
Income under the head Business/Profession 6,00,000.00
Income under the head House Property 4,00,000.00
Income under the head Capital Gains
Long term capital gains 4,50,000
Short term capital gains u/s 111A 3,50,000 8,00,000.00
Income under the head Other Sources (Casual Income) 3,50,000.00
Gross Total Income 21,50,000.00
Less: Deduction u/s 80C to 80U (1,25,000.00)
Total Income 20,25,000.00
Computation of Tax Liability
Tax on LTCG `4,50,000 @ 20% u/s 112 90,000.00
Tax on STCG `3,50,000 @ 15% u/s 111A 52,500.00
Tax on Casual income `3,50,000 @ 30% u/s 115BB 1,05,000.00
Tax on `8,75,000 at slab rate 87,500.00
Tax before health & education cess 3,35,000.00
Add: HEC @ 4% 13,400.00
Tax Liability 3,48,400.00

Solution 11:
Computation of Total Income `
Income under the head Salary 3,00,000
Income under the head House Property 16,00,000
Income under the head Capital Gains (Long term capital gains) 1,00,000
Gross Total Income 20,00,000
Less: Deduction u/s 80C to 80U (60,000)
Total Income 19,40,000
Agricultural Income 4,00,000
Computation of Tax Liability
Tax on long term capital gain `1,00,000 @ 20% u/s 112 20,000
Normal income `18,40,000
Step 1. Tax on (18,40,000 + 4,00,000) 4,84,500
Step 2. Tax on (`2,50,000 + 4,00,000) at slab rates (42,500)
Step 3. Deduct Tax at Step 2 from Tax at Step 1 4,42,000
Tax before health & education cess 4,62,000
Add: HEC @ 4% 18,480
Tax Liability 4,80,480
Computation of Total Income And Tax Liability 85

Solution 11(b):
Computation of Total Income `
Income under the head Salary 3,00,000
Income under the head House Property 16,00,000
Income under the head Capital Gains (Long term capital gains) 1,00,000
Gross Total Income 20,00,000
Less: Deduction u/s 80C to 80U (60,000)
Total Income 19,40,000
Agricultural Income 4,00,000
Computation of Tax Liability
Tax on long term capital gain `1,00,000 @ 20% u/s 112 20,000
Step 1. Tax on (18,40,000 + 4,00,000) 4,82,000
Step 2. Tax on (`3,00,000 + 4,00,000) at slab rates (50,000)
Step 3. Deduct Tax at Step 2 from Tax at Step 1 4,32,000
Tax before health & education cess 4,52,000
Add: HEC @ 4% 18,080
Tax Liability 4,70,080

Solution 12:
Computation of Total Income `
Income under the head House Property 3,00,000
Income under the head Capital gains (long term capital gains) 5,00,000
Gross Total Income 8,00,000
Less: Deduction u/s 80C to 80U (1,00,000)
Total Income 7,00,000
Agricultural income 3,00,000
Computation of Tax Liability
Tax on normal income `2,00,000 at slab rate Nil
Tax on long term capital gain `4,50,000 (`5,00,000 – `50,000) @ 20% u/s 112 90,000
Tax before health & education cess 90,000
Add: HEC @ 4% 3,600
Tax Liability 93,600

Solution 12(b):
Computation of Total Income `
Income under the head House Property 3,00,000
Income under the head Capital gains (long term capital gains) 5,00,000
Gross Total Income 8,00,000
Less: Deduction u/s 80C to 80U (1,00,000)
Total Income 7,00,000
Agricultural income 3,00,000
Computation of Tax Liability
Tax on normal income `2,00,000 at slab rate Nil
Tax on long term capital gain `2,00,000 (`5,00,000 – `3,00,000) @ 20% u/s 112 40,000
Tax before health & education cess 40,000
Add: HEC @ 4% 1,600
Tax Liability 41,600
Note: If non-agricultural income is upto the limit not chargeable to tax (`2,50,000/3,00,000/5,00,000),
partial integration is not applicable.

Solution 13:
Computation of Total Income `
Income under the head Other Sources (Casual income) 5,00,000
Computation of Total Income And Tax Liability 86

Income under the head Capital gains (STCG u/s 111A) 7,00,000
Gross Total Income 12,00,000
Less: Deduction u/s 80C to 80U Nil
Total Income 12,00,000
Agricultural income 3,00,000
Computation of Tax Liability
Tax on STCG `4,50,000 (`7,00,000 – 2,50,000) @ 15% u/s 111A 67,500
Tax on casual income `5,00,000 @ 30% 1,50,000
Tax before health & education cess 2,17,500
Add: HEC @ 4% 8,700
Tax Liability 2,26,200
Note: On Long term capital gain, casual income and short term capital gain u/s 111A partial integration
shall not be applicable.

Solution 13(b):
Computation of Total Income `
Income under the head Other Sources (Casual income) 5,00,000
Income under the head Capital gains (STCG u/s 111A) 7,00,000
Gross Total Income 12,00,000
Less: Deduction u/s 80C to 80U Nil
Total Income 12,00,000
Agricultural income 3,00,000
Computation of Tax Liability
Tax on STCG `7,00,000 @ 15% u/s 111A 1,05,000
Tax on casual income `5,00,000 @ 30% 1,50,000
Tax before health & education cess 2,55,000
Add: HEC @ 4% 10,200
Tax Liability 2,65,200
Note: On Long term capital gain, casual income and short term capital gain u/s 111A partial integration
shall not be applicable.

Solution 14:
Computation of Total Income `
Income under the head Business/Profession 12,00,000
Income under the head Other Sources (Casual income) 5,00,000
Gross Total Income 17,00,000
Less: Deduction u/s 80C to 80U Nil
Total Income 17,00,000
Agricultural income 10,00,000
Computation of Tax Liability
Tax on casual income `5,00,000 @ 30% u/s 115BB 1,50,000
Normal income `12,00,000
Step 1. Tax on (12,00,000 + 10,00,000) 4,60,000
Step 2. Tax on (`5,00,000 + 10,00,000) at slab rates (2,50,000)
Step 3. Deduct Tax at Step 2 from Tax at Step 1 2,10,000
Tax before health & education cess 3,60,000
Add: HEC @ 4% 14,400
Tax Liability 3,74,400

Solution 14(b):
Computation of Total Income `
Income under the head Business/Profession 12,00,000
Income under the head Other Sources (Casual income) 5,00,000
Computation of Total Income And Tax Liability 87

Gross Total Income 17,00,000


Less: Deduction u/s 80C to 80U Nil
Total Income 17,00,000
Agricultural income 10,00,000
Computation of Tax Liability
Tax on casual income `5,00,000 @ 30% u/s 115BB 1,50,000
Normal income `12,00,000
Step 1. Tax on (12,00,000 + 10,00,000) 4,70,000
Step 2. Tax on (`3,00,000 + 10,00,000) at slab rates (2,00,000)
Step 3. Deduct Tax at Step 2 from Tax at Step 1 2,70,000
Tax before health & education cess 4,20,000
Add: HEC @ 4% 16,800
Tax Liability 4,36,800

Solution 15:
Computation of Total Income `
Income under the head Salary 3,00,000
Income under the head House Property 1,00,000
Income under the head Capital Gains
Short Term Capital Gain 50,000
Short Term Capital Gain111A 2,00,000
Long Term Capital Gain 1,50,000
Income under the head Capital Gains 4,00,000
Casual Income 70,000
Gross Total Income 8,70,000
Less: Deduction u/s 80C to 80U (1,10,000)
Total Income 7,60,000
Agricultural Income 5,00,000
Computation of Tax Liability
Tax on casual income `70,000 @ 30% u/s 115BB 21,000
Tax on long term capital gain `1,50,000 @ 20% u/s 112 30,000
Tax on short term capital gain `2,00,000 @ 15% u/s 111A 30,000
Normal income `3,40,000
Step 1. Tax on (3,40,000 + 5,00,000) 80,500
Step 2. Tax on (`2,50,000 + 5,00,000) at slab rates (62,500)
Step 3. Deduct Tax at Step 2 from Tax at Step 1 18,000
Tax before health & education cess 99,000
Add: HEC @ 4% 3,960
Tax Liability 1,02,960

Solution 15(b):
Computation of Total Income `
Income under the head Salary 3,00,000
Income under the head House Property 1,00,000
Income under the head Capital Gains
Short Term Capital Gain 50,000
Short Term Capital Gain111A 2,00,000
Long Term Capital Gain 1,50,000
Income under the head Capital Gains 4,00,000
Casual Income 70,000
Gross Total Income 8,70,000
Less: Deduction u/s 80C to 80U (1,10,000)
Total Income 7,60,000
Computation of Total Income And Tax Liability 88

Agricultural Income 5,00,000


Computation of Tax Liability
Tax on casual income `70,000 @ 30% u/s 115BB 21,000
Tax on long term capital gain Nil (`1,50,000-1,50,000) @ 20% u/s 112 Nil
Tax on short term capital gain ` 1,90,000(`2,00,000-`10,000) @ 15% u/s 111A 28,500
Tax on normal income `3,40,000 at slab rate Nil
Tax before health & education cess 49,500
Add: HEC @ 4% 1,980
Tax Liability 51,480
Note: If non-agricultural income is upto the limit not chargeable to tax (`2,50,000/ 3,00,000/5,00,000),
partial integration is not applicable.

Solution 15(c):
Computation of Total Income `
Income under the head Salary 3,00,000
Income under the head House Property 1,00,000
Income under the head Capital Gains
Short Term Capital Gain 50,000
Short Term Capital Gain111A 2,00,000
Long Term Capital Gain 1,50,000
Income under the head Capital Gains 4,00,000
Casual Income 70,000
Gross Total Income 8,70,000
Less: Deduction u/s 80C to 80U (3,00,000)
Total Income 5,70,000
Agricultural Income 5,00,000
Computation of Tax Liability
Tax on casual income `70,000 @ 30% u/s 115BB 21,000
Tax on long term capital gain `1,50,000 @ 20% u/s 112 30,000
Tax on short term capital gain `2,00,000 @ 15% u/s 111A 30,000
Tax on normal income `1,50,000 at slab rate Nil
Tax before health & education cess 81,000
Add: HEC @ 4% 3,240
Tax Liability 84,240
Note: If non-agricultural income is upto the limit not chargeable to tax (`2,50,000/ 3,00,000/5,00,000),
partial integration is not applicable.

Solution 16:
Computation of Total Income `
Income under the head House Property 2,00,000
Income under the head capital gains (long term capital gains) 10,00,000
Gross Total Income 12,00,000
Less: Deduction u/s 80C to 80U (60,000)
Total Income 11,40,000
Agricultural income 7,00,000
Computation of Tax Liability
Tax on normal income `1,40,000 at slab rate Nil
Tax on long term capital gain `8,90,000 (`10,00,000 – `1,10,000) @ 20% u/s 112 1,78,000
Tax before health & education cess 1,78,000
Add: HEC @ 4% 7,120
Tax Liability 1,85,120
Note: If non-agricultural income is upto the limit not chargeable to tax (`2,50,000/ 3,00,000/5,00,000),
partial integration is not applicable.
Computation of Total Income And Tax Liability 89

Solution 16(b):
Computation of Total Income `
Income under the head House Property 10,00,000
Income under the head capital gains (long term capital gains) 10,00,000
Gross Total Income 20,00,000
Less: Deduction u/s 80C to 80U (60,000)
Total Income 19,40,000
Agricultural income 7,00,000
Computation of Tax Liability
Tax on long term capital gain `10,00,000 @ 20% u/s 112 2,00,000
Normal income `9,40,000
Step 1. Tax on (9,40,000 + 7,00,000) at slab rates 3,02,000
Step 2. Tax on (`3,00,000 + 7,00,000) at slab rates (1,10,000)
Step 3. Deduct Tax at Step 2 from Tax at Step 1 1,92,000
Tax before health & education cess 3,92,000
Add: HEC @ 4% 15,680
Tax Liability 4,07,680

Solution 17:
Tax liability and additional tax liability of the company shall be as given below: `
Profit before tax 500,00,000.00
Income tax on `500,00,000 @ 30 % 150,00,000.00
Add: Surcharge @ 7% 10,50,000.00
Add: HEC @ 4% 6,42,000.00
Income tax liability 166,92,000.00
Dividend 65,00,000.00
65,00,000 + (65,00,000 / 82.528% x 17.472%) 78,76,114.77
Additional Income Tax 13,76,114.77
78,76,114.77 x 17.472%
Rounded off u/s 288B 13,76,110.00
Tax liability of the shareholder shall be nil.

Solution 17(b):
Tax liability and additional tax liability of the company shall be as given below: `
Profit before tax 500,00,000.00
Income tax on `500,00,000 @ 40% 200,00,000.00
Add: Surcharge @ 2% 4,00,000.00
Add: HEC @ 4% 8,16,000.00
Income tax liability 212,16,000.00
Additional income tax of the foreign company is nil.
Tax liability of the shareholder shall be as given below:
Dividend from foreign company 5,00,000.00
Tax on `5,00,000 at slab rate 12,500.00
Add: HEC @ 4% 500.00
Tax Liability 13,000.00
Computation of Total Income And Tax Liability 90

EXAMINATION QUESTIONS
NOV – 2011
Question 7 (4 Marks)
Discuss the taxability of agricultural income under the Income Tax Act, 1961. How will income be
computed where an individual derives agricultural and non-agricultural income?
Answer: Under section 10(1), any agricultural income in India is fully exempt from income tax but if the
agricultural income is from outside India, it is chargeable to tax.
Indirect taxing of agricultural income or partial integration of agricultural income (Under the
constitution, the power to levy a tax on agricultural income vests in the states. However, parliament
has also levied a tax on such income. Explain how this has been achieved?)
If any person has agricultural income as well as non-agricultural income, his tax liability shall be computed
in the manner given below:
1. Compute tax on the total of agricultural income and non- agricultural income considering it to be total
income of the assessee.
2. Compute tax on exemption limit (`2,50,000 / 3,00,000 / 5,00,000) and agricultural income considering
it to be total income.
3. Deduct tax computed under Step 2 from Step 1 and apply health & education cess.
4. Long term capital gain, casual income and short term capital gain u/s 111A shall not be taken into
consideration for the purpose of partial integration
5. If Agricultural income is upto `5,000, or non-agricultural income is upto the limit not chargeable to tax
(`2,50,000/3,00,000/5,00,000), partial integration is not applicable.
6. Partial integration is not applicable in case of a partnership firm or a company.

MAY – 2007
Question 3 (4 Marks)
The broad break-up of tax and allied details of Mrs. X, born on 31st March, 1959 are as under: `
Long-term capital gains on sale of house 2,00,000
Short-term capital gains on sale of shares in B Ltd. (STT paid) 30,000
Prize winning from a T.V. show 20,000
Business income 2,90,000
Net agricultural income 4,40,000
Deduction allowed under section 80C to 80U 60,000
Compute the tax payable by Mrs. X for the assessment year 2019-20.
Answer. Computation of Total Income
Business Income 2,90,000
Long term capital gain on sale of house 2,00,000
Short-term capital gains on sale of shares in B Ltd. (STT paid) 30,000
Casual Income (Prize winning from a T.V. show) 20,000
Gross Total Income 5,40,000
Less: Deduction u/s 80C to 80U (60,000)
Total Income 4,80,000
Computation of tax payable by Mrs. X for the A.Y. 2019-20
Particulars `
(i) Tax on long-term capital gain of `1,30,000 (2,00,000 – 70,000) @ 20% 26,000
(ii) Tax on short term capital gain of `30,000 @ 15% 4,500
(iii) Tax on winnings of `20,000 from a T.V. show @ 30% 6,000
(iv) Tax on balance income of `2,30,000 at slab rate Nil
Deficiency of `70,000 has been allowed from LTCG
Amount of tax before HEC 36,500
Add: HEC @ 4% 1,460
Tax payable by Mrs. X 37,960
Computation of Total Income And Tax Liability 91

(i) Mrs. X has completed 60 years of age on 31st March, 2019 i.e. she has completed the age of 60 years on
the last day of the previous year. Therefore, she is entitled to the higher basic exemption limit of `3,00,000.
(ii) Partial integration is not applicable because her non-agricultural income is not exceeding the exemption
limit of `3,00,000.
(b) Presume income from business is `5,00,000.
Computation of Gross Total Income
Business Income 5,00,000
Long term capital gain on sale of house 2,00,000
Short-term capital gains on sale of shares in B Ltd. (STT paid) 30,000
Casual Income (Prize winning from a T.V. show) 20,000
Gross Total Income 7,50,000
Less: Deduction u/s 80C to 80U (60,000)
Total Income 6,90,000
Computation of tax payable by Mrs. X for the A.Y. 2019-20 `
(i) Tax on long-term capital gain of `2,00,000 @ 20% 40,000
(ii) Tax on short term capital gain of `30,000 @ 15% 4,500
(iii) Tax on winnings of `20,000 from a T.V. show @ 30% 6,000
(iv) Tax on balance income of `4,40,000
1. Tax on 4,40,000 + 4,40,000 = 8,80,000 86,000
2. Tax on 3,00,000 + 4,40,000 = 7,40,000 (58,000)
Tax at step no.1 minus tax at step no.2 28,000
Amount of tax before HEC 78,500
Add: HEC @ 4% 3,140
Tax payable by Mrs. X 81,640
Mrs. X has completed 60 years of age on 31st March, 2019 i.e. she has completed the age of 60 years on the
last day of the previous year. Therefore, she is entitled to the higher basic exemption limit of `3,00,000.

MAY – 2006
Question 2 (3 Marks)
(a) The total income of Mrs. X computed for the assessment year 2019-20 is `3,80,000, which includes the
following: `
Long-term capital gains 30,000
Winnings from lotteries 20,000
Short-term capital gains covered by Sec. 111A 10,000
Agricultural income earned by her was `50,000.
Compute the tax payable by Mrs. X.
Solution: Computation of tax payable by Mrs. X for the A.Y. 2019-20 `
Tax on lottery income of `20,000 @ 30% 6,000
Tax on long-term capital gain of `30,000 @ 20% 6,000
Tax on short-term capital gain covered u/s 111A @ 15% of `10,000 1,500
Normal income `3,20,000
Tax on (`3,20,000 + `50,000) at slab rate 6,000
Tax on (`2,50,000 + `50,000) at slab rate (2,500)
Tax on normal income (12,000 – 5,000) 3,500
Tax before health & education cess 17,000
Add: HEC @ 4% 680
Tax payable by Mrs. X 17,680
Taxability of Gift 92

TAXABILITY OF GIFT
SECTION 56
TAXABILITY OF GIFT
SECTION 56(2)(x)
Question 1: Explain taxability of gift.
Answer:
Gift received by any person shall be taxable and the gifts shall be divided into 3 parts.
1. Gift of sum of money
2. Gift of any property other than immovable property
3. Gift of immovable property
Taxability is as given below:
1. Gift of sum of money
If any person has received any sum of money from one or more persons without consideration and the
aggregate value of all such gifts received during the year exceeds fifty thousand rupees, the whole of the
aggregate value of such sum shall be taxable under the head Other Sources but if the aggregate value is upto
`50,000, entire amount shall be exempt from income tax. E.g. Mr. X has received 3 gifts of `15,000 each
from his 3 friends, entire amount of `45,000 is exempt from income tax but if he has received 3 gifts of
`20,000 each, entire amount of `60,000 shall be taxable. Further it will be considered to be normal income.

2. Gift of any property other than immovable property


If any person has received gift of any property other than immovable property without consideration and the
aggregate fair market value of such properties received during a particular year exceeds `50,000, it will be
taxable under the head Other Sources but if aggregate value of all such properties is upto `50,000, it will be
exempt from income tax.

If the consideration is less than the aggregate fair market value of such properties by an amount exceeding
`50,000, aggregate fair market value as exceeds such consideration shall be taxable under the head Other
Sources. Further it will be considered to be normal income.

3. Gift of immovable property


If any person has received any immovable property without consideration, it will be exempt if stamp duty
value is upto `50,000 but if the stamp duty value exceeds fifty thousand rupees, entire stamp duty value
shall be taxable under the head Other Sources. Value of individual immovable property shall be taken into
consideration instead of aggregate value of all such properties.
(If any person is selling immovable property, its Conveyance Deed shall be prepared in the office of
Registrar and some tax has to be paid to the State Government for transferring the property and it is called
stamp duty and the value on which such duty is charged is called stamp duty value (also called circle rate).
A person may not disclose the right value hence the value is determined by State Government.)
If immovable property has been received for a consideration which is less than the stamp duty value of the
property by an amount exceeding fifty thousand rupees and also amount is exceeding 5% of the
consideration, in such cases taxable amount shall be the stamp duty value of such property as exceeds such
consideration.
Example
(i) Mr. X purchased immovable property for `3,00,000 but stamp duty value is `5,00,000, taxable amount
shall be `2,00,000
Taxability of Gift 93

(ii) Mr. X has sold immovable property to Mr. Y for `100,00,000 but stamp duty value is `105,00,000, in
this taxable amount shall be Nil because stamp duty value is not exceeding the actual consideration
maximum by 5% but if stamp duty value is ` 106,00,000, taxable amount shall be ` 6,00,000 because stamp
duty is exceeding by more than 5% of actual consideration.

If the date of the agreement fixing the amount of consideration for the transfer of immovable property and
the date of registration are not the same and in such cases, the stamp duty value on the date of the agreement
shall be taken into consideration but part of consideration should have been paid by account payee cheque,
an account payee bank draft or by use of electronic clearing system through a bank account on or before
the date of agreement. E.g. Mr. X has entered into agreement with a builder ABC Limited on 01.07.2015 for
purchase of one building for `20,00,000 but stamp duty value was `27,00,000 and advance of `3,00,000 was
given by cheque but property was transferred in his name on 01.07.2018 and on that date stamp duty value
was `35,00,000, in this case amount of gift shall be `7,00,000 (27,00,000 – 20,00,000). (Difference amount
is more than `50,000 and more than 5% of the consideration). Similarly, it will also be considered to be
normal income.

The gift is exempt in the following cases


(a) If any individual has received any gift from any of his relative, it will be exempt from income tax. The
term relative shall include
(a) spouse of the individual;
(b) brother or sister of the individual;
(c) brother or sister of the spouse of the individual;
(d) brother or sister of either of the parents of the individual;
(e) any lineal ascendant or descendant of the individual; (ascendant means mother/ father/ grand mother /
grand father and so on: Descendant means son / daughter / grand son / grand daughter etc.
(f) any lineal ascendant or descendant of the spouse of the individual;
(g) spouse of the person referred to in items (b) to (f)
(b) If any individual has received any gift from any person of any amount on the occasion of his/her
marriage. If gift is received by the parents of such individual, in that case it will be taxable. If any individual
has received gift on the occasion of anniversary, it will be taxable.
(c) If any person has received any gift under a will/ inheritance, it will be exempt from income tax.
(d) in contemplation of death of the payer or donor (Contemplation of Death means the apprehension of an
individual that his life will end in the immediate future by a particular illness etc.)
(e) from any local authority or charitable hospital or charitable educational institution or charitable trust or
other similar organisation.

Question 2: Explain meaning of property.


Answer:
"PROPERTY" means the following capital asset of the assessee, namely:—
(i) immovable property being land or building or both;
(ii) shares and securities;
(iii) jewellery;
(iv) archaeological collections (relating to past/ ancient)
(v) drawings (a picture or diagram made with a pencil, pen, or crayon without paint.)
(vi) paintings;
(vii) sculptures;
(viii) any work of art; or
(ix) bullion (Gold and silver in the form of biscuits / bricks / bars)

If any person has received gift of any other property, it will not be taxable e.g. motor car or plant and
machinery or a watch or a mobile phone etc.
E.g. Mr. X received a mobile phone valued `70,000 from his friend, in this case, it will be exempt from
income tax.
Taxability of Gift 94

Question 3: Write a note on Taxability of gift received by HUF from its members.
Answer:
If any Hindu undivided family has received any gift from any of its members, it will be exempt from
income tax. E.g. One HUF has received cash gift of `10,00,000 from one of its members, it will be exempt
from income tax.
If HUF has given gift to its member, it will be taxable.

Question 4: Write a note on Taxability of stock-in-trade.


Answer:
If any person has received any asset as stock-in-trade, it will not be taxable as gift e.g. Mr. X is a dealer in
gold and he has purchased gold for `20 lakhs but market value is ` 27 lakhs, in this case it will not be
taxable as gift (because cost will be shown in the books as `20 lakhs and entire profit on sale shall be
taxable under the head business/profession.)
Illustration 1: Mr. X, a dealer in shares, received the following without consideration during the P.Y.2018-
19 from his friend Mr. Y, -
(1) Cash gift of ` 75,000 on his anniversary, 15th April, 2018.
(2) Bullion, the fair market value of which was ` 60,000, on his birthday, 19th June, 2018.
(3) A plot of land at Faridabad on 1st July, 2018, the stamp value of which is ` 5 lakh on that date. Mr. Y had
purchased the land in April, 2013.
Mr. X purchased from his friend Z, who is also a dealer in shares, 1000 shares of X Ltd. @ ` 400 each on
19th June, 2018, the fair market value of which was ` 600 each on that date.
(4) Mr. X sold these shares in the course of his business on 23rd June, 2018.
(5) On 1st November, 2018, Mr. X took possession of property (building) booked by him two years back at `
20 lakh. The stamp duty value of the property as on 1st November, 2018 was ` 32 lakh and on the date of
booking was ` 23 lakh. He had paid ` 1 lakh by cheque as down payment on the date of booking.
Compute the income of Mr. X chargeable under the head “Income from other sources” for A.Y.2019-20.
Solution:
Particulars `
(1) Cash gift is taxable 75,000
(2) Since bullion is included in the definition of property, it is taxable. 60,000
(3) Stamp duty value of plot of land at Faridabad is taxable. 5,00,000
(4) Difference of ` 2 lakh in the value of shares of X Ltd. purchased from Mr. Z, a dealer in -
shares, is not taxable as it represents the stock-in-trade of Mr. X. Since Mr. X is a dealer in
shares and it has been mentioned that the shares were subsequently sold in the course of
his business, such shares represent the stock-in-trade of Mr. X.
(5) Difference between the stamp duty value of `23 lakh on the date of booking and the actual 3,00,000
consideration of `20 lakh paid is taxable. (Difference amount is more than `50,000 and
more than 5% of the consideration)
Income from Other Sources 9,35,000

Illustration 2: Discuss the taxability or otherwise of the following in the hands of the recipient under
section 56(2)(x) the Income-tax Act, 1961 -
(i) X HUF received ` 75,000 in cash from niece of Mr. X (i.e., daughter of Mr. X’s sister). Mr. X is the
Karta of the HUF.
(ii) Miss. X, a member of her father’s HUF, transferred a house property to the HUF without consideration.
The stamp duty value of the house property is ` 9,00,000.
(iii) Mr. X received 100 shares of A Ltd. from his friend as a gift on occasion of his 25th marriage
anniversary. The fair market value on that date was ` 100 per share. He also received jewellery worth
`45,000 (FMV) from his nephew on the same day.
(iv) X HUF gifted a car to son of Karta for achieving good marks in XII board examination. The fair market
value of the car is ` 5,25,000.
Taxability of Gift 95

Solution:
Taxable/ Amount Reason
Non-taxable liable to
tax (`)
(i) Taxable 75,000 Sum of money exceeding `50,000 received without consideration from a
non-relative is taxable under section 56(2)(x). Daughter of Mr. X’s sister is
not a relative of X HUF, since she is not a member of X HUF.
(ii) Non-taxable Nil Immovable property received without consideration by a HUF from its
relative is not taxable under section 56(2)(x). Since Miss. X is a member of
the HUF, she is a relative of the HUF.
(iii) Taxable 55,000 As per provisions of section 56(2)(x), in case the aggregate fair market
value of property, other than immovable property, received without
consideration exceeds `50,000, the whole of the aggregate value shall be
taxable. In this case, the aggregate fair market value of shares (`10,000)
and jewellery (`45,000) exceeds `50,000. Hence, the entire amount of
`55,000 shall be taxable.
(iv) Non-taxable Nil Car is not included in the definition of property for the purpose of section
56(2)(x), therefore, the same shall not be taxable.
Illustration 3: Discuss taxability in the following cases:
(i) Mr. X has received three gifts from his three friends
(a) `55,000 in cash
(b) Land with market value `5,00,000 but the value for the purpose of charging stamp duty `4,00,000.
(c) Jewellery with market value `3,00,000
In this case, taxable amount shall be 55,000 + 4,00,000 + 3,00,000 = 7,55,000
(ii) Mr. X has received gift of `50,000 in cash from his friend, in this case it will not be considered to be his
income.
(iii) Mr. X has received gift of `1,50,000 in cash from his brother, in this case it will not be considered to be
his income.
(iv) Mr. X has received gift of `1,50,000 in cash from his mother’s sister, in this case it will not be
considered to be his income.
(v) Mr. X has received gift of `1,50,000 in cash from his father’s brother, in this case it will not be
considered to be his income.
(vi) Mr. X has received gift of `1,50,000 in cash from his cousin, in this case it will be chargeable to tax.
(vii) Mr. X has received gift of `1,50,000 in cash from brother of his spouse, in this case it will not be
considered to be his income.
(viii) Mr. X has received gift of `1,50,000 in cash from his grand father, in this case it will not be considered
to be his income.
(ix) Mr. X has received gift of `1,50,000 in cash from spouse of his brother, in this case it will not be
considered to be his income.
(x) Mr. X has received gift of `1,50,000 in cash from husband of his sister, in this case it will not be
considered to be his income.
(xi) Mr. X has received gift of `1,50,000 in cash from sister of his brother’s wife, in this case it will be
considered to be his income.
(xii) Mr. X has received gift of `1,50,000 in cash from the sister of his spouse, in this case it will not be
considered to be his income.
(xiii) Mr. X has received gift of `5,000 in cash on his birthday from each of his eleven friends, in this case it
will be considered to be his income because the total amount is exceeding `50,000.
(xiv) Mr. X has received gift of property valued `1,50,000 from his friend, in this case it will be considered
to be his income.
(xv) Mr. X has received gift of `1,50,000 in cash from his friend on the occasion of his marriage, in this case
it will not be considered to be his income.
(xvi) Mr. X has received gift of `75,000 in cash and property `75,000 from his fiancee, in this case gift in
cash will be considered to be his income and the gift as property shall also be considered to be his income.
Taxability of Gift 96

Question 5: Explain taxability of gift received from employer.


Answer: Gifts to the Employees Section 17(2)(viii) Rule 3(7)(iv)
Gift given by the employer in kind upto `5,000 in aggregate during a particular year is exempt and excess
over it is taxable. If the employer has given any voucher or token in lieu of which such gift may be received,
it will also be exempt in the similar manner.

Gifts in cash or gifts convertible into cash i.e. gift cheques etc. shall be fully chargeable to tax.
E.g. Mr. X is employed in ABC Ltd. and he has received a cash gift of `11,000 from his employer, in this
case taxable amount shall be `11,000 and it will be income under the head Salary and shall be taxable at the
normal rate but if Mr. X has received one wrist watch of `11,000 from his employer, taxable amount shall
be `6,000.

Question 6: Explain Taxability of gift received in connection with business/profession.


Answer: Gifts or Perquisites from Clients Section 28
The value of any benefit or perquisite, whether convertible into money or not, arising from business or the
exercise of a profession.
If any person has received any gift or perquisite or benefit either in cash or in kind from any of his clients, it
will be considered to be business receipt and shall be taken into consideration while computing income
under the head business/profession.
Example: A Doctor has received a gift of ` 40,000 from one of his clients, in this case it will be considered
to be income under the head business/profession.

Question 7: Explain taxability of scholarship/ award / reward.


Answer: Scholarship Section 10(16)
Any scholarship received by a person for meeting the cost of education shall be exempt from income tax.
Award/ Reward Section 10(17A)
Any award or reward whether in cash or in kind instituted by the Central Government or the State
Government shall be exempt from income tax. Similarly any private award or reward shall be exempt from
income tax if approved by the Central Government.

Question 8: Explain taxability of compensation or other payment in connection with employment.


Answer: As per 56(2)(xi), any compensation or other payment, due to or received by any person,by
whatever name called, in connection with the termination of his employment or the modification of the
terms and conditions relating thereto shall be taxable (if not covered under the head salary).
Taxability of Gift 97

PRACTICE PROBLEMS
TOTAL PROBLEMS 5
Problem 1.
Discuss taxability in the following cases:
(i) Mr. X has received gift of ` 50,000 in cash from his friend.
(ii) Mr. X has received gift of ` 2,50,000 in cash from his brother.
(iii) Mr. X has received gift of ` 2,50,000 in cash from his mother’s sister.
(iv) Mr. X has received gift of ` 2,50,000 in cash from his father’s brother.
(v) Mr. X has received gift of ` 2,50,000 in cash from his cousin.
(vi) Mr. X has received gift of ` 2,50,000 in cash from brother of his spouse.
(vii) Mr. X has received gift of ` 2,50,000 in cash from his grand father.
(viii) Mr. X has received gift of ` 2,50,000 in cash from spouse of his brother.
(ix) Mr. X has received gift of ` 2,50,000 in cash from husband of his sister.
(x) Mr. X has received gift of ` 2,50,000 in cash from sister of his brother’s wife.
(xi) Mr. X has received gift of ` 2,50,000 in cash from the sister of his spouse.
(xii) Mr. X has received gift of `6,000 in cash on his birthday from each of his eleven friends.
(xiii) Mr. X has received gift of ` 2,50,000 as property from his friend.
(xiv) Mr. X has received gift of `2,50,000 in cash from his friend on the occasion of his marriage.
(xv) Mr. X has received gift of `1,00,000 in cash and `1,00,000 as property from his fiancée.

Problem 2.
Mr. X submits the particulars for the previous year 2018-19 as given below:
1. He has received a gift of `27,000 from one of his friend on 01.09.2018.
2. He has received a gift of `11,000 on 01.10.2018 from his wife Mrs. X.
3. He has received a gift of `29,000 from his step daughter on 01.01.2019.
4. He has received a gift of `27,000 from grand mother of Mrs. X on 07.01.2019.
5. He has received a gift of `20,000 in kind from his employer on 01.03.2019.
6. He has received gold as gift from his friend on 01.12.2018 with value `2,00,000.
7. He has received `27,000 as gift from his maternal aunt (mother’s sister) on 10.12.2018.
8. He has received dividend of `2,00,000 from a domestic company on 31.03.2019.
9. He has received two gifts of `30,000 each from his neighbours on 01.06.2018.
Compute his tax liability for assessment year 2019-20.
Answer = Tax Liability: `100

Problem 3.
Mr. X received gift in cash `3,00,000 from son of his father’s brother and gift of `1,00,000 in cash from
brother of father of Mrs. X. He has agricultural income `5,00,000.
Compute his tax liability for Assessment Year 2019-20.
Answer = Tax Liability: `31,200

(b) He is aged 81 years.


Answer = Tax Liability: Nil

(c) He is non-resident and he has completed age of 80 years as on 31.03.2019.


Answer = Tax Liability: `31,200

Problem 4.
Mr. X received jewellery valued `5,00,000 from brother of his grand father and his agricultural income is
`1,00,000.
Compute his income and tax liability for Assessment Year 2019-20.
Answer = Total Income: `5,00,000; Tax Liability: `28,600
Taxability of Gift 98

Problem 5.
Following gifts are received by Mrs. X, who is carrying on jewellery business, during the previous year
2018-19:
(i) On the occasion of her marriage on 07.09.2018, she has received `1,20,000 as gift out of which `85,000
are from relatives and balance from friends.
(ii) On 03.10.2018, she has received cash gift of `2,50,000 from cousin of her mother.
(iii) A mobile phone worth `15,000 is gifted by her friend on 21.09.2018.
(iv) She gets a cash gift of `2,40,000 from the elder brother of her husband's grandfather on 10.12.2018.
(v) She has received a cash gift of `6,00,000 from her friend on 27.01.2019.
(vi) She has received bullion, the fair market value of which was `4,75,000 on her birthday,19.01.2019.
Mrs. X purchased from her friend, who is also carrying jewellery business, jewellery at ` 2,50,000 on
25.01.2019, the fair market value of which was `5,00,000 on that date.
Compute total income and tax liability of Mrs. X for A.Y.2019-20.
Answer = Total Income: `15,65,000; Tax Liability: `2,93,280
Taxability of Gift 99

SOLUTIONS
TO
PRACTICE PROBLEMS
Solution 1:
(i) Mr. X has received gift of `50,000 in cash from his friend, in this case it will not be considered to be his
income.
(ii) Mr. X has received gift of `2,50,000 in cash from his brother, in this case it will not be considered to be
his income.
(iii) Mr. X has received gift of ` 2,50,000 in cash from his mother’s sister, in this case it will not be
considered to be his income.
(iv) Mr. X has received gift of `2,50,000 in cash from his father’s brother, in this case it will not be
considered to be his income.
(v) Mr. X has received gift of `2,50,000 in cash from his cousin, in this case it will be chargeable to tax.
(vi) Mr. X has received gift of `2,50,000 in cash from brother of his spouse, in this case it will not be
considered to be his income.
(vii) Mr. X has received gift of `2,50,000 in cash from his grand father, in this case it will not be considered
to be his income.
(viii) Mr. X has received gift of `2,50,000 in cash from spouse of his brother, in this case it will not be
considered to be his income.
(ix) Mr. X has received gift of `2,50,000 in cash from husband of his sister, in this case it will not be
considered to be his income.
(x) Mr. X has received gift of `2,50,000 in cash from sister of his brother’s wife, in this case it will be
considered to be his income.
(xi) Mr. X has received gift of `2,50,000 in cash from the sister of his spouse, in this case it will not be
considered to be his income.
(xii) Mr. X has received gift of `6,000 in cash on his birthday from each of his eleven friends, in this case it
will be considered to be his income because the total amount is exceeding `50,000.
(xiii) Mr. X has received gift of `2,50,000 in kind from his friend, in this case it will be considered to be his
income.
(xiv) Mr. X has received gift of `2,50,000 in cash from his friend on the occasion of his marriage, in this
case it will not be considered to be his income.
(xv) Mr. X has received gift of `1,00,000 in cash and `1,00,000 as property from his fiancee, in this case
gift in cash will be considered to be his income and the gift in kind shall also be considered to be his income.

Solution 2: `
Computation of income under the head Salary
Gift in kind from his employer (20,000 – 5,000) 15,000.00
Income under the head Salary 15,000.00
Computation of income under the head Other Sources
Gift received from friend 27,000.00
Gifts received from neighbours 60,000.00
Gift received from friend in kind 2,00,000.00
Income under the head Other Sources 2,87,000.00
Gross Total Income 3,02,000.00
Less: Deduction u/s 80C to 80U Nil
Total Income 3,02,000.00
Taxability of Gift 100

Computation of Tax Liability


Tax on `3,02,000 at slab rate 2,600.00
Less: Rebate u/s 87A (2,500.00)
Tax before health & education cess 100.00
Add: HEC @ 4% 4.00
Tax Liability 104.00
Rounded off u/s 288B 100.00
Note: Dividend received by Mr. X from domestic company is exempt u/s 10(34).

Solution 3: `
Computation of income under the head Other Sources
Gift received from son of his father’s brother 3,00,000
Gift received from bother of father’s of Mrs. X 1,00,000
Income under the head Other Sources 4,00,000
Gross Total Income 4,00,000
Less: Deduction u/s 80C to 80U Nil
Total Income 4,00,000
Agricultural Income 5,00,000
Computation of Tax Liability
Step 1. Tax on (4,00,000 + 5,00,000) at slab rates 92,500
Step 2. Tax on (`2,50,000 + 5,00,000) at slab rates (62,500)
Step 3. Deduct Tax at Step 2 from Tax at Step 1 30,000
Tax before health & education cess 30,000
Add: HEC @ 4% 1,200
Tax Liability 31,200

Solution 3(b):
Total Income 4,00,000
Agricultural Income 5,00,000
Tax Liability Nil
Note: If non-agricultural income is upto the limit not chargeable to tax (`2,50,000/ 3,00,000/5,00,000),
partial integration is not applicable.

Solution 3(c):
Total Income 4,00,000
Agricultural Income 5,00,000
Computation of Tax Liability
Step 1. Tax on (4,00,000 + 5,00,000) at slab rates 92,500
Step 2. Tax on (`2,50,000 + 5,00,000) at slab rates (62,500)
Step 3. Deduct Tax at Step 2 from Tax at Step 1 30,000
Add: HEC @ 4% 1,200
Tax Liability 31,200
Solution 4: `
Computation of income under the head Other Sources
Gift in kind from brother of his grand father 5,00,000
Income under the head Other Sources 5,00,000
Gross Total Income 5,00,000
Less: Deduction u/s 80C to 80U Nil
Total Income 5,00,000
Agricultural Income 1,00,000
Computation of Tax Liability
Step 1. Tax on (5,00,000 + 1,00,000) at slab rates 32,500
Step 2. Tax on (`2,50,000 + 1,00,000) at slab rates (5,000)
Taxability of Gift 101

Step 3. Deduct Tax at Step 2 from Tax at Step 1 27,500


Tax before health & education cess 27,500
Add: HEC @ 4% 1,100
Tax Liability 28,600

Solution 5:
Computation of Total Income of Mrs. X for the A.Y. 2019-20 `
Gift received on the occasion of marriage are exempt --
Cash gift received from cousin of Mrs. X’s mother is taxable under section 56(2)(x) 2,50,000
(Cousin of Mrs. X’s mother is not a relative)
Mobile phone gifted by her friend is not taxable since it is not included in the definition
of “property” under section 56(2)(x) --
Cash gift received from elder brother of husband’s grandfather is taxable 2,40,000
(Brother of husband’s grandfather is not a relative)
Cash gift from friend is taxable 6,00,000
Since bullion is included in the definition of property, therefore, when bullion is
received without consideration, the same is taxable, since the aggregate fair
market value exceeds `50,000 4,75,000
Difference of `2.5 lakh in the value of jewellery purchased from her friend, is not
taxable as it represents the stock-in-trade of Mrs. X. Since Mrs. X is carrying
jewellery business and it has been mentioned that the jewellery were subsequently
sold in the course of her business, such jewellery represent the stock-in-trade
of Mrs. X. Nil
Income under the head Other Sources 15,65,000
Gross Total Income 15,65,000
Less: Deduction u/s 80C to 80U Nil
Total Income 15,65,000
Computation of Tax Liability
Tax on ` 15,65,000 at slab rate 2,82,000
Add: HEC @ 4% 11,280
Tax Liability 2,93,280
Taxability of Gift 102

EXAMINATION QUESTIONS
MAY – 2016
Question 4(a) (2 x 2 = 4 Marks)
Discuss the taxability or otherwise in the hands of the recipients, as per the provisions of the Income-tax
Act, 1961:
(i) Mr. N, a member of his father's HUF, transferred a house property to the HUF without
consideration. The value of the house is `10 lacs as per the Registrar of stamp duty.
(ii) Mr. Kumar gifted a car to his sister's son (Sunil) for achieving good marks in CA Final exam. The
fair market value of the car is `5,00,000.
Answer:
(i) Non-Taxable: As per sec 56(2)(x), if HUF has received any Gift from its member, it will be exempt from
Income tax. In the given case, HUF has received a Gift of house property from its member Mr. N hence it
will be exempt from income tax and what is the value of house property shall not matter.
(ii) Non-Taxable: If any person has received a gift from brother of mother, it will be covered in the
definition of relative and shall be exempt from income tax further if a gift is taxable it should be covered in
the definition of property as given u/s 56(2)(x). In the given case gift is from relative and further gift is of
motor car which is not covered in the definition of property hence it will be exempt from Income Tax.
MAY – 2012
Question 1 (1 Marks)
State whether the following are chargeable to tax and the amount liable to tax.
A sum of `1,20,000 was received as gift from non-relatives by Mr. X on the occasion of the marriage of his
son Mr. Y.
Answer: As per section 56(2)(x), if any gift has been received on the occasion of marriage, it will be exempt
from income tax but if gift has been received by the parents of the person getting married, such gift shall be
taxable hence in this case gift received by Mr. X is taxable because marriage is that of his son Mr. Y.
MAY – 2011
Question 7 (4 Marks)
The following details have been furnished by Mrs. X, pertaining to the year ended 31.03.2019:
(i) Cash gift of `51,000 received from her friend on the occasion of her “Shastiaptha Poorthi”, a wedding
function celebrated on her husband completing 60 years of age. This was also her 25th wedding anniversary.
(ii) On the above occasion, a diamond necklace worth `2 lacs was presented by her sister living in Dubai.
(iii) When she celebrated her daughter’s wedding on 21.02.2019, her friend assigned in Mrs. X’s favour, a
fixed deposit held by the said friend in a scheduled bank; the value of the fixed deposit and the accrued
interest on the said date was `51,000.
(iv) She has short term capital gains under section 111A `10 lakhs.
Compute total income and tax liability for assessment year 2019-20.
Answer: (i) Any sum of money received by an individual on the occasion of the marriage of the individual
is exempt. This provision is, however, not applicable to a cash gift received during a wedding function
celebrated on completion of 60 years of age.
The gift of `51,000 received from a non-relative is, therefore, chargeable to tax under section 56(2)(x) in the
hands of Mrs. X.
(ii) The provisions of section 56(2)(x) are not attracted in respect of any sum of money or property received
from a relative. Thus, the gift of diamond necklace received from her sister is not taxable under section
56(2)(x), even though jewellery falls within the definition of “property”.
(iii) To be exempt from applicability of section 56(2)(x), the property should be received on the occasion of
the marriage of the individual, not that of the individual’s son or daughter. Therefore, this exemption
provision is not attracted in this case.
Taxability of Gift 103

Any sum of money received without consideration by an individual is chargeable to tax under section
56(2)(x), if the aggregate value exceeds `50,000 in a year. “Sum of money” has, however, not been defined
under section 56(2)(x).
Therefore, there are two possible views in respect of the value of fixed deposit assigned in favour of Mrs.
X–
(1)The first view is that fixed deposit does not fall within the meaning of “sum of money” and therefore, the
provisions of section 56(2)(x) are not attracted. Fixed deposit is also not included in the definition of
“property”.
(2) However, another possible view is that fixed deposit assigned in favour of Mrs. X falls within the
meaning of “sum of money” received.
Income assessable as “Income from other sources”
If the first view is taken, the total amount chargeable to tax as “Income from other sources” would be
`51,000, being cash gift received from a friend on her Shastiaptha Poorthi.
As per the second view, the provisions of section 56(2)(x) would be attracted in respect of the fixed deposit
assigned and the “Income from other sources” of Mrs. X would be `1,02,000 (`51,000 + `51,000).
Tax liability as per first view
Income under the head Other Sources 51,000.00
Income under the head Capital Gains (STCG u/s 111A) 10,00,000.00
Gross Total Income 10,51,000.00
Less: Deduction u/s 80C to 80U Nil
Total Income 10,51,000.00
Computation of Tax Liability
Tax on `51,000 at slab rate Nil
Tax on `8,01,000 (`10,00,000 – `1,99,000) @ 15% 1,20,150.00
Add: HEC @ 4% 4,806.00
Tax Liability 1,24,956.00
Rounded off u/s 288B 1,24,960.00
Tax liability as per second view
Income under the head Other Sources 1,02,000.00
Income under the head Capital Gains (STCG u/s 111A) 10,00,000.00
Gross Total Income 11,02,000.00
Less: Deduction u/s 80C to 80U Nil
Total Income 11,02,000.00
Computation of Tax Liability
Tax on `1,02,000 at slab rate Nil
Tax on `8,52,000 (`10,00,000 – `1,48,000) @ 15% 1,27,800.00
Add: HEC @ 4% 5,112.00
Tax Liability 1,32,912.00
Rounded off u/s 288B 1,32,910.00

NOV – 2008
Question 2 (4 Marks)
Mrs. X has received the following gifts during previous year 2018-19.
(i) On the occasion of her marriage on 14.08.2018, she has received `90,000 as gift out of which `70,000 are
from relatives and balance from friends.
(ii) On 12.09.2018, she has received gift of `18,000 from cousin of her mother.
(iii) A cell phone of `21,000 is gifted by her employer on 15.08.2018.
(iv) She gets a gift of `25,000 from the elder brother of her husband's grandfather on 25.10.2018.
(v) She has received a gift of `2,000 from her friend on 14.04.2018.
(vi) She has won `4 lakh from a game show on electronic media.
Compute her tax liability for assessment year 2019-20.
Taxability of Gift 104

Answer:
Computation of taxable income of Mrs. X from gifts for A.Y. 2019-20
Particulars Taxable amount Reason for taxability or
` otherwise of each gift
 Relatives and friends Nil Gifts received on the occasion of
marriage are not taxable.
 Cousin of Mrs. X’s mother 18,000 Cousin of Mrs. X’s mother is
not a relative. Hence, the gift is taxable.
 Elder brother of husband’s grandfather 25,000 Brother of husband’s grandfather is
not a relative. Hence, the gift is taxable.
 Friend 2,000 Gift from friend is taxable.
Aggregate value of gifts 45,000
Since the aggregate value of gifts received by Mrs. X during the previous year 2018-19 does not exceed
`50,000, the same is not chargeable to tax under section 56(2)(x) of the Income-Tax Act, 1961.
Gift received from the employer in kind upto `5,000 is exempt from income tax but excess over it is taxable
hence in this case taxable amount of gift shall be `16,000 (21,000 – 5,000) and it will be taxable under the
head Salary.
Income under the head Salary 16,000
Income under the head Other Sources 4,00,000
Gross Total Income 4,16,000
Less: Deduction u/s 80C to 80U Nil
Total Income 4,16,000
Computation of Tax Liability
Tax on `16,000 at slab rate Nil
Tax on `4,00,000 @ 30% 1,20,000
Add: HEC @ 4% 4,800
Tax Liability 1,24,800

MAY – 2008
Question 3 (1 Marks)
Choose the correct answer with reference to the provisions of the Income-tax Act, 1961:
Mr. X received `70,000 from his friend on the occasion of his birthday.
(a) The entire amount of `70,000 is taxable
(b) `25,000 is taxable
(c) The entire amount is exempt
(d) None of the above.
Answer:
(a) The entire amount of `70,000 is taxable.
MAY – 2005
Question 1 (1 Marks)
th
Gift of `5,00,000 received on 10 July, 2018 through account payee cheque from a non-relative regularly
assessed to income-tax, is
(a) A capital receipt not chargeable to tax
(b) Chargeable to tax as income from other sources
(c) Chargeable to tax as business income
(d) Exempt upto `50,000 and balance chargeable to tax as income from other sources.
Answer:
(b) Chargeable to tax as income from other sources
Advance Payment Of Tax 105

ADVANCE PAYMENT OF TAX


OR

PAY AS YOU EARN SCHEME


SECTION 207 TO 219
SECTIONS PARTICULARS
207 Liability for payment of advance tax
208 Conditions of liability to pay advance tax
211 Instalments of advance tax and due dates
234A Computation of interest in case of payment of tax after last date of filing of ROI
234B Computation of interest in case of payment of tax after end of previous year
234C Computation of interest in case of default in payment of advance tax
Rule 119A Rounded off of the amount on which interest is to be computed
Advance Payment Of Tax 106

Question 1: Write a note on advance payment of income tax.


Answer: As per section 207, every person shall pay tax in advance as per the provisions of advance tax i.e.
in general every person should estimate his income and pay tax however exact amount of income tax shall
be calculated at the end of the year.
A senior citizen who do not have income under the head business/profession shall be exempt from payment
of advance tax.
As per section 208, advance tax shall be payable during a financial year in every case where the amount of
such tax payable by the assessee during that year, is ten thousand rupees or more.
An assessee has to estimate his current income and pay advance tax thereon. He need not submit any
estimate or statement of income to the Assessing Officer.

As per section 211, all assessee have to pay advance tax in the manner given below:
Due date of installment Amount payable
Upto 15th June of P.Y. 15% of tax payable
Upto 15th September of P.Y. 45% of tax payable
Upto 15th December of P.Y. 75% of tax payable
Upto 15th March of P.Y. 100% of tax payable
Example
For the previous year 2018-19, ABC Ltd. has estimated its tax payable to be `2,00,000, in this case advance
tax shall be paid by the company as given below:
Upto 15.06.2018 30,000
Upto 15.09.2018 90,000
Upto 15.12.2018 1,50,000
Upto 15.03.2019 2,00,000
If the last day for payment of any installment of advance tax is a day on which the receiving bank is closed,
the assessee can make the payment on the next immediately following working day.

Question 2: Write a note on payment of interest for late payment of income tax
Answer: As per section 234C, if any person has defaulted in payment of advance tax, interest shall be
charged @ 1% per month for a period of 3 months on the amount of default in each installment, but for the
last installment, interest shall be charged only for one month.
Income tax paid upto 31st March of previous year is also called advance tax.
As per section 234B, if advance tax paid is less than 90% of actual tax liability, assessee shall be required to
pay interest @ 1% per month or part of a month from 1st April of assessment year upto the date of payment.
If advance tax paid is 90% or more of actual tax liability, no interest is payable.

As per section 234A, if any person has paid income tax after expiry of the last date of filing of return of
income, interest shall be payable @ 1% p.m. or part of the month for the period subsequent to the last date
of filing of return of income.

Illustration 1: ABC Ltd. has estimated its tax liability for assessment year 2019-20 `4,40,000 and has paid
advance tax accordingly but actual tax liability was found to be `10,00,000.
The company has paid balance amount on 02.01.2020 and filed return of income on the same date.
Compute interest payable under section 234A, 234B, and 234C.
Solution:
Estimated Tax = 4,40,000 Actual Tax = 10,00,000
Interest under section 234C shall be computed in the manner given below:
Tax Payable Tax Paid Default
15.06.2018 1,50,000 66,000 84,000
Interest u/s 234C = 84000 x 1% x 3 = 2,520
15.09.2018 4,50,000 1,98,000 2,52,000
Interest u/s 234C = 2,52,000 x 1% x 3 = 7,560
15.12.2018 7,50,000 3,30,000 4,20,000
Advance Payment Of Tax 107

Interest u/s 234C = 4,20,000 x 1% x 3 = 12,600


15.03.2019 10,00,000 4,40,000 5,60,000
Interest u/s 234C = 5,60,000 x 1% x 1 = 5,600

Total interest payable u/s 234C 28,280

Interest under section 234B shall be computed from 01.04.2019 to 02.01.2020 and is as given below:
10,00,000 – 4,40,000 = 5,60,000 x 1% x 10 = 56,000

Interest under section 234A shall be computed from 01.10.2019 to 02.01.2020 and is as given below:
5,60,000 x 1% x 4 = 22,400
Total interest payable (28,280 + 56,000 + 22,400) 1,06,680

Illustration 2: ABC Ltd. has tax liability of `7,00,000 for the previous year 2018-19 and the company has
not paid any advance tax and entire tax amount was paid by the company on 31.12.2019. In this case,
interest shall be calculated in the manner given below:
1. Interest u/s 234C
`
15.06.2018 1,05,000 x 1% x 3 = 3,150
15.09.2018 3,15,000 x 1% x 3 = 9,450
15.12.2018 5,25,000 x 1% x 3 = 15,750
15.03.2019 7,00,000 x 1% x 1 = 7,000
Total interest payable 35,350
2. Interest u/s 234B (01-04-2019 to 31-12-2019)
7,00,000 x 1% x 9 = 63,000
3. Interest u/s 234A (01-10-2019 to 31-12-2019)
7,00,000 x 1% x 3 = 21,000
Total Interest Payable 1,19,350
Question 3: Explain Special Provision for all assessee.
Answer: Special Provision for all assessee
If the advance tax paid by the assessee upto 15th June is 12% of the tax payable and upto 15th September,
is 36% of the tax payable, in such cases no interest shall be charged for default in such instalment.

Illustration 3: ABC Ltd. has estimated its tax payable to be `5,00,000 for previous year 2018-19 and has
paid advance tax accordingly but actual tax liability of the company was found to be `5,50,000 and
difference of tax amount was paid on 10.12.2019. Compute interest under section 234A, 234B and 234C.
Solution:
Interest under section 234C shall be computed in the manner given below:
Tax Payable Tax Paid Default
15.06.2018 82,500 75,000 7,500
Interest u/s 234C = Nil (because advance tax paid is at least 12%)
15.09.2018 2,47,500 2,25,000 22,500
Interest u/s 234C = Nil (because advance tax paid is at least 36%)
15.12.2018 4,12,500 3,75,000 37,500
Interest u/s 234C = 37,500 x 1% x 3 = 1,125
15.03.2019 5,50,000 5,00,000 50,000
Interest u/s 234C = 50,000 x 1% x 1 = 500
Total interest payable u/s 234C 1,625
Interest under section 234B
Advance tax paid is more than 90% of actual tax liability, no interest is payable
Interest under section 234A shall be computed from 01.10.2019 to 10.12.2019 and is as given below:
50,000 x 1% x 3 = 1,500
Total interest payable (1,625 + 1,500) 3,125
Advance Payment Of Tax 108

Question 4: Explain Payment of advance tax in case of capital gains/casual income/ newly setup
business/ profession/ Dividend Income in excess of ` 10,00,000.
Answer: Payment of advance tax in case of capital gains/casual income/newly setup business/
profession/ Dividend Income in excess of ` 10,00,000 Section 234C
In case of capital gains and casual income, no advance tax is payable on estimated basis but if there is actual
accrual of casual income or capital gains, advance tax is to be paid in the subsequent installments and if such
accrual is after 15th March, advance tax is to be paid upto 31st March of previous year otherwise interest
shall be charged under section 234C.

Illustration 4: Mr. X has paid advance tax as given below:


Upto 15.06.2018 ` 15,000
Upto 15.09.2018 ` 45,000
Upto 15.12.2018 ` 95,000
Upto 15.03.2019 `1,70,000
He had long term capital gains of `3,00,000 on 01.01.2019 and his income under the head
business/Profession is `11,00,000
He has filed return of income on 10.12.2019 and has paid difference of the tax on 10.12.2019.
Last date for filing of return is 31.07.2019.
Compute interest payable under section 234A, 234B and 234C.
Solution:
Computation of Tax Liability `
Normal Income 11,00,000
Long term capital gains 3,00,000
Total Income 14,00,000
Tax on `11,00,000 at slab rate 1,42,500
Tax on `3,00,000 @ 20% 60,000
Add: HEC @ 4% 8,100
Tax Liability 2,10,600
(Tax liability excluding capital gains `11,00,000 at slab rate + HEC @ 4% 1,48,200)
Interest u/s 234C
Since capital gains arises on 1st January 2019, installments for 15th June, 15th September and 15th December
shall be checked without including tax on capital gain and shall be as given below:
Amount payable Amount actually paid Shortfall
` ` `
Upto 15.06.2018 (1,48,200 x 15%) 22,230.00 15,000 7,230.00
Rounded off under rule 119A = 7,200
Interest u/s 234C = 7,200 x 1% x 3 = 216

Upto 15.09.2018 (1,48,200 x 45%) 66,690.00 45,000 21,690.00


Interest u/s 234C = 21,600 x 1% x 3 = 648

Upto 15.12.2018 (1,48,200 x 75%) 1,11,150.00 95,000 16,150.00


Rounded off under rule 119A = 16,100
Interest u/s 234C = 16,100 x 1% x 3 = 483

Installment for 15th March shall be including tax on capital gains and is as given below:
Upto 15.03.2019 (2,10,600 x 100%) 2,10,600 1,70,000 40,600
Interest u/s 234C = 40,600 x 1% x 1 = 406
Total Interest u/s 234C `1,756
Interest u/s 234B (01-04-2019 to 10-12-2019)
40,600 x 1% x 9 `3,654
Interest u/s 234A(01-07-2019 to 10-12-2019)
40,600 x 1% x 5 `2,030
Advance Payment Of Tax 109

Similar provision shall be applicable in case of a newly setup Business/Profession


If any Assessee has started Business/Profession in the current year, assessee shall be exempt from payment
of advance tax prior to commencement of Business/Profession i.e. advance tax has to be paid in installments
subsequent to commencement of Business/Profession.
If Business/Profession has been started after 15th March, advance tax should be paid upto 31st March
otherwise Interest shall be charged under section 234C for one month @ 1%.
Example: Mr. X started his business on 01.10.2018 and had income `10,00,000 upto 31.03.2019, In this
case, he will be required to pay advance tax in the manner given below:
`
Income under the head Business/ Profession 10,00,000
Gross Total Income/Total Income 10,00,000
Computation of Tax Liability
Tax on 10,00,000 at slab rate 1,12,500
Add: HEC @ 4% 4,500
Tax Liability 1,17,000

Advance Tax Payment


15.06.2018 Nil
15.09.2018 Nil
15.12.2018 (1,17,000 x 75%) 87,750.00
15.03.2019 (1,17,000 x 100%) 1,17,000.00

Similar provision shall be applicable in case of Dividend Income in excess of ` 10,00,000


If any Assessee has received Dividend in excess of `10,00,000 then excess amount is taxable u/s 115BBDA
@ 10% and in such case advance tax is payable on actual accrual of Dividend income i.e. advance tax is to
be paid in the subsequent installments and if such accrual is after 15th March, advance tax is to be paid upto
31st March of previous year otherwise interest shall be charged under section 234C and Advance Tax shall
not be payable on estimated basis.
If Dividend has been received after 15th March, advance tax should be paid upto 31st March otherwise
Interest shall be charged under section 234C for one month @ 1%.

Illustration 5: ABC Ltd. started his business on 01.10.2018 and had earning from business from 01.10.2018
to 31.03.2019 `20,00,000, in this case company need not pay advance tax upto 15.09.2018 but advance tax
is to be paid in subsequent installments. Company should pay advance tax on 15.12.2018 equal to
(20,00,000 x 30% + HEC) x 75% = 4,68,000 and company should pay total advance tax on 15.03.2019 equal
to (20,00,000 x 30% + HEC) x 100% = 6,24,000.

Illustration 6: A partnership firm made the following payments of advance tax during the financial year
2018-19: `
Upto June 15, 2018 4,15,000
Upto September 15, 2018 8,25,000
Upto December 15, 2018 16,64,000
Upto March 15, 2019 26,23,000
Return of income filed by the firm is `88,00,000 under the head “profits and gains of business or profession”
and `9,50,000 by way of long term capital gains on sale of a property effected on December 1, 2018. What
is the interest payable by the assessee under section 234B and section 234C for assessment year 2019-20?
Assume that the return of income was filed on 30.09.2019 i.e. the due date and tax was fully paid on self
assessment.
Solution: Computation of Tax Liability
`
Business income 88,00,000
Long term capital gains 9,50,000
Total Income 97,50,000
Advance Payment Of Tax 110

Tax on `88,00,000 @ 30% 26,40,000


Tax on `9,50,000 @ 20% 1,90,000
Add: HEC @ 4% 1,13,200
Tax Liability 29,43,200
(Tax liability excluding capital gains `88,00,000 x 30% + HEC@ 4% 27,45,600)

Interest u/s 234C


Since capital gains arises on 1st December 2018, installment for 15th June and 15th September shall be
checked without including tax on capital gain and shall be as given below:
Amount payable Amount actually paid Shortfall
` ` `
Upto 15.06 2018 (27,45,600 x 15%) 4,11,840 4,15,000 NIL
Interest u/s 234C = Nil

Upto 15.09 2018 (27,45,600 x 45%) 12,35,520 8,25,000 4,10,520


Rounded off under rule 119A = 4,10,500
Interest u/s 234C = 4,10,500 x 1% x 3 month = 12,315

Installments for 15th December and 15th March shall be including tax on capital gains and is as given below:
Upto 15.12.2018 (29,43,200 x 75%) 22,07,400 16,64,000 5,43,400
Interest u/s 234C = 5,43,400 x 1% x 3 month = 16,302

Upto 15.03.2019 (29,43,200 x 100%) 29,43,200 26,23,000 3,20,200


Interest u/s 234C = 3,20,200 x 1% x 1 month = 3,202

Total Interest u/s 234C `31,819

Interest u/s 234B (01-04-2019 to 30-09-2019)


3,20,200 x 1% x 6 `19,212

Rounding off for the purpose of calculating Interest Rule 119A


As per rule 119A, the principal amount shall be rounded off in the multiples of `100 and for this purpose
any fraction of `100 shall be ignored. E.g. `1,60,275 shall be rounded off as 1,60,200.

Illustration 7: ABC Ltd. has paid advance tax for the previous year 2018-19 as given below:
1. Upto 15.06.2018 ` 50,000
2. Upto 15.09.2018 `1,50,000
3. Upto 15.12.2018 `3,00,000
4. Upto 15.03.2019 `6,00,000
Actual tax liability was found to be `7,00,000 and balance tax was paid on 10.12.2019. Compute interest
payable under section 234A, 234B, 234C.
Solution:
Interest under section 234C shall be computed in the manner given below:
Tax Payable Tax Paid Shortfall
15.06.2018 1,05,000 50,000 55,000
Interest u/s 234C = 55,000 x 1% x 3 = 1,650
15.09.2018 3,15,000 1,50,000 1,65,000
Interest u/s 234C = 1,65,000 x 1% x 3 = 4,950
15.12.2018 5,25,000 3,00,000 2,25,000
Interest u/s 234C = 2,25,000 x 1% x 3 = 6,750
15.03.2019 7,00,000 6,00,000 1,00,000
Interest u/s 234C = 1,00,000 x 1% x 1 = 1,000
Total interest payable u/s 234C 14,350
Advance Payment Of Tax 111

Interest under section 234B shall be computed from 01.04.2019 to 10.12.2019 and is as given below:
7,00,000 – 6,00,000 = 1,00,000 x 1% x 9 = 9,000
Interest under section 234A shall be computed from 01.10.2019 to 10.12.2019 and is as given below:
1,00,000 x 1% x 3 = 3,000
Total interest payable (14,350 + 9,000 + 3,000) 26,350

(b) Presume actual tax liability is `6,50,000.


Solution:
Interest under section 234C shall be computed in the manner given below:
Tax Payable Tax Paid Shortfall
15.06.2018 97,500 50,000 47,500
Interest u/s 234C = 47,500 x 1% x 3 = 1,425
15.09.2018 2,92,500 1,50,000 1,42,500
Interest u/s 234C = 1,42,500 x 1% x 3 = 4,275
15.12.2018 4,87,500 3,00,000 1,87,500
Interest u/s 234C = 1,87,500 x 1% x 3 = 5,625
15.03.2019 6,50,000 6,00,000 50,000
Interest u/s 234C = 50,000 x 1% x 1 = 500
Total interest payable u/s 234C 11,825
Interest under section 234B
Advance tax paid is more than 90% of actual tax liability, no interest is payable
Interest under section 234A shall be computed from 01.10.2019 to 10.12.2019 and is as given below:
50,000 x 1% x 3 = 1,500
Total interest payable (11,825 + 1,500) 13,325

Due date for filing the return of income Section 139(1)


Return is to be filed in general upto 31st July of the assessment year, however, in the following cases, the last
date shall be 30th September of the assessment year.
1. Every company assessee
Example
For the previous year 2018-19, ABC Ltd. has to file its return of income upto 30.09.2019.
2. Any other person who is required to get his accounts audited either under Income Tax Act or under any
other Act.
Example
Mr. X has his own business and his turnover for previous year 2018-19 is `102 lakhs. In this case, the last
date of filing the return of income shall be 30.09.2019, but if turnover is `97 lakhs, the last date shall be
31.07.2019.
Similarly if a partnership firm XY has turnover of its business ` 65 lakhs for previous year 2018-19, in this
case, the last date of filing of return of income shall be 31.07.2019.

Audit u/s 44AB


As per Section 44AB, Audit is compulsory for every person carrying on business and sales turnover or
gross receipts exceeds ` 1 crore during the previous year and every person carrying on profession and gross
receipts is exceeding `50 lakh during the previous year.

If tax liability is less than ` 10,000 then interest u/s 234B & 234C shall not be payable but Interest u/s 234A
shall be payable.
Eg. Mr. X has Income under the head house property is ` 4 Lakh and he paid income tax on 10/12/2019.
Compute Interest u/s 234A,234B & 234C. In this case, His Tax Liability shall be :
`
Income under the head House Property 4,00,000
Gross Total Income/Total Income 4,00,000
Advance Payment Of Tax 112

Computation of Tax Liability


Tax on `4,00,000 at slab rate 7,500
Add: HEC @ 4% 300
Tax Liability 7,800

Since Tax Liability is less than ` 10,000 hence Interest u/s 234B & 234C is not payable.
Interest u/s 234A = 7,800 x 1% x 5 = `390

Question 5: Explain Powers of Assessing officer to direct the Assessee to pay Advance Tax.
Answer: Powers of Assessing Officer to direct the Assessee to pay Advance Tax Section 209, 210
If any person has not paid advance tax and Assessing Officer is of the opinion that such person has to pay
advance tax, in such cases Assessing Officer may issue him a notice in Form No.28 directing such person to
pay advance tax but notice can be given only if such person has already been assessed through regular
assessment in any of the earlier years. Regular assessment shall include scrutiny assessment under section
143(3) or Best judgement assessment under section 144. If Assessing Officer has issue notice, estimated
income for such year shall be the income of the latest previous year in respect of which the assessee has
been assessed by way of regular assessment but if assessee has filed any return subsequently and income
reported in such return is higher than the income selected above, in that case income reported by the
assessee shall be considered to be estimated income of current year.
If the assessee do not pay the advance tax even after receiving such notice, he will be considered to be
assessee in-default as per section 218 and penalty can be imposed equal to the amount not paid as per
section 221.
If the assessee finds that his tax liability shall be less than the amount computed by the Assessing Officer,
assessee may give a reply in Form No.28A and can pay tax as per his own estimate.
Example
For the previous year 2018-19, ABC Ltd. has not paid any advance tax till 10.10.2018 and in the earlier
years the company was assessed in the manner given below:
2015-16 143(3) (Scrutiny Assessment) 7,00,000
2016-17 144 (Best Judgement Assessment) 10,00,000
2017-18 ROI 8,00,000
In this case Assessing officer shall have the powers to give notice to the assessee and its estimated income
shall be considered to be `10,00,000. If any assessee has received a notice in form no. 28 but he finds that
his tax liability shall be less than the amount computed by the Assessing Officer, in that case he can give a
reply in form no. 28A and can pay tax as per his own estimate.
Advance Payment Of Tax 113

PRACTICE PROBLEMS
TOTAL PROBLEMS 3
Problem 1.
Mr. X has incomes as given below:
1. Income under the head house property `15,00,000
2. Gift of a painting from a friend with market value `2,00,000
3. Gift of shares and securities from Mrs. X valued `3,00,000
4. Agricultural income `3,00,000
He has paid advance tax as given below:
Upto 15th June 2018 `15,000
Upto 15th Sept 2018 `30,000
Upto 15th Dec 2018 `50,000
th
Upto 15 March 2019 `60,000
Balance amount of tax was paid and return of income was filed on 10th Sept 2019.
Compute his tax liability for the Assessment Year 2019-20 and also interest under section 234A, 234B and
234C.
Answer = Tax Liability: `4,05,600; Interest under section 234A: `6,912; Interest under section 234B:
`20,736; Interest under section 234C: `17,031

Problem 2.
ABC Ltd. (an Indian Company) has income as given below:
1. Income from Business `20,00,000
2. Income under the head House Property `7,00,000
The company has paid advance tax as given below:
Upto 15th June 2018 ` 20,000
Upto 15th Sept 2018 ` 30,000
Upto 15th Dec 2018 ` 80,000
th
Upto 15 March 2019 `1,00,000
Balance amount of tax was paid and return of income was filed on 10th Dec 2019.
Due date for filing of ROI in case of ABC Ltd. is 30.09.2019.
Compute tax liability for the Assessment Year 2019-20 and also interest under section 234A, 234B and
234C.
Answer = Tax Liability: `8,42,400; Interest under section 234A: `22,272; Interest under section 234B:
`66,816; Interest under section 234C: `37,637

Problem 3.
Mrs. X has income under the head house property `18,00,000 and she has received gift of `3,00,000 in cash
from her husband’s sister and `1,00,000 from her sister’s husband and `1,20,000 from sister of her mother
in law. She has agricultural income of `4,00,000. She has paid advance tax as given below:
Upto 15th June 2018 ` 15,000
Upto 15th Sept 2018 ` 45,000
th
Upto 15 Dec 2018 ` 75,000
Upto 15th March 2019 `1,00,000
Balance amount of tax was paid on 10th Dec 2019 and return of income filed on the same date and due date
for filing return of income is 31.07.2019.
Compute her tax liability for the Assessment Year 2019-20 and also interest under section 234A, 234B and
234C.
Answer = Tax Liability: `4,84,640; Interest under section 234A: `19,230; Interest under section 234B:
`34,616; Interest under section 234C: `19,416
Advance Payment Of Tax 114

SOLUTIONS
TO
PRACTICE PROBLEMS
Solution 1: `
Computation of Total Income
Income under the head House Property 15,00,000
Income under the head Other Sources
Gift in kind received from a friend 2,00,000
Gross Total Income 17,00,000
Less: Deduction u/s 80C to 80U Nil
Total Income 17,00,000
Agricultural Income 3,00,000
Computation of Tax Liability
Step 1. Tax on (17,00,000 + 3,00,000) at slab rates 4,12,500
Step 2. Tax on (`2,50,000 + 3,00,000) at slab rates (22,500)
Step 3. Deduct Tax at Step 2 from Tax at Step 1 3,90,000
Add: HEC @ 4% 15,600
Tax Liability 4,05,600

Interest u/s 234C


Amount payable Amount actually paid Shortfall
` ` `
Upto 15.06 2018 (4,05,600 x 15%) 60,840 15,000 45,840
Rounded off under rule 119A = 45,800
Interest u/s 234C = 45,800 x 1% x 3 month = 1,374

Upto 15.09 2018 (4,05,600 x 45%) 1,82,520 30,000 1,52,520


Rounded off under rule 119A = 1,52,500
Interest u/s 234C = 1,52,500 x 1% x 3 month = 4,575

Upto 15.12.2018 (4,05,600 x 75%) 3,04,200 50,000 2,54,200


Interest u/s 234C = 2,54,200 x 1% x 3 month = 7,626

Upto 15.03.2019 (4,05,600 x 100%) 4,05,600 60,000 3,45,600


Interest u/s 234C = 3,45,600 x 1% x 1 month = 3,456

Interest liability under section 234C `17,031

Interest under section 234B (01-04-2019 to 10-09-2019)


3,45,600 x 1% x 6 `20,736
Interest under section 234A (01-08-2019 to 10-09-2019)
3,45,600 x 1% x 2 ` 6,912
Solution 2: `
Total Income 27,00,000
Computation of Tax Liability
Tax on `27,00,000 @ 30% 8,10,000
Add: HEC @ 4% 32,400
Tax Liability 8,42,400
Advance Payment Of Tax 115

Interest u/s 234C


Amount payable Amount actually paid Shortfall
` ` `
Upto 15.06 2018 (8,42,400 x 15%) 1,26,360 20,000 1,06,360
Rounded off under rule 119A = 1,06,300
Interest u/s 234C = 1,06,300 x 1% x 3 month = 3,189

Upto 15.09 2018 (8,42,400 x 45%) 3,79,080 30,000 3,49,080


Rounded off under rule 119A = 3,49,000
Interest u/s 234C = 3,49,000 x 1% x 3 month = 10,470

Upto 15.12.2018 (8,42,400 x 75%) 6,31,800 80,000 5,51,800


Interest u/s 234C = 5,51,800 x 1% x 3 month = 16,554

Upto 15.03.2019 (8,42,400 x 100%) 8,42,400 1,00,000 7,42,400


Interest u/s 234C = 7,42,400 x 1% x 1 month = 7,424

Interest liability under section 234C `37,637

Interest under section 234B (01-04-2019 to 10-12-2019)


7,42,400 x 1% x 9 `66,816

Interest under section 234A (01-10-2019 to 10-12-2019)


7,42,400 x 1% x 3 `22,272

Solution 3: `
Computation of Total Income
Income under the head House Property 18,00,000
Income under the head Other Sources
Gift received from sister of her mother in law 1,20,000
Gross Total Income 19,20,000
Less: Deduction u/s 80C to 80U Nil
Total Income 19,20,000
Agricultural Income 4,00,000
Computation of Tax Liability
Step 1. Tax on (19,20,000 + 4,00,000) at slab rates 5,08,500
Step 2. Tax on (`2,50,000 + 4,00,000) at slab rates (42,500)
Step 3. Deduct Tax at Step 2 from Tax at Step 1 4,66,000
Add: HEC @ 4% 18,640
Tax Liability 4,84,640
Interest u/s 234C
Amount payable Amount actually paid Shortfall
` ` `
Upto 15.06 2018 (4,84,640 x 15%) 72,696 15,000 57,696
Rounded off under rule 119A = 57,600
Interest u/s 234C = 57,600 x 1% x 3 month = 1,728

Upto 15.09 2018 (4,84,640 x 45%) 2,18,088 45,000 1,73,088


Rounded off under rule 119A = 1,73,000
Interest u/s 234C = 1,73,000 x 1% x 3 month = 5,190

Upto 15.12.2018 (4,84,640 x 75%) 3,63,480 75,000 2,88,480


Rounded off under rule 119A = 2,88,400
Advance Payment Of Tax 116

Interest u/s 234C = 2,88,400 x 1% x 3 month = 8,652

Upto 15.03.2019 (4,84,640 x 100%) 4,84,640 1,00,000 3,84,640


Interest u/s 234C = 3,84,600 x 1% x 1 month = 3,846

Interest liability under section 234C `19,416

Interest under section 234B (01-04-2019 to 10-12-2019)


3,84,600 x 1% x 9 `34,614
Interest under section 234A (01-08-2019 to 10-12-2019)
3,84,600 x 1% x 5 `19,230
Advance Payment Of Tax 117

EXAMINATION QUESTIONS
NOV – 2017
Question 5(a) (i) (Marks 2)
Under section 208, obligation to pay advance tax arises in every case where the advance tax payable is `
10,000 or more, State exception to this rule.
Answer: As per Section 208, Advance Tax shall be payable during a financial year in every case where the
amount of such tax payable by the assessee is `10,000 or more however as per section 207, a senior citizen
who do not have income under the head business/profession shall be exempt from payment of advance tax,
eg. Mr. X is a senior citizen and he has income under the head business/ profession and also under the head
house property, in this case he will be required to pay advance tax but if he do not have income under the
head business / profession, he will be exempt from payment of advance tax.

NOV – 2016
Question 6(a) (2 Marks)
Discuss the provision under Income Tax Act for Payment of Advance Tax in case of Capital Gain.
Answer: Refer Answer given in the book

MAY – 2016
Question 7(a)(ii). (4 Marks)
Briefly discuss the provisions of Section 234B of the Income-tax Act,1961 for the short - payment or non-
payment of advance tax.
Refer Answer given in the Chapter

NOV – 2015
Question 7(a). (4 Marks)
Explain briefly the provisions of advance tax on capital gains and casual income.
Refer Answer given in the Chapter

MAY – 2014
(2 x 2 = 4 Marks)
Question. Who is liable to pay Advance Tax? What is the procedure to compute the Advance Tax payable?
Solution: As per section 207, every person is liable to pay advance tax however a senior citizen is exempt
from payment of advance tax provided he do not have any income under the head Business/Profession. As
per section 208, every person in whose case tax payable is less than `10,000 shall also be exempt from
payment of advance tax. For this purpose, the assessee himself shall estimate his tax liability / tax payable
and shall pay advance tax in installments given under section 211. An assessee is exempt from payment of
advance tax with regard to capital gains and casual income but advance tax should be paid after accrual of
such income in subsequent installments. Net agricultural income is also to be considered for the purpose of
computing advance tax.
A person may increase or reduce the amount of advance tax payable in subsequent installment(s) in
accordance with his estimate of current income.

MAY – 2013
Question 7(a) (4 Marks)
Briefly discuss the provisions relating to payment of advance tax in case of capital gains and casual income.
Solution: Refer answer given in the chapter
Advance Payment Of Tax 118

JUNE – 2009
Question 5 (4 Marks)
Enlist the installments of advance tax and due dates thereon in case of companies.
Answer. Refer answer given in the chapter

MAY – 2008
Question 5 (4 x 2 = 8 Marks)
(a) Briefly discuss about the interest chargeable under Section 234A for delay or default in furnishing return
of income.
(b) What are the due dates of instalments and the quantum of advance tax payable by companies?
Answer 5(a)/5(b) . Refer answer given in the chapter

NOV – 2007
Question 5 (4 Marks)
Briefly discuss the provisions relating to payment of advance tax on income arising from capital gains and
casual income.
Answer. Refer answer given in the chapter
Advance Payment Of Tax 119

CHALLAN Tax Applicable (Tick One)*


NO./ (0020) INCOME-TAX ON COMPANIES Assessment Year
ITNS 280 (CORPORATION TAX) -
(0021) INCOME TAX (OTHER THAN
COMPANIES)
Permanent Account Number

Full Name

Complete Address with City & State

Tel. No. Pin


Type Of Payment (Tick One)
Advance Tax (100)
Self Assessment Tax (300) Tax on Distributed Profits of Domestic Companies (106)
Tax on Regular Assessment (400) Tax on Distributed Income to Unit Holders (107)
DETAILS OF PAYMENTS Amount (in Rs. Only) FOR USE IN RECEIVING BANK
Income Tax Debit to A/c / Cheque credited on
Surcharge - -
Health & education cess D D M M Y Y
Interest
Penalty
Others SPACE FOR BANK SEAL
Total
Total (in words)
CRORES LACS THOUSANDS HUNDREDS TENS UNITS

Paid in Cash/Debit to A/c /Cheque No. Dated


Drawn on
(Name of the Bank and Branch)
Date: Rs.
Signature of person making payment
Taxpayers Counterfoil (To be filled up by tax payer) SPACE FOR BANK SEAL
PAN
Received from
(Name)
Cash/ Debit to A/c /Cheque No. For Rs.
Rs. (in words)
Drawn on
(Name of the Bank and Branch)
on account of Companies/Other than Companies/Tax
(Strike out whichever is not
Income Tax on applicable)
(To be filled up by person making the
Type of Payment payment)
for the Assessment Year - Rs.
Residential Status & Scope of Total Income 120

RESIDENTIAL STATUS
&
SCOPE OF TOTAL INCOME
SECTION 5 TO 9
PARTICULARS SECTIONS

Scope of total income/incidence of tax 5


Rules for determining the residential status of an individual 6(1), 6(6)(a)
Rules for determining the residential status of an Hindu Undivided Family 6(2), 6(6)(b)
Rules for Determining the residential status of Firm/ Association of person/Body of 6(2)
individual
Rules for determining the residential status of a company 6(3)
Rules for determining the residential status of any other person 6(4)
Incomes deemed to be received 7
Income deemed to accrue or arise in India 9

Residential Status Section 6


Whether a particular income shall be taxed or not shall depend on the residential status and the type of
income. Residential status infact explains connection of the person with the country and types of income
explains the connection of the income with the country. If the person do not have any connection and also
the incomes do not have any connection with the country, the income shall not be taxable but if either the
person or the income has any connection, the income is taxable.
For this purpose, taxpayers are classified into three broad categories on the basis of their residential status.
(1) Resident and ordinarily resident
(2) Resident but not ordinarily resident
(3) Non-resident
The residential status of an assessee is ascertained with reference to each previous year.
Question 1: How to determine Residential status of individuals Section 6(1)/6(6)(a)
Answer: Under section 6(1), an individual is said to be resident in India in any previous year, if he satisfies
any one of the following conditions:
(i) He stays in India for 182 days or more during the relevant previous year
(ii) He stays in India for 60 days or more and also for 365 days or more during 4 years preceding the
relevant previous year.
If the individual satisfies any one of the conditions mentioned above, he is a resident, otherwise the
individual is a non-resident.
e.g. Mr. X stayed in India for 200 days in previous year 2018-19, in this case he will be considered to be
resident in India.
If Mr. X stayed in India for 100 days in previous year 2018-19 and also for 365 days during 4 years
preceding the previous year 2018-19, he will be considered to be resident in India.
If Mr. X stayed in India during previous year 2018-19 for 59 days, he will be considered to be non-resident
in previous year 2018-19.
The term “stay in India” includes stay in the territorial waters of India (i.e. 12 nautical miles into the sea
from the Indian coastline). Even the stay in a ship or boat in the territorial waters of India shall be
considered to be stay in India. (1 nautical mile = 1.1515 miles = 1.852 Kms).
Residential Status & Scope of Total Income 121

It is not necessary that the period of stay must be continuous nor is it essential that the stay should be at the
usual place of residence, business or employment of the individual.
For the purpose of counting the number of days stayed in India, both the date of departure as well as the date
of arrival are considered to be in India.
Meaning of Not-ordinarily resident Section 6(6)(a)
An individual who is resident of India shall be considered to be NOR if he has complied with at least one of
the conditions given below:
(i) If such individual has during the 7 previous years preceding the relevant previous year been in India
for a period of 729 days or less or
(ii) If such individual has been non-resident in India in 9 years out of 10 previous years preceding the
relevant previous year
If he has not complied with even a single condition, he will be considered to be ROR.
Income Tax Act has defined NOR but we can define ROR in the manner given below:
An individual is said to be a resident and ordinarily resident if he satisfies both the following conditions:
(i) He is a resident in any 2 out of the last 10 years preceding the relevant previous year, and
(ii) His total stay in India in the last 7 years preceding the relevant previous year is 730 days or more.
If the individual satisfies both the conditions mentioned above, he is a resident and ordinarily resident but if
only one or none of the conditions are satisfied, the individual is a resident but not ordinarily resident.
Illustration 1: Determine residential status of Mr. X for the assessment year 2019-20, who stays in India
during various financial years asunder:
Previous Years Stay 2011-12 91
2018-19 100 2010-11 90
2017-18 200 2009-10 88
2016-17 91 2008-09 89
2015-16 90 2007-08 86
2014-15 89 2006-07 87
2013-14 87 2005-06 89
2012-13 82 2004-05 90
Solution:
As per section 6(1), Stay in India is 60 days plus 365 days during 4 years preceding the relevant previous
year, hence he is resident.
His stay during 7 years is 730 hence he is not able to comply with first condition of section 6(6)(a). He is
able to comply with second condition of 6(6)(a) i.e. he is non-resident in atleast 9 years out of 10 years
preceding the relevant previous year hence he is NOR.
Years Status 2013-14 Non-resident
2018-19 Resident 2012-13 Non-resident
2017-18 Resident 2011-12 Non-resident
2016-17 Non-resident 2010-11 Non-resident
2015-16 Non-resident 2009-10 Non-resident
2014-15 Non-resident 2008-09 Non-resident

Illustration 2: Determine residential status of Mr. X for the assessment year 2019-20, who stays in India
during various financial years asunder:
Previous Years Stay 2011-12 80
2018-19 75 2010-11 91
2017-18 197 2009-10 86
2016-17 94 2008-09 85
2015-16 89 2007-08 89
2014-15 90 2006-07 72
Residential Status & Scope of Total Income 122

2013-14 89 2005-06 69
2012-13 91 2004-05 92
Solution:
As per section 6(1), Stay in India is 60 days plus 365 days during 4 years preceding the relevant previous
year, hence he is resident.
His stay during 7 years is 730 hence he is not able to comply with first condition of section 6(6)(a). He is
able to comply with second condition of 6(6)(a) i.e. he is non-resident in atleast 9 years out of 10 years
preceding the relevant previous year hence he is NOR.

Years Status 2013-14 Non-resident


2018-19 Resident 2012-13 Non-resident
2017-18 Resident 2011-12 Non-resident
2016-17 Non-resident 2010-11 Non-resident
2015-16 Non-resident 2009-10 Non-resident
2014-15 Non-resident 2008-09 Non-resident
Illustration 3: Mr. X an American citizen has come to India for the first time on 10.07.2014, as an
employee of a multinational company. The particulars of his arrival and departure are as given below:
Date of arrival Date of departure
10.07.2014 07.08.2015
07.10.2015 27.11.2016
01.03.2017 01.02.2018
10.05.2018 30.03.2019
Not yet returned
Determine his residential status for previous year 2014-15 to 2018-19.
Solution:
Previous Year 2014-15
{July – 22, August – 31, September 30, October 31, November 30, December 31, January 31, February – 28,
March – 31}
Days of stay in India are 265.
As per section 6(1), Stay in India is 182 days or more during the relevant previous year, hence he is resident.
and his stay during 7 years is Nil hence he is able to comply with first condition of section 6(6)(a). hence he
is NOR.
Previous Year 2015-16
{April – 30, May – 31, June – 30, July – 31,August – 7,October – 25, November – 30, December – 31,
January – 31, February – 29, March – 31}
Days of stay in India are 306.
As per section 6(1), Stay in India is 182 days or more during the relevant previous year, hence he is resident.
and his stay during 7 years is 265 days hence he is able to comply with first condition of section 6(6)(a).
hence he is NOR.
Previous Year 2016-17
{April – 30, May – 31, June – 30, July – 31,August – 31,September – 30 , October – 31, November – 27,
March – 31}
Days of stay in India are 272, so Mr. X is Resident.
As per section 6(1), Stay in India is 182 days or more during the relevant previous year, hence he is resident.
and his stay during 7 years is 571 days hence he is able to comply with first condition of section 6(6)(a).
hence he is NOR.
Previous Year 2017-18
{April – 30, May – 31, June – 30, July – 31,August – 31, September – 30, October – 31, November – 30,
December – 31, January – 31, February – 1}
Days of stay in India are 307.
As per section 6(1), Stay in India is 182 days or more during the relevant previous year, hence he is resident.
and his stay during 7 years is more than 729 days hence he is not able to comply with first condition of
Residential Status & Scope of Total Income 123

section 6(6)(a) and he is not able to comply second condition also i.e. non-resident in atleast 9 years out of
10 years preceding the relevant previous year. hence he is ROR.
Previous Year 2018-19
{May – 22, June – 30, July – 31,August – 31, September – 30, October – 31, November – 30, December –
31, January – 31, February – 28, March - 30}
Days of stay in India are 325.
As per section 6(1), Stay in India is 182 days or more during the relevant previous year, hence he is resident.
and his stay during 7 years is more than 729 days hence he is not able to comply with first condition of
section 6(6)(a) and he is not able to comply second condition of section 6(6)(a) also i.e. non-resident in
atleast 9 years out of 10 years preceding the relevant previous year. hence he is ROR.
Illustration 4: Mr. X an American citizen has come to India for the first time on 01.07.2015 as an executive
of a multinational company. His employer has allowed him to visit USA every year and for this purpose he
will be leaving India every year on 1st November and shall come back on 31st December, besides that he has
visited Hong Kong on several occasions in connection with the official work, because he is looking after the
employer’s operations in Hong Kong also, with details asunder:
Date of leaving India Date of arriving in India
10.09.2015 30.09.2015
07.02.2016 08.05.2016
11.07.2016 21.10.2016
10.02.2017 23.07.2017
11.02.2018 12.06.2018
01.02.2019 10.04.2019
Determine his residential status for the previous years 2015-16 to 2018-19.
Solution:
Previous Year 2015-16
{July – 31, August – 31, September – 11, October – 31, November – 1, December – 1, January – 31,
February – 7}
Days of stay in India are 144
As per section 6(1), Stay in India is less than 182 days but more than 60 days during the relevant previous
year but Stay in India is less than 365 days during 4 years preceding the relevant previous year, hence he is
not complying even a single condition of section 6(1) hence he is Non – resident.
Previous Year 2016-17
{May – 24, June – 30, July – 11, October – 11, November – 1, December – 1, January – 31, February – 10}
Days of stay in India are 119.
As per section 6(1), Stay in India is less than 182 days but more than 60 days during the relevant previous
year but Stay in India is less than 365 days during 4 years preceding the relevant previous year, hence he is
not complying even a single condition of section 6(1) hence he is Non – resident.
Previous Year 2017-18
{July – 9, August – 31, September – 30, October – 31, November – 1, December – 1, January – 31, February
– 11}
Days of stay in India are 145
As per section 6(1), Stay in India is less than 182 days but more than 60 days during the relevant previous
year but Stay in India is less than 365 days during 4 years preceding the relevant previous year, hence he is
not complying even a single condition of section 6(1) hence he is Non – resident.
Previous Year 2018-19
{June – 19, July – 31, August – 31, September – 30, October – 31, November – 1, December – 1, January –
31, February – 1}
Days of stay in India are 176. During the preceding 4 years, his stay is for 365 days or more so he is
resident. His stay during 7 years is 729 days or less, hence he is resident but not ordinarily resident.
Illustration 5: Mr. X, the Australian cricketer comes to India for 105 days every year. Find out his
residential status for the A.Y. 2019-20.
Residential Status & Scope of Total Income 124

Solution: He has complied with condition of 60 + 365 days hence he is resident further stay in 7 years is
more than 729 days and also condition of non-resident in 9 years out of 10 years is not complied with hence
he is ROR.
Illustration 6: Mr. X, a Canadian citizen, comes to India for the first time during the P.Y.2014-15. During
the financial years 2014-15, 2015-16, 2016-17, 2017-18 and 2018-19 he was in India for 55 days, 60 days,
90 days, 150 days and 70 days respectively. Determine his residential status for the A.Y.2019-20.
Solution:
During the previous year 2018-19, Mr. B was in India for 70 days and during the 4 years preceding the
previous year 2018-19, he was in India for 355 days (i.e. 55 + 60 + 90 + 150 days).
Thus, he does not satisfy section 6(1). Therefore, he is a non-resident for the previous year 2018-19.
Illustration 7: On 01.06.2016 Mr. X, a Malaysian citizen leaves India after stay of 10 years. During the
financial year 2017-18 he comes to India for a period of 46 days. Later, he returns to India for one year on
10.10.2018.
Determine Mr. X’s residential status for the assessment year 2019-20.
Solution:
No. of days of stay in India
P.Y. 2018-19 173 Days
{22 + 30 + 31 + 31 + 28 + 31}
P.Y. 2017-18 46 Days
P.Y. 2016-17 62 Days
{30 + 31 + 1}
P.Y. 2015-16 366 Days
P.Y. 2014-15 365 Days
P.Y. 2013-14 365 Days
P.Y. 2012-13 365 Days
P.Y. 2011-12 366 Days
P.Y. 2010-11 365 Days
P.Y. 2009-10 365 Days
P.Y. 2008-09 365 Days
The person is resident and ordinarily resident. Mr. X was in India for 60 days in 2018-19 and for 365 days
or more in the 4 years immediately preceding the relevant previous year and he does not satisfy even a
single condition of section 6(6)(a).
NOV – 2011 (2 Marks)
Brett Lee, an Australian cricket player visits India for 100 days in every financial year. This has been his
practice for the past 10 financial years. Find out his residential status for the assessment year 2019-20.
Answer: An individual is said to be resident in India in any previous year, if he complies with at least one of
the following conditions:-
(a) He is in India in that year for a period amounting in all to 182 days or more, or
(b) He is in India in that year for a period amounting in all to 60 days or more and also for 365 days or more
during four years preceding the relevant previous year.
Since, Brett Lee has complied with the second condition hence he is resident.
Further more, an individual shall be considered to be not ordinarily resident in India in case his stay in India
is 729 days or less during preceding seven years or he is non-resident in atleast 9 years during preceding 10
years.
Since stay of Brett Lee during preceding seven years is 700 days. Hence, he is NOR.
MAY – 1998 (4 Marks)
Mr. Nixon, an American citizen, is appointed by a multi-national company to its branch in New Delhi in
2015. Mr. Nixon has never been to India before this appointment. He arrives in Bombay on 15th April, 2015
and joins the New Delhi office on 20th April, 2015. His wife and children join him in India on 20th October,
2015. The company allotted him a leased residence for purposes of his stay. This residence is occupied by
him from the beginning of October, 2015.
On 10th February, 2016, he is transferred by his employer, on deputation basis, to be the Regional Chief of
his employer’s operations in South East Asia having headquarters in Hongkong. He leaves New Delhi on
Residential Status & Scope of Total Income 125

11th February and arrives in Hongkong on 12th February, 2016. Mr. Nixon leaves behind his wife and
children in India till 14th August, 2017, when they leave along with him for Hongkong. Mr. Nixon had come
to India earlier on 15th June, 2017, on two months’ leave. The members of the family occupied the residence
till date of departure to Hongkong.
At the end of the period of deputation, Mr. Nixon is reposted to India and joins the New Delhi office of his
employer as Chief of Indian operations on 2nd February, 2019.
In what residential status Mr. Nixon will be assessable, for the various years, to income tax in India?
Answer:
The period of stay of Mr. Nixon for various years is given below:
P.Y. 2015-16/ A.Y. 2016-17 Period of stay 303 days
(April – 16, May -31, June – 30, July – 31, August – 31, September – 30, October – 31, November – 30,
December – 31, January – 31, February – 11)
P.Y. 2016-17/ A.Y. 2017-18 Period of stay Nil
P.Y. 2017-18/ A.Y. 2018-19 Period of stay 61 days
(June – 16, July – 31, August – 14)
P.Y. 2018-19/ A.Y. 2019-20 Period of stay 58 days
(February – 27, March – 31)
Under section 6(1) of the Act, an individual is said to be resident in India in any previous year if he satisfies
one of the following basic conditions:
(i) is in India in the previous year for a period of 182 days or more ;
(ii) is in India for a period of 60 days or more in the previous year and 365 days or more during the four
years preceding the previous year.
A person will be considered to be ‘not ordinarily resident’ if he satisfies any of the following two conditions
viz;
(i) he has been in India for a period of 729 days or less in 7 previous years preceding the relevant previous
year.
(ii) he has been a non resident in India in 9 out of 10 previous years preceding the relevant previous year ; or
Maintenance of a residence in India or the stay of the wife and children in India are not relevant for
determining the residential status of Mr. Nixon.
In the above background, Mr. Nixon’s case will be decided as under:
(i) P.Y. 2015-16/ A.Y. 2016-17: has been in India for 303 days. He will be a resident under the basic
conditions. Since his stay in seven years preceding the relevant previous year is Nil i.e. 729 days or less,
hence he will be NOR
(ii) P.Y. 2016-17/A. Y. 2017-18: has not been in India at all ; though his wife and children continue to reside
in New Delhi, he will be a non-resident for this year.
(iii) P.Y. 2017-18/A.Y. 2018-19: has been in India for 61 days and for 303 days in 4 years preceding the
relevant previous year hence he will be non-resident.
(iv) P.Y. 2018-19/A.Y. 2019-20: has been in India for 58 days, he will be non-resident.
Question 2: Write a note on determination of residential status of Individual covered in special
category.
Answer: Special category
As per Section 6(1),Certain individuals are covered in the special category and they will be considered to be
resident only if they stay in India for 182 days or more i.e. second condition of 60 plus 365 days shall not be
applicable and such individuals are:
1. Any individual who is a citizen of India and has left India for taking up any business or profession or
employment outside India e.g. Mr. X is a citizen of India and has left India on 01.09.2018 for taking up an
employment in Germany, in this case he will be covered in the special category and his status shall be non-
resident. If any such person is employed in India and he has been transferred outside India, he will also be
covered in the special category. E.g. Mr. X is employed in Punjab National Bank in India and he has been
transferred to the London branch, in this case he will be covered in the special category. If any person has
business or profession in India and he is going out of India in connection with business or profession, he will
not be covered in special category.
Residential Status & Scope of Total Income 126

2. Any individual who is a citizen of India or is a person of Indian origin and is having
business/profession/employment outside India and has come to India on a visit shall also be covered in the
special category e.g. Mr. X is a citizen of India and is settled as a doctor in USA and has come to India on a
visit for 181 days, he will be covered in the special category and his status shall be non-resident.
A person is said to be of Indian origin if he or either of his parents or either of his grandparents (including
parents of mother) were born in undivided India. e.g. Mr. X has taken birth in UK and is a citizen of UK but
his grand father has taken birth in India in 1942, in this case Mr. X will be considered to be person of Indian
origin.
3. Any individual who is a citizen of India and has left India as a member of crew of an Indian ship, shall
also be covered in special category. The time period mentioned in Continuous Discharge Certificate shall be
considered to be the period of stay outside India and remaining time period shall be considered to be stay in
India.
Example
Mr. Anand is an Indian citizen and a member of the crew of a Singapore bound Indian ship engaged in
international traffic departing from Chennai port on 6th June, 2018. From the following details for the
P.Y.2018-19, determine the residential status of Mr. Anand for A.Y.2019-20, assuming that his stay in India
in the last 4 previous years (preceding P.Y.2018-19) is 400 days and last seven previous years (preceding
P.Y.2018-19) is 750 days:

Particulars Date
Date entered into the Continuous Discharge Certificate in respect of joining the 6th June, 2018
ship by Mr. Anand
Date entered into the Continuous Discharge Certificate in respect of signing off the 9th December, 2018
ship by Mr. Anand
Answer.
In this case, the voyage is undertaken by an Indian ship engaged in international traffic, originating from a
port in India (i.e., the Chennai port) and having its destination at a port outside India (i.e., the Singapore
port). Hence, the voyage is an eligible voyage for the purposes of section 6(1). Therefore, the period
beginning from 6th June, 2018 and ending on 9th December, 2018, being the dates entered into the
Continuous Discharge Certificate in respect of joining the ship and signing off from the ship by Mr. Anand,
an Indian citizen who is a member of the crew of the ship, has to be excluded for computing the period of
his stay in India. Accordingly, 187 days [25+31+31+30+31+30+9] have to be excluded from the period of
his stay in India. Consequently, Mr. Anand’s period of stay in India during the P.Y.2018-19 would be 178
days [i.e., 365 days – 187 days]. Since his period of stay in India during the P.Y.2018-19 is less than 182
days, he is a non-resident for A.Y.2019-20.
Note - Since the residential status of Mr. Anand is “non-resident” for A.Y.2019-20 consequent to his
number of days of stay in P.Y.2018-19 being less than 182 days, his period of stay in the earlier previous
years become irrelevant.
Illustration 8: Mr. X, an Indian citizen, leaves India on 22.09.2018 for the first time, to work as an officer
of a company in France. Determine his residential status for the A.Y. 2019-20.
Solution:
During the previous year 2018-19, Mr. X, an Indian citizen, was in India for 175 days (i.e. 30 + 31+ 30 + 31
+ 31 + 22 days). He does not satisfy the minimum criteria of 182 days. Also, since he is an Indian citizen
leaving India for the purposes of employment, the second condition under section 6(1) is not applicable to
him. Therefore, Mr. X is a non-resident for the A.Y.2019-20.
Illustration 9: Mr. X and Mrs. X are settled outside India for the purpose of employment and they came to
India on 15.10.2018 on a visit for 7 months. Both of them are Indian citizens. In the earlier years they were
in India as follows:
Year Mr. X Mrs. X
2017 – 2018 235 Days 365 Days
2016 – 2017 330 Days 30 Days
2015 – 2016 Nil 28 Days
2014 – 2015 118 Days 120 Days
Residential Status & Scope of Total Income 127

Find out the residential status of Mr. X and Mrs. X for the assessment year 2019-20.
Solution:
Both are NR for the assessment year 2019-20
Stay of Mr. X in India
Previous Year 2018-19 168 Days
{17 + 30 + 31 + 31 + 28 + 31}
Stay of Mrs. X in India
Previous Year 2018-19 168 Days
{17 + 30 + 31 + 31 + 28 + 31}
Since they are covered in special category they will be resident only if their stay in India in relevant previous
year is 182 days or more, hence they are non–resident.

NOV – 2010 (3 Marks)


Mr. Ram an Indian citizen left India on 22.09.2018 for the first time to work as an officer of a company in
Germany.
Determine the residential status of Ram for the assessment year 2019-20 and explain the conditions to be
fulfilled for the same under the Income-tax Act, 1961.
Answer: Under section 6(1), an individual is said to be resident in India in any previous year if he satisfies
any one of the following conditions –
(i) He has been in India during the previous year for a total period of 182 days or more, or
(ii) He has been in India for a period of 60 days or more during the relevant previous year and also for 365
days or more during 4 years preceding the relevant previous year.
In the case of Indian citizens leaving India for employment, the period of stay during the previous year must
be 182 days to become resident.
During the previous year 2018-19, Mr. Ram, an Indian citizen, was in India for 175 days only (i.e. 30 + 31 +
30 + 31 + 31 + 22 days). Thereafter, he left India for employment purposes. Since he does not satisfy the
minimum criteria of 182 days, he is a non-resident for the A.Y. 2019-20.
NOV – 2002 (3 Marks)
In the year P.Y. 2018-2019, a sailor has remained on ship as a crew member of an Indian Ship as follows:
(1) Outside the territorial waters of India for 183 days.
(2) Inside the territorial waters of India for 182 days.
Is he considered to be resident or not for the Assessment Year 2019-20. Comment.
Answer: As per section 6(1) he is covered in the special category and he will be considered to be resident if
he stays in India for 182 days or more.
In the given case, period of stay in India is 182 days. Therefore, the sailor is treated as a resident in India.

Question 3: Explain how to determine residential status of HUF Section 6(2)/6(6)(b).


Answer: As per section 6(2), an HUF would be resident in India if the control and management of its
affairs is situated wholly or partly in India. If the control and management of the affairs is situated wholly
outside India it will be considered to be non-resident. Since control and management of HUF is in the hands
of its Karta hence place of stay of Karta shall be taken into consideration i.e. if Karta is out of India
throughout the year, HUF shall be Non-resident but if Karta has come to India for a few days, HUF shall be
resident.
The expression ‘control and management’ refers to the central control and management and not to the
carrying on of day-to-day business by servants, employees or agents.
Meaning of Not-ordinarily resident Section 6(6)(b)
If an HUF is resident, as per section 6(6)(b), it will be considered to be NOR if its Karta has complied with
at least one of the conditions given below:
(i) If the karta is in India during the 7 previous years preceding the relevant previous year for a period of
729 days or less.
(ii) If the karta is non-resident in India in any 9 out of the 10 previous years preceding the relevant
previous year, or
If karta has not complied with even a single condition, HUF shall be ROR.
Residential Status & Scope of Total Income 128

Illustration 10: Karta of one Hindu Undivided Family comes to India every year for minimum 60 days and
maximum 91 days. Determine residential status of the Hindu Undivided Family and also that of the Karta
for the assessment year 2019-20.
Solution:
Hindu Undivided Family is resident since the Karta has come to India for at least 60 days but the stay of
Karta during seven years can be maximum 637 days hence Hindu Undivided Family shall be considered to
be resident but not ordinarily resident.
Karta in his individual capacity is non-resident because he cannot comply with even one of the two
conditions given under section 6(1).
Illustration 11: One Hindu Undivided Family is being managed partly from Mumbai and partly from
Nepal. Mr. X (a foreign citizen), Karta of Hindu Undivided Family, comes on a visit to India every year
since 1982 in month of April for 105 days.
Determine residential status of the Hindu Undivided Family and also that of the Karta in his individual
capacity for the assessment year 2019-20.
Solution:
For the previous year 2018-19, the control and management of the affairs of Hindu Undivided Family is
being partly managed from India. Hence Hindu Undivided Family is resident but Mr. X cannot comply with
any of the conditions of section 6(6)(b), hence Hindu Undivided Family is resident and ordinarily resident.
Karta shall be considered to be resident and ordinarily resident because his stay during 7 years is 735 days.
Also, he will not be non-resident in nine years out of ten years preceding the relevant previous year.
JUNE – 2009 (2 Marks)
State with reason, whether the following statements are True or False:
Mr. X, Karta of HUF, claims that the HUF is non-resident as the business of HUF is transacted from UK and
all the policy decisions are taken there.
Answer. True, A HUF is considered to be a non-resident where the control and management of its affairs
are situated wholly outside India. In the given case, since all the policy decisions of HUF are taken from
UK, the HUF is a non-resident.
Question 4: Explain how to determine residential status of partnership firm or Body of Individual or
Association of Persons.
Answer:
Firms and Association of Persons Section 6(2)
A firm and an AOP would be resident in India if the control and management of its affairs is situated wholly
or partly in India. Where the control and management of the affairs is situated wholly outside India, the firm
and AOP would become a non-resident. There are no ROR or NOR in case of persons other than individual
or HUF. E.g. XY partnership firm has two partners Mr. X and Mr. Y and Mr. X is working partner and is in
USA throughout the year and Mr. Y is a dormant partner and is in India throughout the year, in this case
partnership firm shall be non-resident but if Mr. X has come to India for a few days, partnership firm shall
be resident.
Question 5: Explain how to determine residential status of a Company.
Answer:
Companies Section 6(3)
An Indian company is always resident in India even if its control and management is outside India or its
business is outside India.
A foreign company shall be resident in India if its place of effective management, at any time in that year, is
in India.
“Place of effective management” means a place where key management and commercial decisions are made
for the conduct of the business of an entity.
e.g. Micromax Informatics Ltd. was incorporated in India and it has business in many countries outside
India, in this case company shall be considered to be resident.
e.g. HCL Technologies Ltd. was incorporated in India and it has its control and management outside India
also, in this case company shall be considered to be resident.
e.g. ABC Ltd. was incorporated outside India and place of effective management is in India, in this case
company shall be considered to be resident.
Residential Status & Scope of Total Income 129

e.g. Videocon Industries Ltd. was incorporated in India, in this case company shall be considered to be
resident.
e.g. Samsung Electronics Co., Ltd. was incorporated in South Korea and place of effective management is
also in South Korea , in this case company shall be considered to be non-resident.
e.g. BlackBerry Ltd. was incorporated in Canada and place of effective management is also in Canada, in
this case company shall be considered to be non-resident.
Illustration 12: ABC Inc., a Swedish company headquartered at Stockholm, not having a permanent
establishment in India, has set up a liaison office in Mumbai in April, 2018 in compliance with RBI
guidelines to look after its day to day business operations in India, spread awareness about the company’s
products and explore further opportunities. The liaison office takes decisions relating to day to day routine
operations and performs support functions that are preparatory and auxiliary in nature. The significant
management and commercial decisions are, however, in substance made by the Board of Directors at
Sweden. Determine the residential status of ABC Inc. for A.Y.2019-20.
Answer.
Section 6(3) has been substituted by the Finance Act, 2016 with effect from A.Y.2017-18 to provide that a
company would be resident in India in any previous year, if-
(i) it is an Indian company; or
(ii) its place of effective management, in that year, is in India .
In this case, ABC Inc. is a foreign company. Therefore, it would be resident in India for P.Y.2018-19 only if
its place of effective management, in that year, is in India.
Explanation to section 6(3) defines “place of effective management” to mean a place where key
management and commercial decisions that are necessary for the conduct of the business of an entity as a
whole are, in substance made. In the case of ABC Inc., its place of effective management for P.Y.2018-19 is
not in India, since the significant management and commercial decisions are, in substance, made by the
Board of Directors outside India in Sweden.
ABC Inc. has only a liaison office in India through which it looks after its routine day to day business
operations in India. The place where decisions relating to day to day routine operations are taken and
support functions that are preparatory or auxiliary in nature are performed are not relevant in determining
the place of effective management.
Hence, ABC Inc., being a foreign company is a non-resident for A.Y.2019-20, since its place of effective
management is outside India in the P.Y.2018-19.
Illustration 13: Wipro Ltd. an Indian company has most of its business outside India. Determine its
residential status.
Solution:
An Indian company shall always be considered to be resident in India.
Illustration 14: Afcons Infreastructure International Ltd. is incorporated in Mauritius and its place of
effective management is in Mauritius. Determine its residential status for the assessment year 2019-20.
Solution:
Foreign company shall be resident in India only if its place of effective management, at any time in that
year, is in India. Hence, Afcons Infreastructure International Ltd. is a non-resident company.
Illustration 15: Bista Ltd., a foreign company and it carries on majority of its operations and decision
making activities from Calcutta and Assam but some part of operational activities and few decisions are
being taken from the place at which registered office of Bista Ltd. is located, i.e. Dhaka.
Determine its residential status for the assessment year 2019-20.
Solution:
Bista Ltd. is a foreign company and its place of effective management is in India. Hence Bista Ltd. is
resident in India for the assessment year 2019-20.
Question 6: Explain how to determine residential status of other persons.
Answer:
Local Authorities and Artificial Juridical Persons Section 6(4)
Local authorities and artificial juridical persons would be resident in India if the control and management of
its affairs is situated wholly or partly in India. Where the control and management of the affairs is situated
wholly outside India, they would become non-residents.
Residential Status & Scope of Total Income 130

TAX INCIDENCE / SCOPE OF TOTAL INCOME


Question 7: Write a note on scope of total income or tax incidence.
Answer: As per section 5, scope of total income or tax incidence in various status shall be as given
below:
(1) Resident and ordinarily resident – In case of ROR, the following incomes shall be taxable.
(i) income accruing / arising in India.
(ii) income received or deemed to be received in India even if accruing /arising abroad.
(iii) income accruing / arising aboard and received aboard.
In simpler terms, ROR has to pay tax on his world income in India.
Meaning of income received in India
Income shall be considered to be received in India if it has been received directly in India from its source i.e.
if the income has been received outside India and after that it was transferred to India, it will not be
considered to be income received in India rather it is income received abroad.
Therefore, when once an amount is received as income, remittance or transmission of that amount from one
place or person to another does not constitute receipt of income in the hands of the subsequent recipient or at
the place of subsequent receipt.
Example
Mr. X has one house in USA and rent has been received directly in India. It will be considered to be income
received in India and it is chargeable to tax in case of all the three status, but if Mr. X has one bank account
with Bank of America, New York and rent has been deposited in that account and subsequently the bank has
transferred the amount to Mr. X in India, it will be considered to be income received outside India, because
income has already been received outside India and subsequently it was remitted to India.
Similarly, if Mr. X has income in Nepal and it was deposited in the branch of an Indian bank in Nepal,
subsequently the amount was remitted in India, it will be considered to be income received outside India.
(2) Resident but not ordinarily resident – The following incomes shall be taxable.
(i) income accruing / arising in India.
(ii) income received or deemed to be received in India even if accruing /arising abroad.
(iii) income accruing / arising aboard and received aboard but from a business controlled from India or
from a profession which was set up in India.
Meaning of profession setup in India
Profession set up in India means that it was originally setup in India and subsequently there was an
expansion outside India. E.g. Mr. X started his profession of an advocate in Delhi and subsequently he
opened his branch outside India, it will be called profession setup in India.
(3) Non-resident –The following incomes shall be taxable.
(i) income accruing /arising in India.
(ii) income received or deemed to be received in India even if accruing /arising abroad.
Illustration 16: Mr. X has income asunder:
1. He has income from a business in Germany amounting to `3,00,000 and half of it was received in
India.
2. He has interest income of `1,00,000 from UK Development Bond and entire interest income was
credited to a bank account in UK. Subsequently, the amount was transferred in India.
3. He has a business in Bombay and entire income of `3,00,000 was received in UK.
4. He has one house property in Ghaziabad and income of `5,00,000 was received in UK.
5. He has received salary income of `5,00,000 (computed) in India and half of the services were
rendered in UK and half in India.
(Presume all the above incomes are computed incomes)
Compute his income presuming that he is NOR, NR and ROR.
Solution:
ROR NOR NR
1. Income received in India 1,50,000 1,50,000 1,50,000
Income accruing/arising abroad and received abroad 1,50,000 xxxxx xxxxx
Residential Status & Scope of Total Income 131

2. Income accruing/arising abroad and received abroad 1,00,000 xxxxx xxxxx


3. Income accruing/arising in India 3,00,000 3,00,000 3,00,000
4. Income accruing/arising in India 5,00,000 5,00,000 5,00,000
5. Income received in India 5,00,000 5,00,000 5,00,000
Total 17,00,000 14,50,000 14,50,000
Illustration 17: ABC partnership firm has an income of `3 lakhs in India and income accruing/arising
abroad and also received abroad `23 lakhs. It consists of two partners. Mr. X who is an active partner, is
staying outside India throughout the year. Mr. Y is a dormant partner and is staying in India throughout the
year.
Compute tax liability of the partnership firm in India for the assessment year 2019-20.
(b) Also compute tax liability of the firm if Mr. Y is also an active partner.
Solution: `
(a) Partnership firm is non-resident
Income from business/profession in India 3,00,000
Gross Total Income 3,00,000
Less: Deduction u/s 80C to 80U Nil
Total Income 3,00,000
Tax @ 30% + HEC @ 4% 93,600
(b) Partnership firm is resident
Income from business/profession 26,00,000
Gross Total Income 26,00,000
Less: Deduction u/s 80C to 80U Nil
Total Income 26,00,000
Tax @ 30% + HEC @ 4% 8,11,200
Illustration 18: Mr. X had following income during the previous year ended 31st March, 2019: `
(1) Salary received in India for three months (being computed income) 25,000
(2) Income from house property in India 18,000
(3) Interest on savings bank deposit in SBI, in India 4,000
(4) Amount brought into India out of the past-untaxed profits earned in Germany 20,500
(5) Income from business in Bangladesh, being controlled from India 12,542
(6) Dividends received in Belgium from French companies, out of which `2,500 were
remitted to India 23,150
You are required to compute his gross total income for the assessment year 2019-20, if he is a
(a) resident and ordinarily resident;
(b) not ordinarily resident; and
(c) non-resident.
Presume all the above income is computed income.
Solution:
ROR NOR NR
(1) Salary received in India 25,000 25,000 25,000
  Taxable on receipt basis
(2) Income from house property in India 18,000 18,000 18,000
   Income accruing/arising in India
(3) Interest on savings bank deposit in SBI, in India 4,000 4,000 4,000
   Income accruing/arising in India
(4) Past untaxed profits brought in India — — —
   Not an income of the previous year 2018-19 hence not taxable
(5) Income from business in Bangladesh being controlled from India 12,542 12,542 —
   Not taxable in case of non resident
(6) Dividend received in Belgium 23,150 — —
   Income accrued & received outside India
Residential Status & Scope of Total Income 132

Gross Total Income 82,692 59,542 47,000


Illustration 19: Mr. X earns the following income during the financial year 2018-19: `
(1) Income from house property in London, received in India 60,000
(2) Profits from business in Japan and managed from there (received in Japan) 9,00,000
(3) Dividend from foreign company, received in India 30,000
(4) Dividend from Indian company, received in England 50,000
(5) Profits from business in Kenya, controlled from India, Profits received in Kenya 3,00,000
(6) Profits from business in Delhi, managed from Japan 7,00,000
(7) Capital gains on transfer of shares of Indian companies, sold in USA and gains were
received there 2,00,000
(8) Pension from former employer in India, received in Japan 50,000
(9) Profits from business in Pakistan, deposited in bank there 20,000
(10) Profit on sale of asset in India but received in London 8,000
(11) Past untaxed profits of UK business of 2017-18 brought into India in 2018-19 90,000
(12) Interest on Government securities accrued in India but received in Paris 80,000
(13) Interest on USA Government securities, received in India 20,000
(14) Salary earned in Bombay, but received in UK 60,000
(15) Income from property in Paris, received there 1,00,000
(Presume all the above incomes are computed incomes)
Determine the gross total income of Mr. X if he is (i) resident and ordinarily resident, resident but not
ordinarily resident, non-resident in India during the financial year 2018-19.
Solution:
ROR NOR NR
(1) Income received in India 60,000 60,000 60,000
(2) Income accruing/arising and received outside India 9,00,000 — —
(3) Income received in India 30,000 30,000 30,000
(4) Income accruing in India but exempt under section 10(34) — — —
(5) Income accruing/arising and received outside India, but 3,00,000 3,00,000 —
business controlled from India
(6) Income accruing/arising in India 7,00,000 7,00,000 7,00,000
(7) Income accruing/arising in India 2,00,000 2,00,000 2,00,000
(8) Income accruing/arising in India 50,000 50,000 50,000
(9) Income accruing/arising and received outside India 20,000 — —
(10) Income accruing/arising in India 8,000 8,000 8,000
(11) Past untaxed profits — — —
(12) Income accruing/arising in India 80,000 80,000 80,000
(13) Income received in India 20,000 20,000 20,000
(14) Income accruing/arising in India 60,000 60,000 60,000
(15) Income accruing/arising and received outside India 1,00,000 — —
Gross Total Income 25,28,000 15,08,000 12,08,000
Illustration 20: ABC Pvt. Ltd., an Indian company has an income of `30 lakhs from a business in India.
This company has a business income of `12 lakhs from outside India. Out of which 7 lakhs were received in
India and balance outside India.
Compute tax liability of the Indian company for the assessment year 2019-20.
Solution: `
Income from business in India 30,00,000
Income from outside India 12,00,000
Income under the head Business/Profession 42,00,000
Gross Total Income 42,00,000
Less: Deductions u/s 80C to 80U Nil
Residential Status & Scope of Total Income 133

Total Income 42,00,000


Computation of Tax Liability
Tax on `42,00,000 @ 30% 12,60,000
Add: HEC @ 4% 50,400
Tax Liability 13,10,400
Note: Indian company is always considered to be resident in India and its incomes even earned and received
outside India shall be chargeable to tax in India.
Illustration 21: Mr. X, a foreign citizen (not being a person of Indian origin) came to India for the first time
on 2nd December, 2018 for a visit of 210 days. Mr. X had the following income during the previous year
ended 31st March, 2019: `
(1) Salary (computed) received in India for three months 1,00,000
(2) Income from house property in London (received there) 2,75,200
(3) Amount brought into India out of the past-untaxed profits earned in Germany 80,000
(4) Income from agriculture in Sri Lanka, received and invested there 12,300
(5) Income from business in Nepal, being controlled from India 35,000
(6) Income from house property in USA received in USA
(`76,000 is used in Canada for meeting the educational expenses of Mr. X’s daughter and
` 10,000 is later on remitted in India) 86,000
You are required to compute his total income for the assessment year 2019-20.
Solution:
Mr. X is a foreign citizen. He was in India during the previous year 2018-19 for 120 (30 + 31 + 28 + 31)
days. Thus, he does not satisfy the first condition of 182 days. The second condition is also not satisfied as
Mr. X came to India for first time during the previous year 2018-19.
Mr. X is therefore non–resident in India. The total income of Mr. X for the assessment year 2019-20 will be:
`
(1) Salary (computed) received in India for three months
Taxable on receipt basis 1,00,000
(2) Income from house property in London (received there)
   Not taxable as income is accruing & arising outside India and is also received outside India —
(3) Amount brought in India out of the past untaxed-profits earned in Germany
   Not taxable as it is not income —
(4) Income from agriculture in Sri Lanka being invested there
   Income accrued and received outside India —
(5) Income from business in Nepal, being controlled from India
    Not taxable in the case of non- resident —
(6) Income from house property in USA received in USA
(` 76,000 is used in Canada or meeting the educational expenses of Mr. X’s daughter and
` 10,000 is later on received in India)
   Income accrued and received outside India —
Gross Total Income 1,00,000
Less: Deduction u/s 80C to 80U Nil
Total Income 1,00,000
Illustration 22: Mr. X earns the following incomes during the financial year 2018-19. `
(1) Profits from a business in Japan, controlled from India,
(half of the profits received in India) 40,000
(2) Income from property in Bombay, received in UK 70,000
(3) Income from a property in USA, received there but subsequently remitted to India 2,00,000
(4) Income from property in USA, received there (`50,000 remitted in India) 80,000
(5) Salary received in India for services rendered in USA 50,000
(6) Income from profession in Paris, which was set up in India, received in Paris 80,000
(7) Interest from deposit with an Indian company, received in Japan 9,000
(8) Income from profession in Bombay received in Paris 30,000
Residential Status & Scope of Total Income 134

(9) Profits of business in Iran, deposited in a bank there, business controlled from India
(out of `4,00,000, ` 1,00,000 is remitted in India) 4,00,000
(10) Interest on German development bonds, half of which is received in India 10,000
(11) Income from property in Canada, one-fifth is received in India 50,000
(Presume all the above incomes are computed income i.e. all the exemptions and deductions have already
been allowed)
Determine the gross total income of Mr. X if he is (i) resident and ordinarily resident, (ii) resident but not
ordinarily resident, (iii) non-resident in India during the financial year 2018-19.
Solution:
ROR NOR NR
(1) Income accruing/arising outside India from a business controlled in 40,000 40,000 20,000
India, half of the income received in India
(2) Income accruing/arising in India 70,000 70,000 70,000
(3) Income accruing/arising outside India and received outside India 2,00,000 — —
(4) Income accruing/arising outside India and received outside India 80,000 — —
(5) Income received in India 50,000 50,000 50,000
(6) Income accruing/arising and received outside India, but profession 80,000 80,000 —
set up in India
(7) Income accruing/arising in India 9,000 9,000 9,000
(8) Income accruing/arising in India 30,000 30,000 30,000
(9) Income accruing/arising outside India and received outside India,
but business controlled from India 4,00,000 4,00,000 —
(10) Income accruing/arising outside India, half received outside India 10,000 5,000 5,000
and half in India
(11) Income accruing/arising outside India, 4/5th received outside India 50,000 10,000 10,000
and 1/5th in India
Gross Total Income 10,19,000 6,94,000 1,94,000
Illustration 23: Mr. X is a citizen of India and is employed in ABC Limited and getting salary `1,00,000
p.m. and he was transferred out of India on 01.09.2018 and he left India for first time from 01.09.2018 and
he visiting in India from 26.01.2019 to 15.02.2019 and salary for January 2019 was received in India and at
the time of departure he received 3 gifts `20,000 from 3 friends each and also a phone of `70,000.
He has agricultural income in India `4,00,000
Compute his tax liability for assessment year 2019-20.
Solution:
Mr. X shall be covered in special category so therefore his status shall be non-resident. `
(30 + 31 + 30 + 31 + 31 + 1 + 6 + 15) 175 days.
Income under the head Salary accruing / arising in India
1,00,000 x 5 5,00,000
Income received in India
1,00,000 x 1 1,00,000
Income under the head Salary 6,00,000
Income under the head Other Sources 60,000
Gross Total Income 6,60,000
Less: Deduction 80C to 80U Nil
Total Income 6,60,000
Agricultural Income 4,00,000
Computation of Tax Liability
Step 1. Tax on (6,60,000 + 4,00,000 = 10,60,000) at slab rate 1,30,500
Step 2. Tax on (4,00,000 + 2,50,000 = 6,50,000) at slab rate (42,500)
Deduct step 2 from step 1 88,000
Add: HEC 4% 3,520
Residential Status & Scope of Total Income 135

Tax Liability 91,520


Question 8: Explain meaning of income deemed to accrue or arise in India Section 9.
Answer:
Income deemed to the accruing/arising in India shall be taxable in all the three status i.e. ROR/NOR/NR.
As per section 9, the following incomes shall be deemed to be accruing / arising in India.
1. If any income has its source in India, such income shall be considered to be accruing / arising in India i.e.
employment/house property/business/profession/capital asset or any other source of income is in India but if
source is partly in India and partly outside India, income shall be accruing / arising in India only to the
extent the source is in India e.g. Mr. X is employed in Punjab National Bank and is posted in Delhi branch
on a salary of `1,00,000 p.m. In this case, his income shall be deemed to be accruing/arising in India but if
he is transferred to the London branch w.e.f 01.01.2019, his income accruing/arising in India shall be
`9,00,000 i.e. salary upto 31.12.2018 and the income which is accruing/arising abroad shall be `3,00,000
(i.e. salary from 01.01.2019 to 31.03.2019)
Business connection
If any person has business in India as well as outside India, it will be called business connection and in case
of such business, the income of the business deemed to accrue or arise in India shall be only such part of the
income as is reasonably attributable to the operations carried out in India and as per rule 10, assessing
officer shall have the powers to determine the extent upto which income is accruing/arising in India.
There will be a business connection if any non-resident has business outside India but has agent in India who
(a) habitually secures orders in India, for the non-resident.
(b) habitually maintains in India a stock of goods from which he regularly delivers goods on behalf of the
non-resident or
(c) habitually concludes contracts on behalf of the non-resident or plays the principal role leading to
conclusion of contracts and the contracts are in the name of non - resident or the contracts are for the transfer
of ownership or for granting of right to use property owned by that non- resident or the provision of services
by the non - resident.
Significant economic presence of a non-resident in India shall also be considered to be business connection
and SEP means -
(a) transaction in respect of any goods, services or property carried out by a non-resident in India including
provision of download of data or software in India, if the aggregate of payments arising from such
transaction or transactions during the previous year exceeds such amount as may be prescribed; or
(b) systematic and continuous soliciting of business activities or engaging in interaction with such number of
users as may be prescribed, in India through digital means:
Provided that the transactions or activities shall constitute significant economic presence in India, whether or
not,—
(i) the agreement for such transactions or activities is entered in India;
(ii) the non-resident has a residence or place of business in India; or
(iii) the non-resident renders services in India:
Provided further that only so much of income as is attributable to the
transactions or activities referred to in clause (a) or clause (b) shall be deemed to accrue or arise in India.’.
There is no business connection in the following three cases:
(a) If any non-resident has business outside India but such person is purchasing goods from India and do not
have any other activity in India, in this case there is no business connection but if such person has any other
activity in India, it will be considered to be business connection. e.g. Mr. X a non-resident has one shop in
New York for selling Indian goods and all these goods are purchased from India. In this case, there is no
business connection. However, if assessee is carrying out any other activity in India, it will be considered to
be business connection.
If in the above case the assessee has manufacturing unit in India, it will be considered to be a business
connection.
(b) If any non-resident has the business of running a news agency or of publishing newspapers,
magazines or journals etc. outside India, no income shall be deemed to accrue or arise in India to him
from activities which are confined to the collection of news and views in India for transmission out of India
but if newspaper etc. is being sold in India, there will be business connection or if there is telecasting or
Residential Status & Scope of Total Income 136

broadcasting of such news/views etc. in India, there will be business connection and income shall be taxable
to that extent.
(c) If any non-resident is doing shooting of any cinematograph film in India, there is no business
connection but if such film is being shown in India, there will be business connection.
2. If any person is holding shares of any Indian company, any capital gain on transfer of such shares shall be
considered to be income accruing/arising in India even if shares were sold outside India.
In case of shares of a foreign company, capital gains shall be accruing / arising in India if the value of the
shares is because of the assets located in India or because of business in India (the amendment is to overrule
the judgment in Vodafone case).
3. If any individual is a citizen of India and is an employee of the government and is posted outside India,
his salary income shall be accruing / arising in India e.g. Mr. X is citizen of India and is an IFS. He is posted
in Indian embassy in USA, in this case, his salary income shall be accruing/arising in India.
4. If any loan has been taken by the government from outside India, interest paid by the government shall be
considered to be income of the person who has received such interest and it is accruing / arising in India and
it do not matter whether loan was used in India or outside India. e.g. If Central Government has taken a loan
from an agency in USA, equivalent to Indian `1,000 lakh @ 10%, in this case, interest of `100 lakhs paid by
the Government to such agency shall be considered to be the income of such agency accruing/arising in
India.
If such loan has been taken by a person who is resident in India, interest income shall be accruing / arising in
India only if loan amount has been used in India but if loan amount has been utilized outside India it will be
accruing / arising abroad. E.g. ABC Ltd. an Indian company has taken a loan from an agency in USA and
the amount was utilized in USA. In this case, interest income shall be accruing/arising in USA but if loan
amount is used in India in any source, it will be accruing / arising in India
If such loan has been taken by a non-resident, interest income shall be accruing / arising in India only if loan
amount has been utilised in India in business/profession but if loan amount is utilised in any other source in
India or it has been used outside India, interest income shall be accruing / arising abroad. E.g. X Ltd. a non-
resident company has taken a loan from outside India and loan amount was utilized in India in house
property. In this case, interest paid by the company shall be income of the recipient accruing/arising abroad
but if loan amount was utilised in India in business/profession, interest income shall be considered to be
accruing/arising in India. The person receiving interest shall be liable to pay income tax on such income
even if such person do not have any Territorial Nexus with India i.e. such non-resident do not have a
residence or place of business or business connection in India
MAY – 2006 (2 Marks)
Mr. X, left for USA on 01.05.2018. He has not visited India thereafter. Mr. X borrows money from his
friend Mr. Y, who left India one week before Mr. X's departure, to the extent of `10 lakhs and buys shares in
X Ltd., an Indian company. Discuss the taxability of the interest charged @ 10% in Mr. Y's hands where the
same has been received in New York.
Answer: Stay of Mr. X and Mr. Y during the previous year 2018-19 is less than 60 days hence both of them
are non-residents as per section 6(1).
As per section 9, if any non-resident has taken loan from outside India and the loan was utilized in India in
any source other than business or profession, interest received by the person who has given the loan shall not
be considered to be accruing/arising in India and is not taxable in India. In the given case, loan amount was
invested in the shares of an Indian company hence interest received by Mr. Y shall not be considered to be
income accruing/arising in India.
5. If government has taken any patent right or any technical services from outside India and has paid royalty
or technical fee for such patent right etc., it will be considered to be income of the person who has received
it and it is accruing / arising in India even if the patent right etc. has been used outside India.
If such payment is being given by any resident or non-resident, it will be income of the recipient accruing /
arising in India only if such patent right etc. has been used in India otherwise it will be accruing / arising
abroad.
Royalty means amount payable in connection with patent, invention, model, design, formula, process, trade
marks etc.
Fees for Technical Services means any consideration for the rendering of Managerial, Technical or
Residential Status & Scope of Total Income 137

Consultancy Services.
If any income is accruing and arising in India relating to royalty or technical fees etc., it will be taxable in
India even if the person receiving income is non-resident and even if such non-resident do not have any
Territorial Nexus with India i.e. such non-resident do not have a residence or place of business or business
connection in India and also the non-resident has not rendered services in India.
6. If any person has received pension, it will be deemed to be accruing/arising in India if the employer is in
India. E.g. Mr. X is settled in Canada and is getting a pension of `30,000 p.m. from Punjab National Bank,
in this case his pension income shall be accruing/arising in India.
MAY-2012 (3 Marks)
Discuss the correctness or otherwise of the statement – “Income deemed to accrue or arise in India to a non-
resident by way of interest, royalty and fees for technical services is to be taxed irrespective of territorial
nexus”.
Answer: As per section 9, if any non-resident has provided any patent right or any managerial, technical
services and such patent right etc was used in India, in such cases any royalty or fee received by non-
resident shall be considered to be income accruing/arising in India and shall be taxable and it do not matter
that the non-resident do not have residence or place of business or business connection in India i.e. there is
no territorial nexus or non-resident has not rendered services in India. E.g. If Suzuki Incorporation of Japan
a non-resident company has provided technical know-how in Japan to Maruti Udyog Limited for use in
India and has received `300,00,000 in this case, such income is deemed to be accruing/arising in India and is
taxable in India even if Suzuki Incorporation do not have any Territorial Nexus with India i.e. the company
do not have place of residence or place of business in India. Similarly if any loan was given by a non-
resident to some other non-resident and such other non-resident has utilized loan amount in India in
business/profession, interest received by the non-resident shall be considered to be his income
accruing/arising in India even if such non-resident do not have any territorial nexus with India.
MAY – 2011 (3 Marks)
Miss Vivitha paid a sum of 5000 USD to Mr. Kulasekhara, a management consultant practicing in Colombo,
specializing in project financing. The payment was made in Colombo. Mr. Kulasekhara is a non-resident.
The consultancy related to a project in India with possible Ceylonese collaboration. Is this payment
chargeable to tax in India in the hands of Mr. Kulasekhara, since the services were used in India?
Answer: As per section 9, if any non-resident has provided any patent right or any managerial, technical
services and such patent right etc was used in India, in such cases any royalty or fee received by non-
resident shall be considered to be income accruing/arising in India and shall be taxable and it do not matter
that the non-resident do not have residence or place of business or business connection in India i.e. there is
no territorial nexus or non-resident has not rendered services in India. In the instant case, since the services
were utilized in India, the payment received by Mr. Kulasekhara, a non-resident, in Colombo is chargeable
to tax in his hands in India, as it is deemed to accrue or arise in India.
NOV – 2009 (4 Marks)
Determine the taxability of income of US based company ABC Ltd., in India on entering following
transactions during the financial year 2018-19:
(i) `5 lacs received from an Indian domestic company for providing technical know how in India.
(ii) `6 lacs from an Indian firm for conducting the feasibility study for the new project in Finland.
(iii) `4 lacs from a non-resident for use of patent for a business in India.
(iv) `8 lacs from a non-resident Indian for use of know how for a business in Singapore.
(v) `10 lacs for supply of manuals and designs for the business to be established in Singapore.
Explain the rate of tax applicable on taxable income for US based company, ABC Ltd., in India.
Answer: A non resident is chargeable to tax in India in respect of following incomes:
(i) Income received or deemed to be received in India.
(ii) Income accruing or arising or deemed to accrue or arise in India.
In view of the above provisions, taxability of income is determined in following manner:
S. No. Transaction details Amount (`)
(i) Amount received from an Indian domestic company for providing technical 5 Lacs
know how in India is taxable in India
Residential Status & Scope of Total Income 138

(ii) Conducting the feasibility study for the new project in Finland for the Indian firm Nil
is not taxable in India as it is for the business outside India.
(iii) Money received from a non resident for use of patent for a business in India is 4 Lacs
taxable in India
(iv) Money received from a non resident Indian for use of know-how for a business in Nil
Singapore is for the business outside India, therefore not taxable in India.
(v) Payment received for supply of manuals and designs for the business to be Nil
established in Singapore is not taxable in India.
Total Income in India 9 Lacs
The basic normal rate applicable for the US based company who is a foreign company is 40% In case the
taxable income is more than 1 crore but upto `10 crore in the previous year, the surcharge @ 2% is
applicable. The HEC is payable @ 4%.
Question 9: Explain income deemed to be received in India.
Answer: Such incomes is taxable in all the three status. Under section 7, employer’s contribution to
Recognised Provident Fund in excess of 12% of salary of the employee shall be considered to be income
deemed to be received in India. Similarly, interest on the provident fund balance in excess of 9.5% p.a. shall
be considered to be income deemed to be received in India.
As per provisions of employees provident fund and miscellaneous provisions Act,1952, every employee and
employer shall contribute to Recognised provident fund @ 12% of salary of employee and for this purpose
account shall be opened in the name of the employee with provident fund commissioner. Interest earned on
such amount shall be credited to the RPF. Employer’s contribution in excess of 12% of salary of employee
shall be taxable under the head salary. Interest on employer’s contribution in excess of 9.5% per annum shall
be taxable under the head salary. Interest on employees contribution in excess of 9.5% per annum shall be
taxable under the head other sources.
Deduction shall be allowed under section 80C from gross total income for employees contribution but
maximum deduction allowed shall be ` 1,50,000 (Details given under section 80C)
If any Seafarer (crew member of ship) is Non-resident and Income is accruing/arising abroad and his
income has been received directly in his bank account in India, such income shall not be taxable.

Illustration 24: Mrs. X is a citizen of India and is employed in ABC Ltd. in India and is getting salary of
`60,000 p.m. and she was transferred out of India w.e.f 01.09.2018 and for this purpose she left India on
01.09.2018 for the first time and she visited India from 27.12.2018 to 07.01.2019 and her salary for the
month of Dec’ 2018 was received in India. Employer and employee both have contributed @ 13% (each) of
salary to the recognized provident fund and during the year interest of `50,000 was credited to the
recognized provident fund @ 10% p.a.
Compute her total income and tax liability in India for assessment year 2019-20.
(b) Presume she was transferred w.e.f 01.11.2018 and she left India on 01.11.2018 for the first time.

Solution:
As per section 6(1), in this case, Mrs. X is covered in special category and her stay in India is less than 182
days hence she will be non-resident and her incomes taxable in India shall be
`
Income accruing/arising in India 3,00,000.00
60,000 x 5
Income received in India 60,000.00
60,000 x 1
Income deemed to be received in India
Employer contribution 7,200.00
(60,000 x 12) x 1% (13% - 12%)
Interest in excess of 9.5%
50,000 /10% x 0.5% = 2,500
Interest on employer contribution 1,250.00
2,500 /2
Residential Status & Scope of Total Income 139

(Interest on employee contribution i.e. `1,250


shall be taxable under the head Other Sources)
Gross Salary 3,68,450.00
Less: Standard Deduction u/s 16(ia) (40,000.00)
Income under the head Salary 3,28,450.00
Income under the head Other Sources 1,250.00
Gross Total Income 3,29,700.00
Less: Deduction u/s 80C (93,600.00)
Contribution to recognized provident fund
(60,000 x 12) x 13%
Total Income 2,36,100.00

Computation of Tax Liability


Tax on `2,36,100 at slab rate Nil
Tax Liability Nil

Solution (b):
In this case, Mrs. X is covered in special category and her stay in India is more than 182 days hence she will
be ROR and her incomes taxable in India shall be `
Income accruing/arising in India 4,20,000.00
60,000 x 7
Income received in India 60,000.00
60,000 x 1
Income accruing/arising abroad / received abroad 2,40,000.00
60,000 x 4
Income deemed to be received in India
Employer contribution 7,200.00
(60,000 x 12) x 1% (13% - 12%)
Interest in excess of 9.5%
50,000 /10% x 0.5% = 2,500
Interest on employer contribution 1,250.00
2,500 /2
(Interest on employee contribution i.e. `1,250
shall be taxable under the head Other Sources)
Gross Salary 7,28,450.00
Less: Deduction u/s 16(ia) (40,000.00)
Income under the head Salary 6,88,450.00
Income under the head Other Sources 1,250.00
Gross Total Income 6,89,700.00
Less: Deduction u/s 80C (93,600.00)
Contribution to recognized provident fund
(60,000 x 12) x 13%
Total Income 5,96,100.00

Computation of Tax Liability


Tax on `5,96,100 at slab rate 31,720.00
Add: HEC @ 4% 1,268.80
Tax Liability 32,988.80
Rounded off u/s 288B 32,990.00
Residential Status & Scope of Total Income 140

Any past untaxed profits shall not be considered to be the income of the current year in any status i.e.
ROR, NOR, NR.
Example
Mr. X had income of `3,00,000 in the year 2015-16 but he has not disclosed the income. It was detected in
the previous year 2018-19. In this case, it will not be considered to be income of 2018-19 in any status,
rather it will be considered to be income of the year 2015-16.
Illustration 25: Determine the taxability of the following incomes in the hands of a resident and ordinarily
resident, resident but not ordinarily resident, and non-resident for the A.Y. 2019-20 –
Particulars Amount (` )
(1) Interest on UK Development Bonds, 50% of interest received in India 10,000
(2) Income from a business in Chennai (50% is received in India) 20,000
(3) Profits on sale of shares of an Indian company received in London 20,000
(4) Dividend from British company received in London 5,000
(5) Profits on sale of plant at Germany 50% of profits are received in India 40,000
(6) Income earned from business in Germany which is controlled from Delhi (`40,000 is 70,000
received in India)
(7) Profits from a business in Delhi but managed entirely from London 15,000
(8) Income from property in London deposited in a Indian Bank at London, brought to 50,000
India
(9) Interest for debentures in an Indian company received in London. 12,000
(10) Fees for technical services rendered in India but received in London 8,000
(11) Profits from a business in Bombay managed from London 26,000
(12) Pension for services rendered in India but received in Burma 4,000
(13) Income from property situated in Pakistan received there 16,000
(14) Past foreign untaxed income brought to India during the previous year 5,000
(15) Income from agricultural land in Nepal received there and then brought to India 18,000
(16) Income from profession in Kenya which was set up in India, received there but spent 5,000
in India
(17) Gift received on the occasion of his wedding 20,000
(18) Interest on savings bank deposit in State Bank of India 10,000
(19) Income from a business in Russia, controlled from Russia 20,000
(20) Dividend from Reliance Petroleum Limited, an Indian Company 5,000
(21) Agricultural income from a land in Rajasthan 15,000

Solution: Computation of Gross Total Income for the A.Y.2019-20


Particulars Resident and Resident but not Non
ordinarily resident ordinarily resident resident
` ` `
(1) Interest on UK Development Bonds, 50% of 10,000 5,000 5,000
interest received in India
(2) Income from a business in Chennai (50% is 20,000 20,000 20,000
received in India)
(3) Profits on sale of shares of an Indian company 20,000 20,000 20,000
received in London
(4) Dividend from British company received in 5,000 - -
London
(5) Profits on sale of plant at Germany 50% of 40,000 20,000 20,000
profits are received in India
(6) Income earned from business in Germany 70,000 70,000 40,000
which is controlled from Delhi, out of which
`40,000 is received in India
(7) Profits from a business in Delhi but managed 15,000 15,000 15,000
Residential Status & Scope of Total Income 141

entirely from London


(8) Income from property in London deposited in 50,000 - -
a Bank at London, later on remitted to India
(9) Interest for debentures in an Indian company 12,000 12,000 12,000
received in London.
(10) Fees for technical services rendered in India 8,000 8,000 8,000
but received in London
(11) Profits from a business in Bombay managed 26,000 26,000 26,000
from London
(12) Pension for services rendered in India but 4,000 4,000 4,000
received in Burma
(13) Income from property situated in Pakistan 16,000 - -
received there
(14) Past foreign untaxed income brought to India - - -
during the previous year
(15) Income from agricultural land in Nepal 18,000 - -
received there and then brought to India
(16) Income from profession in Kenya which was 5,000 5,000 -
set up in India, received there but spent in India
(17) Gift received on the occasion of his wedding - - -
[not an income]
(18) Interest on savings bank deposit in State 10,000 10,000 10,000
Bank of India
(19) Income from a business in Russia, controlled 20,000 - -
from Russia
(20) Dividend from Reliance Petroleum Limited, - - -
an Indian Company [it is exempt u/s 10(34)]
(21) Agricultural income from a land in Rajasthan - - -
[it is exempt u/s 10(1)]
Gross Total Income 3,49,000 2,15,000 1,80,000
Residential Status & Scope of Total Income 142

PRACTICE PROBLEMS
TOTAL PROBLEMS 24
Problem 1 TO 10
Determine residential status of Mr. X for the assessment year 2019-20, who stays in India during various
financial years asunder:
Previous 1 2 3 4 5 6 7 8 9 10
Years
2018-19 65 183 181 69 300 70 72 95 180 93
2017-18 91 90 87 110 97 99 94 92 91 90
2016-17 190 78 98 91 103 104 101 100 99 80
2015-16 89 120 189 196 110 98 97 96 95 90
2014-15 87 91 92 93 94 95 94 93 92 100
2013-14 86 99 92 95 99 100 101 100 99 90
2012-13 84 66 93 94 365 210 209 208 207 80
2011-12 105 210 91 93 — 0 91 92 91 90
2010-11 110 110 92 92 362 300 200 100 — 100
2009-10 112 94 93 91 10 99 88 77 66 110
2008-09 100 96 91 90 310 100 99 92 94 120
2007-08 91 199 90 89 210 92 94 96 98 130
2006-07 94 81 89 8 92 80 70 60 50 100
2005-06 97 82 88 87 88 55 65 75 85 80
2004-05 99 83 87 86 84 40 50 60 70 60
Answer = (1) ROR; (2) ROR; (3) ROR; (4) ROR; (5) ROR; (6) NOR; (7) ROR; (8) ROR; (9) ROR; (10)
NR

Problem 11.
Mr. X, a citizen of USA, has come to India for the first time on 01.07.2014. The particulars of his arrival and
departure are as given below:
Date of arrival Date of departure
01.07.2014 11.12.2014
27.03.2015 21.07.2015
10.09.2015 01.03.2016
01.01.2017 23.09.2017
01.02.2018 01.07.2018
11.02.2019 ——
Determine his residential status for various years.
Answer = 2014-15 – Non-Resident (NR)
2015-16 – Resident but not ordinarily resident (NOR)
2016-17 – Resident but not ordinarily resident (NOR)
2017-18 – Resident but not ordinarily resident (NOR)
2018-19– Resident and ordinarily resident (ROR)

Problem 12.
Mr. X, a citizen of U.K., has come to India for the first time on 01.07.2014. The particulars of his arrival and
departure are as given below:
Date of arrival Date of departure
01.07.2014 07.09.2014
01.01.2015 08.03.2015
11.07.2015 20.09.2015
10.02.2016 09.05.2016
Residential Status & Scope of Total Income 143

01.01.2017 20.05.2017
11.03.2018 21.06.2018
27.03.2019 ——
Determine his residential status for various years.
Answer = 2014-15 – Non-Resident (NR)
2015-16 – Non-Resident (NR)
2016-17 – Non-Resident (NR)
2017-18 – Resident but not ordinarily resident (NOR)
2018-19 – Resident but not ordinarily resident (NOR)

Problem 13.
Mr. X goes out of India every year for 274 days.
Determine his residential status for the previous year 2018-19.
Answer = Resident but not ordinarily resident (NOR)

Problem 14.
Mr. X, a citizen of Japan, has come to India for the first time on 01.10.2018 for 200 days.
Determine his residential status for the assessment year 2019-20.
Answer = Resident but not ordinarily resident (NOR)

Problem 15.
Mr. X, a citizen of U.K. came to India for the first time on 01.07.2008 in connection with his employment.
He left India on 01.11.2017 for taking up a job in USA. He again came to India on 01.01.2019 on a visit and
left India on 01.03.2019.
Determine his residential status for the assessment year 2019-20.
Answer = Resident and ordinarily resident (ROR)

Problem 16.
Mr. X, a German citizen, came to India on 23.05.2017 and left India on 30.05.2018.
Determine his residential status for the assessment year 2018-19, 2019-20.
Answer = Assessment Year 2018-19: Resident but not ordinarily resident (NOR)
Assessment Year 2019-20: Non- Resident (NR)

Problem 17.
Mr. X, a citizen of India, is employed in Soliton Technologies, an Indian company. His employer has
transferred him to his branch in Japan. Mr. X left India on 29.09.2018 for his new posting in Japan.
Determine his residential status for the assessment year 2019-20.
Prior to this, Mr. X was posted outside India for 11 months in the previous year 2013-14 and for 10.5
months in the year 2009-10.
Answer = Resident and ordinarily resident (ROR)

Problem 18.
Dr. Reddy’s Labs is an Indian company and has borrowed funds from Bank of America, New York for
investing it in one of its projects in USA. In this case, interest paid by Dr. Reddy’s Labs to Bank of America
shall be accruing/arising __________.
Answer = Outside India

Problem 19.
Calculate taxable income of an individual on the basis of the following informations, for the assessment year
2019-20, if he is:
(a) Ordinarily Resident
(b) Not Ordinarily Resident; and
(c) Non-Resident
Residential Status & Scope of Total Income 144

`
(i) Profit from business in Japan received in India. 10,000
(ii) Income from agriculture in Pakistan – it is all spent on the education of children there 5,000
(iii) Income accrued in India but received in England 10,000
(iv) Income from house property in Pakistan deposited in a bank there 2,000
(v) Profits of business in America deposited in a bank there. This business is controlled from India 50,000
(vi) Profits earned from business in Meerut 12,000
(vii) Past untaxed foreign income brought into India during the previous year 10,000
(Presume that all the incomes are computed incomes)
Answer: Taxable Income: Resident and ordinarily resident (ROR): `89,000;
Resident but not ordinarily resident (NOR) : `82,000;
Non-Resident (NR) : `32,000

Problem 20.
Mr. X earns the following income during the previous year 2018-19.
Compute his gross total income for assessment year 2019-20 if he is
(i) resident and ordinarily resident.
(ii) resident but not ordinarily resident.
(iii) non-resident. `
(1) Income from agricultural land in Bhutan received there and remitted to India later on 40,000
(2) Dividend from foreign company, received in India 50,000
(3) Pension for service rendered in India, but received in Paris 15,000
(4) Past untaxed profits of 2017-18 brought into India in 2018-19 50,000
(5) Profits from business in Paris, deposited in bank there 1,00,000
(6) Profits from business in Canada, controlled from India, profits received there 1,75,000
(7) Interest on saving bank deposit in Punjab National Bank, in India 20,000
(8) Capital gain on sale of a house in Delhi, amount received in Paris 2,00,000
Answer: Resident and ordinarily resident (ROR): `6,00,000
Resident but not ordinarily resident (NOR): `4,60,000
Non-Resident (NR): `2,85,000

Problem 21.
Mr. X earns the following income during the previous year 2018-19.
Compute his Gross total income for assessment year 2019-20 if he is
(i) resident and ordinarily resident.
(ii) resident but not ordinarily resident.
(iii) non-resident. `
(1) Dividend from an Indian company, received in Japan 60,000
(2) Profit on sale of machinery in India, but received in Japan 1,20,000
(3) Profits from business in Bombay, managed from Japan 2,25,000
(4) Profits from business in Japan, managed from there, received there 1,45,000
(5) Income from house property in India 1,50,000
(6) Income from property in Japan and received there 1,50,000
(7) Income from agriculture in Japan being invested there 75,000
(8) Fees for technical services rendered in India but received in Japan 65,000
(9) Interest on Government securities accrued in India but received in Japan 80,000
(10) Interest on Japan Government securities, received in India 40,000
(Presume that all the incomes are computed incomes)
Answer: Resident and ordinarily resident (ROR): `10,50,000
Resident but not ordinarily resident (NOR): `6,80,000
Non-Resident (NR): `6,80,000
Residential Status & Scope of Total Income 145

Problem 22.
Mr. X earns the following incomes during the financial year 2018-19. `
(1) Profits from a business in Japan, controlled from India, half of the profits received in India 60,000
(2) Income from agriculture in Nepal, brought to India 10,000
(3) Income u/h house property in Bombay, received in UK 1,70,000
(4) Income u/h house property in USA, received there but subsequently remitted to India 2,20,000
(5) Income u/h house property in USA, received there (`50,000 remitted in India) 1,00,000
(6) Salary received in India for services rendered in USA 60,000
(7) Income from profession in Paris, which was set up in India, received in Paris 90,000
(8) Interest from deposit with an Indian company, received in Japan 19,000
(9) Income from profession in Bombay received in Paris 39,000
(10) Profits of business in Iran, deposited in a bank there, business controlled from India
(out of `4,80,000, ` 1,00,000 is remitted in India) 4,80,000
(11) Interest on German development bonds, half of which is received in India 12,000
(12) Income under the head house property in Canada, one-fifth is received in India 50,000
(Presume all the above incomes are computed income i.e. all the exemptions and deductions have already
been allowed)
Determine the gross total income of Mr. X if he is
(i) resident and ordinarily resident,
(ii) resident but not ordinarily resident,
(iii) non-resident in India during the financial year 2018-19.
Answer: Resident and ordinarily resident (ROR): `13,10,000
Resident but not ordinarily resident (NOR): `9,34,000
Non-Resident (NR): `3,34,000

Problem 23.
Mr. X is a citizen of India and is employed in ABC Ltd and is getting a salary of `60,000 p.m. He purchased
one building in India on 1st May, 2018 for `10,00,000 and its market value is `22,00,000 and value for the
purpose of charging stamp duty is `13,00,000. He purchased gold for `8,00,000 and its market value is
`11,00,000. He was transferred out of India w.e.f. 1st Sept, 2018 and he left India on 1st Sept, 2018 and one
of his friend gifted him one colour TV on this occasion, market value `1,00,000.
He has gone out of India in earlier years also.
P.Y. 2017-18 100 days
P.Y. 2016-17 200 days
He visited India from 01.02.2019 to 14.02.2019 and salary for January, 2019 was received in India. He has
taken a loan from outside India on 01.01.2019 and amount was invested in shares of an Indian Company and
received dividend of `30,000 outside India.
He has purchased one house property in USA in December 2018 and sold in March 2019 and there were
short term capital gain of `6,00,000 and the amount was received in USA.
Compute his tax liability for the A.Y.2019-20.
Answer: Tax Liability: `1,00,360

Problem 24.
Mrs. X is employed in ABC Ltd in India and she is an American citizen and is getting a salary of `2,00,000
p.m. She purchased shares of a foreign Company on 01.07.2018 and received dividend of `3,00,000 on
01.08.2018 in India and again dividend of `2,00,000 on 01.03.2019 in USA.
She received gift of one painting in India from her friend on 01.07.2018 and its market value is `49,000 and
she also received gift in cash of `49,000 from the same friend and gift of immovable property with value for
the purpose of charging stamp duty is `51,000 from the same friend.
She purchased UK Development bond and interest equivalent of `2,00,000 was received in USA.
She visited USA for 182 days during P.Y.2018-19.
In the earlier year her stay in India was
P.Y. 2017-18 110 days
Residential Status & Scope of Total Income 146

P.Y. 2016-17 120 days


P.Y. 2015-16 300 days
P.Y. 2014-15 182 days
P.Y. 2013-14 185 days
P.Y. 2012-13 200 days
P.Y. 2011-12 300 days
Compute her tax liability in India for the A.Y. 2019-20.
Answer: Tax Liability: `7,75,630
Residential Status & Scope of Total Income 147

SOLUTIONS
TO

PRACTICE PROBLEMS
Solution 1:
2018-19 Resident
2017-18 Resident
2016-17 Resident
2015-16 Non-Resident
2014-15 Resident
2013-14 Resident
2012-13 Resident
2011-12 Resident
2010-11 Resident
2009-10 Resident
2008-09 Resident
Total stay in 7 years preceding the relevant previous year is 732 days.
Since the assessee is not able to comply with any of the conditions of section 6(6)(a), as listed below, he will
be considered to be ROR.
1. He is non resident in India in at least nine out of ten previous years preceding that year.
or
2. He has during the seven previous years preceding that year been in India for a period of 729 days or
less.

Solution 2:
2018-19 Resident
2017-18 Resident
2016-17 Resident
2015-16 Resident
2014-15 Resident
2013-14 Resident
2012-13 Resident
2011-12 Resident
2010-11 Resident
2009-10 Resident
2008-09 Resident
Total stay in 7 years preceding the relevant previous year is 754 days.
Since the assessee is not able to comply with any of the conditions of section 6(6)(a), as listed below, he will
be considered to be ROR.
1. He is non resident in India in at least nine out of ten previous years preceding that year.
or
2. He has during the seven previous years preceding that year been in India for a period of 729 days or
less.

Solution 3:
2018-19 Resident
2017-18 Resident
Residential Status & Scope of Total Income 148

2016-17 Resident
2015-16 Resident
2014-15 Resident
2013-14 Resident
2012-13 Resident
2011-12 Resident
2010-11 Non-Resident
2009-10 Non-Resident
2008-09 Non-Resident
Total stay in 7 years preceding the relevant previous year is 742 days.
Since the assessee is not able to comply with any of the conditions of section 6(6)(a), as listed below, he will
be considered to be ROR.
1. He is non resident in India in at least nine out of ten previous years preceding that year.
or
2. He has during the seven previous years preceding that year been in India for a period of 729 days or
less.

Solution 4:
2018-19 Resident
2017-18 Resident
2016-17 Resident
2015-16 Resident
2014-15 Resident
2013-14 Resident
2012-13 Resident
2011-12 Non-Resident
2010-11 Non-Resident
2009-10 Non-Resident
2008-09 Non-Resident
Total stay in 7 years preceding the relevant previous year is 772 days.
Since the assessee is not able to comply with any of the conditions of section 6(6)(a), as listed below, he will
be considered to be ROR.
1. He is non resident in India in at least nine out of ten previous years preceding that year.
or
2. He has during the seven previous years preceding that year been in India for a period of 729 days or
less.

Solution 5:
2018-19 Resident
2017-18 Resident
2016-17 Resident
2015-16 Resident
2014-15 Resident
2013-14 Resident
2012-13 Resident
2011-12 Non-Resident
2010-11 Resident
2009-10 Non-Resident
2008-09 Resident
Total stay in 7 years preceding the relevant previous year is 868 days.
Since the assessee is not able to comply with any of the conditions of section 6(6)(a), as listed below, he will
be considered to be ROR.
1. He is non resident in India in at least nine out of ten previous years preceding that year.
Residential Status & Scope of Total Income 149

or
2. He has during the seven previous years preceding that year been in India for a period of 729 days or
less.

Solution 6:
2018-19 Resident
2017-18 Resident
2016-17 Resident
2015-16 Resident
2014-15 Resident
2013-14 Resident
2012-13 Resident
2011-12 Non-Resident
2010-11 Resident
2009-10 Non-Resident
2008-09 Non-Resident
Total stay in 7 years preceding the relevant previous year is 706 days.
Since the assessee is able to comply with any of the conditions of section 6(6)(a), as listed below, he will be
considered to be NOR.
1. He is non resident in India in at least nine out of ten previous years preceding that year.
or
2. He has during the seven previous years preceding that year been in India for a period of 729 days or
less.

Solution 7:
2018-19 Resident
2017-18 Resident
2016-17 Resident
2015-16 Resident
2014-15 Resident
2013-14 Resident
2012-13 Resident
2011-12 Resident
2010-11 Resident
2009-10 Non-Resident
2008-09 Non-Resident
Total stay in 7 years preceding the relevant previous year is 787 days.
Since the assessee is not able to comply with any of the conditions of section 6(6)(a), as listed below, he will
be considered to be ROR.
1. He is non resident in India in at least nine out of ten previous years preceding that year.
or
2. He has during the seven previous years preceding that year been in India for a period of 729 days or
less.

Solution 8:
2018-19 Resident
2017-18 Resident
2016-17 Resident
2015-16 Resident
2014-15 Resident
2013-14 Resident
2012-13 Resident
2011-12 Resident
Residential Status & Scope of Total Income 150

2010-11 Non-Resident
2009-10 Non-Resident
2008-09 Non-Resident
Total stay in 7 years preceding the relevant previous year is 781 days.
Since the assessee is not able to comply with any of the conditions of section 6(6)(a), as listed below, he will
be considered to be ROR.
1. He is non resident in India in at least nine out of ten previous years preceding that year.
or
2. He has during the seven previous years preceding that year been in India for a period of 729 days or
less.

Solution 9:
2018-19 Resident
2017-18 Resident
2016-17 Resident
2015-16 Resident
2014-15 Resident
2013-14 Non-Resident
2012-13 Resident
2011-12 Non-Resident
2010-11 Non-Resident
2009-10 Non-Resident
2008-09 Non-Resident
Total stay in 7 years preceding the relevant previous year is 774 days.
Since the assessee is not able to comply with any of the conditions of section 6(6)(a), as listed below, he will
be considered to be ROR.
1. He is non resident in India in at least nine out of ten previous years preceding that year.
or
2. He has during the seven previous years preceding that year been in India for a period of 729 days or
less.

Solution 10:
Mr. X is in India for 60 days or more in 2018-19 but for less than 365 days in 4 years immediately preceding
2018-19, so he is non-resident in 2018-19.

Solution 11:
Stay of Mr. X in various years is as given below.
In P.Y. 2014-15
{July – 31, August – 31, September – 30, October – 31, November – 30, December – 11, March – 5}
Days of stay in India are 169, so Mr. X is non-resident.
In P.Y. 2015-16
{April – 30, May – 31, June – 30, July – 21, September – 21, October – 31, November – 30, December – 31,
January – 31, February – 29, March – 1}
Days of stay in India are 286. So, he is resident and also he is non-resident in at least 9 years out of 10 years
preceding the relevant previous year, hence he is NOR.
In P.Y. 2016-17
{January – 31, February – 28, March – 31}
Days of stay in India are 90. So, he is resident and also he is non-resident in at least 9 years out of 10 years
preceding the relevant previous year, hence he is NOR.
In P.Y. 2017-18
{April – 30, May – 31, June – 30, July – 31, August – 31, September – 23, February – 28, March – 31}
Days of stay in India are 235. So, he is resident and also his stay during seven years preceding the relevant
previous year is 729 days or less, hence he is NOR.
Residential Status & Scope of Total Income 151

In P.Y. 2018-19
{April – 30, May – 31, June – 30, July – 1, February – 18, March – 31}
Days of stay in India are 141 and during the previous 4 years his stay is for 365 days or more so he is
resident and also he is ROR because he is not able to fulfil any of the conditions of section 6(6)(a). i.e.
1. He is non resident in India in at least nine out of ten previous years preceding that year.
or
2. He has during the seven previous years preceding that year been in India for a period of 729 days or
less.
Hence he is ROR .

Solution 12:
In P.Y. 2014-15
{July – 31, August – 31, September – 7, January – 31, February – 28, March – 8}
Days of stay in India are 136, so Mr. Daniel is non-resident.
In P.Y. 2015-16
{July – 21, August – 31, September – 20, February – 20, March – 31}
Days of stay in India are 123, so, he is non-resident.
In P.Y. 2016-17
{April – 30, May – 9, January – 31, February – 28, March – 31}
Days of stay in India are 129, so, he is non-resident.
In P.Y. 2017-18
{April – 30, May – 20, March – 21}
Days of stay in India are 71 and also he stays for 365 days or more during 4 years preceding the relevant
previous year and also he is able to comply with at least one of the conditions of section 6(6)(a) as given
below.
1. He is non resident in India in at least nine out of ten previous years preceding that year.
or
2. He has during the seven previous years preceding that year been in India for a period of 729 days or
less.
Hence he is NOR.
In P.Y. 2018-19
{April – 30, May – 31, June – 21, March – 5}
Days of stay in India are 87 and during the previous 4 years his stay is more than 365 days. So he is resident
but not ordinarily resident because he is able to fulfill at least one of the two condition given u/s 6(6)(a).

Solution 13:
Since he is out of India every year for 274 days so his days of stay in India are –
In 2018-19 91 Days
In 2017-18 91 Days
In 2016-17 91 Days
In 2015-16 92 Days
In 2014-15 91 Days
So his stay in India during the seven years immediately preceding the relevant previous year is less than 729
days, so he is resident but not ordinarily resident.

Solution 14:
Days of stay in India in P.Y. 2018-19 are 182.
{October – 31, November – 30, December – 31, January – 31, February – 28, March – 31}
So he is resident and also he will be able to comply with at least one of the conditions of section 6(6)(a) as
given below.
1. He is non resident in India in at least nine out of ten previous years preceding that year.
or
2. He has during the seven previous years preceding that year been in India for a period of 729 days or
Residential Status & Scope of Total Income 152

less.
Hence he is NOR.

Solution 15:
His days of stay in India are as under –
In P.Y. 2008-09 274 days
{July – 31, August – 31, September – 30, October – 31, November – 30, December – 31, January – 31,
February – 28, March – 31}
In P.Y. 2009-10 365
In P.Y. 2010-11 365
In P.Y. 2011-12 366
In P.Y. 2012-13 365
In P.Y. 2013-14 365
In P.Y. 2014-15 365
In P.Y. 2015-16 366
In P.Y. 2016-17 365
In P.Y. 2017-18 215
{April – 30, May – 31, June – 30, July – 31, August – 31, September – 30, October – 31, November – 1}
In P.Y. 2018-19 60
{January – 31, February – 28, March – 1}
He is resident in 2018-19 but he is not able to comply with any of the conditions of section 6(6)(a) hence he
is resident and ordinarily resident.

Solution 16:
His days of stay in India in year 2017-18 are 313.
{May – 9, June – 30, July – 31, August – 31, September – 30, October – 31, November – 30, December –
31, January – 31, February – 28, March – 31}
So he is resident and he is also able to comply with one of the condition of section 6(6)(a) hence he will be
considered to be resident but not ordinarily resident.
His days of stay in India in 2018-19 are 60.
{April – 30 and May – 30}
So he is non–resident in the year 2018-19.

Solution 17:
His days of stay in India during 2018-19 are 182.
{April – 30, May – 31, June – 30, July – 31, August – 31, September – 29}
So Mr. X is resident in previous year 2018-19 and also he is not able to comply with any of the conditions of
section 6(6)(a) hence he will be considered to be ROR.
Solution 18:
It will be accruing arising abroad because if any loan has been taken by a person resident in India from
outside India then interest income shall be accruing arising in India only if such resident has utilized the loan
amount in India.

Solution 19:
Particulars ROR NOR NR
(i) Income accruing/arising outside India but received in India 10,000 10,000 10,000
(ii) Income accruing/arising outside India and also received abroad. 5,000 -------- --------
(iii) Income accruing/arising in India 10,000 10,000 10,000
(iv) Income accruing/arising outside India and also received abroad. 2,000 -------- --------
(v) Income accruing/arising outside India and also received outside India but 50,000 50,000 --------
from a business controlled from India
(vi) Income accruing/arising in India 12,000 12,000 12,000
Residential Status & Scope of Total Income 153

(vii) Past profits -------- ------- --------


Taxable Income 89,000 82,000 32,000
Solution 20:
Particulars ROR NOR NR
(1) Income accruing/arising outside India and received outside India 40,000 ------- -------
(2) Income received in India 50,000 50,000 50,000
(3) Income accruing/arising in India 15,000 15,000 15,000
(4) Past profits ------- ------- -------
(5) Income accruing/arising and received outside India 1,00,000 ------- -------
(6) Income accruing/arising and received outside India, but business 1,75,000 1,75,000 -------
controlled from India
(7) Income deemed to be accruing/ arising in India 20,000 20,000 20,000
(8) Income deemed to be accruing/ arising in India 2,00,000 2,00,000 2,00,000
Gross Total Income 6,00,000 4,60,000 2,85,000
Solution 21:
Particulars ROR NOR NR
(1) Income accruing/arising in India but exempt u/s 10(34) -------- -------- --------
(2) Income accruing /arising in India 1,20,000 1,20,000 1,20,000
(3) Income accruing/arising in India 2,25,000 2,25,000 2,25,000
(4) Income accruing/arising and received outside India 1,45,000 ------- -------
(5) Income accruing/arising in India 1,50,000 1,50,000 1,50,000
(6) Income accruing/arising outside India and received outside India 1,50,000 ------- -------
(7) Income accruing/arising outside India and received outside India 75,000 ------- -------
(8) Income accruing/arising in India 65,000 65,000 65,000
(9) Income accruing/arising in India 80,000 80,000 80,000
(10) Income received in India 40,000 40,000 40,000
Gross Total Income 10,50,000 6,80,000 6,80,000
Solution 22:
ROR NOR NR
(1) Income accruing/arising outside India from a business controlled in 60,000 60,000 30,000
India, half of the income received in India
(2) Income accruing/arising outside India and received outside India 10,000 ------ ------
(3) Income accruing/arising in India 1,70,000 1,70,000 1,70,000
(4) Income accruing/arising outside India and received outside India 2,20,000 ------ ------
(5) Income accruing/arising outside India and received outside India 1,00,000 ------ ------
(6) Income received in India 60,000 60,000 60,000
(7) Income accruing/arising and received outside India, but profession 90,000 90,000 ------
set up in India
(8) Income accruing/arising in India 19,000 19,000 19,000
(9) Income accruing/arising in India 39,000 39,000 39,000
(10) Income accruing/arising outside India and received outside India, 4,80,000 4,80,000 ------
but business controlled from India
(11) Income accruing/arising outside India, half received outside India 12,000 6,000 6,000
and half in India
(12) Income accruing/arising outside India, 4/5th received outside India 50,000 10,000 10,000
and 1/5th in India
Gross Total Income 13,10,000 9,34,000 3,34,000
Residential Status & Scope of Total Income 154

Solution 23:
Since Mr. X is covered in special category and will be resident, if his stay in India in relevant previous year
is 182 days or more, hence Mr. X is a non–resident as his stay in India is less than 182 days and his income
taxable in India shall be `
Income under the head Salary
Income accruing/arising in India 3,00,000.00
(60,000 x 5)
Income received in India 60,000.00
(60,000 x 1)
Gross Salary 3,60,000.00
Less: Standard Deduction u/s 16(ia) (40,000.00)
Gross Salary 3,20,000.00
Income under the head Other Sources
Gift of gold (`11,00,000 – `8,00,000) 3,00,000.00
Gift of building (`13,00,000 – `10,00,000) 3,00,000.00
Income under the head Other Sources 6,00,000.00
Gross Total Income 9,20,000.00
Less: Deduction u/s 80C to 80U Nil
Total Income 9,20,000.00
Computation of Tax Liability
Tax on `9,20,000 at slab rate 96,500.00
Add: HEC @ 4% 3,860.00
Tax Liability 1,00,360.00
Note: STCG is received in USA is not taxable in India as the assessee is a non-resident.
Solution 24:
In this case, Mrs. X stays in India for more than 182 days during the previous year 2018-19 and also she is
not able to comply with any of the conditions of section 6(6)(a), she will be considered to be ROR.
Her incomes taxable in India shall be `
Income under the head Salary
Income accruing/arising in India 24,00,000.00
(2,00,000 x 12)
Gross Salary 24,00,000.00
Less: Standard deduction u/s 16(ia) (40,000.00)
Income under the head Salary 23,60,000.00
Income under the head Other Sources
Dividend from foreign company 3,00,000.00
(Received in India)
Gift from friend (immovable property) 51,000.00
Dividend from foreign company 2,00,000.00
(Received in USA)
Interest from UK Development bond 2,00,000.00
(Received in USA)
Income under the head Other Sources 7,51,000.00
Gross Total Income 31,11,000.00
Less: Deduction u/s 80C to 80U Nil
Total Income 31,11,000.00
Computation of Tax Liability
Tax on `31,11,000 at slab rate 7,45,800.00
Add: HEC @ 4% 29,832.00
Tax Liability 7,75,632.00
Rounded off u/s 288B 7,75,630.00
Residential Status & Scope of Total Income 155

EXAMINATION QUESTIONS
NOV – 2017
Question 2(a) (5 Marks)
DAISY Ltd., a foreign company, incorporated in USA and engaged in the Manufacturing and distribution of
diamonds, set up a branch office in India in June 2018.The branch office was required to purchase uncut and
unassorted diamonds from the dealers of Mumbai and export them to USA.
Out of 20 shareholders of DAISY Ltd., 12 shareholders are non-resident in India. All the major decisions
were taken through Board Meetings held at USA.
(i) Determine the residential status of DAISY Ltd. for the Assessment Year 2019-20.
(ii) Discuss the tax treatment of profit from export business.

Answer:
(i) As per section 6(3), A company would be resident in India if
(a) it is an Indian Company; or
(b) its place of effective management in that year is in India.
In the given case, DAISY Ltd. a foreign company therefore it would be resident in India if its place of
effective management in that year is in India.
Section 6(3) defines “place of effective management” to mean a place where key management and
commercial decisions that are necessary for the conduct of the business of an entity as a whole are, in
substance made. In the case of DAISY Ltd., its place of effective management for P.Y.2018-19 is not in
India, since the significant management and commercial decisions are, in substance, made by the Board of
Directors outside India in USA.
Hence, DAISY Ltd, being a foreign company is a non-resident for A.Y.2019-20, since its place of effective
management is outside India in the P.Y.2018-19.

(ii) If any non - resident is purchasing goods from India for the purpose of export, such income shall not be
accruing/arising in India, hence it is not taxable in India.

MAY – 2017
Question 2(a) (i) (4 Marks)
During the last four years preceding the financial year 2018-19, Mr. Damodhar, a citizen of India, was
present in India for 430 days. During the last seven previous years preceding the previous year 2018-19, he
was present in India for 830 days.
Mr. Damodhar is a member of crew of a Dubai bound Indian ship in the international waters, which left
Kochi port in Kerala, on 12th August, 2018.
Following details are made available to you for the previous year 2018-19:

Particulars Date

Date entered into the continuous discharge certificate in respect of joining the 12th August,2018
ship by Mr. Damodhar.

Date entered into the continuous discharge certificate in respect of signing off 21st January,2019
the ship by Mr. Damodhar.

In May, 2018 he had gone out of India to Singapore and Malaysia on a private tour for a continuous period
of 29 days.
You are required to determine the residential status of Mr. Damodhar for the previous year 2018-19.
Residential Status & Scope of Total Income 156

Answer:
As per section 6(1), Mr. Damodhar is covered in special category.

Period of stay in India shall exclude time period from 12th August 2018 to 21st January, 2019 and also 29
days for which he stays out of India.

His stay in India shall be 173 days [April – 30 + May – 2 + June – 30 + July – 31 + August – 11 + January –
10 + February – 28 + March – 31]

Since his period of stay in India during the P.Y.2018-19 is less than 182 days, he is a non-resident for
A.Y.2019-20.

Note - Since the residential status of Mr. Damodhar is “non-resident” for A.Y.2019 –20 consequent to his
number of days of stay in P.Y.2018–19 being less than 182 days, his period of stay in the earlier previous
years become irrelevant.

Question 4(a) (4 Marks)


A Korean Company Damjung Ltd. entered in to the following transactions during the financial year 2018-
19:
(a) Received `20 lakhs from a non-resident for use of patent for a business in India.
(b) Received `15 lakhs from a non-resident Indian for use of know-how for a business in Sri Lanka and this
amount was received in Japan. [Assume that the above amount is converted/stated in Indian Rupees].
(c) Received `7 lakhs from RR Co. Ltd., an Indian company for providing technical know-how in India.
(d) Received `5 lakhs from R & Co. Mumbai for conducting the feasibility study for a new project in Nepal
and the payment was made in Nepal.

Explain briefly, whether the above receipts are chargeable to tax in India.

Solution:
(a) Taxable in India: As per section 9, Amount received for use of Patent for a Business in India shall
be deemed as Income accruing arising in India and chargeable to tax in India.
(b) Not Taxable in India: As per section 9, Amount received in Japan for use of know - how for a
business in Sri Lanka shall not be deemed as Income accruing arising from India and not chargeable
to tax in India.
(c) Taxable in India: As per section 9, Amount received for use of Technical know - how in India shall
be deemed as Income accruing arising in India and shall be taxable in India.
(d) Not Taxable in India: As per section 9,Amount received in Nepal for Feasibility study conducted
for the new project in Nepal shall not be deemed as Income accruing arising from India and not
chargeable to tax in India.

NOV – 2016
Question 4(a) (2 Marks)
State with reasons whether the following receipts are taxable or not under the provisions of Income-tax Act,
1961?
Mr. Frdie, a non-resident residing in Sweden, has received rent from Mr. Nadal, also a non-resident residing
in France in respect of a property taken on lease at Mumbai. Since this income is received outside India from
a non-resident, Frdie claims that his income is not chargeable to Tax in India.
Answer:
As per section 9, if source of income is in India, income shall be accruing/arising in India and shall be
taxable in all the three status even if income has been received outside India, hence in the given case Income
is chargeable to tax in India as income is accruing and arising from India as property is situated in India.
Residential Status & Scope of Total Income 157

MAY – 2016
Question 2(a)(i) (4 Marks)
How is the residential status of a Company determined for the purpose of Income - Tax Act, 1961, for the
Assessment Year 2019-20?
Answer: Refer answer given in the book
NOV – 2015
Question 2(a). (4 Marks)
Mr. X, an Indian Citizen left India on 20.04.2016 for the first time to setup a software firm in Singapore. On
10.04.2018, he entered into an agreement with XYZ Limited, an Indian Company for the transfer of
technical documents and designs to setup an automobile factory in Faridabad. He reached India along with
his team to render the requisite services on 15.05.2018 and was able to complete his assignment on
20.08.2018. He left for Singapore on 21.08.2018. He charged `50 Lakhs for his services from XYZ Limited.
Determine the residential status of Mr. X for the Assessment Year 2019-20 and explain as to the taxability
of the fees charged from XYZ Limited as per Income Tax Act, 1961.
Solution:
Residential status of Mr. X for the Assessment Year 2019-20
As per Section 6(1), any individual who is a citizen of India and has left India for taking up any business or
profession or employment outside India are covered in the special category and they will be considered to be
resident only if they stay in India for 182 days or more i.e. second condition of 60 plus 365 days shall not be
applicable.
In the given case Mr. X is covered under special category and stay in the current year is 99 days only which
is less than 182 days. So he is non-resident.
As per section 9, if any non-resident has provided any patent right or any managerial, technical services and
such patent right etc. was used in India, in such cases any royalty or fee received by non-resident shall be
considered to be income accruing/arising in India and shall be taxable and it do not matter that the non-
resident do not have residence or place of business or business connection in India i.e. there is no territorial
nexus or non-resident has not rendered services in India.
In the given case Fees charged from XYZ Limited is taxable in India because the assesse has transferred
technical documents and designs to setup an automobile factory in Faridabad (i.e. in India).

MAY – 2015
Question 2(a)(i). (4 Marks)
Explain with reasons whether the following transactions attract income-tax in India in the hands of
recipients?
(a) Salary paid to Mr. David, a citizen of India `15,00,000 by the Central Government for the services
rendered in Canada.
(b) Legal charges of `7,50,000 paid to Mr. Johnson, a lawyer of London, who visited India to represent a
case at the Supreme Court.
(c) Royalty paid to Rajeev, a non-resident by Mr. Mukesh, a resident for a business carried on in Sri Lanka.
(d) Interest received of `1,00,000, on money borrowed from France, by Ms. Dyana, a non-resident for the
business at Bangalore.
Solution:
(a) Taxable in India: As Mr. David is an Central Government employee and salary paid by Central
Government for the services rendered outside India is taxable in India.
(b) Taxable in India: As service is rendered in India means income accruing and arising from India.
(c) Taxable outside India: As Royalty is paid for business carried outside India means Income accruing
outside India.
(d) Taxable in India: As the loan is used for business in India.

Question 2(a)(ii). (4 Marks)


Residential Status & Scope of Total Income 158

Ms. Bindu, a non-resident, residing in New York since 2000, came back to India on 19.02.2017 for
permanent settlement in India. Explain her residential status of Ms. Bindu for the Assessment Year 2019-
20. In accordance with the various provision of Indian Income tax Act.
Solution:
Ms. Bindu is a resident since her stay in the previous year 2018-19 is 365 days.

2017-18 - 365
2016-17 - 41 (i.e. 10 + 31)
2015-16 - Nil (since she is residing in New York)
2014-15 - Nil (since she is residing in New York)
2013-14 - Nil (since she is residing in New York)
2012-13 - Nil (since she is residing in New York)
2011-12 - Nil (since she is residing in New York)
406
Since stay in 7 years preceding the relevant previous year is less than 729 days and She is non-resident in 9
years preceding the relevant previous year, she will be considered to be NOR.

NOV – 2014
Question 2(a). (5 Marks)
Mrs. X and Mrs. Y are sisters and they earned the following income during the Financial Year 2018-19.
Mrs. X is settled in Malaysia since 2006 and visits India for a month every year. Mrs. Y is settled in Indore
since her marriage in 2006. Compute the total income of Mrs. X and Mrs. Y for the assessment year 2019-
20:
Sl. Particulars Mrs. X Mrs. Y
No. ` `
(i) Income from Profession in Malaysia, (set up in India) received there 15,000 -
(ii) Profit from business in Delhi, but managed directly from Malaysia 40,000 -
(iii) Rent (computed) from property in Malaysia deposited in a Bank at Malaysia, 1,20,000 -
later on remitted to India through approved banking channels.
(iv) Dividend from PQR Ltd. an Indian Company 5,000 9,000
(v) Dividend from a Malaysian company received in Malaysia 15,000 8,000
(vi) Cash gift received from a friend on Mrs. Y’s 50th birthday - 51,000
(vii) Agricultural income from land in Maharashtra 7,500 4,000
(viii) Past foreign untaxed income brought to India 5,000 -
(ix) Fees for technical services rendered in India received in Malaysia 25,000 -
(x) Income from a business in Pune (Mrs. X receives 50% of the income in India) 12,000 15,000
(xi) Interest on debentures in an Indian company (Mrs. X received the same in 18,500 14,000
Malaysia)
(xii) Short-term capital gain on sale of shares of an Indian company 15,000 25,500
(xiii) Interest on Fixed Deposit with SBI in India 12,000 8,000

Solution:
Computation of Total Income of Mrs. X and Mrs. Y for the A.Y. 2019-20
Sl.No. Particulars Mrs. X Mrs. Y
NR ROR
(i) Income from Profession in Malaysia, (set up in India) received - -
there
(ii) Profit from business in Delhi, but managed directly from Malaysia 40,000 -
(iii) Rent (computed) from property in Malaysia deposited in a Bank at - -
Malaysia, later on remitted to India through approved banking
channels.
(iv) Dividend from PQR Ltd. an Indian Company - -
Residential Status & Scope of Total Income 159

(v) Dividend from a Malaysian company received in Malaysia - 8,000


(vi) Cash gift received from a friend on Mrs. Y’s 50th birthday - 51,000
(vii) Agricultural income from land in Maharashtra (exempt u/s 10(1)) - -
(viii) Past foreign untaxed income brought to India - -
(ix) Fees for technical services rendered in India received in Malaysia 25,000 -
(x) Income from a business in Pune (Mrs. X receives 50% of the 12,000 15,000
income in India)
(xi) Interest on debentures in an Indian company (Mrs. X received the 18,500 14,000
same in Malaysia)
(xii) Short-term capital gain on sale of shares of an Indian company 15,000 25,500
(xiii) Interest on fixed deposit with SBI in India 12,000 8,000
Total Income 1,22,500 1,21,500

MAY – 2013
Question 2(a). (4 Marks)
Mr. X and Mr. Y are brothers and they earned the following incomes during the financial year 2018-19.
Mr. X settled in America in the year 2006 and Mr. Y settled in Mumbai. Mr. X visits India for 20 days
every year. Mr. Y also visits America every year for a month. Compute their total income for the
Assessment year 2019-20 from the following information.

Sl. Particulars Mr. X Mr. Y


No. ` `
1. Interest on American Development bonds, 50% of interest received in 46,000 18,000
India.
2. Dividend from a Japanese Company received in America. 10,000 15,000
3. Short term capital gains on sale of shares of an Indian company 45,000 75,000
received in India.
4. Profit from a business in Mumbai, but managed directly from America. 10,000 -
5. Income from a business in Mumbai. 32,000 28,000
6. Fees for technical services rendered in America and received in America. 1,50,000 -
The services were, however, utilized in India.
7. Interest on fixed deposit with State Bank of India, Mumbai. 4,500 12,000
8. Income from house property at Mumbai. 67,200 38,500

Solution:
Computation of Total Income of Mr. X & Mr. Y for the A.Y. 2019-20
Sl. Particulars Mr. X Mr. Y
No. NR ROR
(`) (`)
1. Interest on American Development Bonds 23,000 18,000
2. Dividend from Japanese Company received in America - 15,000
3. Short term capital gains on sale of shares of an Indian company 45,000 75,000
received in India
4. Profit from a business in Mumbai but managed directly from America 10,000 -
5. Income from a business in Mumbai 32,000 28,000
6. Fees for technical services rendered in America, and received in 1,50,000 -
America, but services utilized in India
7. Interest on fixed deposit with State Bank of India, Mumbai 4,500 12,000
8. Income from house property at Mumbai 67,200 38,500
Total Income 3,31,700 1,86,500
MAY – 2012
Residential Status & Scope of Total Income 160

Question 2 (5 Marks)
Mr. X & Mr. Y are brothers and they earned the following incomes during the financial year 2018-19. Mr. X
settled in Canada in the year 2006 and Mr. Y settled in Delhi. Compute the total income for the assessment
year 2019-20.
Sl. Particulars Mr. X Mr. Y
No.
1. Interest on Canada Development Bond, (only 50% of interest 35,000 40,000
received in India)
2. Dividend from British company received in London 28,000 20,000
3. Profit from a business in Nagpur, but managed directly from 1,00,000 1,40,000
London
4. Short term capital gain on sale of shares of an Indian 60,000 90,000
company received in India
5. Income from a business in Chennai 80,000 70,000
6. Fees for technical services rendered in India, but received in 1,00,000 -----
Canada
7. Interest on fixed deposit in UCO Bank, Delhi 7,000 12,000
8. Agricultural income from a land situated in Andhra Pradesh 55,000 45,000
9. Income under the head house property at Bhopal 1,00,000 60,000
Answer:
Computation of Total Income of Mr. X and Mr. Y for the A.Y. 2019-20
Sl. Particulars Mr. X Mr. Y
No. Non-Resident ROR
1. Interest on Canada Development Bond, (only 50% of interest received 17,500 40,000
in India)
2. Dividend from British company received in London ---- 20,000
3. Profit from a business in Nagpur, but managed directly from London 1,00,000 1,40,000
4. Short term capital gain on sale of shares of an Indian company 60,000 90,000
received in India
5. Income from a business in Chennai 80,000 70,000
6. Fees for technical services rendered in India, but received in Canada 1,00,000 -----
7. Interest on fixed bank deposit in UCO Bank, Delhi 7,000 12,000
8. Agricultural income from a land situated in Andhra Pradesh ------ ------
9. Income under the head house property at Bhopal 1,00,000 60,000
Total Income 4,64,500 4,32,000
Notes:
1. Dividend received from British company in London, by a non-resident assessee is not taxable income,
while the same received by an ROR assessee is taxable and is not exempt under section 10(34) of Income
Tax Act, 1961.
2. Agricultural income from a land situated in Andhra Pradesh(in India), is exempted under section 10(1) of
Income tax Act, 1961 in case of both non-resident and resident assessee.
NOV – 2011
Question 5 (3 Marks)
Mr. X (Citizen of India) a Government employee serving in the Ministry of External Affairs left India for
the first time on 31.03.2018 due to his transfer to Indian High Commission in Canada. He did not visit any
time during previous year 2018-19. He has received the following income for the Previous Year 2018-19.
`
(i) Income under the head Salary (Computed) 5,00,000
(ii) Interest on fixed deposit from bank in India 1,00,000
(iii) Income from agriculture in Pakistan 2,00,000
(iv) Income from house property in Pakistan 2,50,000
Compute his gross total income for Assessment Year 2019-20.
Residential Status & Scope of Total Income 161

Answer:
Mr. X is a non-resident in the previous year 2018-19 as he doesn’t come to India during the year.
Computation of Gross total income of Mr. X for A.Y. 2019-20 `
Income under the head Salary
Income under the head Salary {Taxable as it is deemed to be earned in India u/s 9} 5,00,000
Income under the head Other Sources
Interest on FDR 1,00,000
Income from agriculture in Pakistan Nil
(assumed to be received outside India)
Income under the head Other Sources 1,00,000
Income under the head House Property
Income from house property in Pakistan Nil
(assumed to be received outside India)
Gross Total Income 6,00,000

MAY – 2010
Question 3 (4 Marks)
From the following particulars of Income furnished by Mr. X pertaining to the year ended 31.03.2019,
compute the total income for the assessment year 2019-20, if he is:
(i) Resident and ordinary resident;
(ii) Resident but not ordinary resident;
(iii) Non-resident:
Particulars Amount (`)
(a) Profit on sale of shares in Indian Company received in Germany 15,000
(b) Dividend from a Japanese Company received in Japan 10,000
(c) Income from business in London deposited in a bank in London,
later on remitted to India through approved banking channels 75,000
(d) Dividend from RP Ltd., an Indian Company 6,000
(e) Agricultural income from lands in Gujarat 25,000
Answer.
Computation of Total Income of Mr. X for the A.Y. 2019-20
Particulars Resident & Resident but Non-
ordinarily not ordinarily Resident
resident resident
a) Profit on sale of shares of an Indian company, received in 15,000 15,000 15,000
Germany
b) Dividend from a Japanese company, received in Japan. 10,000 - -
c) Income from business in London deposited in a bank in 75,000 - -
London
d) Dividend from RP Ltd., an Indian Company [See Note (i) - - -
below]
e) Agricultural income from land in Gujarat [See Note (ii) - - -
below]
TOTAL INCOME 1,00,000 15,000 15,000
Notes
(i) Dividend from Indian company is exempt under section 10(34)
(ii) Agricultural income is exempt under section 10(1).

MAY – 2010
Question 1 (2 Marks each)
Answer the following with reasons having regard to the provisions of the Income-Tax Act, 1961 for the
Assessment Year 2019-20:
Residential Status & Scope of Total Income 162

(i) State the Scope of total income in the case of an individual, whose residential status is ‘non-resident’ with
reference to section 5.
(ii) Mr. X a citizen of India received salary from the Government of India for the services rendered outside
India. Is the salary income chargeable to tax?
Answer:
(i) As per section 5, in case of non-resident, incomes taxable shall be:
(a) any income deemed to be accruing/arising in India.
(b) any income which is received or is deemed to be received in India.

(ii) As per Section 9, salaries payable by the Government to a citizen of India for services rendered outside
India is deemed to accrue or arise in India. Hence, salary received by Mr. X, a citizen of India, from the
Government of India for services rendered outside India is chargeable to tax under the head ‘Salaries’.

NOV – 2009
Question 1 (4 Marks)
Mr. X and Mr. Y are brothers and they earned the following incomes during the financial year 2018-19. Mr.
X settled in U.K. in the year 2005 and Mr. Y settled in Surat. Compute the gross total income for the
Assessment Year 2019-20.
Sr. No. Particulars Mr. X Mr. Y
1. Interest on U.K. development bonds, 50% of interest received in India 25,000 20,000
2. Dividend from British Company received in London 8,000 10,000
3. Profit from a business in Mumbai, but managed directly from London 10,000 12,000
4. Profit on sale of shares of an Indian company received in India 50,000 80,000
5. Income from a business in Delhi 20,000 20,000
6. Fees for technical services rendered in India, but received in London 1,00,000 -
7. Interest on fixed deposit in SBI, Bangalore 5,000 15,000
8. Agricultural income from a land situated in Rajasthan 25,000 25,000
9. Income under the head House Property at Bangalore 50,400 33,600

Answer:
Computation of Total Income of Mr. X and Mr. Y for the A.Y. 2019-20
Sl. Particulars Mr. X Mr. Y
No. Non-Resident ROR
1. Interest on U.K. Development Bonds 12,500 20,000
2. Dividend from British Company received in London - 10,000
3. Profit from a business in Mumbai but managed directly from London 10,000 12,000
4. Profit on sale of shares of an Indian company received in India 50,000 80,000
5. Income from a business in Delhi 20,000 20,000
6. Fees for technical services rendered in India but received in London 1,00,000 -
7. Interest on fixed Deposit in SBI Bangalore 5,000 15,000
8. Agricultural income from a land in Rajasthan [(Exempt u/s.10(1)] - -
9. Income under the head House property at Bangalore 50,400 33,600
Gross Total Income 2,47,900 1,90,600

Notes:
1. Dividend received from British company in London, by a non-resident assessee is not taxable income,
while the same received by an ROR assessee is taxable and is not exempt under section 10(34) of Income
Tax Act, 1961.
2. Agricultural income from a land situated in the State of Rajasthan, is exempted under section 10(1) of
Income tax Act, 1961 in case of both non-resident and resident assessee.
Residential Status & Scope of Total Income 163

MAY – 2008
Question 3 (1 Marks)
Choose the correct answer with reference to the provisions of the Income-tax Act, 1961:
Income accruing in Japan and received there is taxable in India in the case of –
(a) Resident and ordinarily resident only
(b) Both resident and ordinarily resident and resident but not ordinarily resident
(c) Both resident and non-resident
(d) Non-resident
Answer: (a) Resident and ordinarily resident only

MAY – 2007
Question 1 (2 Marks)
State with reasons, whether the following statement is true or false, with regard to the provisions of the
Income-tax Act, 1961:
Only individuals and HUFs can be resident, but not ordinarily resident in India; firms can be either a
resident or non-resident.
Answer: True: A person is said to be “not-ordinarily resident” in India if he satisfies either of the
conditions given in sub-section (6) of section 6. This sub-section relates to only individuals and Hindu
Undivided Families. Therefore, only individuals and Hindu undivided families can be resident, but not
ordinarily resident in India. All other classes of assessees can be either a resident or non-resident for the
purpose of income-tax. A firm can, therefore, either be a resident or non-resident.
.
Income Under The Head House Property 164

INCOME UNDER THE HEAD


HOUSE PROPERTY
SECTION 22 TO 27
PARTICULARS SECTIONS
Income from house property—Chargeability and meaning of house property 22
Annual value—
House let out throughout the year/partly let out/partly vacant 23(1)
One house which is self-occupied 23(2)
House partly self-occupied/partly let out, may or may not be vacant 23(3)
More than one house which are self-occupied 23(4)
Municipal tax Proviso to sec 23(1)
Treatment of unrealised rent Explanation to 23(1)
Statutory Deduction/Standard Deduction 24(a)
Interest on capital borrowed 24(b)
Interest in case of loan taken from outside India 25
Recovery of unrealised rent / Arrears of rent 25A
Property owned by co-owners 26
Deemed ownership 27
Meaning of unrealised rent Rule 4

Chargeability of income under the head house property Section 22


Income from letting out of house property is chargeable to tax under the head House Property. If the income
is from sale or purchase of house property, it will be taxable under the head Capital Gains, however if the
sale or purchase is part of a business, income is taxable under the head Business/Profession.
House property shall include all types of house properties i.e. residential houses, shops, godowns, cinema
building, workshop building, hotel buildings etc.
The term house property shall include not only the buildings but also the lands appurtenant thereto i.e. the
term house property shall include even any open land which is part and parcel of the building. E.g. Mr. X
has one big house and it includes vast open area within its boundaries. The house has been let out at a rent of
`1,00,000 p.m., out of which rent of `25,000 p.m. is attributable to the open land. In this case, entire rental
income is taxable under the head house property.
If any person has let out only land, which is not essential part of any building, income is taxable under the
head other sources. E.g. Mr. X has one big piece of land which is let out for arranging exhibitions or for the
purpose of marriage parties etc., rent received or receivable is taxable under the head other sources.
Income from business of letting out house property/Property held as stock-in-trade
If any person is holding house property as stock-in-trade or for the purpose of letting out as business, income
shall be taxable under the head business/profession. E.g. ABC Ltd. is holding 500 flats for the purpose of
letting out, income shall be taxable under the head business/profession.
Computation of Income under the head House Property
Gross Annual Value (GAV) `…………….
Less: Municipal Taxes `.……………
Net Annual Value (NAV) `.….………..
Less:
Income Under The Head House Property 165

Deduction allowed under section 24


- Statutory deduction / standard deduction @ 30% of NAV [Section 24(a)] `……..………
- Interest on borrowed capital for construction etc. [Section 24(b)] `……………..
Income under the head “House Property” `……………..

Question 1: Write a note on computation of income of a house property which is let out throughout
the year.
Answer: As per section 23(1)(a)/(b), gross annual value i.e. reasonable rental value shall be computed in
the manner given below:
1. Compare Fair Rent and Municipal Valuation and select the higher.
2. Compare the rent so selected with Standard Rent and the lower of the two shall be considered to be
Expected Rent. (It is also called Annual Letting Value)
3. Compare Expected Rent with Rent Received or Receivable and the higher shall be considered to be Gross
Annual Value.
Fair rent i.e. the rent of similar types of buildings in the same locality.
Municipal valuation i.e. rental value determined by the municipality for the purpose of charging municipal
tax. It is also called rateable value.
Standard rent i.e. the highest possible rent as per Rent Control Act.
Rent received or receivable
Illustration 1: Mr. X has one house property which is let out @ `80,000 p.m. Fair rent ` 90,000 p.m.,
Municipal Valuation `70,000 p.m., Standard Rent `81,000 p.m. Municipal tax paid `60,000 and interest
paid on loan for construction of house property is ` 50,000.
Compute his Income Tax Liability for A.Y 2019-20.
Solution: `
Computation of income under the head House Property
Gross Annual Value 9,72,000.00
Working Note: `
(a) Fair Rent (90,000 x 12) 10,80,000
(b) Municipal Value (70,000 x 12) 8,40,000
(c) Higher of (a) or (b) 10,80,000
(d) Standard Rent (81,000 x 12) 9,72,000
(e) Expected Rent {Lower of c or d} 9,72,000
(f) Rent received /receivable (80,000 x 12) 9,60,000
GAV shall be higher of (e) or (f) 9,72,000
Less: Municipal Tax (60,000.00)
Net Annual Value 9,12,000.00
Less: 30% of NAV u/s 24(a) (2,73,600.00)
Less: Interest on capital borrowed u/s 24(b) (50,000.00)
Income under the head House Property 5,88,400.00
Gross Total Income 5,88,400.00
Less: Deduction u/s 80C to 80U NIL
Total Income 5,88,400.00
Computation of Tax Liability
Tax on `5,88,400 at slab rate 30,180.00
Add: HEC @ 4% 1,207.20
Tax Liability 31,387.20
Rounded off u/s 288B 31,390.00
Illustration 2: Mrs. X has let out one House property @ `62,000 p.m., Municipal Valuation `72,000 p.m.,
Fair Rent `90,000 p.m., Standard Rent `1,00,000 p.m., Municipal Tax paid `40,000 and Interest on loan
taken for construction `60,000
She has completed the age of 60 years on 01.04.2019.
Compute Income Tax Liability for the A.Y 2019-20.
Income Under The Head House Property 166

Solution: `
Computation of income under the head House Property
Gross Annual Value 10,80,000.00
Working Note: `
(a) Fair Rent (90,000 x 12) 10,80,000
(b) Municipal Value (72,000 x 12) 8,64,000
(c) Higher of (a) or (b) 10,80,000
(d) Standard Rent (1,00,000 x 12) 12,00,000
(e) Expected Rent {Lower of c or d} 10,80,000
(f) Rent received /receivable (62,000 x 12) 7,44,000
GAV shall be higher of (e) or (f) 10,80,000
Less: Municipal Tax (40,000.00)
Net Annual Value 10,40,000.00
Less: 30% of NAV u/s 24(a) (3,12,000.00)
Less: Interest on capital borrowed u/s 24(b) (60,000.00)
Income from house property 6,68,000.00
Gross Total Income 6,68,000.00
Less: Deduction u/s 80C to 80U NIL
Total Income 6,68,000.00
Computation of Tax Liability
Tax on `6,68,000 at slab rate 46,100.00
Add: HEC @ 4% 1,844.00
Tax Liability 47,944.00
Rounded off u/s 288B 47,940.00
Illustration 3: Mr. X owns five houses in Chennai, all of which are let-out. Compute the GAV of each
house from the information given below –
Particulars House I House II House III House IV House V
Municipal Value 80,000 55,000 65,000 24,000 75,000
Fair Rent 90,000 60,000 65,000 25,000 80,000
Standard Rent N.A. 75,000 58,000 N.A. 78,000
Actual rent received/ receivable 72,000 72,000 60,000 30,000 72,000
Solution:
GAV 90,000 72,000 60,000 30,000 78,000

MAY – 2012 (5 Marks)


Mr. X owns five houses at Cochin. Compute the gross annual value of each house from the information
given below:
House-I House-II House-III House-IV House –V
Municipal value 1,20,000 2,40,000 1,10,000 90,000 75,000
Fair rent 1,50,000 2,40,000 1,14,000 84,000 80,000
Standard rent 1,08,000 N.A. 1,44,000 N.A. 78,000
Actual rent received/ 1,80,000 2,10,000 1,20,000 1,08,000 72,000
receivable
Answer:
House I `
Computation of Gross Annual Value
(a) Fair Rent 1,50,000
(b) Municipal Valuation 1,20,000
(c) Higher of (a) or (b) 1,50,000
(d) Standard Rent 1,08,000
(e) Expected Rent {Lower of (c) or (d)} 1,08,000
Income Under The Head House Property 167

(f) Rent Received/Receivable 1,80,000


(g) Higher of (e) or (f) shall be GAV 1,80,000
House II `
Computation of Gross Annual Value
(a) Fair Rent 2,40,000
(b) Municipal Valuation 2,40,000
(c) Higher of (a) or (b) 2,40,000
(d) Standard Rent N.A
(e) Expected Rent {Lower of (c) or (d)} 2,40,000
(f) Rent Received/Receivable 2,10,000
(g) Higher of (e) or (f) shall be GAV 2,40,000
House III `
Computation of Gross Annual Value
(a) Fair Rent 1,14,000
(b) Municipal Valuation 1,10,000
(c) Higher of (a) or (b) 1,14,000
(d) Standard Rent 1,44,000
(e) Expected Rent {Lower of (c) or (d)} 1,14,000
(f) Rent Received/Receivable 1,20,000
(g) Higher of (e) or (f) shall be GAV 1,20,000
House IV `
Computation of Gross Annual Value
(a) Fair Rent 84,000
(b) Municipal Valuation 90,000
(c) Higher of (a) or (b) 90,000
(d) Standard Rent N.A
(e) Expected Rent {Lower of (c) or (d)} 90,000
(f) Rent Received/Receivable 1,08,000
(g) Higher of (e) or (f) shall be GAV 1,08,000
House V `
Computation of Gross Annual Value
(a) Fair Rent 80,000
(b) Municipal Valuation 75,000
(c) Higher of (a) or (b) 80,000
(d) Standard Rent 78,000
(e) Expected Rent {Lower of (c) or (d)} 78,000
(f) Rent Received/Receivable 72,000
(g) Higher of (e) or (f) shall be GAV 78,000
Question 2: Explain deductibility of property taxes (municipal taxes).
Answer: Property Taxes (municipal taxes) Proviso to Section 23(1)
In order to maintain any particular town or city, there is always some authority and it is called local authority
e.g. MCD in Delhi and such authority is allowed to charge certain tax in connection with house property and
such tax are called municipal tax or house tax or property tax. If an assessee has paid such tax, deduction
shall be allowed for the tax so paid from GAV but if tax is due but not paid, deduction is not allowed.
If tax has been paid by the tenant, in that case tax shall not be allowed to be deducted.
Example
During the previous year 2018-19 municipality has levied taxes `20,000, but the assessee has paid `15,000.
In this case, amount allowed to be deducted is `15,000. In the next year, municipality has levied taxes of
`45,000 but the assessee has paid ` 55,000 which includes `5,000 for the earlier year and `5,000 for the
subsequent year. In this case, amount allowed to be deducted in previous year 2019-20 shall be `55,000.
Income Under The Head House Property 168

Question 3: Write a note on set off and carry forward of losses under the head house property.
Answer: Set off and carry forward of losses under the head house property Section 70/71/71B
Inter Source adjustment Section 70
As per section 70, if any person has loss from any house property, such loss can be set off from income of
any other house property and it is called inter-source adjustment or intra-head adjustment. E.g. Mr. X has
two houses: there is loss of `5,00,000 from one house and income of `8,00,000 from the other house, in this
case, loss of one source (house) can be set off from income of the other source (house).
Inter Head adjustment Section 71
As per section 71, unadjusted loss can be set off from incomes of other heads but as per section 58(4), such
loss can not be set off from casual income and it is called inter-head adjustment. E.g. Mr. X has loss from
house property `1,50,000 and income from business/profession `5,00,000, in this case, loss is allowed to be
set off but if he has any casual income, loss can not be set off from casual income.

Where in respect of any assessment year, the net result of the computation under the head “Income from
house property” is a loss and the assessee has income assessable under any other head of income, the
assessee shall not be entitled to set off such loss, to the extent the amount of the loss exceeds `2,00,000,
against income under the other head.’.

Carry Forward and Set Off Section 71B


As per section 71B, unadjusted loss is allowed to be carried forward to the subsequent years but for a
maximum period of 8 years starting from the year subsequent to the year in which the loss was incurred and
in the subsequent years, loss can be set off only from income under the head house property. E.g. Mr. X has
incurred loss under the head house property in the previous year 2018-19/assessment year 2019-20 and it
could not be set off in the same year, it can be carried forward upto Previous Year 2026-27/Assessment Year
2027-28 (as shown below)
Year 1 Previous year 2019-20 Assessment Year 2020-21
Year 2 Previous year 2020-21 Assessment Year 2021-22
Year 3 Previous year 2021-22 Assessment Year 2022-23
Year 4 Previous year 2022-23 Assessment Year 2023-24
Year 5 Previous year 2023-24 Assessment Year 2024-25
Year 6 Previous year 2024-25 Assessment Year 2025-26
Year 7 Previous year 2025-26 Assessment Year 2026-27
Year 8 Previous year 2026-27 Assessment Year 2027-28
E.g. Mr. X has loss under the head house property of the previous year 2010-11/assessment year 2011-12
`5,00,000 and income under the head house property `5,00,000 in previous year 2018-19/assessment year
2019-20, in this case, loss shall be allowed to be set off because it will be allowed to be carried forward upto
a period of 8 years starting from Previous Year 2011-12/Assessment Year 2012-13 and is as shown below:
Year 1 Previous year 2011-12 Assessment Year 2012-13
Year 2 Previous year 2012-13 Assessment Year 2013-14
Year 3 Previous year 2013-14 Assessment Year 2014-15
Year 4 Previous year 2014-15 Assessment Year 2015-16
Year 5 Previous year 2015-16 Assessment Year 2016-17
Year 6 Previous year 2016-17 Assessment Year 2017-18
Year 7 Previous year 2017-18 Assessment Year 2018-19
Year 8 Previous year 2018-19 Assessment Year 2019-20

Additional Points
1. If the loss can be set off, it has to be set off compulsorily i.e. it is not voluntary. E.g. Mr. X has loss under
the head house property `1,50,000 in previous year 2018-19/assessment year 2019-20 and income under the
head business/profession `1,50,000 in the same year, in this case loss has to be set off.
2. Any loss has to be set off first within the same head and after that under some other heads and after that
carry forward is allowed.
Income Under The Head House Property 169

3. Loss of current year shall be set off first and only after that brought forward losses can be adjusted, eg.
Mr. X has income from one house ` 10,00,000 and loss from other house ` 10,00,000 in P.Y. 2018-19 and
also unadjusted loss of ` 10,00,000 under the head house property of P.Y. 2010-11, in this case loss of
current year is to be adjusted first.
Illustration 4: Mr. X has Loss under the head House Property `13,00,000 and income under the head Salary
`8,00,000 and income under the head Business/Profession `6,00,000 and LTCG `20,00,000 and Casual
income `5,00,000. Compute his tax liability for A.Y. 2019-20.

Solution:
In this case, Mr. X has the option to set off the loss under the head House Property either from normal
income or from LTCG and tax liability in two options shall be:
Option-1: Set off from normal income:
Computation of Total Income `
Income under the head Salary 8,00,000
Less: Loss under the head House Property (2,00,000)
Income under the head Salary 6,00,000
Income under the head Business/Profession 6,00,000
Long term capital gain 20,00,000
Casual income 5,00,000
Gross Total Income 37,00,000
Less: Deduction u/s 80C to 80U Nil
Total Income 37,00,000
Computation of Tax Liability
Tax on Casual income `5,00,000 @ 30% u/s 115BB 1,50,000
Tax on LTCG `20,00,000 @ 20% u/s 112 4,00,000
Tax on normal income `12,00,000 at slab rate 1,72,500
Tax before HEC 7,22,500
Add: HEC @ 4% 28,900
Tax Liability 7,51,400
Loss under the head house property of `11,00,000 shall be carried forward.
Note: As per section 71, Maximum loss of `2,00,000 is allowed to be set off from other heads.
Option-2: Set off from LTCG:
Computation of Total Income `
Income under the head Salary 8,00,000
Income under the head Business/Profession 6,00,000
Long term capital gain 20,00,000
Less: Loss under the head House Property (2,00,000)
Long term capital gain 18,00,000
Casual income 5,00,000
Gross Total Income 37,00,000
Less: Deduction u/s 80C to 80U Nil
Total Income 37,00,000
Computation of Tax Liability
Tax on Casual income `5,00,000 @ 30% u/s 115BB 1,50,000
Tax on LTCG `18,00,000 @ 20% u/s 112 3,60,000
Tax on normal income `14,00,000 at slab rate 2,32,500
Tax before HEC 7,42,500
Add: HEC @ 4% 29,700
Tax Liability 7,72,200
Loss under the head house property of `11,00,000 shall be carried forward.
Note: As per section 71, Maximum loss of `2,00,000 is allowed to be set off from other heads.
Option-1 is better since Total Tax Liability is lower in this option.
Income Under The Head House Property 170

Illustration 5: Mr. X has Loss under the head House Property ` 20,00,000 and income under the head
Salary ` 10,00,000 and income under the head Business/Profession ` 11,00,000 and LTCG ` 10,00,000 and
deduction u/s 80C to 80U is ` 2,00,000. Compute his tax liability for A.Y. 2019-20.

Solution:
In this case, Mr. X has the option to set off the loss under the head House Property either from normal
income or from LTCG and tax liability in two options shall be:
Option-1: Set off from normal income:
Computation of Total Income `
Income under the head Salary 10,00,000
Less: Loss under the head House Property (2,00,000)
Income under the head Salary 8,00,000
Income under the head Business/Profession 11,00,000
Long term capital gain 10,00,000
Gross Total Income 29,00,000
Less: Deduction u/s 80C to 80U (2,00,000)
Total Income 27,00,000
Computation of Tax Liability
Tax on LTCG `10,00,000 @ 20% u/s 112 2,00,000
Tax on normal income 17,00,000 at slab rate 3,22,500
Tax before HEC 5,22,500
Add: HEC @ 4% 20,900
Tax Liability 5,43,400
Loss under the head house property of `18,00,000 shall be carried forward.
Note: As per section 71, Maximum loss of `2,00,000 is allowed to be set off from other heads.

Option-2: Set off from LTCG:


Computation of Total Income `
Income under the head Salary 10,00,000
Income under the head Business/Profession 11,00,000
Long term capital gain 10,00,000
Less: Loss under the head House Property (2,00,000)
Long term capital gain 8,00,000
Gross Total Income 29,00,000
Less: Deduction u/s 80C to 80U (2,00,000)
Total Income 27,00,000
Computation of Tax Liability
Tax on LTCG `8,00,000 @ 20% u/s 112 1,60,000
Tax on normal income `19,00,000 at slab rate 3,82,500
Tax before HEC 5,42,500
Add: HEC @ 4% 21,700
Tax Liability 5,64,200
Loss under the head house property of `18,00,000 shall be carried forward.
Note: As per section 71, Maximum loss of `2,00,000 is allowed to be set off from other heads.
Option-1 is better since Total Tax Liability is lower in this option.

Question 4: Write a note on computation of income of house lying vacant for some period.
Answer: House lying vacant for some period Section 23(1)(c)
If the house is partly let out and partly vacant, in such cases expected rent shall be computed for 12 months
but while computing rent received /receivable, rent for the period for which the house was vacant shall be
excluded and GAV shall be higher of expected rent and rent received/receivable but if the rent
received/receivable is less than the expected rent owing to vacancy, in that case rent received/receivable
shall be gross annual value. e.g. If expected rent is `20,000 p.m. and rent received/receivable is `15,000
Income Under The Head House Property 171

p.m. and there is vacancy for 5 months, in this case GAV shall be the expected rent because even if there
was no vacancy, still rent received/receivable was less than expected rent.
If in this case rent received/receivable is `25,000 p.m. and it is vacant for 5 months, gross annual value shall
be the rent received/receivable because if there was no vacancy, rent R/R would have been higher than
expected rent accordingly in the given case, R/R is lower than expected rent owing to vacancy.

Illustration 6: Compute gross annual value in the following cases for the assessment year 2019-20:
Particulars Situation 1 Situation 2 Situation 3 Situation 4
Fair Rent (p.m.) 9,000 13,000 12,000 16,000
Municipal Valuation (p.m.) 10,000 9,000 9,000 18,000
Standard Rent (p.m.) 12,000 11,000 7,000 16,000
Rent received/ receivable (p.m.) 7,000 11,500 20,000 16,500
Vacancy 1 month 1 month 2 month 2 months
Solution:
Situation 1 `
Computation of Gross Annual Value
(a) Fair Rent 1,08,000
(9,000 x 12)
(b) Municipal Valuation 1,20,000
(10,000 x 12)
(c) Higher of (a) or (b) 1,20,000
(d) Standard Rent 1,44,000
(12,000 x 12)
(e) Expected Rent {Lower of (c) or (d)} 1,20,000
(f) Rent Received/Receivable 77,000
(7,000 x 11)
If there was no vacancy, in that case rent received/receivable would have been `84,000 and
It was still less than expected rent, therefore GAV shall be expected rent.
Gross Annual Value 1,20,000
Situation 2 `
Computation of Gross Annual Value
(a) Fair Rent 1,56,000
(13,000 x 12)
(b) Municipal Valuation 1,08,000
(9,000 x 12)
(c) Higher of (a) or (b) 1,56,000
(d) Standard Rent 1,32,000
(11,000 x 12)
(e) Expected Rent {Lower of (c) or (d)} 1,32,000
(f) Rent Received/Receivable 1,26,500
(11,500 x 11)
In this case, if there was no vacancy, rent received/receivable would have been `1,38,000
hence rent received/receivable is lower in this case due to vacancy, therefore GAV
shall be the rent received/receivable.
Gross Annual Value 1,26,500
Situation 3 `
Computation of Gross Annual Value
(a) Fair Rent 1,44,000
(12,000 x 12)
(b) Municipal Valuation 1,08,000
(9,000 x 12)
(c) Higher of (a) or (b) 1,44,000
Income Under The Head House Property 172

(d) Standard Rent 84,000


(7,000 x 12)
(e) Expected Rent {Lower of (c) or (d)} 84,000
(f) Rent Received/Receivable 2,00,000
(20,000 x 10)
In this case, rent R/R is higher than the expected rent, GAV shall be Rent R/R
Gross Annual Value 2,00,000
Situation 4 `
Computation of Gross Annual Value
(a) Fair Rent 1,92,000
(16,000 x 12)
(b) Municipal Valuation 2,16,000
(18,000 x 12)
(c) Higher of (a) or (b) 2,16,000
(d) Standard Rent 1,92,000
(16,000 x 12)
(e) Expected Rent {Lower of (c) or (d)} 1,92,000
(f) Rent Received/Receivable 1,65,000
(16,500 x 10)
In this case, if there was no vacancy, rent received/receivable would have been `1,98,000
hence rent received/receivable is lower in this case owing to vacancy, therefore GAV
shall be the rent received/receivable.
Gross Annual Value 1,65,000

Question 5: Write a note on House lying vacant for full year.


Answer: As per section 23(1)(c), if any House Property is lying vacant throughout the year, it will be
considered to be deemed to be let out and income shall be computed in the similar manner as in case of a let
out house. Expected Rent shall be considered to be Gross annual value.

As per section 23 (5), Where the property consisting of any building or land appurtenant thereto is held as
stock-in trade and the property or any part of the property is not let during the whole of the previous year,
the annual value of such property or part of the property, for the period up to one year from the end of the
financial year in which the certificate of completion of construction of the property is obtained from the
competent authority, shall be taken to be nil.

Question 6: Write a note on unrealized rent.


Answer: Treatment of unrealised rent Explanation to Section 23(1)/Rule 4
Unrealised rent means such rent which is irrecoverable and is considered to be loss i.e. bad debt and in such
cases, expected rent shall be computed for full year and while computing rent received or receivable, such
unrealised rent shall be excluded and GAV shall be higher of expected rent and rent received/receivable (no
special treatment like vacancy).
e.g. Mr. X has let out one house `50,000 p.m. , fair rent `45,000 p.m., municipal valuation `40,000 p.m.
standard rent `70,000 p.m. and there was unrealized rent for 3 months, in this case GAV of the house shall
be
Expected rent (45,000 x 12) 5,40,000
Rent received /receivable (50,000 x 9) 4,50,000
GAV 5,40,000
Rent shall be considered to be unrealised rent only if all the conditions of Rule 4 have been complied with
and such conditions are:
(a) the defaulting tenant has vacated, or steps have been taken to compel him to vacate the property;
(b) the defaulting tenant is not in occupation of any other property of the assessee;
(c) the assessee has taken all reasonable steps to institute legal proceedings for the recovery of the unpaid
rent or satisfies the Assessing Officer that legal proceedings would be useless.
Income Under The Head House Property 173

(d) the tenancy is bona fide (genuine)


Recovery of unrealised rent Section 25A
If any assessee has recovered unrealized rent in subsequent years, rent so recovered shall be considered to be
income of the assessee under the head house property and it do not matter whether the assessee has any
house property in his name in that year or not. If assessee has received any interest, it will be considered to
be income of the assessee under the head other sources. If assessee has incurred any expenses on legal
proceedings, it will not be allowed to be deducted.
A sum equal to thirty per cent of the unrealised rent shall be allowed as deduction.

Illustration 7: Mr. X has let out one house `70,000 per month fair rent `80,000 per month municipal
valuation `60,000 per month, Standard Rent ` 65,000 per month. Municipal tax paid `40,000, Interest u/s 24
(b) `50,000. Assessee has recovered unrealized rent of `60,000 plus interest `7,000. He has incurred legal
expenses `12,000 compute his Income and Tax Liability A.Y.2019-20.
Solution:
Computation of income under the head house property `
Gross Annual Value 8,40,000.00
Working Note: `
(a) Fair Rent (80,000 x 12) 9,60,000
(b) Municipal Valuation (60,000 x 12) 7,20,000
(c) Higher of (a) or (b) 9,60,000
(d) Standard Rent (65,000 x 12) 7,80,000
(e) Expected Rent {Lower of (c) or (d)} 7,80,000
(f) Rent received /receivable (70,000 x 12) 8,40,000
(g) Higher of (e) or (f) shall be GAV 8,40,000
Less: Municipal Tax (40,000.00)
Net Annual Value 8,00,000.00
Less: 30% of NAV u/s 24(a) (2,40,000.00)
Less: Interest on capital borrowed u/s 24(b) (50,000.00)
Income under the head House Property 5,10,000.00
Add: Recovery of Unrealised rent u/s 25A 60,000.00
Less: Deduction @ 30% (18,000.00) 42,000.00
Income under the head House Property 5,52,000.00

Income under the head other sources


Interest from unrealized rent 7,000.00

Gross Total Income 5,59,000.00


Less: Deduction u/s 80C to 80U NIL
Total Income 5,59,000.00
Computation of Tax Liability
Tax on ` 5,59,000 at slab rate 24,300.00
Add: HEC @ 4% 972.00
Tax Liability 25,272.00
Rounded off u/s 288B 25,270.00

Illustration 8: Compute gross annual value in the following cases for the assessment year 2019-20:
Particulars Situation 1 Situation 2 Situation 3 Situation 4
Fair Rent (p.m.) 11,000 13,000 16,000 14,000
Municipal Valuation (p.m.) 12,000 11,000 18,000 9,000
Standard Rent (p.m.) 13,000 12,000 17,000 8,000
Rent received/ receivable 8,000 12,500 17,000 21,000
(p.m.)
Income Under The Head House Property 174

Vacancy - 2 months 3 month 1 month


Unrealised rent 1 month - 1 month 3 month
Solution:
Situation 1 `
Computation of Gross Annual Value
(a) Fair Rent 1,32,000
(11,000 x 12)
(b) Municipal Valuation 1,44,000
(12,000 x 12)
(c) Higher of (a) or (b) 1,44,000
(d) Standard Rent 1,56,000
(13,000 x 12)
(e) Expected Rent {Lower of (c) or (d)} 1,44,000
(f) Rent Received/Receivable 88,000
(8,000 x 11)
GAV = Higher of (e) or (f) 1,44,000
Gross Annual Value 1,44,000
Situation 2
Computation of Gross Annual Value
(a) Fair Rent 1,56,000
(13,000 x 12)
(b) Municipal Valuation 1,32,000
(11,000 x 12)
(c) Higher of (a) or (b) 1,56,000
(d) Standard Rent 1,44,000
(12,000 x 12)
(e) Expected Rent {Lower of (c) or (d)} 1,44,000
(f) Rent Received/Receivable 1,25,000
(12,500 x 10)
In this case, if there was no vacancy, rent received/receivable would have been `1,50,000
hence rent received/receivable is lower in this case due to vacancy, therefore GAV
shall be the rent received/receivable.
Gross Annual Value 1,25,000
Situation 3
Computation of Gross Annual Value
(a) Fair Rent 1,92,000
(16,000 x 12)
(b) Municipal Valuation 2,16,000
(18,000 x 12)
(c) Higher of (a) or (b) 2,16,000
(d) Standard Rent 2,04,000
(17,000 x 12)
(e) Expected Rent {Lower of (c) or (d)} 2,04,000
(f) Rent Received/Receivable 1,36,000
(17,000 x 8)
If there was no vacancy, in that case rent received/receivable would have been `1,87,000
and It was still less than expected rent, therefore GAV shall be expected rent.
Gross Annual Value 2,04,000
Situation 4
Computation of Gross Annual Value
(a) Fair Rent 1,68,000
(14,000 x 12)
(b) Municipal Valuation 1,08,000
Income Under The Head House Property 175

(9,000 x 12)
(c) Higher of (a) or (b) 1,68,000
(d) Standard Rent 96,000
(8,000 x 12)
(e) Expected Rent {Lower of (c) or (d)} 96,000
(f) Rent Received/Receivable 1,68,000
(21,000 x 8)
In this case, rent R/R is higher than the expected rent, GAV shall be Rent R/R
Gross Annual Value 1,68,000
Illustration 9: Mr. X has let out one house property to Mr. Y @ ` 80,000 p.m. Fair rent `90,000 p.m.
Municipal valuation `80,000 p.m. and Standard rent of the house `76,000 p.m. The house remained vacant
for 2 months and there was unrealised rent for 3 months. Mr. X has paid municipal tax of `60,000 and
interest on loan for construction of house property is `69,000.
Compute his Income Tax Liability for A.Y.2019-20.
Solution: `
Computation of income under the head house property
Gross Annual Value 9,12,000.00
Working Note: `
(a) Fair Rent (90,000 x 12) 10,80,000
(b) Municipal Valuation (80,000 x 12) 9,60,000
(c) Higher of (a) or (b) 10,80,000
(d) Standard Rent (76,000 x 12) 9,12,000
(e) Expected Rent {Lower of (c) or (d)} 9,12,000
(f) Rent received /receivable (80,000 x 7) 5,60,000
If there was no vacancy, in that case rent received receivable would have been
`7,20,000 and it was still less than expected rent ,therefore GAV shall be expected rent
GAV 9,12,000
Less: Municipal Tax (60,000.00)
Net Annual Value 8,52,000.00
Less: 30% of NAV u/s 24(a) (2,55,600.00)
Less: Interest on capital borrowed u/s 24(b) (69,000.00)
Income under the head House Property 5,27,400.00
Gross Total Income 5,27,400.00
Less: Deduction u/s 80C to 80U NIL
Total Income 5,27,400.00
Computation of Tax Liability
Tax on ` 5,27,400 at slab rate 17,980.00
Add: HEC @ 4% 719.20
Tax Liability 18,699.20
Rounded off u/s 288B 18,700.00
Illustration 10: Mr. X has a property whose municipal valuation is ` 1,30,000 p.a. The fair rent is `1,10,000
p.a. and the standard rent fixed by the Rent Control Act is ` 1,20,000 p.a. The property was let out for a rent
of ` 11,000 p.m. throughout the previous year. Unrealised rent was ` 11,000 and all conditions prescribed by
Rule 4 are satisfied. He paid municipal taxes @ 10% of municipal valuation. Interest on borrowed capital
was ` 2,00,000 for the year. He has LTCG `6,00,000. Compute his tax liability for A.Y. 2019-20.
Solution:
Computation of Income from house property of Mr. X for A.Y. 2019-20
Gross Annual Value 1,21,000.00
Working Note: `
(a) Fair Rent 1,10,000
(b) Municipal Valuation 1,30,000
(c) Higher of (a) or (b) 1,30,000
Income Under The Head House Property 176

(d) Standard Rent 1,20,000


(e) Expected Rent {Lower of (c) or (d)} 1,20,000
(f) Rent received /receivable (11,000 x 11) 1,21,000
GAV 1,21,000
Less: Municipal Tax ( 10% of `1,30,000) (13,000.00)
Net Annual Value 1,08,000.00
Less: 30% of NAV u/s 24(a) (32,400.00)
Less: Interest on capital borrowed u/s 24(b) (2,00,000.00)
Loss under the head House Property (1,24,400.00)
Loss shall be adjusted from LTCG and balance LTCG shall be (6,00,000 – 1,24,400) 4,75,600.00
Total Income 4,75,600.00
Income Tax (`4,75,600 – 2,50,000) x 20% 45,120.00
Add: HEC @ 4% 1,804.80
Tax Liability 46,924.80
Rounded off u/s 288B 46,920.00
Illustration 11: Mr. X has a property whose municipal valuation is ` 2,50,000 p.a. The fair rent is `2,00,000
p.a. and the standard rent fixed by the Rent Control Act is ` 2,10,000 p.a. The property was let out for a rent
of ` 20,000 p.m. However, the tenant vacated the property on 31.01.2019. Unrealised rent was `20,000 and
all conditions prescribed by Rule 4 are satisfied. He paid municipal taxes @ 8% of municipal valuation.
Interest on borrowed capital was `1,65,000 for the year. He has casual income `3,00,000. Compute his tax
liability for A.Y. 2019-20.
(b) Presume he is non-resident and his date of birth 31.03.1959.
Solution: Computation of income from house property of Mr. X for A.Y. 2019-20
Gross Annual Value 1,80,000.00
Working Note: `
(a) Fair Rent 2,00,000
(b) Municipal Valuation 2,50,000
(c) Higher of (a) or (b) 2,50,000
(d) Standard Rent 2,10,000
(e) Expected Rent {Lower of (c) or (d)} 2,10,000
(f) Rent received /receivable (20,000 x 9) 1,80,000
If there was no vacancy, in that case rent received receivable would have been
(20,000 x 11) `2,20,000 which is exceeding expected rent hence GAV shall be rent
received /receivable i.e. `1,80,000
GAV 1,80,000
Less: Municipal Tax (8% of `2,50,000) (20,000.00)
Net Annual Value 1,60,000.00
Less: 30% of NAV u/s 24(a) (48,000.00)
Less: Interest on capital borrowed u/s 24(b) (1,65,000.00)
Loss under the head House Property (53,000.00)
Loss shall be carried forward and as per section 58(4), loss shall not be adjusted from casual income
Casual income 3,00,000.00
Total Income 3,00,000.00
Income Tax `3,00,000 x 30% 90,000.00
Less: Rebate u/s 87A (2,500.00)
Tax before health & education cess 87,500.00
Add: HEC @ 4% 3,500.00
Tax Liability 91,000.00
(b)
Income Tax `3,00,000 x 30% 90,000.00
Add: HEC @ 4% 3,600.00
Tax Liability 93,600.00
Income Under The Head House Property 177

MAY – 2012 (4 Marks)


Explain the treatment of unrealized rent and its recovery in subsequent years under the provisions of Income
Tax Act, 1961.
Answer: Refer answer given in the book

Question 7: Write a note on Statutory Deduction or Standard Deduction.


Answer: Statutory Deduction or Standard Deduction Section 24(a)
Under section 24(a), every assessee shall be allowed a notional expenditure equal to thirty per cent of the
net annual value of the house for the various expenditures incurred by him.
Actual expenditure incurred by the assessee shall not be taken into consideration.
Example
Net annual value of one house is `3,00,000 and actual expenditure incurred on repairs are `75,000,
deduction allowed under section 24(a) shall be `90,000.

Question 8: Write a note on deduction for interest on the capital borrowed.


Answer: Interest on borrowed capital is allowed as deduction under section 24(b)
If any assessee has taken a loan or advance for purchase/ construction / renovation / addition / alteration /
substitution or repair etc. of the house property, interest on such loan shall be allowed to be deducted under
section 24(b) from NAV and interest is allowed on due basis but only simple interest is allowed i.e. interest
on interest is not allowed. The assessee can take any number of loan. Interest for the year for which income
is being computed shall be allowed in the same year and shall be called current period interest. Interest for
the period prior to the year in which the house was purchased or constructed shall be called prior period
interest and such interest shall be allowed in 5 annual equal instalments starting from the year in which the
house was purchased or constructed. E.g. If Mr. X had taken a loan of `5,00,000 for construction of property
on 01.10.2017 and interest is payable @ 10% p.a. and the construction was completed on 30.06.2018, in this
case interest allowed under section 24(b) shall be:
Interest for the year (01.04.2018 to 31.03.2019) = 10% of ` 5,00,000 = ` 50,000
Prior period interest =10% of ` 5,00,000 for 6 months (from 01.10.2017 to 31.03.2018)=` 25,000
Prior period interest to be allowed in 5 equal annual installments of ` 5,000 from the year of completion of
construction i.e. in this case, P.Y.2018-19.
Therefore, total interest deduction under section 24(b) = 50,000 + 5000 = ` 55,000.
If any assessee has taken a new loan to repay the original loan, in such cases interest for such new loan shall
be allowed in the similar manner.
As per section 25 if loan is taken from outside India, in that case also interest is allowed but the person
making payment of interest should deduct tax at source or the person receiving interest should have an agent
in India.
Illustration 12: Mr. X took a loan of `5,00,000 on 01.10.2015 @ 10% p.a. for construction of house which
was completed on 31.03.2019.
Compute interest on capital borrowed for the previous year 2018-19.
Solution:
Prior period interest
From 01.10.2015 to 31.03.2018
= 5,00,000 x 10% x 30/12 = `1,25,000
Installment = `1,25,000/5 = `25,000
Current year interest
From 01.04.2018 to 31.03.2019
= 5,00,000 x 10% = `50,000
Total Interest = `25,000 + `50,000 = `75,000
Illustration 13: Mr. X has taken a loan of `15,00,000 on 01.07.2014 from State Bank of India @ 12% p.a.
for construction of one house which was completed on 01.05.2018 and was let out @ `90,000 p.m. w.e.f
01.07.2018 and Fair rent is `1,00,000 p.m. and the assessee has paid municipal tax of `30,000 in P.Y. 2018-
19 and the assessee has repaid the loan amount in annual instalment of `1,00,000 starting from 01.01.2017.
Income Under The Head House Property 178

Compute his income tax liability for the assessment year 2019-20.
Solution: `
Computation of income under the head House Property
Gross Annual Value 11,00,000.00
Working Note: `
(a) Fair Rent (1,00,000 x 11) 11,00,000
(b) Expected Rent 11,00,000
(c) Rent received /receivable (90,000 x 9) 8,10,000
If there was no vacancy, in that case rent received/receivable would have been
`9,90,000 and it was still less than expected rent ,therefore GAV shall be expected rent
GAV 11,00,000
Less: Municipal Tax (30,000.00)
Net Annual Value 10,70,000.00
Less: 30% of NAV u/s 24(a) (3,21,000.00)
Less: Interest on capital borrowed u/s 24(b) (2,84,400.00)
Working Note: `
Current period Interest
From 01.04.2018 to 31.03.2019
(13,00,000 x 12% x 9/12) + (12,00,000 x 12% x 3 /12) = 1,53,000
Prior period interest
From 01.07.2014 to 31.03.2018
15,00,000 x 12% x 30/12 = 4,50,000
14,00,000 x 12% x 12/12 = 1,68,000
13,00,000 x 12% x 3/12 = 39,000
Instalment = 6,57,000 / 5 = 1,31,400
Total Interest = `1,53,000 + `1,31,400= 2,84,400
Income under the head house property 4,64,600.00
Gross Total Income 4,64,600.00
Less: Deduction u/s 80C (1,00,000.00)
Total Income 3,64,600.00
Computation of Tax Liability
Tax on normal income `3,64,600 at slab rate 5,730.00
Add: HEC @ 4% 229.20
Tax Liability 5,959.20
Rounded off u/s 288B 5,960.00
Illustration 14: Mr. X has taken a loan of `15,00,000 on 01.07.2014 from State Bank of India @ 12% p.a.
for construction of one house which was completed on 01.04.2017 and was let out @ `90,000 p.m. w.e.f
01.05.2017 and Fair rent is `1,00,000 p.m. and the assessee has paid municipal tax of `30,000 in P.Y. 2018-
19 and the assessee has repaid the loan amount in annual instalment of `1,00,000 starting from 01.01.2017.
Compute his income tax liability for the assessment year 2019-20.
Solution: `
Computation of income under the head House Property
Gross Annual Value 12,00,000.00
Working Note: `
(a) Fair Rent (1,00,000 x 12) 12,00,000
(b) Expected Rent 12,00,000
(c) Rent received /receivable (90,000 x 12) 10,80,000
GAV 12,00,000
Less: Municipal Tax (30,000.00)
Net Annual Value 11,70,000.00
Less: 30% of NAV u/s 24(a) (3,51,000.00)
Less: Interest on capital borrowed u/s 24(b) (2,51,400.00)
Income Under The Head House Property 179

Working Note: `
Current period Interest
From 01.04.2018 to 31.03.2019
(13,00,000 x 12% x 9/12) + (12,00,000 x 12% x 3 /12) = 1,53,000

Prior period interest


From 01.07.2014 to 31.03.2017
15,00,000 x 12% x 30/12 = 4,50,000
14,00,000 x 12% x 3/12 = 42,000
Instalment = 4,92,000 / 5 = 98,400
Total Interest = `1,53,000 + `98,400= 2,51,400
Income under the head house property 5,67,600.00
Gross Total Income 5,67,600.00
Less: Deduction u/s 80C (1,00,000.00)
Total Income 4,67,600.00
Computation of Tax Liability
Tax on normal income `4,67,600 at slab rate 10,880.00
Add: HEC @ 4% 435.20
Tax Liability 11,315.20
Rounded off u/s 288B 11,320.00
Illustration 15: Mr. X has constructed one house on 01.09.2018 and it was let out @ `80,000 p.m. and
municipal taxes paid are `35,000. The house was constructed after taking a loan from outside India and
interest allowed under section 24(b) is `2,10,000, but the assessee has not deducted tax at source.
Compute assessee’s tax liability for assessment year 2019-20.
Solution: `
Gross Annual Value (80,000 x 7) 5,60,000.00
Less: Municipal Taxes (35,000.00)
Net Annual Value 5,25,000.00
Less: 30% of NAV u/s 24(a) (1,57,500.00)
Less: Interest on capital borrowed u/s 24(b) Nil
Income under the head House Property 3,67,500.00

Computation of Tax Liability


Tax on `3,67,500 at slab rate 5,875.00
Add: HEC @ 4% 235.00
Tax Liability 6,110.00
(b) Presume in the above question, the person who has given the loan has one agent in India as per section
163.
Compute tax liability for the assessment year 2019-20.
Solution: `
Gross Annual Value (80,000 x 7) 5,60,000
Less: Municipal Taxes (35,000)
Net Annual Value 5,25,000
Less: 30% of NAV u/s 24(a) (1,57,500)
Less: Interest on capital borrowed u/s 24(b) (2,10,000)
Income under the head House Property 1,57,500
Tax Liability Nil
(c) Presume in the above question, the assessee has deducted tax at source.
Compute tax liability for the assessment year 2019-20.
Solution: `
Gross Annual Value (80,000 x 7) 5,60,000
Less: Municipal Taxes (35,000)
Net Annual Value 5,25,000
Income Under The Head House Property 180

Less: 30% of NAV u/s 24(a) (1,57,500)


Less: Interest on capital borrowed u/s 24(b) (2,10,000)
Income under the head House Property 1,57,500
Tax Liability Nil

MAY – 2007 (6 Marks)


Miss Charlie, an American national, got married to Mr. Radhey of India in USA on 02.03.2018 and came to
India for the first time on 16.03.2018. She left for USA on 23.09.2018.
She returned to India again on 27.03.2019. While in India, she had purchased a show room in Mumbai on
22.04.2018, which was leased out to a company on a rent of `25,000 p.m. from 01.05.2018. She had taken
loan from a bank for purchase of this show room on which bank had charged interest of `97,500 upto
31.03.2019.
She had received the following gifts from her relatives and friends during 01.04.2018 to 30.06.2018:
- From parents of husband ` 51,000
- From married sister of husband ` 11,000
- From two very close friends of her husband, `1,51,000 and `21,000 `1,72,000
Determine her residential status and compute the total income chargeable to tax alongwith the amount of tax
payable on such income for the Assessment Year 2019-20.
Answer.
As per section 6(1), an individual is considered to be resident in India if he stays in India for 182 days or
more or he stays in India for 60 days or more during the relevant previous year and also for 365 days or
more during 4 years preceding the relevant previous year.
Since Miss Charlie is not able to comply with any of the condition mentioned above, she is non-resident in
previous year 2018-19.
Her stay in India during the previous year 2018-19 and in the preceding four years is as under:-
P.Y. 2018-19
01.04.2018 to 23.09.2018 - 176 days
27.03.2019 to 31.03.2019 - 5 days
Total 181 days
Four preceding previous years
P.Y. 2017- 2018 [16.03.2018 to 31.03.2018] - 16 days
P.Y. 2016- 2017 [01.04.2016 to 31.03.2017] - Nil
P.Y. 2015- 2016 [01.04.2015 to 31.03.2016] - Nil
P.Y. 2014- 2015 [01.04.2014 to 31.03.2015] - Nil
Total 16 days

Computation of total income of Miss. Charlie for the A.Y. 2019-20 `


Income from house property
Gross Annual Value [25,000 x 11] 2,75,000
Less: Municipal taxes Nil
Net Annual Value 2,75,000
Less: Standard deduction 30% of NAV u/s 24(a) (82,500)
Less: Interest on loan 24(b) (97,500)
Income under the Head House Property 95,000
Income from other sources
- `51,000 received from parents of husband would be exempt Nil
- `11,000 received from married sister of husband is exempt Nil
- From two friends of husband `1,51,000 and `21,000
aggregating to `1,72,000. 1,72,000
Income under the head Other Sources 1,72,000
Total Income 2,67,000
Computation of tax payable by Miss. Charlie for the A.Y. 2019-20
Tax on total income of `2,67,000 850.00
Income Under The Head House Property 181

Add: HEC @ 4% 34.00


Tax Liability 884.00
Rounded off u/s 288B 880.00
Notes –
1. Actual rent received has been taken as the gross annual value in the absence of other information (i.e.
Municipal value, fair rental value and standard rent) in the question.
2. Rebate under section 87A is not allowed to non-resident.
Question 9: Write a note on one house which is self-occupied.
Answer: One house which is self-occupied Section 23(2)
If any person has one house which is self-occupied, its GAV shall be nil and municipal tax are not allowed
to be deducted and NAV shall also be nil and deduction under section 24(a) is not allowed but deduction
under section 24(b) is allowed but maximum `30,000 however it will be maximum `2,00,000 if loan has
been taken w.e.f 01st April 1999 onwards for purchase or construction and house has been purchased or
constructed within 5 years from the end of the year in which the assessee has taken loan and also assessee
should submit a certificate from the lender certifying the amount of interest. If loan is taken for repairs /
renovations etc. maximum interest allowed shall be `30,000.
If the house is self occupied as well as vacant, its income shall be computed as if it is self occupied house.
E.g. Mr. X has one house which is vacant for 3 months and self occupied for 9 months, its income shall be
computed considering it to be self occupied house.
The ceiling prescribed for one self-occupied property as above in respect of interest on loan borrowed does
not apply to a deemed let-out property.

Illustration 16: Mrs. X has one house property at Indira Nagar in Bangalore. She stays with her family in
the house. The rent of similar property in the neighbourhood is `25,000 p.m. The municipal valuation is
`23,000 p.m. Municipal taxes paid is `8,000. The loan of `20,00,000 was taken on 01.01.2012 from SBI
Housing Finance Ltd. The construction was completed on 30.11.2014. The accumulated interest up to
31.03.2014 is `3,00,000. During the previous year 2018-19, Mrs. X paid ` 1,88,000 which included
`1,44,000 as interest. Compute Mrs. X’s income from house property for A.Y. 2019-20. All the conditions
for higher deduction of interest in case of self-occupied property is satisfied.
Solution:
Computation of income from house property of Mrs. X for A.Y. 2019-20
Particulars Amount
GAV Nil
Less: Municipal Tax Nil
NAV Nil
Less: Statutory Deduction u/s 24(a) Nil

Less: Interest on capital u/s 24(b)


Current period interest `1,44,000
Prior period installment `3,00,000 / 5 = `60,000
Maximum allowed (2,00,000)
Loss from house property (2,00,000)
Illustration 17(a): Mr. X has taken a loan of `5,00,000 on 01.10.1998 @ 10% p.a. for construction of a
house which was completed on 01.10.2017 and the house remained self-occupied throughout the previous
year 2018-19.
The assessee has income under the head salary `4,00,000.
Compute tax liability for assessment year 2019-20.
Solution: `
Net Annual Value Nil
Less: Interest on capital borrowed u/s 24(b) (30,000)
Working Note: `
Current period Interest
From 01.04.2018 to 31.03.2019
Income Under The Head House Property 182

5,00,000 x 10% = 50,000


Prior period interest
From 01.10.1998 to 31.03.2017
5,00,000 x 10% x 222/12 = 9,25,000
Instalment = 9,25,000 / 5 = 1,85,000
Total Interest = `50,000 + `1,85000 = 2,35,000
Subject to maximum `30,000
Loss under the head House Property (30,000)
Income under the head Salary 4,00,000
Gross Total Income 3,70,000
Less: Deduction u/s 80C to 80U Nil
Total Income 3,70,000
Computation of Tax Liability
Tax on `3,70,000 at slab rate 6,000
Add: HEC @ 4% 240
Tax Liability 6,240
Illustration 17(b): Presume in above question, the loan was taken on 01.10.2014. The assessee has
submitted a certificate confirming the amount of interest.
Solution: `
Net Annual Value Nil
Less: Interest on capital borrowed u/s 24(b) (75,000)
Working Note: `
Current period Interest
From 01.04.2018 to 31.03.2019
5,00,000 x 10% = 50,000
Prior period interest
From 01.10.2014 to 31.03.2017
5,00,000 x 10% x 30/12 = 1,25,000
Instalment = 1,25,000 / 5 = 25,000
Total Interest = `50,000 + `25,000 = 75,000
Loss under the head House Property (75,000)
Income under the head Salary 4,00,000
Gross Total Income 3,25,000
Less: Deduction u/s 80C to 80U Nil
Total Income 3,25,000
Computation of Tax Liability
Tax on `3,25,000 at slab rate 3,750.00
Less: Rebate u/s 87A (2,500.00)
Tax before health & education cess 1,250.00
Add: HEC @ 4% 50.00
Tax Liability 1,300.00
NOV – 2013 (5 Marks)
Mr. X owns a residential house in Delhi. The house is having two identical units. First unit of the house is
self-occupied by Mr. X and another unit is rented for `55,000 p.m. The rented unit was vacant for three
months during the year. The particulars of the house for the previous year 2018-19 are as under:
Standard Rent `11,20,000 p.a.
Municipal Valuation `10,44,000 p.a.
Fair Rent `11,35,000 p.a.
Municipal tax paid by Mr. X 12% of the Municipal Valuation
Light and water charges `800 p.m.
Interest on borrowed capital `2,000 p.m.
Insurance charges `3,500 p.a.
Income Under The Head House Property 183

Painting expenses `16,000 p.a.


Compute his income and tax liability of Mr. X for the assessment year 2019-20.
Answer:
Computation of Income from house property of Mr. X for A.Y. 2019-20
Rented unit (50% of total area)
Gross Annual Value (GAV) 4,95,000
(a) Municipal valuation (` 10,44,000 x ½) 5,22,000
(b) Fair rent (` 11,35,000 x ½) 5,67,500
(c) Higher of (a) or (b) 5,67,500
(d) Standard rent (` 11,20,000 x ½) 5,60,000
(e) Expected rent lower of (c) or (d) 5,60,000
(f) Rent receivable for the whole year (` 55,000 x 9) 4,95,000
If there was no vacancy, rent received/receivable would have been 55,000 x 12 =
6,60,000, which is higher than Expected rent, hence GAV shall be rent
received/receivable
Less: Municipal taxes (12% of ` 5,22,000) (62,640)
Net Annual Value (NAV) 4,32,360
Less : Deductions
(a) 30% of NAV under section 24(a) (1,29,708)
(b) Interest on borrowed capital (` 1,000 x 12) u/s 24(b) (12,000)
Taxable income from let out portion 2,90,652
Self occupied unit (50% of total area)
Annual value Nil
Less : Deduction under section 24(b):
Interest on borrowed capital (` 1,000 x 12) (12,000)
Income from house property 2,78,652
Gross Total Income 2,78,652
Less: Deduction u/s 80C to 80U Nil
Total Income (Rounded off u/s 288A) 2,78,650
Computation of Tax Liability
Tax on `2,78,650 at slab rate 1,432.50
Less: Rebate u/s 87A (1,432.50)
Tax Liability Nil
Note: No deduction will be allowed separately for light and water charges, insurance charges and painting
expenses.
NOV – 2008 (5 Marks)
Mr. X owns one residential house in Mumbai. The house is having two units. First unit of the house is self
occupied by Mr. X and another unit is rented for `55,000 p.m. The rented unit was vacant for 2 months
during the year.
The particulars of the house for the previous year 2018-19 are as under:
Standard rent ` 10,62,000 p.a.
Municipal valuation ` 8,90,000 p.a.
Fair rent ` 10,85,000 p. a
Municipal tax 15% of municipal valuation
Light and water charges paid by the tenant ` 500 p.m.
Interest on borrowed capital ` 1,500 p.m.
Insurance charges paid by Mr. X ` 3,000 p.a.
Repairs ` 12,000 p.a.
Compute income from house property of Mr. X and tax liability for the A.Y. 2019-20.
Answer.
Computation of Income from house property for A.Y. 2019-20
(A) Rented unit (50% of total area) `
Gross Annual Value 5,50,000
Income Under The Head House Property 184

Working note: `
(a) Fair rent (`10,85,000 x ½) 5,42,500
(b) Municipal valuation (`8,90,000 x ½) 4,45,000
(c) Higher of (a) or (b) 5,42,500
(d) Standard rent (`10,62,000 x ½) 5,31,000
(e) Expected rent (lower of (c) or (d) 5,31,000
(f) Rent received or receivable (`55,000 x 10) 5,50,000
Since, the actual rent received is higher than the annual letting value, the actual
rent received is the Gross Annual value i.e. `5,50,000
Less: Municipal taxes (15% of `4,45,000) (66,750)
Net Annual value 4,83,250
Less: Deductions under section 24
(i) 30% of net annual value u/s 24(a) 1,44,975
(ii) Interest on borrowed capital (`750 x 12) u/s 24(b) 9,000 (1,53,975)
Taxable income from let out portion 3,29,275
(B) Self occupied unit (50% of total area)
Annual value Nil
Less: Deduction under section 24
Interest on borrowed capital (`750 x 12) u/s 24(b) 9,000 (9,000)
Income from House property 3,20,275
Income under the head House Property 3,20,275
Gross Total Income 3,20,275
Less: Deduction u/s 80C to 80U Nil
Total Income (rounded off u/s 288A) 3,20,280
Computation of Tax Liability
Tax on `3,20,280 at slab rate 3,514.00
Less: Rebate u/s 87A (2,500.00)
Tax before health & education cess 1,014.00
Add: HEC @ 4% 40.56
Tax Liability 1,054.56
Rounded off u/s 288B 1,050.00
Notes:
(i) It is assumed that both the units are of identical size. Therefore, the rented unit would represent 50% of
total area and the self-occupied unit would represent 50% of total area.
(ii) No deduction will be allowed separately for light and water charges, insurance charges and repairs.
MAY – 2000 (3 Marks)
Mr. X commenced construction of a residential house intended exclusively for his residence, on 01.11.2017.
He raised a loan from PNB of `5,00,000 at 16 per cent interest for the purpose of construction on
01.11.2017. Finding that there was an over-run in the cost of construction he raised a further loan of
`8,00,000 at the same rate of interest on 01.10.2018. The assessee has submitted a certificate confirming the
amount of interest. What is the interest allowable under section 24, assuming that the construction was
completed by 31.03.2019?
Answer:
Since the house was for self-occupation only, the annual value of the property would be ‘nil’ under section
23(2). The interest allowable for the current year has to be considered with respect to both the loans.
Interest on loan borrowed after 01.04.1999 is eligible for deduction subject to a maximum of `2,00,000 in
the case of self occupied property.
Prior period interest (upto 31.03.2018)
(5,00,000 x 16% x 5 / 12 ) 33,333
This is to be allowed over 5 years beginning with assessment year 2019-20. Amount
allowable for each year 6,667
Income Under The Head House Property 185

Interest eligible for deduction for the assessment year 2019-20


Prior period interest : One-fifth of `33,333 6,667
Interest on first loan : Current year interest : 5,00,000 @ 16% 80,000
Interest on second loan : 8,00,000 @ 16% x 6/12 64,000
Total interest 1,50,667
Therefore, interest allowable under section 24 would be `1,50,667

Question 10: Write a note on more than one house which are self-occupied (deemed to be let out
property).
Answer: More than one house which are self-occupied (deemed to be let out property) Section 23(4)
If any assessee has more than one house which are self-occupied, in such cases only one of these houses
shall be considered to be self-occupied and income shall be computed under section 23(2) and all other
houses shall be deemed to be let out and income shall be computed in the similar manner as in case of let out
house. Expected rent shall be considered to be GAV of the house.
Illustration 18: Mr. X has 3 houses which are self occupied and the details of these houses is as under.
Particulars House I House II House III
(In `) (In `) (In `)
Fair rent 11,00,000 12,00,000 11,50,000
Municipal valuation 11,24,000 11,78,000 11,25,000
Standard rent 13,00,000 12,50,000 11,40,000
Municipal taxes paid 1,00,000 80,000 90,000
Interest on capital borrowed on 3,20,000 2,90,000 1,90,000
01.04.2013 and all the necessary
conditions are complied with to avail
higher amount of interest.
Repair charges 10,000 3,000 8,000
Date of completion of house 01.10.2015 01.10.2015 01.10.2015
Compute income under the head house property.

Solution:
Option I `
House I is Self Occupied
Loss (2,00,000)
House II is deemed to be Let Out
Gross Annual Value 12,00,000
Working Note: `
(a) Fair rent 12,00,000
(b) Municipal valuation 11,78,000
(c) Higher of (a) or (b) 12,00,000
(d) Standard rent 12,50,000
(e) Expected rent {Lower of (c) or (d)} 12,00,000
GAV = Expected rent 12,00,000
Less: Municipal Taxes (80,000)
Net Annual Value 11,20,000
Less: 30% of NAV u/s 24(a) (3,36,000)
Less: Interest on capital borrowed u/s 24(b) (2,90,000)
Income 4,94,000
House III is deemed to be Let Out
Gross Annual Value 11,40,000
Working Note: `
(a) Fair rent 11,50,000
Income Under The Head House Property 186

(b) Municipal valuation 11,25,000


(c) Higher of (a) or (b) 11,50,000
(d) Standard rent 11,40,000
(e) Expected rent {Lower of (c) or (d)} 11,40,000
GAV = Expected rent 11,40,000
Less: Municipal Taxes (90,000)
Net Annual Value 10,50,000
Less: 30% of NAV u/s 24(a) (3,15,000)
Less: Interest on capital borrowed u/s 24(b) (1,90,000)
Income 5,45,000
Income under Option I [(2,00,000) + 4,94,000 + 5,45,000] 8,39,000
Option II
House I is deemed to be Let Out
Gross Annual Value 11,24,000
Working Note: `
(a) Fair rent 11,00,000
(b) Municipal valuation 11,24,000
(c) Higher of (a) or (b) 11,24,000
(d) Standard rent 13,00,000
(e) Expected rent {Lower of (c) or (d)} 11,24,000
GAV = Expected rent 11,24,000
Less: Municipal Taxes (1,00,000)
Net Annual Value 10,24,000
Less: 30% of NAV u/s 24(a) (3,07,200)
Less: Interest on capital borrowed u/s 24(b) (3,20,000)
Income 3,96,800
House II is Self Occupied
Loss (2,00,000)
House III is deemed to be Let Out
Income 5,45,000
Income under Option II [3,96,800 + (2,00,000) + 5,45,000] 7,41,800
Option III
House I is deemed to be Let Out
Income 3,96,800
House II is deemed to be Let Out
Income 4,94,000
House III is Self Occupied
Loss (1,90,000)
Income under Option III [3,96,800 + 4,94,000 + (1,90,000)] 7,00,800
Third Option is the best
Income under the head House Property 7,00,800
NOV – 1999 (6 Marks)
Mr. X occupied two flats for his residential purposes, particulars of which are as follows:
Particulars Flat I(in `) Flat II(in `)
Municipal Valuation 90,000 45,000
Fair Rent 1,20,000 40,000
Fair rent under Rent Control Act
(i.e. Standard Rent) 80,000 Not available
Municipal taxes paid 10% of municipal valuation 10% of municipal valuation
Fire insurance paid 1,000 600
Interest payable on capital borrowed
for purchase of flat 40,000 Nil
Income Under The Head House Property 187

Income of Mr. X from his Proprietary business– XYZ Warehousing Corporation is `6,50,000.
Determine the taxable income and tax liability for the assessment year 2019-20. You are informed that Mr.
X could not occupy flat for 2 months commencing from December 1st, 2018 and that he has attained the age
of 65 on 23.08.2018.
Answer: `
Income shall be computed as per Section 23(4)
Option I
Flat I is Self Occupied Sec 23(2)
Income (30,000.00)
Flat II is Let out Sec 23(1)
Gross Annual Value 45,000.00
Working Note: `
(a) Fair Rent 40,000
(b) Municipal Valuation 45,000
(c) Expected Rent {Higher of (a) or (b) 45,000
GAV = Expected Rent 45,000
Less: Municipal taxes (4,500.00)
Net Annual Value 40,500.00
Less: 30% of NAV u/s 24(a) (12,150.00)
Less: Interest on capital borrowed u/s 24(b) Nil
Income 28,350.00
Income under Option I (1,650.00)

Option II
Flat I is Let out Sec 23(1)
Gross Annual Value 80,000.00
Working Note: `
(a) Fair Rent 1,20,000
(b) Municipal Valuation 90,000
(c) Higher of (a) or (b) 1,20,000
(d) Standard Rent 80,000
(e) Expected Rent {Lower of (c) or (d)} 80,000
GAV = Expected Rent 80,000
Less: Municipal taxes (9,000.00)
Net Annual Value 71,000.00
Less: 30% of NAV u/s 24(a) (21,300.00)
Less: Interest on capital borrowed u/s 24(b) (40,000.00)
Income 9,700.00
Flat II is Self occupied Sec 23(2)
Income Nil
Income under Option II is 9,700.00
Hence Option I is better.
Computation of Gross Total Income
Income under the head House Property (1,650.00)
Income under the head Business/Profession 6,50,000.00
Gross Total Income 6,48,350.00
Less: Deduction u/s 80C to 80U Nil
Total Income 6,48,350.00
Computation of Tax Liability
Tax on `6,48,350 at slab rate 39,670.00
Add: HEC @ 4% 1,586.80
Tax Liability 41,256.80
Rounded off u/s 288B 41,260.00
Income Under The Head House Property 188

MAY – 2014 (5 Marks)


Mrs. X has two houses, both of which are self occupied. The particulars of these are given below:
Particulars (Value in `)
House — I House — II
Municipal Valuation per annum 1,20,000 1,15,000
Fair Rent per annum 1,50,000 1,75,000
Standard rent per annum 1,00,000 1,65,000
Date of completion 31-03-1999 31-03-2001
Municipal taxes payable during the year (paid for House II only) 12% 8%
Interest on money borrowed for repair of property during current year - 55,000
Compute Mrs. X’s income from the House Property for the Assessment Year 2019-20 and suggest which
house should be opted by Mrs. X to be assessed as self occupied so that her Tax liability is minimum.
Solution:
In this case, Mrs. X has more than one house property for self-occupation. As per section 23(4), Mrs. X can
avail the benefit of self-occupation (i.e., benefit of “Nil” Annual Value) only in respect of one of the house
properties, at her option. The other house property would be treated as “deemed let-out” property, in respect
of which the annual letting value would be the gross annual value. Mrs. X should, therefore, consider the
most beneficial option while deciding which house property should be treated by her as self-occupied.
Option 1
House I – Self-occupied `
Income Nil
House II – Deemed to be let out
Gross Annual Value 1,65,000
Working Note: `
(a) Fair rent 1,75,000
(b) Municipal valuation 1,15,000
(c) Higher of (a) or (b) 1,75,000
(d) Standard rent 1,65,000
(e) Expected rent {Lower of (c) or (d)} 1,65,000
GAV = Expected rent 1,65,000
Less: Municipal Taxes (9,200)
Net Annual Value 1,55,800
Less: 30% of NAV u/s 24(a) (46,740)
Less: Interest on capital borrowed u/s 24(b) (55,000)
Income 54,060
Income under Option I [Nil + 54,060] 54,060
Option 2
House I – Deemed to be let out
Gross Annual Value 1,00,000
Working Note: `
(a) Fair rent 1,50,000
(b) Municipal valuation 1,20,000
(c) Higher of (a) or (b) 1,50,000
(d) Standard rent 1,00,000
(e) Expected rent {Lower of (c) or (d)} 1,00,000
GAV = Expected rent 1,00,000
Less: Municipal Taxes Nil
Net Annual Value 1,00,000
Less: 30% of NAV u/s 24(a) (30,000)
Less: Interest on capital borrowed u/s 24(b) Nil
Income 70,000
Income Under The Head House Property 189

House II – Self-occupied
Loss (30,000)
Income under Option II [70,000 + (30,000)] 40,000
Second Option is better
Income under the head House Property 40,000

Question 11: Write a note on computation of Income of Unoccupied House.


Answer: Income of unoccupied house section 23(2)(b)
As per section 23(2)(b),if any house cannot actually be occupied by the owner by reason of the fact that
owing to his employment, business or profession carried on at any other place, he has to reside at that other
place in a building not belonging to him, in such cases assessee shall have the option to compute income of
such house as if it is self-occupied i.e. it will not be considered to be deemed to be let out.

Question 12: Write a note on house property which is divided into different portions/units.
Answer: If any house property is divided into different portions, every portion shall be considered to be a
separate house and income shall be computed accordingly. There is no need to treat the whole property as a
single unit for computation of income from house property.
Municipal valuation/fair rent/standard rent, if not given separately, shall be apportioned between the let-out
portion and self-occupied portion either on plinth area or built-up floor space or on such other reasonable
basis.
Property taxes, if given on a consolidated basis can be bifurcated as attributable to each portion or floor on a
reasonable basis.
Illustration 19: Mr. X owns a house in Madras. During the previous year 2018-19, 2/3rd portion of the
house was self-occupied and 1/3rd portion was let out for residential purposes at a rent of ` 8,000 p.m.
Municipal value of the property is `3,00,000 p.a., fair rent is `2,70,000 p.a. and standard rent is ` 3,30,000
p.a. He paid municipal taxes @ 10% of municipal value during the year. A loan of `25,00,000 was taken by
him during the year 2014 for acquiring the property. Interest on loan paid during the previous year 2018-19
was `1,20,000. Compute Mr. X’s income from house property for the A.Y. 2019-20. All the conditions for
higher deduction of interest in case of self-occupied property is satisfied.
Solution:
There are two units of the house. Unit I with 2/3rd area is used by Mr. X for self-occupation throughout the
year and no benefit is derived from that unit, hence it will be treated as self-occupied and its annual value
will be nil. Unit 2 with 1/3rd area is let-out through out the previous year and its annual value has to be
determined as per section 23(1).
Computation of income from house property of Mr. X for A.Y. 2019-20
Particulars `
Unit I (2/3rd area – self-occupied)
Annual Value Nil
Less: Deduction under section 24(b) 2/3rd of ` 1,20,000 (80,000)
Loss from Unit I (self-occupied) (80,000)
Unit II (1/3rd area – let out)
Gross Annual Value 1,00,000
Working Note: `
(a) Fair rent 90,000
(b) Municipal valuation 1,00,000
(c) Higher of (a) or (b) 1,00,000
(d) Standard rent 1,10,000
(e) Expected rent {Lower of (c) or (d)} 1,00,000
(f) Rent received/ receivable (8,000 x 12) 96,000
GAV = Expected rent 1,00,000
Less: Municipal Taxes (10,000)
Net Annual Value 90,000
Less: 30% of NAV u/s 24(a) (27,000)
Income Under The Head House Property 190

Less: Interest on capital borrowed u/s 24(b) (40,000)


Income under the head House Property 23,000
Net Loss (57,000)
Question 13: Write a note on house property owned by the assessee and used for own
business/profession.
Answer: House property owned by the assessee and used for own business/profession Section
22/section 30
If any person owns any house property and it is being used by him in his own business/profession, income of
such building shall not be computed under the head house property rather income shall be computed under
the head business/profession and as per section 30, for this purpose, while computing the income under the
head business/profession, no rent shall be allowed to be debited to the profit & loss account in connection
with such building. All the expenses of the house property shall be debited to the profit and loss account and
deduction under section 24(a) is not allowed rather actual expenditure shall be debited to the profit and loss
account. Such expenditure may be municipal tax, repairs, depreciation, land revenue, ground rent etc.
Illustration 20: Mrs. X aged 62 years is engaged in a business in her own building and furnishes the
following information.
Market rent of the building is `1,00,000 p.m. and expenses incurred on repairs are `37,000 and interest on
loan taken for construction of the building is `65,000 and depreciation `30,000 and municipal tax paid
`30,000 and land revenue paid `10,000 and premium paid for insurance of the house `7,000. ground rent
paid `8,000.
Income from business before debiting any expense of house property is `16,00,000.
Compute her income tax liability for Assessment Year 2019-20.
Solution: `
Income from business before debiting any expense of house property 16,00,000.00
Less: Repair of Building (37,000.00)
Less: Interest on loan taken for construction of building (65,000.00)
Less: Depreciation (30,000.00)
Less: Municipal Taxes (30,000.00)
Less: Land revenue (10,000.00)
Less: Insurance premium of the house (7,000.00)
Less: Ground rent (8,000.00)
Income under the head Business/Profession 14,13,000.00
Gross Total Income 14,13,000.00
Less: Deduction u/s 80C to 80U Nil
Total Income 14,13,000.00
Computation of Tax Liability
Tax on `14,13,000 at slab rate 2,33,900.00
Add: HEC @ 4% 9,356.00
Tax Liability 2,43,256.00
Rounded off u/s 288B 2,43,260.00
Question 14: Write a note on a house property which is let-out for part of the year and self-occupied
for part of the year and may or may not be vacant.
Answer: A house property which is let-out for part of the year and self-occupied for part of the year
and may or may not be vacant Section 23(3)
If any house property is let out as well as self-occupied, in such cases expected rent (also called annual
letting value) shall be computed for full year but Rent received/receivable shall be only for the period the
house was let out and GAV shall be the higher. There will not be any such adjustment as in case of vacancy.
Illustration 21: Mr. X constructed one house in 2017 and it is let out for 4 months and self occupied for 6
months and vacant for 2 months during previous year 2018-19. Municipal valuation of the house is `40,000
p.m. and fair rent `30,000 p.m. Standard rent of the house is `38,000 p.m. It was let out @ `32,000 p.m.
Municipal tax levied is `6,000 out of which `2,000 was paid by the tenant and `2,000 by the assessee and
balance `2,000 yet to be paid.
Interest on the capital borrowed for construction of the house is `30,000.
Income Under The Head House Property 191

Long Term Capital Gains is `2,10,000


Compute his income and tax Liability for the assessment year 2019-20.
Solution: `
Computation of income from House Property of Mr. X
Gross Annual Value 4,56,000
Working Note: `
(a) Fair Rent (30,000 x 12) 3,60,000
(b) Municipal Valuation (40,000 x 12) 4,80,000
(c) Higher of (a) or (b) 4,80,000
(d) Standard Rent (38,000 x 12) 4,56,000
(e) Expected Rent {Lower of (c) or (d)} 4,56,000
(f) Rent Received/Receivable (32,000 x 4) 1,28,000
If there was no vacancy, in that case rent received/receivable would have been
`1,92,000 and it was still less than expected rent, therefore GAV shall be expected
rent.
GAV 4,56,000
Less: Municipal Taxes (2,000)
Net Annual Value 4,54,000
Less: 30% of NAV u/s 24(a) (1,36,200)
Less: Interest on capital borrowed u/s 24(b) (30,000)
Income under the head House Property 2,87,800
Long Term Capital Gains 2,10,000
Gross Total Income 4,97,800
Less: Deduction u/s 80C to 80U Nil
Total Income 4,97,800
Computation of Tax Liability
Tax on `2,10,000 @ 20% u/s 112 42,000.00
Tax on `2,87,800 at slab rate 1,890.00
Tax before health & education cess 43,890.00
Add: HEC @ 4% 1,755.60
Tax Liability 45,645.60
Rounded off u/s 288B 45,650.00
Illustration 22: Mrs. X owns a house property at Adyar in Chennai. The municipal value of the property is
`5,00,000, fair rent is ` 4,20,000 and standard rent is ` 4,80,000. The property was let-out for `50,000 p.m.
up to December 2018. Thereafter, the tenant vacated the property and Mrs. X used the house for self-
occupation. Rent for the months of November and December 2018 could not be realised in spite of the
owner’s efforts. All the conditions prescribed under Rule 4 are satisfied. She paid municipal taxes @ 12%
during the year. She had paid interest of ` 25,000 during the year for amount borrowed for repairs for the
house property. She has LTCG `110,00,000. She has completed age of 80 years as on 31.03.2019. Compute
her tax liability for the A.Y. 2019-20.
Solution:
Computation of income from house property of Mrs. X for the A.Y.2019-20
Gross Annual Value 4,80,000.00
Working Note: `
(a) Fair rent 4,20,000
(b) Municipal valuation 5,00,000
(c) Higher of (a) or (b) 5,00,000
(d) Standard rent 4,80,000
(e) Expected rent {Lower of (c) or (d)} 4,80,000
(f) Rent received/ receivable (50,000 x 7) 3,50,000
(unrealised rent 2 months and self occupied 3 months)
GAV = Expected rent 4,80,000
Income Under The Head House Property 192

Less: Municipal Taxes (60,000.00)


Net Annual Value 4,20,000.00
Less: 30% of NAV u/s 24(a) (1,26,000.00)
Less: Interest on capital borrowed u/s 24(b) (25,000.00)
Income under the head House Property 2,69,000.00
LTCG 110,00,000.00
Total Income 112,69,000.00
Normal income 2,69,000.00
LTCG (110,00,000 – 2,31,000) x 20% 21,53,800.00
Add: Surcharge @ 15% 3,23,070.00
Tax before health & education cess 24,76,870.00
Add: HEC @ 4% 99,074.80
Tax Liability 25,75,944.48
Rounded off u/s 288B 25,75,940.00
NOV – 2001 (4 Marks)
From the following particulars furnished by Mr. X for the previous year ending 31.03.2019. Compute the
taxable income and tax liability for assessment year 2019-20:
(i) He owns a house property at metro city. The fair rental value per annum is ` 27,000 and the municipal
value is ` 24,000.
(ii) The house was let out from 01.04.2018 to 31.08.2018 @ `2,100 per month. From 01.09.2018 Mr. X
occupies the house for his residence.
(iii) Expenditure incurred on property and paid:
(a) Municipal tax `4,000
(b) Fire insurance `2,500
(c) Land revenue `4,600
(d) Repairs `1,000
(iv) Interest paid on borrowings for construction:
(a) For the current year `21,600
(b) Instalment of prior period `12,960
He has long term capital gains of `5,00,000.
Answer:
Computation of income under the head House Property `
Gross Annual Value 27,000.00

Working Note: `
(a) Fair Rent 27,000
(b) Municipal Valuation 24,000
(c) Higher of (a) or (b) 27,000
(d) Expected rent 27,000
(e) Rent Received/Receivable (2,100 x 5) 10,500
GAV = Higher of (d) or (e) 27,000
Less: Municipal taxes (4,000.00)
Net Annual Value 23,000.00
Less: 30% of NAV u/s 24(a) (6,900.00)
Less: Interest on capital borrowed u/s 24(b) (21,600 +12,960) (34,560.00)
Loss from house property (18,460.00)
Income under the head Capital Gains (LTCG) 5,00,000.00
Income under the head Capital gains after adjusting loss from house property 4,81,540.00
Gross Total Income 4,81,540.00
Less: Deduction u/s 80C to 80U Nil
Total Income 4,81,540.00
Computation of Tax Liability
Tax on `2,31,540 (`4,81,540 – `2,50,000) @ 20% u/s 112 46,308.00
Income Under The Head House Property 193

Add: HEC @ 4% 1,852.32


Tax Liability 48,160.32
Rounded off u/s 288B 48,160.00
Illustration 23: Mr. X has one big house. 25% of it is being used by the assessee in his own
business/profession and 50% of the house is let out @ `10,000 p.m. However, it remained vacant for one
month and there is unrealised rent for 1½ month. Remaining 25% is self occupied throughout the year. Fair
rent of the entire house is `25,000 p.m., municipal valuation `22,000 p.m. and municipal tax paid is
`22,000. Insurance premium paid is `6,000, repair charges `8,000, land revenue paid `4,000, ground rent is
`3,000 and depreciation of the house is `12,000. Assessee’s income under the head business/profession
before charging expenditure relating to house property is `2,00,000.
Compute his total income and tax liability for assessment year 2019-20.
Solution: `
Computation of income under the head House Property
Income from self occupied portion
Income of self occupied portion Nil
Income of let out portion
Gross Annual Value 1,50,000.00
Working Note: `
(a) Fair Rent (12,500 x 12) 1,50,000
(b) Municipal Valuation (11,000 x 12) 1,32,000
(c) Expected rent {Higher of (a) or (b)} 1,50,000
(d) Rent Received/Receivable (10,000 x 9.5) 95,000
If there was no vacancy, in that case rent received/receivable would have been
`1,05,000 and it was still less than expected rent, therefore GAV shall be expected
rent
GAV 1,50,000
Less: Municipal taxes (11,000.00)
Net Annual Value 1,39,000.00
Less: 30% of NAV u/s 24(a) (41,700.00)
Less: Interest on capital borrowed u/s 24(b) Nil
Income under the head House Property 97,300.00
Computation of income under the head Business/Profession
Income before debiting any expense of the house property 2,00,000.00
Less: Municipal taxes (5,500.00)
Less: Insurance premium (1,500.00)
Less: Repairs charges (2,000.00)
Less: Land revenue (1,000.00)
Less: Ground Rent (750.00)
Less: Depreciation (3,000.00)
Income under the head Business/Profession 1,86,250.00
Computation of Total Income
Income under the head House Property 97,300.00
Income under the head Business/Profession 1,86,250.00
Gross Total Income 2,83,550.00
Less: Deductions u/s 80C to 80U Nil
Total Income 2,83,550.00
Computation of Tax Liability
Tax on `2,83,550 at slab rate 1,677.50
Less: Rebate u/s 87A (1,677.50)
Tax Liability Nil
Income Under The Head House Property 194

NOV – 2003 (5 Marks)


Mr. A and B constructed their houses on a piece of land purchased by them at New Delhi. The built up area
of each house was 1,000 sq. ft. ground floor and an equal area in the first floor.
A started construction on 01.04.2017 and completed on 31.03.2018. A occupied the entire house on
01.04.2018. A has availed a housing loan of `20 lakhs @ 12% p.a. on 01.04.2017 and has also submitted a
certificate from the lender certifying the amount of interest.
B started construction on 01.07.2017 and completed on 01.07.2018. B occupied the ground floor on
01.07.2018 and let out the first floor for a rent of `15,000 per month. However, the tenant vacated the house
on 31.12.2018 and B occupied the entire house during the period 01.01.2019 to 31.03.2019. B has availed a
housing loan of `12 lakhs @ 10% p.a. on 01.07.2017 and has also submitted a certificate from the lender
certifying the amount of interest.
Following are the other information: `
(i) Fair rental value of each unit 6,00,000 per annum
(Ground floor / First floor)
(ii) Municipal value of each unit 3,00,000 per annum
(Ground floor / First floor)
(iii) Municipal taxes paid by A - 8,000
B - 8,000
(iv) Repair and maintenance charges paid by A - 28,000
B - 30,000
No repayment was made by either of them till 31.03.2019.
Compute tax liability for the assessment year 2019-20.
Answer:
Computation of income from House Property of Mr. A `
Net Annual Value Nil
Less: 30% of NAV 24(a) Nil
Less: Interest on capital borrowed u/s 24(b) (2,00,000)
Interest paid on borrowed capital
= 20,00,000 @ 12 % = `2,40,000
Interest deduction restricted to `2,00,000
There is no prior period interest
Loss under the head “House Property” (2,00,000)
Tax Liability Nil
Computation of income from House Property of Mr. B
Ground floor (self occupied)
Net Annual Value Nil
Less: 30% of NAV u/s 24(a) Nil
Less: Interest on capital borrowed u/s 24(b) (69,000)
Working Note:
Prior period interest
From 01.07.2017 to 31.03.2018
= 12,00,000 x 10% x 9 / 12 = 90,000
90,000 allowed in 5 equal instalments
= 90,000 / 5 = ` 18,000 per annum
= 18,000 / 2 = `9,000
Current period interest
From 01.04.2018 to 31.03.2019
= 12,00,000 x 10% x 1/2 = `60,000
Total Interest = `60,000 + ` 9,000 = `69,000
Loss from House Property (69,000)
First floor (Let out)
Gross Annual Value 4,50,000
Income Under The Head House Property 195

Working Note: `
(a) Fair Rent (6,00,000 x 9/12) 4,50,000
(b) Municipal Valuation (3,00,000 x 9/12) 2,25,000
(c) Higher of (a) or (b) 4,50,000
(d) Expected Rent 4,50,000
(e) Rent Received/Receivable (15,000 x 6) 90,000
GAV = Higher of (d) or (e) 4,50,000
Less: Municipal taxes (4,000)
Net Annual Value 4,46,000
Less: 30% of NAV u/s 24(a) (1,33,800)
Less: Interest on capital borrowed u/s 24(b) (69,000)
Working Note:
Prior period interest
From 01.07.2017 to 31.03.2018
= 12,00,000 x 10% x 9 / 12 = 90,000
90,000 allowed in 5 equal instalments
= 90,000 / 5 = ` 18,000 per annum
= 18,000 / 2 = `9,000
Current period interest
From 01.04.2018 to 31.03.2019
= 12,00,000 x 10% x 1/2 = `60,000
Total Interest = `60,000 + ` 9,000 = `69,000
Income from House Property 2,43,200
Income under the head “Income from House Property” of Mr. B 1,74,200
(Both ground floor and first floor)
Tax Liability Nil
Question 15: Write a note on composite rent.
Answer: Composite Rent
A person may let out a house property alongwith some facilities like generator or security etc. and rent may
be charged combined for the house property as well as facility, such rent is called composite rent and in such
cases rent for house property shall be taxable under the head house property and rent for facilities shall be
taxable under the head other sources but after deducting all the expenses relating to such facility.
If there are so many facilities and bifurcation is not possible, income shall be taxable under the head
business/profession or other sources e.g. in case of hotel business/paying guest accommodation/warehousing
or auditorium etc., income is taxable under the head business/profession.
Illustration 24: Mr. X has let out one house alongwith generator facility and has charged a sum of `40,000
p.m. as rent, out of which `3,000 p.m. is attributable to the generator. He has paid `2,300 and the tenant has
paid `900 towards municipal taxes. The interest on the capital borrowed for construction of the house is
`7,000. Mr. X has paid repair charge of the generator `3,400, fuel charges `5,600 and operator’s salary `300
p.m.
Compute the tax liability of Mr. X for assessment year 2019-20.
Solution: `
Computation of income under the head House Property
Gross Annual Value (37,000 x 12) 4,44,000.00
Less: Municipal Taxes (2,300.00)
Net Annual Value 4,41,700.00
Less: 30% of NAV u/s 24(a) (1,32,510.00)
Less: Interest on capital borrowed u/s 24(b) (7,000.00)
Income under the head House Property 3,02,190.00
Computation of income under the head Other Sources
Income from generator 36,000.00
(3,000 x 12)
Income Under The Head House Property 196

Less: Repair charges (3,400.00)


Less: Fuel charges (5,600.00)
Less: Operator Salary (3,600.00)
(300 x 12)
Income under the head Other Sources 23,400.00
Computation of Total Income
Income under the head House Property 3,02,190.00
Income under the head Other Sources 23,400.00
Gross Total Income 3,25,590.00
Less: Deduction u/s 80C to 80U Nil
Total Income 3,25,590.00
Computation of Tax Liability
Tax on `3,25,590 at slab rate 3,779.50
Less: Rebate u/s 87A (2,500.00)
Tax before health & education cess 1,279.50
Add: HEC @ 4% 51.18
Tax Liability 1,330.68
Rounded off u/s 288B 1,330.00

Question 16: Write a note on letting out of building which is supplementary to the business.
Answer: Letting out of building which is supplementary to the business
If any person has let out any house property for any purpose which is supplementary to the business of the
assessee, in such cases rental income shall be taxable under the head business/profession and all expenses of
such house property shall be debited to the profit and loss account. E.g. If a Public school has let out a part
of its building to a Bank, in this case rent received shall be considered to be income under the head
Business/Profession and all expenses of such house property shall be debited to profit and loss account.
Similarly, if any company has constructed houses for the employees in their premises and it is let out to the
employees, rental income is taxable under the head Business/Profession.

Question 17: Write a note on tax liability in respect of arrears of rent.


Answer: Tax liability in respect of arrears of rent Section 25A
Sometimes rent of a house property may be increased from retrospective effect i.e. from back date and the
assessee may receive arrear of rent, such arrears are taxable in the year in which arrears have been received,
however deduction shall be allowed @ 30% of such arrears and only the balance amount shall be taxable. It
do not matter that the assessee do not have any house property in his name in that year.
There is no specific provision given in the Income Tax Act relating to advance payment of rent.
Illustration 25: Mr. X has let out his house to State Bank @ `20,000 p.m. The bank has increased the rent
on 1st July, 2018 to `27,000 p.m. retrospectively w.e.f. 01.11.2017. The assessee has paid municipal taxes of
`7,000 during the previous year 2018-19.
Compute income under the head House Property for assessment year 2019-20.
Solution:
Computation of income under the head House Property ` `
Gross Annual Value (27,000 x 12) 3,24,000
Less: Municipal Taxes (7,000)
Net Annual Value 3,17,000
Less: 30% of NAV u/s 24(a) (95,100)
Less: Interest on capital borrowed u/s 24(b) Nil
2,21,900
Add: Arrears of rent (Sec 25A) (7,000 x 5) 35,000
Less: 30% of `35,000 (10,500) 24,500
Income under the House Property 2,46,400
Income Under The Head House Property 197

MAY – 2002 (4 Marks)


Mr. X owns a house property which is let out. During the previous year ending 31.03.2019 he receives (i)
arrears of rent of `30,000 and (ii) unrealised rent of `20,000.
You are requested to
(a) state, how they should be dealt with as per the provisions of the Act, and
(b) compute the income chargeable under the head “Income from house property”.
Answer:
(a) As per provisions of section 25A, arrears of rent will be charged to tax as income from house property in
the previous year in which such rent is received, after deducting a sum equal to 30% of such amount. The
taxability shall be there whether Mr. X remains as the owner of the property in the concerned year or not. In
this case, it shall be taxed as income from house property in the year of receipt of such arrear rent.
(b) As per the provisions of section 25A, the unrealised rent when received, it shall be deemed to be the
income chargeable under the head “Income from house property” and shall be charged to tax in the year of
receipt, after deducting a sum equal to 30% of such amount. In this case also, the taxability shall be there,
irrespective of the fact whether Mr. X is the owner of property or not in the year of receipt.
Computation of income from house property
`
Arrears of rent 30,000
Less : Deduction @ 30% of `30,000/- u/s 25A (9,000)
21,000
Add : Unrealised rent received 20,000
Less : Deduction @ 30% of `20,000/- u/s 25A (6,000)
14,000
Income from house property 35,000

Question 18: Write a note on sub-letting of house property.


Answer: Sub-letting of house property Section 56
If any person has let out any house property and the tenant has further given the same house property on
rent, it is called sub-letting and in this case, income of the person who has sub-let the house property, shall
be computed under the head Other Sources and income shall be equal to gross rent received – expenses
incurred for such house property.
Example
Mr. X has let out one house to Mr. Y at a rent of `1,00,000 p.m. and has paid municipal tax of `1,00,000.
Mr. Y has sub-let 50% of the house property to Mr. Z at a rent of `70,000 p.m., in this case income of Mr. X
and Mr. Y shall be computed in the manner given below:
Income of Mr. X shall be computed under the head House Property
GAV (1,00,000 x 12) 12,00,000
Less: Municipal Tax (1,00,000)
NAV 11,00,000
Less: Deduction u/s 24(a) (3,30,000)
Income under the head House Property 7,70,000
Income of Mr. Y shall be computed under the head Other Sources
Gross Rent received (70,000 x 12) 8,40,000
Less: Rent paid by him (1,00,000 x 50% x 12) (6,00,000)
Income under the head Other Sources 2,40,000

Question 19: Write a note on computation of income from house property situated outside India.
Answer:
Income of house property situated outside India shall be computed in the similar manner as in case of house
property situated in India and such income shall be taxable in the case of ROR. In case of NR or NOR such
income is exempt provided income is received outside India i.e. if income is received in India, it will be
taxable in case of NR/NOR also.
Income Under The Head House Property 198

Illustration 26: Mr. X, a British national, is a resident and ordinarily resident in India during the P.Y.2018-
19. He owns a house in London, which he has let out at £ 10,000 p.m. The municipal taxes paid to the
Municipal Corporation of London is £ 8,000 during the P.Y.2018-19. The value of one £ in Indian rupee to
be taken at ` 82.50. Compute Mr. X’s taxable income for the A.Y. 2019-20.

Solution:
For the P.Y.2018-19, Mr. X, a British national, is resident and ordinarily resident in India. Therefore,
income received by him by way of rent of the house property located in London is to be included in the total
income in India. Municipal taxes paid in London is be to allowed as deduction from the gross annual value.
Computation of Income from house property of Mr. X for A.Y. 2019-20
Particulars `
Gross Annual Value (£ 10,000 × 12 × 82.50) 99,00,000
Less: Municipal taxes paid (£ 8,000 × 82.50) (6,60,000)
Net Annual Value (NAV) 92,40,000
Less: Deduction under section 24
(a) 30% of NAV = 30% of ` 92,40,000 (27,72,000)
Income from house property 64,68,000

NOV – 2009 (4 Marks)


Mrs. X, a resident and ordinarily resident individual, owns a house in U.S.A. She receives rent @ $ 2,000
per month. She paid municipal taxes of $ 1,500 during the financial year 2018-19.
She also owns a two storied house in Mumbai, ground floor is used for her residence and first floor is let out
at a monthly rent of `10,000.
Municipal taxes paid for the house amounts to `7,500. Mrs. X had constructed the house by taking a loan
from a nationalized bank on 20.06.2015. She repaid the loan of `54,000 including interest of `24,000 in the
current year.
The value of one dollar is to be taken as `45.
Compute total income from house property and also tax liability of Mrs. X for assessment year 2019-20.

Answer.
Computation of Income from House Property of Mrs. X for the Assessment Year 2019-20 `
GAV of the house in USA
($2000 p.m. x `45 per USD x 12 months) 10,80,000.00
Less: Municipal taxes paid ($1500 x `45 per USD) (67,500.00)
Net Annual Value 10,12,500.00
Less: Statutory deduction under section 24(a) @ 30% of NAV (3,03,750.00)
Income from House property 7,08,750.00
GAV of house at Mumbai (let out portion)- Ist Floor
Gross Annual Value (10,000 x 12) 1,20,000.00
Less: Municipal taxes paid (1/2 of `7,500) (3,750.00)
Net Annual Value (NAV) 1,16,250.00
Less: Statutory deduction under section 24(a) @ 30% of NAV (34,875)
Less: Interest on Housing loan (1/2 of `24,000) 24(b) (12,000)
Income from House property 69,375.00
GAV of house at Mumbai (self occupied portion) – Ground Floor Nil
Less: Municipal taxes Nil
Net Annual Value Nil
Less: Statutory deduction under section 24(a) @ 30% of NAV Nil
Less: Interest on Housing Loan (1/2 of `24,000) 24(b) (12,000.00)
Loss from House property (12,000.00)
Income from House property 7,66,125.00
Gross Total Income 7,66,125.00
Less: Deduction u/s 80C (Repayment of housing loan) (30,000.00)
Income Under The Head House Property 199

Total Income (rounded off u/s 288A) 7,36,130.00


Computation of Tax Liability
Tax on `7,36,130 at slab rate 59,726.00
Add: HEC @ 4% 2,389.04
Tax Liability 62,115.04
Rounded off u/s 288B 62,120.00
Question 20: Write a note on treatment of income from co-owned property.
Answer: Treatment of income from co-owned property Section 26
If any house property is owned by co-owners and their shares are given, in such cases each such co-owner
has to pay tax on his share in the income of house property but if shares are not given, it will be considered
to be income of co-owners (BOI/AOP) e.g. Mr. X and Mr. Y are co-owners of a house property and their
shares are not given and income is `20 lakhs, in this case it will be taxable as income of co-owners but if
share of each one is 50%, Mr. X will pay tax on income of `10 lakh and Mr. Y will pay tax on income of
`10 lakh.
Where the house property owned by co-owners is self occupied by each of the co-owners, the annual value
of the property of each co-owner will be Nil and each co-owner shall be entitled to a deduction of `30,000 /
` 2,00,000, as the case may be, under section 24(b) on account of interest on borrowed capital e.g. Mr. X
and Mr. Y are co-owners of a particular house property and it is self occupied by each one of them and each
one has share of 50% and interest on loan taken for construction of a house is `5 lakh and loan was taken
w.e.f 01.04.1999 onwards and house was completed within 5 years and a certificate has been submitted
confirming the amount of interest, in this case loss of Mr. X shall be `2 lakh and loss of Mr. Y shall be `2
lakh.
If share of individual, co-owner is not given, it will be considered to be loss of BOI and loss can be
maximum ` 2,00,000.
If any house property is owned by a partnership firm or company, it will be considered to be income of
partnership firm or company and not that of partners or shareholders.

NOV – 2009 (4 Marks)


Mr. X is a co-owner of a house property alongwith his brother.
Municipal value of the Property 1,60,000
Fair Rent 1,50,000
Standard Rent under the Rent Control Act 1,70,000
Rent received 15,000 p.m.
The loan for the construction of this property is jointly taken and the interest charged by the bank is `25,000
out of which `21,000 have been paid. Interest on the unpaid interest is `450. To repay this loan, X and his
brother have taken a fresh loan and interest charged on this loan is `5,000.
The Municipal taxes of `5,100 have been paid by the tenant.
Mr. X has 50% share in the house property.
Mr. X has income from Other Sources `2,60,000.
Compute the income from this property chargeable in the hands of Mr. X and tax liability for A.Y. 2019-20.
Answer. Computation of income from house property of Mr. X for A.Y. 2019-20
` `
Gross Annual Value 1,80,000
Working Note:
(a) Municipal value of property 1,60,000
(b) Fair rent 1,50,000
(c) Higher of (a) and (b) 1,60,000
(d) Standard rent 1,70,000
(e) Annual Letting Value / Expected Rent [lower of (c) and (d)] 1,60,000
(f) Actual rent [15,000 x 12] 1,80,000
(g) Gross Annual Value [higher of (e) and (f)] 1,80,000
Less: Municipal taxes – paid by the tenant, hence not deductible Nil
Net Annual Value (NAV) 1,80,000
Income Under The Head House Property 200

Less: Standard deduction 30% of NAV u/s 24(a) (54,000)


Less: Interest on housing loan u/s 24(b)
Interest on loan taken from bank (25,000)
Interest on fresh loan to repay old loan for this property (5,000)
Income under the head house property 96,000
50% share taxable in the hands of Mr. X 48,000
Income under the head House Property 48,000
Income under the head Other Sources 2,60,000
Gross Total Income 3,08,000
Less: Deduction u/s 80C to 80U Nil
Total Income 3,08,000
Computation of Tax Liability
Tax on `3,08,000 at slab rate 2,900
Less: Rebate u/s 87A (2,500)
Tax before health & education cess 400
Add: HEC @ 4% 16
Tax Liability 416
Rounded off u/s 288B 420

Note: Interest on housing loan is allowable as a deduction under section 24 on accrual basis. Further,
interest on fresh loan taken to repay old loan is also allowable as deduction. However, interest on unpaid
interest is not allowable as deduction under section 24.

Question 21: Write a note on owner / deemed owner.


Answer: Owner / deemed owner Section 22 / 27
As per section 22, the owner of house property shall be liable to pay income tax and if the title of the
ownership of the property is under dispute in a court of law, the decision as to who will be the owner
chargeable to income-tax under section 22 will be of the Income-tax Department till the court gives its
decision to the suit filed in respect of such property. E.g. Ownership of a house property is disputed among
two brothers Mr. X and Mr. Y and rent is being received by Mr. X, in this case Mr. X shall be considered to
be the owner of the house property till decision is given by the Court and after that amount of tax shall be
adjusted as per court decision.

As per section 27, the following persons, are deemed to be the owners.
(i) Transfer of a house property to spouse – In case of transfer of house property by an individual to his or
her spouse otherwise than for adequate consideration, the transferor is deemed to be the owner of the
transferred property. In case of transfer to spouse in connection with an agreement to live apart, the
transferor will not be deemed to be the owner. The transferee will be the owner of the house property. In
case of transfer of house property by an individual to his or her minor child, the transferor is deemed to be
the owner of the house property transferred. In case of transfer to a minor married daughter, the transferor is
not deemed to be the owner. E.g. Mr. X has two house property each having income of `10 lakh and Mr. X
has gifted one house property to Mrs. X, in this case income from such house property shall be taxable in the
hands of Mr. X but if Mr. X has sold the house property to Mrs. X and has taken full payment, in that case
income from house property shall be taxable in the hands of Mrs. X.
(ii) Person in possession of a property – If any person has given possession of house property and has
taken full payment but ownership in documents has not yet been transferred, in such cases the proposed
buyer is the deemed owner and shall be liable to pay income tax and it is called part performance of a
contract of the nature referred to in section 53A of the Transfer of Property Act. E.g. Mr. X has sold his
house property to Mr. Y for `50 lakhs and has taken full payment and possession has been given to Mr. Y
but conveyance deed is not prepared in the name of Mr. Y, in this case Mr. Y is the deemed owner.
(iii) Member of a co-operative society etc. – A member of a co-operative society, company or other
association of persons to whom a building or part thereof is allotted or leased under a House Building
Income Under The Head House Property 201

Scheme of a society/company/association, shall be deemed to be owner of that building or part thereof


allotted to him although the co-operative society/company/ association is the legal owner of that building.
(iv) Holder of an impartible estate – The impartible estate is a property which is not divisible. The holder
of an impartible estate shall be deemed to be the individual owner of all properties comprised in the estate.
NOV – 2010 (2 Marks each)
Explain briefly the applicability of section 22 for chargeability of income tax for:
(i) House property situated in foreign country and
(ii) House property with disputed ownership.
Answer:
Applicability of section 22 for chargeability of income-tax for –
(i) House property situated in foreign country
A resident and ordinarily assessee is taxable under section 22 in respect of a house property situated in
foreign country. A resident but not ordinarily resident or a non resident is taxable in respect of income from
such property if the income is received in India during the previous year.
(ii) House property with disputed ownership
If the title of ownership of the house property is under dispute in a court of law, the decision about who is
the owner lies with the Court but till then income tax shall be required from the person who is the beneficial
owner of the house property.
Illustration 27: Mr. Anand sold his residential house property in March, 2018. In June, 2018, he recovered
rent of `10,000 from Mr. Gaurav, to whom he had let out his house for two years from April 2012 to March
2014. He could not realise two months rent of `20,000 from him and to that extent his actual rent was
reduced while computing income from house property for A.Y.2014-15.
Further, he had let out his property from April, 2014 to February, 2018 to Mr. Satish. In April, 2016, he had
increased the rent from `12,000 to `15,000 per month and the same was a subject matter of dispute. In
September, 2018, the matter was finally settled and Mr. Anand received `69,000 as arrears of rent for the
period April 2016 to February, 2018. Would the recovery of unrealised rent and arrears of rent be taxable in
the hands of Mr. Anand, and if so in which year?
Solution:
Since the unrealised rent was recovered in the P.Y.2018-19, the same would be taxable in the A.Y.2019-20
under section 25A, irrespective of the fact that Mr. Anand was not the owner of the house in that year.
Further, the arrears of rent was also received in the P.Y.2018-19, and hence the same would be taxable in the
A.Y.2019-20 under section 25A, even though Mr. Anand was not the owner of the house in that year. A
deduction of 30% of unrealised rent recovered and arrears of rent would be allowed while computing
income from house property of Mr. Anand for A.Y.2019-20.
Computation of income from house property of Mr. Anand for A.Y.2019-20
Recovery of Unrealised Rent 10,000
Add: Arrear of Rent Received 69,000
Total 79,000
Less: Deduction @ 30% (23,700)
Income under the head House Property 55,300
Income Under The Head House Property 202

PRACTICE PROBLEMS
TOTAL PROBLEMS 30
Problem 1:
Mr. X has let out one building @ ` 90,000 p.m. and fair rent is ` 80,000 p.m. standard rent ` 1,00,000 p.m.
Municipal valuation ` 81,000 p.m., Municipal Tax paid ` 70,000 p.a., Interest on loan for construction of
house property ` 82,000.
Compute his tax liability for assessment year 2019-20.
Answer: Tax Liability: `39,000

Problem 2:
X Ltd. has let out one building to ABC Ltd. @ `3,00,000 p.m. and X Ltd. has paid municipal tax of
`6,00,000 p.a. X Ltd. has paid interest of `3,00,000 on loan taken for construction of building. Fair rent of
the building is `2,50,000 p.m. and Municipal Valuation is `2,75,000 p.m. and Standard Rent is `2,80,000
p.m.
Compute Income Tax Liability for assessment year 2019-20.
Answer: Income Tax Liability: `5,61,600

Problem 3:
XYZ Ltd. has let out one building to ABC Ltd. @ `2,00,000 p.m. Fair rent is `1,80,000 p.m. and standard
rent `2,20,000 p.m. The company paid municipal tax of `6,00,000 during the year.
Compute income tax Liability of XYZ Ltd.
Answer: Income Tax Liability: `3,93,120

Problem 4:
Mr. X has let out one house at a rent of `50,000 p.m. Fair rent `55,000 p.m. Municipal Valuation `52,000
p.m., standard rent `60,000 p.m.. The house remain vacant for 3 months. The assessee paid municipal tax
`30,000. Interest on loan u/s 24(b) is `20,000.Compute Income and Tax Liability A.Y. 2019-20.
Answer: Income Tax Liability: `8,890
(b) Presume it is let out at a rent of `60,000 P.m.
Answer: Income Tax Liability: `1,920
(c) Presume it is let out at a rent of `55,000 P.m.
Answer: Income Tax Liability: `290
(d) Presume it is let out at a rent of `1,00,000 P.m.
Answer: Income Tax Liability: `31,510

Problem 5:
Mr. X has let out one house at a rent of `15,00,000 p.m. Fair rent `15,50,000 p.m. Municipal Valuation
`15,20,000 p.m., standard rent `16,00,000 p.m.. The house remain vacant for 3 months. The assessee paid
municipal tax `13,00,000. Interest on loan u/s 24(b) is `12,00,000.Compute Income and Tax Liability A.Y.
2019-20.
Answer: Income Tax Liability: `36,90,260
(b) Presume it is let out at a rent of `16,00,000 P.m.
Answer: Income Tax Liability: `25,20,800
(c) Presume it is let out at a rent of `15,60,000 P.m.
Answer: Income Tax Liability: `24,34,320
(d) Presume it is let out at a rent of `20,00,000 P.m.
Answer: Income Tax Liability: `35,39,560

Problem 6.
Mr. X has loss under the head house property `3,00,000 and LTCG `4,00,000 and casual income `1,00,000
Compute his income and tax liability for A.Y. 2019-20.
Answer: Income Tax Liability: `28,600
Income Under The Head House Property 203

Problem 7.
Mr. X has income under the head house property `3,00,000 and LTCG `4,00,000 and casual income `
1,00,000. He has loss under the head house property `4,00,000 of P.Y. 2011-12 and it could not be adjusted.
Compute his income and tax liability for A.Y. 2019-20.
Answer: Income Tax Liability: `62,400

Problem 8.
Mr. X has income under the head house property `3,00,000 and income under the head business/profession
`5,00,000 and LTCG `4,00,000 and casual income `1,00,000. He has loss under the head house property `
4,00,000 of P.Y. 2011-12 and it could not be adjusted. Compute his income and tax liability for A.Y. 2019-
20.
Answer: Income Tax Liability: `1,27,400

Problem 9.
Mr. X has income from house - 1 `8,00,000 and loss from house - 2 `3,00,000 and STCG 111A ` 1,50,000
and casual income `1,00,000 and he received Dividend of `2,00,000 from Domestic Company and he has
Agriculture Income from India `2,00,000. He has loss under the head house property `4,00,000 of P.Y.
2011-12 and it could not be adjusted. Compute his income and tax liability for A.Y. 2019-20.
Answer: Income Tax Liability: `28,600

Problem 10.
Compute gross annual value in the following cases for the assessment year 2019-20:
Particulars Situation 1 Situation 2 Situation 3 Situation 4
Fair Rent (p.m.) 10,000 12,000 13,000 15,000
Municipal Valuation (p.m.) 11,000 10,000 8,000 17,000
Standard Rent (p.m.) 12,000 11,000 7,000 16,000
Rent received/ receivable (p.m.) 7,000 11,500 20,000 16,000
Vacancy - 2 months 1 month 3 month
Unrealised rent 1 month - 3 month 1 month
Answer = Gross Annual Value: Situation 1: `1,32,000; Situation 2: `1,15,000; Situation 3: `1,60,000;
Situation 4: `1,92,000

Problem 11.
Mr. X has let out one house property @ `70000 per month and there is unrealised Rent of 2 months and
there is vacancy of 3 month. Fair rent `60,000 per month, municipal valuation `55,000 per month and
standard rent `80,000 per month. Municipal tax paid `62,000. Interest on loan for construction of the house
property is `75,000.The assessee has unrealised Rent of `2,00,000 in P.Y. 2015-16 and he has recovered
`1,50,000 in P.Y. 2018-19 and interest of `18,000 and he has incurred `11,000 as legal expense.
Compute his tax liability for assessment year 2019-20.
Answer: Tax Liability: `14,790

Problem 12.
Mr. X (non-resident) has one house with fair rent `20,000 p.m., municipal valuation `10,000 p.m., standard
rent `18,000 p.m. It was let out for `12,000 p.m. but it remains vacant for 1½ months and there was
unrealised rent for 2 months. Municipal taxes paid are `11,000 and interest on capital borrowed for
construction of the house is `3,00,000.
Mr. X has income under the head other sources `7,00,000.
Compute his total income and tax liability for the assessment year 2019-20.
Answer = Total Income: `5,43,500; Tax Liability: `22,050

Problem 13.
Mrs. X has taken a loan of ` 11,00,000 on 01.07.2012 at a rate of 10% per annum from SBI for construction
Income Under The Head House Property 204

of one house which was completed on 31.03.2014 and the house was let out at a rate of `80,000 per month
w.e.f. 01.11.2017 and fair rent is `1,00,000 per month. Municipal taxes paid in previous year 2018-2019
`30,000. She has taken a fresh loan of `11,00,000 on 01.07.2017 @ 11% per annum and it was utilized to
repay the original amount.
Compute her income tax liability for assessment year 2019-20.
Answer: Income Tax Liability: `54,180

Problem 14.
Mr. X took a loan of ` 6,10,500 @ 7% p.a. on 01.09.2015 from his friend for construction of one house
which was completed on 01.06.2018 and it was let out @ `9,000 p.m. It remained vacant for 1½ month and
there is unrealised rent of `1,000. The fair rent of house is `10,000 p.m. Assessee has repaid half of the loan
amount on 01.07.2017 and remaining amount on 01.02.2019. He has also paid municipal tax of `3,000. His
income from salary `2,65,000.
Compute his total income and tax liability for the assessment year 2019-20.
Answer = Total Income: `2,96,220; Tax Liability: Nil
Problem 15.
Mr. X has taken a loan on 01.07.2015 from SBI @ 11% p.a. of `15,00,000 for construction of one house
which was completed on 01.11.2017 and was self occupied and municipal taxes paid in previous year
2018-19 `32,000. He has given repayment of loan of `70,000 on 01.01.2019. He has submitted a certificate
confirming the amount of interest.
He has income under the head Salary `6,00,000
Compute income tax liability for assessment year 2019-20.
Answer: Tax Liability: `1,560

Problem 16.
Mrs. X has taken a loan on 01.11.2014 from PNB @ 10% p.a. of `10,00,000 for purchase of one house
which was purchased on 01.01.2015 and was self occupied and municipal taxes paid in previous year 2018-
2019 `30,000. She has repaid the loan amount in annual installments of `50,000 starting from 01.01.2016.
The house was vacant for 1 month in previous year 2018-19. She has submitted a certificate confirming the
amount of interest.
She has short term capital gains under section 111A `10,00,000.
Compute Income Tax Liability for assessment year 2019-20.
Answer: Tax Liability: `1,03,940

Problem 17.
Mr. X has taken a loan of `15,00,000 from State Bank on 01.07.2016 @ 10% p.a. and the residential house
was completed on 01.05.2018 and was let out w.e.f. 01.06.2018 @ 80,000 p.m. and fair rent of the house is
`90,000 p.m.
He repaid half of the loan amount on 01.01.2019.
Compute his Income Tax Liability for assessment year 2019-20.
Answer = Total Income: `3,59,250; Tax Liability: `5,680

Problem 18.
Mr. X has taken a loan of `11,00,000 on 01.07.2015 @ 10% p.a. from his friend for construction of one
house which was completed on 01.09.2017 and the house is self occupied during the previous year 2018-19
and Mr. X has paid municipal tax of `12,000.
The assessee has submitted a certificate confirming the amount of interest. Mr. X has short term capital
gains under section 111A `120 lakhs.
Compute his income and Tax Liability for the assessment year 2019-20.
Answer: Total Income: ` 118,51,500; Tax Liability: `20,81,310
Income Under The Head House Property 205

Problem 19.
Mr. X has 2 houses. First is self occupied with fair rent `20,000 p.a., municipal valuation is `55,000 p.a..
Fair rent as per Rent Control Act is `50,000 p.a.. However the house remains vacant for 2 months Architect
has issued completion certificate on 01.07.2016. Mr. X has taken loan for addition to house `3,50,000 on
01.04.2018 @ 13% p.a. The loan was repaid on 01.03.2019 and assessee has submitted a certificate from the
person from whom he has taken the loan certifying that the amount of interest claimed by Mr. X is correct.
In the earlier years, the house was let out and the assessee has recovered unrealised rent of `2,000 in the
previous year 2018-19. The assessee has also incurred legal expenses of `350.
The second house is also self-occupied. However its similar building rent is `64,000 p.a. and rent
determined by municipality for charging house tax is `66,000 p.a. Its standard rent is `6,000 p.m. municipal
tax payable are `5,000.
He has long term capital gains `20,00,000.
Compute his income tax liability for Assessment Year 2019-20.
Answer = Income Tax Liability: `3,62,900

Problem 20.
Mr. X has let out one house @ `45,000 p.m., but this house was vacated on 01.11.2018. The house was self
occupied w.e.f. 01.01.2019. Fair rent of this house is `50,000 p.m., municipal valuation is `47,000 p.m. and
standard rent is `48,000 p.m. The assessee has paid municipal taxes @ 10% of municipal valuation. Interest
on capital borrowed is `42,000. Land revenue paid by the assessee is `11,000 and ground rent paid by him is
`3,000. The assessee has taken a loan for payment of municipal tax and interest paid on loan is `500.
Compute his income under the head house property and tax liability for assessment year 2019-20.
Answer = Income under the head House Property: `3,21,720; Tax Liability: `1,130

Problem 21.
Mr. X has two houses one of which is self occupied throughout the year. Its fair rent is `10,000 p.m.,
municipal valuation `11,000 p.m. and standard rent is `10,500 p.m. Municipal taxes paid are `6,000 and
interest on capital borrowed is `41,000. The assessee has taken the loan for construction of the house on
01.04.1998.
Second house is self occupied for 4 months and let out for 8 months @ of `45,000 p.m. Its fair rent is
`20,000 p.m., municipal valuation is `18,000 p.m. and standard rent `15,000 p.m. Municipal taxes paid are
`20,000 and interest on capital borrowed is `45,000. The assessee has taken the loan for construction of the
house on 01.04.1998.
Compute his income under the head house property for the assessment year 2019-20.
Answer = Income under the head House Property: ` 1,63,000

Problem 22.
Mr. X has let out one showroom building in Pitam Pura @ 1,00,000 p.m. and has paid municipal tax
`85,000 and fair rent of the house is `98,000 p.m.
He has received arrears of rent `3,00,000 relating to the previous year 2017-18.
He has also received unrealized rent of `4,00,000 of previous year 2016-17 and also interest of `20,000 on
such unrealised rent and he has paid `27,000 to the advocate in connection with recovery of unrealized rent.
Compute his income tax liability for assessment year 2019-20.
Answer: Tax Liability: `2,07,640

Problem 23.
Mr. X occupied two flats for his residential purposes, particulars of which are as follows:
Particulars Flat I Flat II
(in `) (in `)
Municipal Valuation 95,000 p.a. 50,000 p.a.
Fair Rent 1,25,000 p.a. 45,000 p.a.
Fair Rent under Rent Control Act 85,000 p.a. Not available
Income Under The Head House Property 206

Municipal taxes paid 10% 10%


Fire Insurance paid 1,500 650
Ground rent due 700 900
Land revenue paid 600 800
Interest payable on capital borrowed for purchase of flat 45,000 Nil
Income of Mr. X from his proprietary business–warehousing corporation is `7,00,000. Determine the total
income and tax liability for the assessment year 2019-20, you are informed that Mr. X could not occupy flat
for 2 months commencing from December 1st, 2018 and that he has attained the age of 82 on 23.08.2018.
Answer = Total Income: `7,01,500; Tax Liability: `41,910

Problem 24.
Mr. X and Mr. Y constructed their houses on a piece of land purchased by them at New Delhi. The built up
area of each house was 1,000 sq. ft. ground floor and an equal area at the first floor.
Mr. X started construction of the house on 01.04.2017 and completed it on 31.03.2018. Mr. X occupied the
entire house on 01.04.2018. Mr. X has availed a housing loan of `25 lakhs @ 12% p.a. on 01.04.2017 and
has also submitted a certificate from the lender certifying the amount of interest.
Mr. Y started construction on 01.04.2017 and completed it on 30.06.2018. Mr. Y occupied the ground floor
on 01.07.2018 and let out the first floor for a rent of `20,000 per month. However, the tenant vacated the
house on 31.12.2018 and Mr. Y occupied the entire house during the period 01.01.2019 to 31.03.2019. Mr.
Y has availed a housing loan of `15 lakhs @ 10% p.a. on 01.07.2017 and has also submitted a certificate
from the lender certifying the amount of interest.
Following are the other information: `
(i) Fair rental value of each unit 1,20,000 Per annum
(Ground floor / first floor)
(ii) Municipal value of each unit 92,000 Per annum
(Ground floor / first floor)
(iii) Municipal taxes paid by X - 10,000
Y - 10,000
(iv) Repair and maintenance charges paid by X - 30,000
Y - 32,000
No repayment was made by either of them till 31.03.2019. Compute income from house property for Mr. X
and Mr. Y for the previous year 2018-19 (assessment year 2019-20).
Answer = Mr. X: ` (2,00,000); Mr. Y: ` (92,000)

Problem 25.
Mrs. X is the owner of a house property. She borrowed `60,000 from life insurance corporation of India on
1st September 2011 @ 15% p.a. for the construction of this house. The construction was completed on
31.03.2014. Since then the house is under her self-occupation. On 1st June 2018 the house was let out @
`3,000 p.m. The tenant vacated the house on 1st August 2018. She occupied the house for self-occupancy.
The house is again let out @ `3,500 p.m. from 1st October 2018.
Other particulars of the house for the previous year 2018-19. `
Municipal Valuation 22,000 p.a.
Municipal taxes disputed, hence not paid 2,200 p.a.
Ground rent for the previous year 2018-19 outstanding 3,200
Insurance premium paid 1,200
Refund of first loan instalment to LIC on 01.10.2018 15,000
Compute the income from house property for assessment year 2019-20.
Answer = Income under the head House Property: `11,025

Problem 26.
Mr. X owns a residential house property. It has two identical units—unit I and unit II. Unit I is self–occupied
by Mr. X and his family members, unit II is let out (rent being `7,500 per month, this unit remained vacant
for one month during which it was self-occupied). Municipal value of the property is `1,30,000. Standard
Income Under The Head House Property 207

rent is `1,40,000 and fair rent is `1,53,000. Municipal taxes is imposed @ 12% (on municipal value) which
is paid by Mr. X. Other expenses for the previous year 2018-19 being repairs `5,100 and insurance `6,300.
Mr. X borrowed `9,00,000 on 01.07.2015 from LIC @ 12% p.a. to construct the property. Construction of
the house was completed on 30.06.2017. The entire loan is still unpaid.
Compute the total income and tax liability of Mr. X for the assessment year 2019-20 on the assumption that
income of Mr. X from other sources is `3,90,000.
Answer = Total Income: `3,39,390; Tax Liability: `2,050

Problem 27.
Mr. X has a house property situated in Mumbai which has two units. Unit I has a floor area of 70% whereas
the unit II has a floor area of 30%. Both the units were self-occupied by the assessee. As the assessee was
allowed a rent free accommodation by his employer w.e.f. 01.04.2018, he vacated both of the units and let
out unit I at a rent of `13,000 p.m. and unit II for `5,000 p.m. unit I remained vacant for 1½ months whereas
unit II was vacant for one month. Other particulars of the house property are asunder: `
Municipal Valuation 1,55,000
Fair Rent 1,75,000
Standard Rent 1,65,000
Municipal taxes paid 35,000
Ground rent due 15,000
Compute income from house property for the assessment year 2019-20.
Answer = Income under the head House Property: `1,09,550

Problem 28.
Mr. X is the owner of a residential house whose construction was completed on 31.08.2014. It has been let
out from 01.01.2015 for residential purposes. Its particulars for the financial year 2018-19 are given below:
`
(i) Municipal Valuation (p.a.) 68,000
(ii) Expected Fair Rent (p.a.) 75,000
(iii) Standard Rent under the Rent Control Act (p.m.) 7,200
(iv) Actual Rent (p.m.) 7,200
(v) Municipal taxes paid (including `7,000 paid by tenant) 21,000
(vi) Water/sewerage benefit tax, levied by State Government paid under protest 5,100
(vii) Interest on loan taken for the construction of the house. The interest has been paid
outside India to a non-resident without deduction of tax at source 20,000
(viii) Stamp duty and registration charges incurred in respect of the lease agreement of the house 2,500
(ix) The unrealised rent for previous year 2017-18 amounts to `42,000.
There is recovery of `22,000 from the defaulting tenant.
Legal charges for the recovery of rent 4,500
Compute income from house property for the assessment year 2019-20.
Answer = Income under the head House Property: `66,080

Problem 29.
Mr. X has three houses with details given below:
House I
It is self occupied with fair rent of `20,000, municipal valuation `55,000, rent as per Rent Control Act is
`50,000. However the house remains vacant for 2 months. Architect has issued completion certificate on
01.07.2016. Loan taken for addition to the house `5,00,000 on 01.04.2018 @ 13% p.a. and loan amount was
repaid on 01.03.2019. The assessee has submitted a certificate from the person from whom he has taken the
loan certifying the amount of the interest claimed.
In the earlier years the house was let out and the assessee has recovered unrealised rent of `2,000 in the
previous year 2018-19 and interest on such unrealised rent also amounting to `250. However the assessee
has incurred legal expenses of `350.
Income Under The Head House Property 208

House II
It is self occupied. Its similar building rent is `64,000 and rent determined by municipality for charging
house tax is `66,000 and its fair rent under Rent Control Act (p.m.) is `6,000. Municipal taxes payable
`5,000.
The assessee has also recovered unrealised rent of `2,000 in the previous year 2016-17 but the expenses
thereon are paid in the year 2018-19 amounting to `200.
House III
It is let out @ `50,000 p.m. and fair rent is `60,000 p.m. Water tax and house tax paid to municipality is
`11,000. Insurance premium paid `6,500 and expenses on repairs `3,000.
Interest on capital borrowed for purchase of house is `55,000.
He has long term capital gains of `3,50,000.
Compute his total income and tax liability for assessment year 2019-20.
Answer = Total Income: `7,68,370; Tax Liability: `81,560
Problem 30.
Determine the income head under which the following incomes shall be taxable.
(i) Mr. X has income from letting out house property.
(ii) Mr. X has sold one house property.
(iii) ABC Ltd. has 500 flats for the purpose of sale/purchase.
(iv) Mr. X has let out an open land.
(v) ABC Ltd. has 500 flats for the purpose of letting out.
(vi) ABC Ltd. has constructed flats within its premises for letting out to the employees.
(vii) Mr. X is engaged in the business of providing paying guest accommodation in his own
building.
(viii) Mr. X is engaged in the business of warehousing.
(ix) Mr. X has sublet one house property.
(x) Mr. X has let out his hotel building.
Answer = (i) House Property; (ii) Capital Gains; (iii) Business/Profession; (iv) Other Sources; (v) House
Property; (vi) Business/Profession; (vii) Business/Profession; (viii) Business/Profession; (ix) Other Sources;
(x) House Property
Income Under The Head House Property 209

SOLUTIONS
TO
PRACTICE PROBLEMS
Solution 1: `
Computation of income under the head House Property
Gross Annual Value 10,80,000
Working Note: `
(a) Fair Rent ( ` 80,000 x 12) 9,60,000
(b) Municipal Valuation (` 81,000 x 12) 9,72,000
(c) Higher of (a) or (b) 9,72,000
(d) Standard Rent (` 1,00,000 x 12) 12,00,000
(e) Expected Rent {Lower of (c) or (d)} 9,72,000
(f) Rent received (`90,000 x 12) 10,80,000
GAV = Higher of (e) or (f) 10,80,000
Less: Municipal Tax (70,000)
Net Annual Value 10,10,000
Less: 30% of NAV u/s 24(a) (3,03,000)
Less: Interest on capital borrowed u/s 24(b) (82,000)
Income under the head House Property 6,25,000
Gross Total Income 6,25,000
Less: Deduction u/s 80C to 80U Nil
Total Income 6,25,000
Computation of Tax Liability
Tax on `6,25,000 at slab rate 37,500
Add: HEC @ 4% 1,500
Total tax liability 39,000

Solution 2: `
Computation of income under the head House Property
Gross Annual Value 36,00,000.00
Working Note: `
(a) Fair Rent (2,50,000 x 12) 30,00,000
(b) Municipal Value (2,75,000 x 12) 33,00,000
(c) Higher of (a) or (b) 33,00,000
(d) Standard Rent (2,80,000 x 12) 33,60,000
(e) Expected Rent {Lower of (c) or (d)} 33,00,000
(f) Rent received /receivable (3,00,000 x 12) 36,00,000
GAV shall be higher of (e) or (f) 36,00,000
Less: Municipal Tax (6,00,000.00)
Net Annual Value 30,00,000.00
Less: 30% of NAV u/s 24(a) (9,00,000.00)
Less: Interest on capital borrowed u/s 24(b) (3,00,000.00)
Income under the head House Property 18,00,000.00
Gross Total Income 18,00,000.00
Less: Deduction u/s 80C to 80U Nil
Income Under The Head House Property 210

Total Income 18,00,000.00


Computation of Tax Liability
Tax on `18,00,000 @ 30% 5,40,000.00
Add: HEC @ 4% 21,600.00
Tax Liability 5,61,600.00

Solution 3. `
Computation of income under the head House Property
Gross Annual Value (2,00,000 x 12) 24,00,000.00
Working Note: `
(a) Fair Rent (1,80,000 x 12) 21,60,000
(b) Standard Rent (2,20,000 x 12) 26,40,000
(c) Expected Rent (lower of (a) or (b) 21,60,000
(d) Rent Received/Receivable (2,00,000 x 12) 24,00,000
GAV = Higher of (c) or (d) 24,00,000
Less: Municipal Tax (6,00,000.00)
Net Annual Value 18,00,000.00
Less: 30% of NAV u/s 24(a) (5,40,000.00)
Less: Interest on capital borrowed u/s 24(b) Nil
Income under the head House Property 12,60,000.00
Gross Total Income 12,60,000.00
Less: Deduction u/s 80C to 80U Nil
Total Income 12,60,000.00
Computation of Tax Liability
Tax on `12,60,000 @ 30% 3,78,000.00
Add: HEC @ 4% 15,120.00
Tax Liability 3,93,120.00

Solution 4: Computation of Total Income & Tax Liability of Mr. X `


Gross Annual Value 6,60,000.00
Working Note: `
(a) Fair Rent (`55,000 x 12) 6,60,000
(b) Municipal Valuation (`52,000 x 12) 6,24,000
(c) Higher of (a) or (b) 6,60,000
(d) Standard Rent (`60,000 x 12) 7,20,000
(e) Expected Rent {Lower of (c) or (d)} 6,60,000
(f) Rent received /receivable (50,000 x 9) 4,50,000
If there was no vacancy, in that case rent received/receivable would have been
(50,000 x 12) `6,00,000 which is not exceeding expected rent hence GAV shall be
expected rent i.e. `6,60,000
GAV 6,60,000
Less: Municipal Tax (30,000.00)
Net Annual Value 6,30,000.00
Less: 30% of NAV u/s 24(a) (1,89,000.00)
Less: Interest on capital borrowed u/s 24(b) (20,000.00)
Income under the head House Property 4,21,000.00
Total Income 4,21,000.00

Computation of Tax Liability


Tax on Normal Income `4,21,000 at slab rate 8,550.00
Add: HEC @ 4% 242.00
Tax Liability 8,892.00
Income Under The Head House Property 211

Rounded off u/s 288B 8,890.00

Solution 4(b): Computation of Total Income & Tax Liability of Mr. X `


Gross Annual Value 5,40,000.00
Working Note: `
(a) Fair Rent (`55,000 x 12) 6,60,000
(b) Municipal Valuation (`52,000 x 12) 6,24,000
(c) Higher of (a) or (b) 6,60,000
(d) Standard Rent (`60,000 x 12) 7,20,000
(e) Expected Rent {Lower of (c) or (d)} 6,60,000
(f) Rent received /receivable (60,000 x 9) 5,40,000
If there was no vacancy, in that case rent received/receivable would have been
(60,000 x 12) `7,20,000 which is exceeding expected rent hence GAV shall be Rent
received/receivable i.e. `5,40,000
GAV 5,40,000
Less: Municipal Tax (30,000.00)
Net Annual Value 5,10,000.00
Less: 30% of NAV u/s 24(a) (1,53,000.00)
Less: Interest on capital borrowed u/s 24(b) (20,000.00)
Income under the head House Property 3,37,000.00
Total Income 3,37,000.00

Computation of Tax Liability


Tax on Normal Income `3,37,000 at slab rate 4,350.00
Less: Rebate u/s 87A (2,500.00)
Tax before health & education cess 1,850.00
Add: HEC @ 4% 74.00
Tax Liability 1,924.00
Rounded off u/s 288B 1,920.00

Solution 4(c): Computation of Total Income & Tax Liability of Mr. X `


Gross Annual Value 4,95,000.00
Working Note: `
(a) Fair Rent (`55,000 x 12) 6,60,000
(b) Municipal Valuation (`52,000 x 12) 6,24,000
(c) Higher of (a) or (b) 6,60,000
(d) Standard Rent (`60,000 x 12) 7,20,000
(e) Expected Rent {Lower of (c) or (d)} 6,60,000
(f) Rent received /receivable (55,000 x 9) 4,95,000
If there was no vacancy, in that case rent received/receivable would have been
(55,000 x 12) `6,60,000 which is equal to expected rent hence GAV shall be Rent
Received/ receivable i.e. `4,95,000
GAV 4,95,000
Less: Municipal Tax (30,000.00)
Net Annual Value 4,65,000.00
Less: 30% of NAV u/s 24(a) (1,39,500.00)
Less: Interest on capital borrowed u/s 24(b) (20,000.00)
Income under the head House Property 3,05,500.00
Total Income 3,05,500.00

Computation of Tax Liability


Tax on Normal Income `3,05,500 at slab rate 2,775.00
Income Under The Head House Property 212

Less: Rebate u/s 87A (2,500.00)


Tax before health & education cess 275.00
Add: HEC @ 4% 11.00
Tax Liability 286.00
Rounded off u/s 288B 290.00

Solution 4(d): Computation of Total Income & Tax Liability of Mr. X `


Gross Annual Value 9,00,000.00
Working Note: `
(a) Fair Rent (`55,000 x 12) 6,60,000
(b) Municipal Valuation (`52,000 x 12) 6,24,000
(c) Higher of (a) or (b) 6,60,000
(d) Standard Rent (`60,000 x 12) 7,20,000
(e) Expected Rent {Lower of (c) or (d)} 6,60,000
(f) Rent received /receivable (1,00,000 x 9) 9,00,000
If there was no vacancy, in that case rent received/receivable would have been
(1,00,000 x 12) `12,00,000 which is exceeding expected rent hence GAV shall be Rent
received/receivable i.e. `9,00,000
GAV 9,00,000
Less: Municipal Tax (30,000.00)
Net Annual Value 8,70,000.00
Less: 30% of NAV u/s 24(a) (2,61,000.00)
Less: Interest on capital borrowed u/s 24(b) (20,000.00)
Income under the head House Property 5,89,000.00
Total Income 5,89,000.00
Computation of Tax Liability
Tax on Normal Income `5,89,000 at slab rate 30,300.00
Add: HEC @ 4% 1,212.00
Tax Liability 31,512.00
Rounded off u/s 288B 31,510.00

Solution 5: `
Computation of Total Income & Tax Liability of Mr. X
Gross Annual Value 186,00,000.00
Working Note: `
(a) Fair Rent (`15,50,000 x 12) 186,00,000
(b) Municipal Valuation (`15,20,000 x 12) 182,40,000
(c) Higher of (a) or (b) 186,00,000
(d) Standard Rent (`16,00,000 x 12) 192,00,000
(e) Expected Rent {Lower of (c) or (d)} 186,00,000
(f) Rent received /receivable (15,00,000 x 9) 135,00,000
If there was no vacancy, in that case rent received/receivable would have been
(15,00,000 x 12) `180,00,000 which is not exceeding expected rent hence GAV shall
be expected rent i.e. `186,00,000
GAV 186,00,000
Less: Municipal Tax (13,00,000.00)
Net Annual Value 173,00,000.00
Less: 30% of NAV u/s 24(a) (51,90,000.00)
Less: Interest on capital borrowed u/s 24(b) (12,00,000.00)
Income under the head House Property 109,10,000.00
Total Income 109,10,000.00
Income Under The Head House Property 213

Computation of Tax Liability


Tax on Normal Income `109,10,000 at slab rate 30,85,500.00
Add: Surcharge @ 15% 4,62,825.00
Tax before health & education cess 35,48,325.00
Add: HEC @ 4% 1,41,933.00
Tax Liability 36,90,258.00
Rounded off u/s 288B 36,90,260.00

Solution 5(b): `
Computation of Total Income & Tax Liability of Mr. X
Gross Annual Value 144,00,000.00
Working Note: `
(a) Fair Rent (`15,50,000 x 12) 186,00,000
(b) Municipal Valuation (`15,20,000 x 12) 182,40,000
(c) Higher of (a) or (b) 186,00,000
(d) Standard Rent (`16,00,000 x 12) 192,00,000
(e) Expected Rent {Lower of (c) or (d)} 186,00,000
(f) Rent received /receivable (16,00,000 x 9) 144,00,000
If there was no vacancy, in that case rent received/receivable would have been
(16,00,000 x 12) `192,00,000 which is exceeding expected rent hence GAV shall be
rent received/receivable i.e. `144,00,000
GAV 144,00,000
Less: Municipal Tax (13,00,000.00)
Net Annual Value 131,00,000.00
Less: 30% of NAV u/s 24(a) (39,30,000.00)
Less: Interest on capital borrowed u/s 24(b) (12,00,000.00)
Income under the head House Property 79,70,000.00
Total Income 79,70,000.00
Computation of Tax Liability
Tax on Normal Income `79,70,000 at slab rate 22,03,500.00
Add: Surcharge @ 10% 2,20,350.00
Tax before health & education cess 24,23,850.00
Add: HEC @ 4% 96,954.00
Tax Liability 25,20,804.00
Rounded off u/s 288B 25,20,800.00

Solution 5(c): `
Computation of Total Income & Tax Liability of Mr. X
Gross Annual Value 140,40,000.00
Working Note: `
(a) Fair Rent (`15,50,000 x 12) 186,00,000
(b) Municipal Valuation (`15,20,000 x 12) 182,40,000
(c) Higher of (a) or (b) 186,00,000
(d) Standard Rent (`16,00,000 x 12) 192,00,000
(e) Expected Rent {Lower of (c) or (d)} 186,00,000
(f) Rent received /receivable (15,60,000 x 9) 140,40,000
If there was no vacancy, in that case rent received/receivable would have been
(15,60,000 x 12) `187,20,000 which is exceeding expected rent hence GAV shall be
rent received/ receivable i.e. `140,40,000
GAV 140,40,000
Less: Municipal Tax (13,00,000.00)
Net Annual Value 127,40,000.00
Income Under The Head House Property 214

Less: 30% of NAV u/s 24(a) (38,22,000.00)


Less: Interest on capital borrowed u/s 24(b) (12,00,000.00)
Income under the head House Property 77,18,000.00
Total Income 77,18,000.00

Computation of Tax Liability


Tax on Normal Income `77,18,000 at slab rate 21,27,900.00
Add: Surcharge @ 10% 2,12,790.00
Tax before health & education cess 23,40,690.00
Add: HEC @ 4% 93,627.60
Tax Liability 24,34,317.60
Rounded off u/s 288B 24,34,320.00

Solution 5(d): `
Computation of Total Income & Tax Liability of Mr. X
Gross Annual Value 180,00,000.00
Working Note: `
(a) Fair Rent (`15,50,000 x 12) 186,00,000
(b) Municipal Valuation (`15,20,000 x 12) 182,40,000
(c) Higher of (a) or (b) 186,00,000
(d) Standard Rent (`16,00,000 x 12) 192,00,000
(e) Expected Rent {Lower of (c) or (d)} 186,00,000
(f) Rent received /receivable (20,00,000 x 9) 180,00,000
If there was no vacancy, in that case rent received/receivable would have been
(20,00,000 x 12) `240,00,000 which is exceeding expected rent hence GAV shall be
rent received/ receivable i.e. `180,00,000
GAV 180,00,000
Less: Municipal Tax (13,00,000.00)
Net Annual Value 167,00,000.00
Less: 30% of NAV u/s 24(a) (50,10,000.00)
Less: Interest on capital borrowed u/s 24(b) (12,00,000.00)
Income under the head House Property 104,90,000.00
Total Income 104,90,000.00

Computation of Tax Liability


Tax on Normal Income `104,90,000 at slab rate 29,59,500.00
Add: Surcharge @ 15% 4,43,925.00
Tax before health & education cess 34,03,425.00
Add: HEC @ 4% 1,36,137.00
Tax Liability 35,39,562.00
Rounded off u/s 288B 35,39,560.00

Solution 6:
Computation of Total Income of Mr. X for A.Y. 2019-20
Income under the head Capital Gains
Long Term Capital Gain 4,00,000
Less: Loss under the head House Property (2,00,000)
Long Term Capital Gain 2,00,000
Income under the head other sources
Casual Income 1,00,000

Gross Total Income 3,00,000


Less: Deduction u/s 80C to 80U Nil
Income Under The Head House Property 215

Total Income 3,00,000

Computation of Tax Liability


Tax on LTCG Nil (`2,00,000- `2,00,000) @ 20% Nil
Tax on casual Income `1,00,000 @ 30% 30,000.00
Less: Rebate u/s 87A (2,500.00)
Tax before health & education cess 27,500.00
Add: HEC @ 4% 1,100.00
Tax Liability 28,600.00

Note: As per section 71, Maximum loss of `2,00,000 is allowed to be set off from other heads.

Solution 7:
Computation of Total Income of Mr. X for A.Y. 2019-20
Income under the head House Property
Income under the head House Property 3,00,000
Less: Set off of Loss of P.Y. 2011-12 (3,00,000)
Income under the head House Property Nil

Income under the head Capital Gains


Long Term Capital Gain 4,00,000

Income under the head other sources


Casual Income 1,00,000

Gross Total Income 5,00,000


Less: Deduction u/s 80C to 80U Nil
Total Income 5,00,000
Computation of Tax Liability
Tax on LTCG ` 1,50,000 (`4,00,000 - `2,50,000) @ 20% u/s 112 30,000.00
Tax on casual Income `1,00,000 @ 30% 30,000.00
Tax before health & education cess 60,000.00
Add: HEC @ 4% 2,400.00
Tax Liability 62,400.00

Note : Brought forward loss cannot be set off from income of other heads hence remaining loss of `1,00,000
of House Property of P.Y. 2011-12 shall be carried forward.

Solution 8:
Computation of Total Income of Mr. X for A.Y. 2019-20
Income under the head House Property
Income under the head House Property 3,00,000
Less: Set off of Loss of P.Y. 2011-12 (3,00,000)
Income under the head House Property Nil

Income under the head Business/Profession 5,00,000

Income under the head Capital Gains


Long Term Capital Gain 4,00,000

Income under the head other sources


Casual Income 1,00,000
Income Under The Head House Property 216

Gross Total Income 10,00,000


Less: Deduction u/s 80C to 80U Nil
Total Income 10,00,000

Computation of Tax Liability


Tax on Normal Income `5,00,000 at slab rate 12,500.00
Tax on LTCG ` 4,00,000 @ 20% u/s 112 80,000.00
Tax on casual Income `1,00,000 @ 30% 30,000.00
Tax before health & education cess 1,22,500.00
Add: HEC @ 4% 4,900.00
Tax Liability 1,27,400.00
Note : Brought forward loss cannot be set off from income of other heads hence remaining loss of `1,00,000
of House Property of P.Y. 2011-12 shall be carried forward.

Solution 9:
Computation of Total Income of Mr. X for A.Y. 2019-20
Income under the head House Property
Income from House -1 8,00,000
Less: Loss from House – 2 (3,00,000)
Less: Set off of Loss of P.Y. 2011-12 (4,00,000)
Income under the head House Property 1,00,000

Income under the head Capital Gains


Short Term Capital Gain 111A 1,50,000

Income under the head other sources


Casual Income 1,00,000
Dividend Income exempt u/s 10(34) Nil
Income under the head other sources 1,00,000

Gross Total Income 3,50,000


Less: Deduction u/s 80C to 80U Nil
Total Income 3,50,000
Agriculture Income 2,00,000

Computation of Tax Liability


Tax on Normal Income `1,00,000 at slab rate Nil
Tax on STCG 111A `Nil (`1,50,000 – `1,50,000) @ 15% u/s 111A Nil
Tax on casual Income `1,00,000 @ 30% 30,000.00
Less: Rebate u/s 87A (2,500.00)
Tax before health & education cess 27,500.00
Add: HEC @ 4% 1,100.00
Tax Liability 28,600.00

Note : 1. Partial Integration is not applicable as Normal Income is less than basic exemption limit.
2. Current year loss of house shall be fully allowed to set off from income of another house in the
same year however such loss shall be allowed to set off from income of other heads maximum
upto `2,00,000

Solution 10:
Situation 1 `
Computation of Gross Annual Value
(a) Fair Rent 1,20,000
Income Under The Head House Property 217

(10,000 x 12)
(b) Municipal Valuation 1,32,000
(11,000 x 12)
(c) Higher of (a) or (b) 1,32,000
(d) Standard Rent 1,44,000
(12,000 x 12)
(e) Expected Rent {Lower of (c) or (d)} 1,32,000
(f) Rent Received/Receivable 77,000
(7,000 x 11)
GAV = Higher of (e) or (f) 1,32,000
Gross Annual Value 1,32,000
Situation 2
Computation of Gross Annual Value
(a) Fair Rent 1,44,000
(12,000 x 12)
(b) Municipal Valuation 1,20,000
(10,000 x 12)
(c) Higher of (a) or (b) 1,44,000
(d) Standard Rent 1,32,000
(11,000 x 12)
(e) Expected Rent {Lower of (c) or (d)} 1,32,000
(f) Rent Received/Receivable 1,15,000
(11,500 x 10)
In this case, if there was no vacancy, rent received/receivable would have been `1,38,000
hence rent received/receivable is lower in this case due to vacancy, therefore GAV
shall be the rent received/receivable.
Gross Annual Value 1,15,000
Situation 3
Computation of Gross Annual Value
(a) Fair Rent 1,56,000
(13,000 x 12)
(b) Municipal Valuation 96,000
(8,000 x 12)
(c) Higher of (a) or (b) 1,56,000
(d) Standard Rent 84,000
(7,000 x 12)
(e) Expected Rent {Lower of (c) or (d)} 84,000
(f) Rent Received/Receivable 1,60,000
(20,000 x 8)
In this case, rent R/R is higher than the expected rent, GAV shall be Rent R/R
Gross Annual Value 1,60,000
Situation 4
Computation of Gross Annual Value
(a) Fair Rent 1,80,000
(15,000 x 12)
(b) Municipal Valuation 2,04,000
(17,000 x 12)
(c) Higher of (a) or (b) 2,04,000
(d) Standard Rent 1,92,000
(16,000 x 12)
(e) Expected Rent {Lower of (c) or (d)} 1,92,000
(f) Rent Received/Receivable 1,28,000
(16,000 x 8)
Income Under The Head House Property 218

If there was no vacancy, in that case rent received/receivable would have been `1,76,000
and it was still less than expected rent, therefore GAV shall be expected rent.
Gross Annual Value 1,92,000

Solution 11: `
Income under the head House Property
Gross annual value 7,20,000.00
Working Note: `
(a) Fair rent (60,000 x 12) 7,20,000
(b) Municipal valuation (55,000 x 12) 6,60,000
(c) Higher of (a) or (b) 7,20,000
(d) Standard Rent (80,000 x 12) 9,60,000
(e) Expected Rent {Lower of (c) or (d)} 7,20,000
(f) Rent Received (70,000 x 7) 4,90,000
If there was no vacancy , then Rent Receivable shall be 70,000 x 10 =
7,00,000, which is lower than the expected rent , hence the GAV shall
be 7,20,000
Less: Municipal taxes paid (62,000.00)
Net Annual Value 6,58,000.00
Less: 30% of NAV u/s 24(a) (1,97,400.00)
Less: Interest on capital borrowed u/s 24(b) (75,000.00)
3,85,600.00
Unrealised rent recovered of 2015-16 section 25A 1,05,000.00
(1,50,000 – 45,000)
Income under the head House Property 4,90,600.00
Income from other sources 18,000.00
Gross Total Income 5,08,600.00
Less: Deduction u/s 80C to 80U NIL
Total Income 5,08,600.00
Computation of Tax Liability
Tax on `5,08,600 at slab rate 14,220.00
Add: HEC @ 4% 568.80
Tax Liability 14,788.80
Rounded off u/s 288B 14,790.00

Solution 12: `
Gross Annual Value 2,16,000.00
Working Note: `
(a) Fair Rent (20,000 x 12) 2,40,000
(b) Municipal Valuation (10,000 x 12) 1,20,000
(c) Higher of (a) or (b) 2,40,000
(d) Standard Rent (18,000 x 12) 2,16,000
(e) Expected Rent {Lower of (c) or (d)} 2,16,000
(f) Rent Receivable = (12,000 x 8.5) 1,02,000
If there was no vacancy, in that case rent received/receivable would have been
`1,20,000 and it was still less than expected rent, therefore GAV shall be expected
rent.
GAV 2,16,000
Less: Municipal Tax (11,000.00)
Net Annual Value 2,05,000.00
Less: 30% of NAV u/s 24(a) (61,500.00)
Less: Interest on capital borrowed u/s 24(b) (3,00,000.00)
Income Under The Head House Property 219

Loss under the head House Property (1,56,500.00)


Income under the head Other Sources 7,00,000.00
Gross Total Income 5,43,500.00
Less: Deduction u/s 80C to 80U Nil
Total Income 5,43,500.00
Computation of Tax Liability
Tax on `5,43,500 at slab rate 21,200.00
Add: HEC @ 4% 848.00
Tax Liability 22,048.00
Rounded off u/s 288B 22,050.00

Solution 13: `
Income under the head House Property
Gross annual value 12,00,000.00
Working Note: `
Fair rent (1,00,000 x 12) 12,00,000
Rent received (80,000 x 12) 9,60,000
Higher shall be the GAV i.e. 12,00,000
Less: Municipal taxes paid (30,000.00)
Net Annual Value 11,70,000.00
Less: 30% of NAV u/s 24(a) (3,51,000.00)
Less: Interest on capital borrowed u/s 24(b) (1,21,000.00)
Working Note:
Prior period interest Nil
Current year interest 11,00,000 x 11% = 1,21,000
Income under the head House Property 6,98,000.00
Gross Total Income 6,98,000.00
Less: Deduction u/s 80C to 80U Nil
Total Income 6,98,000.00

Computation of Tax Liability


Tax on `6,98,000 at slab rate 52,100.00
Add: HEC @ 4% 2,084.00
Tax Liability 54,184.00
Rounded off u/s 288B 54,180.00
Solution 14: `
Gross Annual Value 1,00,000.00
Working Note: `
(a) Fair Rent (10,000 x 10) 1,00,000
(b) Expected Rent 1,00,000
(c) Received/Receivable = 9,000 x 8.5 = 76,500 – 1,000 = 75,500
If there was no vacancy, in that case rent received/receivable would have been
`89,000 and it was still less than expected rent, therefore GAV shall be expected
rent.
GAV 1,00,000
Less: Municipal taxes (3,000.00)
Net Annual Value 97,000.00
Less: 30% of NAV u/s 24(a) (29,100.00)
Less: Interest on capital borrowed u/s 24(b) (36,680.88)
Working Note:
Current Period interest
Income Under The Head House Property 220

From 01.04.2018 to 31.01.2019


= 3,05,250 x 7% x 10/12 = `17,806.25
Prior period interest
From 01.09.2015 to 31.03.2018
From 01.09.2015 to 30.06.2017
= 6,10,500 x 7% x 22/12 = `78,347.5
From 01.07.2017 to 31.03.2018
= 3,05,250 x 7% x 9/12 = `16,025.63
Total = `94,373.13
Instalment = `94,373.13/5 = `18,874.63
Total interest = `17,806.25 + `18,874.63 = `36,680.88
Income under the head House Property 31,219.12
Income under the head Salary 2,65,000.00
Gross Total Income 2,96,219.12
Less: Deductions u/s 80C Nil
Total Income (rounded off u/s 288A) 2,96,220.00
Computation of Tax Liability
Tax on `2,96,220 at slab rate 2,311.00
Less: Rebate u/s 87A (2,311.00)
Tax Liability Nil

Solution 15: `
Income under the head House Property
Gross Annual Value NIL
Less: Municipal taxes paid NIL
Net Annual Value NIL
Less: 30% of NAV u/s 24(a) NIL
Less: Interest on capital borrowed u/s 24(b) (2,00,000.00)

Working Note: `
Prior period interest
01.07.2015 to 31.03.2017
15, 00,000 x 11 % x 21/12= 2,88,750
Installment 2,88,750/5 = 57,750
Current year interest
15,00,000 x 11% x 9/12 = 1,23,750
14,30,000 x 11% x 3/12 = 39,325 1,63,075
2,20,825
Subject to maximum of 2,00,000.
Loss under the head House Property (2,00,000.00)
Income under the head Salary 6,00,000.00
Gross Total Income 4,00,000.00
Less: Deduction u/s 80C (70,000.00)
Total Income 3,30,000.00
Computation of Tax Liability
Tax on `3,30,000 at slab rate 4,000.00
Less: Rebate u/s 87A (2,500.00)
Tax before health & education cess 1,500.00
Add: HEC @ 4% 60.00
Tax Liability 1,560.00
Income Under The Head House Property 221

Solution 16: `
Income under the head House Property
Gross Annual Value NIL
Less: Municipal taxes paid NIL
Net Annual Value NIL
Less: 30% of NAV u/s 24(a) NIL
Less : Interest on capital borrowed u/s 24(b) (83,750.00)
Working Note: `
Prior period interest Nil
Current year interest
From 01.04.2018 to 31.03.2019
8,50,000 x 10% x 9/12 = 63,750
8,00,000 x 10% x 3/12 = 20,000 83,750
Loss under the head House Property (83,750.00)
Income under the head capital gains (STCG u/s 111A) 10,00,000.00
Gross Total Income 9,16,250.00
Less: Deduction u/s 80C Nil
Total Income 9,16,250.00
Computation of Tax Liability
Tax on `6,66,250 (`9,16,250 – 2,50,000) @ 15% 99,937.50
Add: HEC @ 4% 3,997.50
Tax Liability 1,03,935.00
Rounded off u/s 288B 1,03,940.00
Solution 17:
Computation of income under the head House Property
Gross Annual Value 9,90,000.00
Working Note: `
(a) Fair Rent (90,000 x 11) 9,90,000
(b) Expected Rent 9,90,000
(c) Rent Received/Receivable (80,000 x 10) 8,00,000
If there was no vacancy, in that case rent received/receivable would have
been `8,80,000 and it was still less than expected rent, therefore GAV
shall be expected rent.
GAV 9,90,000
Less: Municipal Tax Nil
Net Annual Value 9,90,000.00
Less: 30% of NAV u/s 24(a) (2,97,000.00)
Less: Interest on capital borrowed u/s 24(b) (1,83,750.00)
Working Note:
Prior period interest
From 01.07.2016 to 31.03.2018
= (15,00,000 x 10% x 1) + (15,00,000 x 10% x 9/12)
= `1,50,000 + `1,12,500 = `2,62,500
Installment = `2,62,500/5 = `52,500
Current period interest
From 01.04.2018 to 31.03.2019
= (15,00,000 x 10% x 9/12) + (7,50,000 x 10% x 3/12)
= `1,12,500 + `18,750 = `1,31,250
Total interest on capital borrowed
Income Under The Head House Property 222

= `52,500 + ` 1,31,250 = `1,83,750


Income under the head House Property 5,09,250.00
Gross Total Income 5,09,250.00
Less: Deduction u/s 80C {Repayment of housing loan} (1,50,000.00)
Total Income 3,59,250.00
Computation of Tax Liability
Tax on `3,59,250 at slab rate 5,462.50
Add: HEC @ 4% 218.50
Tax Liability 5,681.00
Rounded off u/s 288B 5,680.00
Solution 18: `
Computation of income under the head House Property
Net Annual Value Nil
Less: Interest on capital borrowed u/s 24(b) (1,48,500.00)
Working Note: `
Current period interest
01.04.2018 to 31.03.2019
11,00,000 x 10% = 1,10,000
Prior period interest
From 01.07.2015 to 31.03.2017
11,00,000 x 10% x 21/12 = 1,92,500
Instalment = 1,92,500/5 = 38,500
Total interest = 1,10,000 + 38,500 = 1,48,500
Loss under the head House Property (1,48,500.00)
Income under the head capital gains
Short term capital gains u/s 111A 120,00,000.00
Gross Total Income 118,51,500.00
Less: Deduction u/s 80C to 80U Nil
Total Income 118,51,500.00
Computation of Tax Liability
Tax on `116,01,500 (`118,51,500 – 2,50,000) @ 15% 17,40,225.00
Add: Surcharge @ 15% 2,61,033.75
Tax before health & education cess 20,01,258.75
Add: HEC @ 4% 80,050.35
Tax liability 20,81,309.10
Rounded off u/s 288B 20,81,310.00

Solution 19: `
Option I
House I is self-occupied
Income from house I (30,000.00)
House II is deemed to be let out
Gross Annual Value 66,000.00
Working Note: `
(a) Fair Rent 64,000
(b) Municipal Valuation 66,000
(c) Higher of (a) or (b) 66,000
(d) Standard Rent (6,000 x 12) 72,000
(e) Expected rent {Lower of (c) or (d)} 66,000
GAV = Expected Rent 66,000
Less: Municipal taxes Nil
Net Annual Value 66,000.00
Income Under The Head House Property 223

Less: 30% of NAV u/s 24(a) (19,800.00)


Less: Interest on capital borrowed u/s 24(b) Nil
Income from house II 46,200.00
Income under option I [46,200 + (30,000)] 16,200.00
Option II
House I is deemed to be let out
Gross Annual Value 50,000.00
Working Note: `
(a) Fair Rent 20,000
(b) Municipal Valuation 55,000
(c) Higher of (a) or (b) 55,000
(d) Standard Rent 50,000
(e) Expected Rent {Lower of (c) or (d)} 50,000
GAV = Expected Rent 50,000
Less: Municipal taxes Nil
Net Annual Value 50,000.00
Less: 30% of NAV u/s 24(a) (15,000.00)
Less: Interest on capital borrowed u/s 24(b) (41,708.33)
Working Note:
Current period interest
From 01.04.2018 to 28.02.2019
3,50,000 x 13% x 11/12 = `41,708.33
Income from house I (6,708.33)
House II is self occupied
Income from house II Nil
Income under option II (6,708.33)
Option II is better
Income under the head House Property (6,708.33)
Add: Unrealised rent received (2,000 – 600) 1,400.00
Loss under the head House Property (5,308.33)
Income under the head Capital Gains (LTCG) 20,00,000.00
Gross Total Income 19,94,691.67
Less: Deduction u/s 80C Nil
Total Income 19,94,691.67
Rounded off u/s 288A 19,94,690.00
Computation of Tax Liability
Tax on `17,44,690 (`19,94,690 – `2,50,000) @ 20% 3,48,938.00
Add: HEC @ 4% 13,957.52
Tax Liability 3,62,895.52
Rounded off u/s 288B 3,62,900.00

Solution 20: `
Gross Annual Value 5,76,000.00
Working Note: `
(a) Fair Rent (50,000 x 12) 6,00,000
(b) Municipal Valuation (47,000 x 12) 5,64,000
(c) Higher of (a) or (b) 6,00,000
(d) Standard Rent (48,000 x 12) 5,76,000
(e) Expected rent {Lower of (c) or (d)} 5,76,000
(f) Rent Receivable (45,000 x 7) 3,15,000
If there was no vacancy, in that case rent received/receivable would have been
`4,05,000 and it was still less than expected rent, therefore GAV shall be expected
Income Under The Head House Property 224

rent
GAV 5,76,000
Less: Municipal Tax (56,400.00)
Net Annual Value 5,19,600.00
Less: 30% of NAV u/s 24(a) (1,55,880.00)
Less: Interest on capital borrowed u/s 24(b) (42,000.00)
Income under the head House Property 3,21,720.00
Gross Total Income 3,21,720.00
Less: Deduction u/s 80C to 80U Nil
Total Income 3,21,720.00
Computation of Tax Liability
Tax on `3,21,720 at slab rate 3,586.00
Less: Rebate u/s 87A (2,500.00)
Tax before health & education cess 1,086.00
Add: HEC @ 4% 43.44
Tax Liability 1,129.44
Rounded off u/s 288B 1,130.00
Solution 21: `
Income from self occupied house
Net Annual Value Nil
Less: 30% of NAV u/s 24(a) Nil
Less: Interest on capital borrowed u/s 24(b) (30,000)
Income from self occupied house (30,000)
Income from partly self occupied and partly let out house
Gross Annual Value 3,60,000
Working Note: `
(a) Fair Rent (20,000 x 12) 2,40,000
(b) Municipal Valuation (18,000 x 12) 2,16,000
(c) Higher of (a) or (b) 2,40,000
(d) Standard Rent (15,000 x 12) 1,80,000
(e) Expected Rent 1,80,000
(f) Rent Receivable (45,000 x 8) 3,60,000
GAV = Higher of (e) or (f) 3,60,000
Less: Municipal taxes (20,000)
Net Annual Value 3,40,000
Less: 30% of NAV u/s 24(a) (1,02,000)
Less: Interest on capital borrowed u/s 24(b) (45,000)
Income from House Property 1,93,000
Income under the head House Property 1,63,000
[`1,93,000 + `(30,000)]
Solution 22: ` `
Gross Annual Value 12,00,000.00
Working Note: `
(a) Fair rent (98,000 x 12) 11,76,000
(b) Rent receivable (1,00,000 x 12) 12,00,000
GAV {Higher of (a) or (b)} 12,00,000
Less: Municipal Taxes (85,000.00)
Net Annual Value 11,15,000.00
Less: 30% of NAV u/s 24(a) (3,34,500.00)
Less: Interest on capital borrowed u/s 24(b) Nil
Income Under The Head House Property 225

7,80,500.00
Add: Arrears of rent (Sec 25A) 3,00,000
Less: 30% of `3,00,000 (90,000) 2,10,000.00
9,90,500.00
Add: Unrealised Rent (4,00,000 – 1,20,000) 2,80,000.00
Income under the head House Property 12,70,500.00
Income under the head Other Sources 20,000.00
Gross Total Income 12,90,500.00
Less: Deduction u/s 80C to 80U Nil
Total Income 12,90,500.00
Computation of Tax Liability
Tax on `12,90,500 at slab rate 1,99,650.00
Add: HEC @ 4% 7,986.00
Tax Liability 2,07,636.00
Rounded off u/s 288B 2,07,640.00
Solution 23: `
Option I
Flat I is self occupied
Income (30,000.00)
Flat II is deemed to be let out
Gross Annual Value 50,000.00

Working Note: `
(a) Fair Rent 45,000
(b) Municipal Valuation 50,000
(c) Higher of (a) or (b) 50,000
Expected Rent 50,000
GAV = Expected Rent 50,000
Less: Municipal taxes (5,000.00)
Net Annual Value 45,000.00
Less: 30% of NAV u/s 24(a) (13,500.00)
Less: Interest on capital borrowed u/s 24(b) Nil
Income 31,500.00
Income under option I [(30,000) + 31,500] 1,500.00
Option II
Flat I is deemed to be let out
Gross Annual Value 85,000.00
Working Note: `
(a) Fair Rent 1,25,000
(b) Municipal Valuation 95,000
(c) Higher of (a) or (b) 1,25,000
(d) Standard Rent 85,000
(e) Expected Rent {Lower of (c) or (d)} 85,000
GAV = Expected Rent 85,000
Less: Municipal taxes (9,500.00)
Net Annual Value 75,500.00
Less: 30% of NAV u/s 24(a) (22,650.00)
Less: Interest on capital borrowed u/s 24(b) (45,000.00)
Income 7,850.00
Flat II is self occupied
Income Nil
Income under option II is `7,850
Income Under The Head House Property 226

Hence Option I is better.


Computation of Total Income
Income under the head House Property 1,500.00
Income under the head Business Profession 7,00,000.00
Gross Total Income 7,01,500.00
Less: Deduction u/s 80C to 80U Nil
Total Income 7,01,500.00
Computation of Tax Liability
Tax on `7,01,500 at slab rate 40,300.00
Add: HEC @ 4% 1,612.00
Tax Liability 41,912.00
Rounded off u/s 288B 41,910.00

Solution 24: `
Computation of income from House Property of Mr. X
Net annual value is Nil Nil
(Since house is self – occupied)
Less: Deduction u/s 24(b) (2,00,000)
Interest paid on borrowed capital
25,00,000 @ 12% = `3,00,000
As per second proviso to section 24(b)
interest deduction restricted to `2,00,000
Loss under the head “House Property” (2,00,000)
Computation of income from house property of Mr. Y
Ground Floor (Self Occupied)
Net Annual Value Nil
Less: 30% of NAV u/s 24(a) Nil
Less: Interest on capital borrowed u/s 24(b) (86,250)
Working Note:
Current period interest
From 01.04.2018 to 31.03.2019
= 15,00,000 x 10% x 1/2 = `75,000
Prior period interest
From 01.07.2017 to 31.03.2018
= 15,00,000 x 10% x 9 / 12 = 1,12,500
1,12,500 allowed in 5 equal instalments
= 1,12,500 / 5 = ` 22,500 per annum
= 22,500 / 2 = `11,250
Total interest = `75,000 + ` 11,250 = `86,250
Loss from house property (86,250)
First Floor (Let Out)
Gross Annual Value 1,20,000
Working Note: `
(a) Fair Rent (1,20,000 x 9/12) 90,000
(b) Municipal Value (92,000 x 9/12) 69,000
(c) Higher of (a) or (b) 90,000
(d) Expected Rent 90,000
(e) Rent Received/Receivable = 20,000 x 6 1,20,000
GAV = Higher of (d) or (e) 1,20,000
Less: Municipal taxes (5,000)
Net Annual Value 1,15,000
Less: 30% of NAV u/s 24(a) (34,500)
Income Under The Head House Property 227

Less: Interest on capital borrowed u/s 24(b) (86,250)


Working Note:
Current period interest
From 01.04.2018 to 31.03.2019
= 15,00,000 x 10% x 1/2 = `75,000
Prior period interest
From 01.07.2017 to 31.03.2018
= 15,00,000 x 10% x 9 / 12 = 1,12,500
1,12,500 allowed in 5 equal instalments
= 1,12,500 / 5 = ` 22,500 per annum
= 22,500 /2 = `11,250
Total Interest = `75,000 + `11,250 = `86,250
Loss from house property (5,750)
Loss under the head “income from house property” of Mr. Y
(Both ground floor and first floor) (92,000)

Solution 25: `
Computation of income under the head House Property
Gross Annual Value 27,000
Working Note: `
(a) Municipal Valuation 22,000
(b) Expected Rent 22,000
(c) Rent Received/Receivable (3,000 x 2) + (3,500 x 6) 27,000
GAV = Higher of (b) or (c) 27,000
Less: Municipal taxes Nil
Net Annual Value 27,000
Less: 30% of NAV u/s 24(a) (8,100)
Less: Interest on capital borrowed u/s 24(b) (7,875)
Working Note:
= [(60,000 x 15% x 6/12) + (45,000 x 15% x 6/12)] = `7,875
Income under the head House Property 11,025

Solution 26:
Computation of income of Unit-I
Since the unit is self-occupied throughout the year. Hence its income shall be computed under section 23(2),
accordingly there will be loss `30,000.
Computation of income of Unit-II
It will be considered to be partially self-occupied and partially let out and income shall be computed under
section 23(3) in the manner given below: `
Gross Annual Value 82,500.00
Working Note: `
(a) Fair Rental Value 76,500
(b) Municipal Valuation 65,000
(c) Higher of (a) or (b) 76,500
(d) Standard Rent 70,000
Expected Rent {Lower of (c) or (d) 70,000
(e) Expected Rent 70,000
(f) Rent Received/Receivable (7,500 x 11) 82,500
GAV = Higher of (e) or (f) 82,500
Less: Municipal taxes (7,800.00)
Net Annual Value 74,700.00
Less: 30% of NAV u/s 24(a) (22,410.00)
Income Under The Head House Property 228

Less: Interest on capital borrowed u/s 24(b) (72,900.00)


Working note:
Current period interest
From 01.04.2018 to 31.03.2019
= 9,00,000 x 12% = `1,08,000
Prior period interest
From 01.07.2015 to 31.03.2017
= 9,00,000 x 12% x 21 / 12 =1,89,000
Installment = 1,89,000 / 5 = 37,800
Total interest= 1,08,000 + 37,800 = 1,45,800
Interest allowed for one unit = 1,45,800 / 2 = `72,900
Loss from house property (20,610.00)
Loss under the head House Property is
(`20,610) + (`30,000) (50,610.00)
Income under the head Other Sources 3,90,000.00
Gross Total Income 3,39,390.00
Less: Deductions u/s 80C to 80U Nil
Total Income 3,39,390.00
Computation of Tax Liability
Tax on `3,39,390 at slab rate 4,469.50
Less: Rebate u/s 87A (2,500.00)
Tax before health & education cess 1,969.50
Add: HEC @ 4% 78.78
Tax Liability 2,048.28
Rounded off u/s 288B 2,050.00
Note: Since condition regarding certificate has not been complied with hence interest has been allowed
maximum to the extent of `30,000.

Solution 27: `
Unit I
Gross Annual Value 1,36,500
Working Note: `
(a) Fair Rental Value (1,75,000 x 70%) 1,22,500
(b) Municipal Valuation (1,55,000 x 70%) 1,08,500
(c) Higher of (a) or (b) 1,22,500
(d) Standard Rent (1,65,000 x 70%) 1,15,500
(e) Expected Rent {Lower of (c) or (d)} 1,15,500
(f) Rent Received/Receivable (13,000 x 10.5) 1,36,500
In this case, rent R/R is higher than the expected rent, GAV shall be Rent R/R
GAV 1,36,500
Less: Municipal taxes (35,000 x 70%) (24,500)
Net Annual Value 1,12,000
Less: 30% of NAV u/s 24(a) (33,600)
Less: Interest on capital borrowed u/s 24(b) Nil
Income under the head House Property 78,400
Unit II
Gross Annual Value 55,000
Working Note: `
(a) Fair Rental Value (1,75,000 x 30%) 52,500
(b) Municipal Valuation (1,55,000 x 30%) 46,500
(c) Higher of (a) or (b) 52,500
(d) Standard Rent (1,65,000 x 30%) 49,500
Income Under The Head House Property 229

(e) Expected Rent {Lower of (c) or (d)} 49,500


(f) Rent Received/Receivable (5,000 x 11) 55,000
In this case, rent R/R is higher than the expected rent, GAV shall be Rent R/R
GAV 55,000
Less: Municipal taxes (35,000 x 30%) (10,500)
Net Annual Value 44,500
Less: 30% of NAV u/s 24(a) (13,350)
Less: Interest on capital borrowed u/s 24(b) Nil
Income under the head house property 31,150
Total income from house property (78,400 + 31,150) 1,09,550
Solution 28: `
Gross Annual Value 86,400
Working Note: `
(a) Fair Rental Value 75,000
(b) Municipal Valuation 68,000
(c) Higher of (a) or (b) 75,000
(d) Standard Rent (7,200 x 12) 86,400
(e) Expected Rent {Lower of (c) or (d)} 75,000
(f) Rent Received/Receivable (7,200 x 12) 86,400
GAV = Higher of (e) or (f) 86,400
Less: Municipal taxes (14,000)
Net Annual Value 72,400
Less: 30% of NAV u/s 24(a) (21,720)
Less: Interest on capital borrowed u/s 24(b) Nil
Interest paid to non-resident without deducting tax at source is not deductible
Income from house property 50,680
Add: Recovery of unrealised rent sec 25A 15,400
(22,000 – 6,600)
Income under the head House property 66,080
Solution 29: `
Option I
House I
It is assumed to be self-occupied
Income from house property I (30,000.00)
Interest on the capital borrowed = `59,583.33
(5,00,000 x 13% x 11/12 = 59,583.33).
But subject to maximum of `30,000
Interest upto `2,00,000 is allowed only if the loan is taken for purchase or construction of the house i.e. if
the loan is taken for reconstruction, higher amount is not allowed.
House II
Assumed to be let out house
Gross Annual Value 66,000.00
Working Note: `
(a) Fair rent 64,000
(b) Municipal Valuation 66,000
(c) Higher of (a) or (b) 66,000
(d) Standard Rent (6,000 x 12) 72,000
(e) Expected Rent {Lower of (c) or (d)} 66,000
GAV = Expected Rent 66,000
Less: Municipal taxes Nil
Net Annual Value 66,000.00
Income Under The Head House Property 230

Less: 30% of NAV u/s 24(a) (19,800.00)


Less: Interest on capital borrowed u/s 24(b) Nil
Income from house property II 46,200.00

Option II
House I
Assumed to be let out
Gross Annual Value 50,000.00
Working Note: `
(a) Fair rent 20,000
(b) Municipal Valuation 55,000
(c) Higher of (a) or (b) 55,000
(d) Standard Rent 50,000
(e) Expected Rent {Lower of (c) or (d)} 50,000
GAV = Expected Rent 50,000
Less: Municipal Taxes Nil
Net Annual Value 50,000.00
Less: 30% of NAV u/s 24(a) (15,000.00)
Less: Interest on capital borrowed u/s 24(b) (59,583.33)
Income from house property I (24,583.33)
House II
Assumed to be self occupied
Income from house II Nil
Income under option I = (30,000) + 46,200 = `16,200
Income under Option II = (`24,583.33)
Hence option II is better.
House III
Gross Annual Value 7,20,000
Working Note: `
(a) Fair rent (60,000 x 12) 7,20,000
(b) Expected Rent 7,20,000
(c) Rent Received/Receivable (50,000 x 12) 6,00,000
GAV = Expected Rent 7,20,000
Less: Municipal Taxes (11,000.00)
Net Annual Value 7,09,000.00
Less: 30% of NAV u/s 24(a) (2,12,700.00)
Less: Interest on capital borrowed u/s 24(b) (55,000.00)
Income from house III 4,41,300.00
Income under the head House Property
House I and II option II (24,583.33)
House III 4,41,300.00
Recovery of unrealised rent (house I) 1,400.00
(2,000 – 600)
Income under the head House Property 4,18,116.67
Computation of Total Income
Income under the head House Property 4,18,116.67
Income under the head Capital Gains (long term capital gain) 3,50,000.00
Income from Other Sources 250.00
Gross Total Income 7,68,366.67
Less: Deduction u/s 80C to 80U Nil
Total Income 7,68,366.67
Rounded off u/s 288A 7,68,370.00
Income Under The Head House Property 231

Computation of Tax Liability


Tax on `4,18,370 at slab rate 8,418.50
Tax on `3,50,000 @ 20% u/s 112 70,000.00
Tax before health & education cess 78,418.50
Add: HEC @ 4% 3,136.74
Tax Liability 81,555.24
Rounded off u/s 288B 81,560.00
Solution 30:
(i) Income under the head House Property.
(ii) Income under the head Capital Gains.
(iii) Income under the head Business/Profession.
(iv) Income under the head Other Sources.
(v) Income under the head House Property.
(vi) Income under the head Business/Profession.
(vii) Income under the head Business/Profession.
(viii) Income under the head Business/Profession.
(ix) Income under the head Other Sources.
(x) Income under the head House Property.
Income Under The Head House Property 232

EXAMINATION QUESTIONS
NOV – 2017
Question 4(a) (Marks 5)
Mr. Aditya, a resident but not ordinarily resident in India during the Assessment Year 2019-20. He owns
two houses, one in Dubai and the other in Mumbai. The house in Dubai is let out there at a rent of DHS
20,000 p.m. (1DIRHAM=INR 18). The entire rent is received in India. He paid Property tax of DHS 2,500
and Sewerage Tax DHS 1,500 there, for the Financial Year 2018-19. The house in Mumbai is self-
occupied.
He has taken a loan of `25,00,000 to construct the house on 1st June, 2015 @ 12%. The construction was
completed on 31st May, 2017 and he occupied the house on 1st June, 2017. The entire loan is outstanding as
on 31st March, 2019. Property tax paid in respect of the second house is `2,400 for the Financial Year
2018-19. Compute the income chargeable under the head “Income from House property” in the hands of
Mr. Aditya for the Assessment Year 2019-20.

Solution:
In the given Mr. Aditya is NOR Hence Income received in India is taxable in India.
Computation of Income from House Property of Mr. Aditya for the Assessment Year 2019-20
`
GAV of the house in Dubai
(20,000 p.m. x `18 per DHS x 12 months) 43,20,000.00
Less: Municipal taxes paid (1500 +2500) x `18 per DHS (72,000.00)
Net Annual Value 42,48,000.00
Less: Statutory deduction under section 24(a) @ 30% of NAV (12,74,400.00)
Income from House property 29,73,600.00

GAV of house at Mumbai (self occupied)


Gross Annual Value Nil
Less: Municipal taxes paid Nil
Net Annual Value (NAV) Nil
Less: Statutory deduction under section 24(a) @ 30% of NAV Nil
Less: Interest on capital borrowed u/s 24(b) (2,00,000)
Working Note:
Prior period interest
From 01.06.2015 to 31.03.2017
= 25,00,000 x 12% x 22 / 12 = 5,50,000
5,50,000 allowed in 5 equal instalments
= 5,50,000 / 5 = 1,10,000 per annum
Current period interest
From 01.04.2018 to 31.03.2019
= 25,00,000 x 12% x 1 = 3,00,000
Total Interest = 1,10,000 + 3,00,000 = 4,10,000 but maximum upto ` 2,00,000
(assuming all the conditions for higher Interest is satisfied.)
Loss from House property (2,00,000.00)
Income from House property 27,73,600.00

MAY – 2017
Question 4(a) (ii) (4 Marks)
Income Under The Head House Property 233

Mr. Ganesh owns a commercial building whose construction got completed in June 2017. He took a loan of
`15 lakhs from his friend on 1-8-2016 and had been paying interest calculated at 15% per annum. He is
eligible for pre-construction interest as deduction as per the provisions of the Income Tax Act.
Mr. Ganesh has let out the commercial building at a monthly rent of `40,000 during the financial year 20l8-
19. He paid municipal tax of `18,000 each for the financial year 2017-18 and 2018-19 on 1-5-2018 and 5-4-
2019 respectively.
Compute income under the head. 'House Property' of Mr. Ganesh for the Assessment Year 2019-20.

Solution:
Computation of Income under the head House Property `
Gross Annual Value (40,000 x 12) 4,80,000
Less: Municipal Taxes (18,000)
Net Annual Value 4,62,000
Less: 30% of NAV u/s 24(a) (1,38,600)
Less: Interest on capital borrowed u/s 24(b) (2,55,000)
Working Note:
Prior period interest
From 01.08.2016 to 31.03.2017
= 15,00,000 x 15% x 8 / 12 = 1,50,000
1,50,000 allowed in 5 equal instalments
= 1,50,000 / 5 = 30,000 per annum
Current period interest
From 01.04.2018 to 31.03.2019
= 15,00,000 x 15% x 1 = 2,25,000
Total Interest = 30,000 + 2,25,000 = 2,55,000
Income under the head House Property 68,400

Note: As per proviso to section 23(1), Municipal Taxes actually paid by the owner during the previous year
is allowed to be deducted from GAV.

NOV – 2015
Question 6(a). (8 Marks)
Mr. X constructed a shopping complex. He had taken a loan of `25 lakhs for construction of the said
property on 01.08.2016 from SBI @ 10% for 5 years. The construction was completed on 30.06.2017.
Rental income received from shopping complex `30,000 per month let out for the whole year. Municipal
Taxes paid for shopping complex `8,000.
Arrears of rent received from shopping complex `1,20,000.
Interest paid on loan taken from SBI for purchase of house for use as own residence for the period 2018-
2019 `3 lakhs. The loan was taken after 01.04.1999 and house was purchased within 5 years from the end of
the year in which loan was taken and assessee has submitted certificate certifying the amount of interest.
You are required to compute Income from House property of Mr. X for AY 2019-2020 as per Income Tax
Act, 1961.
Solution 6(a):
Income under the head House Property
Income from shopping Complex
Gross Annual Value 3,60,000.00
Less: Municipal taxes paid (8,000.00)
Net Annual Value 3,52,000.00
Less: 30% of NAV u/s 24(a) (1,05,600.00)
Less: Interest on capital borrowed u/s 24(b) (2,83,333.33)
Working Note: `
Prior period interest
Income Under The Head House Property 234

From 01.08.2016 to 31.03.2017


25,00,000 x 10% x 8/12 = 1,66,666.67
Installment allowed = 1,66,666.67/5 = 33,333.33 33,333.33
Current year interest
From 01.04.2018 to 31.03.2019
25,00,000 x 10% x 1 = 2,50,000 2,50,000.00
Total 2,83,333.33
Loss from shopping complex (36,933.33)
Add: Arrear of Rent Received 1,20,000
Less: Deduct @ 30% (36,000) 84,000.00
Income from shopping complex 47,066.67
Income from self-occupied property
Gross Annual Value NIL
Less: Municipal taxes paid NIL
Net Annual Value NIL
Less: 30% of NAV u/s 24(a) NIL
Less: Interest on capital borrowed u/s 24(b) (2,00,000.00)
Loss from self-occupied property (2,00,000.00)
Loss under the head House Property (47,066.67-(2,00,000)) (1,52,933.33)
Deduction From Gross Total Income 235

DEDUCTION FROM GROSS


TOTAL INCOME
CHAPTER VI-A
SECTION 80C TO 80U

PARTICULARS SECTIONS
1. Deduction in respect of investment in NSC/PPF/Bank FD etc. 80C
2. Deduction in respect of contribution to certain pension funds 80CCC
3. Deduction in respect of medical insurance premia 80D
4. Deduction in respect of maintenance including medical treatment of a 80DD
dependent who is a person with disability
5. Deduction in respect of medical treatment, etc. 80DDB
6. Deduction in respect of interest on loan taken for higher education 80E
7. Deduction in respect of donations to certain funds, charitable institutions, etc. 80G
8. Deduction in respect of rents paid 80GG
9. Deduction in respect of certain donations for scientific research or rural 80GGA
development
10. Deduction in case of donation to political parties 80GGB/80GGC
11. Deduction in respect of employment of new workmen 80JJAA
12. Deduction in respect of royalty income, etc., of authors of certain books 80QQB
13. Deduction in respect of royalty on patents 80RRB
14. Deduction from interest on savings bank account 80TTA
15. Deduction from interest on savings bank account to Senior Citizens 80TTB
16. Deduction in case of disabled person 80U
Deduction From Gross Total Income 236

DEDUCTION FROM GROSS TOTAL INCOME


CHAPTER VI-A
SECTION 80C to 80U
Deductions under section 80C to 80U are allowed from gross total income to compute total income however
such deduction is allowed only from normal income.
 As per section 112, such deductions are not allowed from long term capital gains.
 As per section 58(4), such deductions are not allowed from casual income.
 As per section 111A, such deduction are not allowed from short term capital gains on the sale of
short term equity shares or short term units of equity oriented mutual funds provided securities
transaction tax has been paid.
Example
Mr. X has income under the head salary `75,000, income from long term capital gains `2,10,000 and casual
income `35,000, in this case maximum amount of deductions allowed shall be `75,000.

Question 1 [V. Imp.]. Write a note on deduction under Section 80C.


Answer: Deduction under section 80C shall be allowed only to
(i) an individual
(ii) Hindu Undivided Family
(Deduction under section 80C is not allowed to any partnership firm or a company etc.)
Deduction shall be allowed to the extent of the following investments but as per section 80CCE, maximum
deduction allowed shall be `1,50,000 (Including deduction under section 80CCC and section 80CCD).
1. Deduction shall be allowed if amount has been invested in National Saving Certificate (NSC) and NSC
are just like a fixed deposit with a bank. Amount can be invested in the name of self, spouse or minor
children and HUF can invest the amount in the name of any of its members. Deduction shall be allowed
equal to the amount invested and amount received on maturity shall be exempt from income tax but
interest shall be taxable every year on accrual basis but payment of interest shall be received on maturity.
Deduction under section 80C shall also be allowed for such accrued interest but no deduction shall be
allowed for accrued interest of the year in which assessee has received payment. NSC are issued for 5
years / 10 years.
Example
Mr. X has income under the head House Property `10 lakh and he invested `50,000 in NSC on
01.10.2018. He has invested `40,000 in previous year 2017-18 also and there is accrued interest of
`4,000 in previous year 2018-19. He has also received `1,00,000 on maturity of NSC which were
invested in the earlier year and original amount is `60,000 and interest for current year is `8,000, in this
case his tax liability shall be
Income under the head House Property 10,00,000
Income under the head Other Sources (4,000+ 8,000) 12,000
Gross Total Income 10,12,000
Less: Deduction u/s 80C
Investment in current year 50,000
Accrued interest 4,000 (54,000)
(no deduction shall be allowed for interest received on maturity)
Total Income 9,58,000
Tax on `9,58,000 at slab rate 1,04,100
Add: HEC @ 4% 4,164
Tax Liability 1,08,264
Rounded off u/s 288B 1,08,260
2. Public provident fund is a deposit scheme run by Central Government and account can be opened in the
bank or post office and maturity shall be after 15 years and the account can be opened in the name of
Deduction From Gross Total Income 237

self, spouse or children. HUF can open the account in the name of any of its members. Amount
received on maturity shall be exempt from income tax and also interest is exempt from income tax. No
deduction is allowed under section 80C for interest.
3. Investment in fixed deposit for a period of 5 years or more with scheduled banks, provided the term
deposit are issued in accordance with a scheme notified by the Central Government. (Bank Term Deposit
Scheme, 2006 – depositor can be individual or Hindu Undivided Family. The deposit should be for a
minimum period of 5 years. Interest income shall be taxable on accrual basis and it will not qualify for
deduction under section 80C.) Principal amount received on maturity shall be exempt. Individual can
deposit the amount only in his own name and HUF can deposit the amount in the name of any of its
member.
4. Five Year Post Office Time Deposit Account.
An assessee is allowed to invest the amount in five year post office time deposit account and deduction
shall be allowed equal to the amount invested. Interest shall be paid on annual basis and it will be
taxable and deduction under section 80C is not allowed. Amount received on maturity shall be
exempt. Individual can invest the amount in his name and HUF can invest the amount in the name of
any of its member.
Pre-mature payment is allowed but amount received on pre-mature payment shall be taxable.
5. If an assessee has taken a loan from a notified organization like banks or financial institution etc. for
purchase or construction of a residential house, in such cases deduction shall be allowed equal to the
amount re-paid by the assessee towards principal (not towards interest).
If loan has been taken for Addition, Alteration, or Repairs etc of the house property, no deduction is
allowed.
If the assessee has transferred the house property before the expiry of 5 years from the end of the
financial year in which possession of such properties was taken by him, no deduction shall be allowable
in the previous year in which the house property has been transferred. The deduction allowed in the past
years shall be considered to be income of the assessee of the previous year in which the house property
is transferred.
6. If any individual or HUF has taken life policy, deduction shall be allowed for the premium paid for such
life policy and individual can take the policy in the name of self, spouse and children and Hindu
Undivided Family can take the policy in the name of any of its members. (Children may be
dependant or independent or may be married or unmarried or step or adopted.)
Deduction is allowed equal to the premium paid but maximum upto 10% of capital sum assured, i.e. if
premium paid is more than 10% of capital sum assured, deduction shall be allowed only for 10% of sum
assured. (In respect of policy issued before 01.04.2012, 10% shall be taken as 20%)
If LIC policy has been taken in the name of a person who is suffering from disability given under section
80U or from a specified disease given under section 80DDB, 10% shall be taken as 15% but it is
applicable for the policies taken w.e.f 01.4.2013 onwards.
If an assessee has discontinued a life insurance policy before paying premium for a period of atleast 2
years, deduction allowed in the earlier years shall be considered to be income of the year in which
policy has been discontinued.
As per section 10(10D), any payment received on maturity of insurance policy shall be exempt from
income tax i.e. even the amount of bonus received shall be exempt from income tax. If the policy
holder has paid premium of more than the specified limit (10% / 15% / 20%) in any of the years,
amount received on maturity shall be chargeable to tax but if the amount has been received on the
death of the policy holder, it will be exempt from income tax.
e.g. Mr. X has paid premium of one life policy `25,000 and sum assured is ` 1,00,000 and policy was
taken on 01.04.2012 onwards, in this case deduction allowed shall be `10,000 but if policy was taken
before 01.04.2012, deduction allowed shall be `20,000. If Mr. X is a handicapped person and policy was
taken w.e.f 01.04.2013 onwards, deduction allowed shall be `15,000
7. Payment of tuition fees to School, College, University or any other Educational Institution in India
provided the fees has been paid in connection with the children of the assessee and further for
maximum two children and it should be whole time education. Children shall include even adopted
and step children also. Deduction is not allowed to HUF. If payment is made outside India, deduction is
Deduction From Gross Total Income 238

not allowed. Similarly if payment is given for part time education or correspondence course, deduction is
not allowed.
8. Employees contribution to statutory provident fund or recognised provident fund or approved
superannuation fund
9. Investment in Units of Unit trust of India or mutual fund including Unit Linked Insurance Plan of
UTI or mutual fund.
10. Subscription to Notified Deposit Schemes of NHB e.g. subscription to Home Loan Account Scheme
of NHB.
11. Investment in equity shares or debentures etc forming part of an eligible issue.
Eligible issue means an issue made by an Indian Public Ltd Company or a Public Financial
Institution, a Mutual Fund etc. and the funds so collected are utilised for Developing, Maintaining
and Operating Infrastructure Facility.
If any such equity shares etc. have been sold within a period of 3 years from the date of purchase, in
such cases deduction earlier allowed shall be considered to be income of such year
12. Investment in notified bonds issued by the National Bank for Agriculture and Rural Development.
13. Senior Citizens Savings Scheme. Amount should be invested in the name of self and amount received
on maturity shall be exempt and interest shall be payable on quarterly basis and interest received is
taxable. Deduction under section 80C for interest is not allowed.
14. Investment in Sukanya Samridhi Account and amount can be invested by an individual as guardian in
the name of girl child who is of the age of 10 years or less. Interest received is exempt. Amount received
on maturity is exempt. Account can be closed after the completion of 21 years of age. In case of
marriage, payment is allowed after completion of 18 years of age.

Deduction shall be allowed only if the amount has been actually paid by the assessee i.e. if the amount
is due but not paid deduction is not allowed. E.g. Premium of `25,000 was due on 27.03.2019 but it
was paid on 10.04.2019, in this case no deduction is allowed in the previous year 2018-19, rather
deduction shall be allowed in the previous year 2019-20.
Illustration 1: Mr. X, aged about 61 years, has earned a lottery income of ` 1,20,000 (gross) during the P.Y.
2018-19. He also has a business income of `6,00,000. He invested an amount of `10,000 in Public Provident
Fund account and ` 24,000 in National Saving Certificates and `60,000 in eligible mutual funds. He has
paid premium of `30,000 for a life policy having sum assured `2,00,000 and policy was taken after
01.04.2012. Compute his tax liability for assessment year 2019-20.
Solution:
Computation of total taxable income of Mr. X for A.Y.2019-20
Income under the head business/profession 6,00,000
Income under the head Other Sources (casual income) 1,20,000
Gross Total Income 7,20,000
Less: Deduction u/s 80C
1. Public Provident Fund (10,000)
2. Investment in NSC (24,000)
3. Mutual Fund (60,000)
4. Payment of premium for LIC policy (20,000)
Total Income 6,06,000
Computation of Tax Liability
Tax on casual income `1,20,000 @ 30% 36,000
Tax on `4,86,000 at slab rate 9,300
Tax before health & education cess 45,300
Add: HEC @ 4% 1,812
Tax Liability 47,112
Rounded off u/s 288B 47,110

Illustration 2: Mr. X has income under the head Business/Profession `19,90,000.


His investments are as given below:
Deduction From Gross Total Income 239

1. Investment in NSC `50,000


2. Investment in PPF in name of Mrs. X `5,000
3. Payment of premium for LIC policy taken in the name of dependent father on 16.06.2018 and its premium
paid is `11,000
4. Payment of premium for LIC policy taken in the name of independent son on 15.04.2018 and its premium
paid is `6,000 (sum assured `1,00,000)
5. Payment of premium for LIC policy taken in the name of independent married daughter on 11.01.2019
and its premium paid is `21,000 (sum assured `1,00,000)
Compute Income Tax liability for the A.Y. 2019-20.
Solution: `
Income under the head business/profession 19,90,000
Gross Total Income 19,90,000
Less: Deduction u/s 80C
1. Investment in NSC (50,000)
2. Investment in PPF in the name of Mrs. X (5,000)
3. Payment of premium for LIC policy taken in the name of dependent father NIL
4. Payment of premium for LIC policy taken in the name of independent son (6,000)
5. Payment of premium for LIC policy taken in the name of independent married
daughter (allowed 10% of sum assured) (10,000)
Total Income 19,19,000
Computation of Tax Liability
Tax on `19,19,000 at slab rate 3,88,200
Add: HEC @ 4% 15,528
Tax Liability 4,03,728
Rounded off u/s 288B 4,03,730

Illustration 3: Mr. X furnishes you the following information:


Raw material purchased `5,00,000. Manufacturing expenses (revenue nature) `2,00,000.
Sale price `18,00,000. Plant & machinery acquired `2,60,000. Depreciation is allowed @ 15%.
He has made the investments as given below:
(i) Fixed deposit with State Bank for two years `5,000.
(ii) Investment in National Saving Certificates `5,000.
(iii) Deposit in Public Provident Fund Account in the name of major married independent son `5,000.
(iv) Deposit in Public Provident Fund Account in the name of minor son `5,000.
(v) Payment of premium for LIC policy in name of major married independent daughter on
15.09.2018 `5,000. (sum assured `1,00,000).
(vi) Payment of premium for LIC policy in name of major married independent son on 11.11.2018
`5,000. (sum assured `20,000)
(vii) Investment in Home Loan Account Scheme of National Housing Bank `5,000 (Investment was
made out of past savings).
(viii) Investment in units of Mutual Funds notified under section 10(23D) `5,000. (Investment was
made out of current income exempt from income tax).
(ix) Investment in Equity Shares of Infrastructure Companies `5,000.
(x) Payment of Tuition fees of his son to a private coaching centre for coaching in taxation `5,000.
Compute his income and tax liability for assessment year 2019-20.
Solution:
Computation of income under the head Business/profession
Sale price 18,00,000.00
Less: Purchase Price (5,00,000.00)
Less: Manufacturing expenses (2,00,000.00)
Less: Depreciation on plant and machinery (2,60,000 x 15%) (39,000.00)
Income under the head Business/profession 10,61,000.00
Gross Total Income 10,61,000.00
Deduction From Gross Total Income 240

Less: Deduction u/s 80C


National Saving Certificate (5,000.00)
Public Provident Fund (10,000.00)
LIC policy in name of major married independent daughter (5,000.00)
LIC policy in name of major married independent son (2,000.00)
Home Loan Account Scheme (5,000.00)
Units of Mutual Funds (5,000.00)
Equity Shares of Infrastructure Companies (5,000.00)
Total Income 10,24,000.00
Computation of Tax Liability
Tax on `10,24,000 at slab rate 1,19,700.00
Add: HEC @ 4% 4,788.00
Tax Liability 1,24,488.00
Rounded off u/s 288B 1,24,490.00

Illustration 4: Compute the eligible deduction under section 80C for A.Y.2019-20 in respect of life
insurance premium paid by Mr. X during the P.Y.2018-19, the details of which are given hereunder –
Date of issue Person insured Actual capital sum Insurance premium
of policy assured (`) paid during 2018-19
(`)
(i) 01.04.2011 Self 2,00,000 50,000
(ii) 01.05.2017 Spouse 1,50,000 20,000
(iii) 01.06.2018 Handicapped Son 3,00,000 60,000
(section 80U disability)
Solution:
Date of issue Person insured Actual Insurance Deduction Remark
of policy capital sum premium u/s 80C for (restricted to
assured paid during A.Y.2019-20 % of sum
2018-19 assured)
(i) 01.04.2011 Self 2,00,000 50,000 40,000 20%
(ii) 01.05.2017 Spouse 1,50,000 20,000 15,000 10%
(iii) 01.06.2018 Handicapped 3,00,000 60,000 45,000 15%
Son (section
80U disability)
Total 1,00,000

Illustration 5 (From RTP): Compute the eligible deduction under section 80C for A.Y.2019-20 in respect
of life insurance premium paid by Mr. X during the P.Y.2018-19, the details of which are given hereunder –
Date of issue Person insured Actual capital sum Insurance premium
of policy Assured (`) paid during 2018-19
(`)
(i) 01.06.2011 Mr. X 4,00,000 75,000
(ii) 01.05.2017 Mrs. X, his wife 1,00,000 25,000
(iii) 01.07.2018 Ms. Y, his handicapped 5,00,000 60,000
daughter (section 80U
disability)
(iv) 01.07.2018 Mr. Z, his son 1,00,000 25,000
Total Premium paid 1,85,000
Solution:
Computation of eligible deduction under section 80C for A.Y. 2019-20
(in respect of life insurance premium paid by Mr. X)
Deduction From Gross Total Income 241

Date of issue Person insured Actual Insurance Restricted Deduction


of policy capital sum premium to % of u/s 80C for
assured paid during sum assured) A.Y.2019-20
2018-19
(i) 01.06.2011 Mr. X 4,00,000 75,000 20% 75,000
(ii) 01.05.2017 Mrs. X 1,00,000 25,000 10% 10,000
(iii) 01.07.2018 Handicapped 5,00,000 60,000 15% 60,000
daughter
(iv) 01.07.2018 Son 1,00,000 25,000 10% 10,000

Total 1,55,000
Maximum deduction u/s 80C restricted to 1,50,000
Note:
In respect of policies issued Maximum deduction u/s 80C
(% of actual capital sum assured)
between 01.04.2003 and 31.03.2012 20%
between 01.04.2012 and 31.03.2013 10%
on or after 01.04.2013
- Insurance on life of person with disability u/s 80U 15%
- Others 10%

NOV – 2014 (2 Marks)


Determine the eligibility and quantum of deduction under Chapter VI-A in the following case:
Mr. X has paid life insurance premium of `70,000 during the year. (Sum Assured 3,50,000 policy issued on
31.03.2011)
Solution:
Deduction for Life Insurance Premium shall be available under section 80C of `70,000 (being maximum
20% of `3,50,000 as the policy was issued before 01.04.2012)
MAY – 2001 (5 Marks)
The particulars of income of Mrs. X. aged 55 years for the financial year 2018-19 are given below: `
(1) Gross salary received from M/s ABC Ltd. for the year 4,00,000
(2) Rental income received from a commercial complex 12,000 p.m.
(3) Arrears of rent received from the complex, which were not charged to
tax in any earlier years 30,000
(4) Interest paid on loan taken for the purchase of a house from a scheduled
bank for use as own residence 1,20,000
(5) Repayment of instalments of loan taken from the bank for the purchase
of the above property 60,000
(6) Deposits in public provident fund account
(i) Towards loan taken from public provident account 20,000
(ii) Out of current year’s income 40,000
(7) Investment made in units of a mutual fund approved by the board under
section 80C of the Income-Tax Act. 40,000
Compute the total income of Mrs. X and the tax payable thereon in respect of assessment year 2019-20.
Answer: ` `
Computation of total income and tax liability Mrs. X
Income from salary
Gross Salary 4,00,000.00
Income under the head Salary 4,00,000.00
Income from house property
Let out commercial complex
Gross Annual Value (12,000 x 12) 1,44,000.00
Deduction From Gross Total Income 242

Less: Municipal taxes Nil


Net Annual Value 1,44,000.00
Less: 30% of NAV u/s 24(a) (43,200.00)
Less: Interest on capital borrowed u/s 24(b) Nil
Income from let out property 1,00,800.00
Property self- occupied for residence
Net Annual Value Nil
Less: Interest on capital borrowed u/s 24(b) (30,000.00)
Loss from self–occupied property (30,000.00)

Arrears of rent Section 25A 30,000


Less: (30% of `30,000) (9,000) 21,000.00
Income under the head House Property 91,800.00
Gross Total Income 4,91,800.00
Less: Deduction u/s 80C (1,40,000.00)
Repayment of loan taken to purchase residential house property 60,000
Deposit in public provident fund out of current income 40,000
Investment made in units of mutual fund for infrastructure facility 40,000
Total Income 3,51,800.00
Computation of Tax Liability
Tax on `3,51,800 at slab rate 5,090.00
Add: HEC @ 4% 203.60
Tax Liability 5,293.60
Rounded off u/s 288B 5,290.00

Question 2: Write a note on deduction in case of contribution to a Pension Fund.


Answer: Deduction in respect of contribution to certain pension Funds Section 80CCC
In general in case of life insurance, lump sum amount is paid to the policyholder but in some of the life
policies pension is given instead of lump sum amount e.g. Jeevan Suraksha policy. If any person has paid
premium for such policy, deduction is allowed under section 80CCC instead of section 80C. Deduction is
allowed only to an individual and individual can take the policy only in his name (and not in name of spouse
or children). Any pension received shall be taxable under the head Other Sources. If the assessee has
surrendered the policy, amount received shall be taxable under the head Other Sources.
e.g. (i) Mr. X has taken Jeevan Suraksha Policy in his name and has paid premium of `10,000, in this case
deduction allowed shall be `10,000
(ii) Mr. X has taken Jeevan Suraksha Policy in the name of Mrs. X and has paid premium of `10,000, in this
case deduction is not allowed.
(iii) Mr. X has taken Jeevan Suraksha Policy in the name of his dependant son and has paid premium of
`10,000, in this case deduction is not allowed.

As per section 80CCE, Maximum Deduction allowed u/s 80C + 80CCC + 80CCD shall be ` 1,50,000.

Illustration 6: Mr. X has income under the head business/profession `3,35,000.


He has made the following investments:-
 NSC `10,000
 Investment in post office 5 year time deposit account `15,000
 Payment of premium for life policy in the name of major married independent son on 10.10.2018
`30,000 (sum assured `90,000)
 Paid premium of ` 11,000 for Jeeven Suraksha policy taken in name of Mr. X on 11.11.2018.
Compute income tax liability for A.Y 2019-20.
Solution: `
Income under the head Business/Profession 3,35,000.00
Gross Total Income 3,35,000.00
Deduction From Gross Total Income 243

Less: Deduction u/s 80C


NSC (10,000.00)
Investment in 5 years post office (15,000.00)
Payment of premium of LIC (9,000.00)
Less: Deduction u/s 80CCC (11,000.00)
Total Income 2,90,000.00

Computation of Tax Liability


Tax on `2,90,000 at slab rate 2,000.00
Less: Rebate u/s 87A (2,000.00)
Tax Liability Nil
Illustration 7: The gross total income of Mr. X for the A.Y.2019-20 is ` 5,00,000. He has made the
following investments/payments during the F.Y.2018-19 – `
(1) Contribution to PPF 90,000
(2) Payment of tuition fees to Apeejay School, New Delhi, for education of his son
studying in Class XI 45,000
(3) Repayment of housing loan taken from Standard Chartered Bank 25,000
(4) Contribution to approved pension fund of LIC 10,000
Compute the eligible deduction under Chapter VI-A for the A.Y.2019-20.
Solution:
Computation of deduction under Chapter VI-A for the A.Y.2019-20
Particular `
Deduction under section 80C
(1) Contribution to PPF 90,000
(2) Payment of tuition fees to Apeejay School, New Delhi, for education of his son studying in 45,000
Class XI
(3) Repayment of housing loan 25,000
1,60,000
Deduction under section 80CCC
(1) Contribution to approved pension fund of LIC 10,000
1,70,000
As per section 80CCE, the aggregate deduction under section 80C, 80CCC and 80CCD has to be
restricted to ` 1,50,000
Deduction allowable under Chapter VI-A for the A.Y.2019-20 1,50,000

Deduction under section 80CCD shall be discussed under the head Salary.

Question 3 [V. Imp.]: Write a note on deduction in case of payment of premium for Medical
Insurance/Mediclaim Policy.
Answer: Deduction in respect of Medical Insurance Premia Section 80D
1. Deduction shall be allowed only to an individual or Hindu Undivided Family.
2. Deduction shall be allowed if the assessee has made payment towards
(i) Medical Insurance or
(ii) Central Government Health Scheme or such other scheme as may be notified by the
Central Government in this behalf
(iii) Preventive Health Check-up
3. Individual can make payment for wife or husband or dependent children and deduction shall be
allowed equal to the amount paid but subject to a maximum of `25,000 but in case of senior
citizen deduction shall be allowed upto `50,000.
If the individual has taken policy in the name of parents (dependent or independent), additional
deduction shall be allowed to the extent of the premium paid but maximum `25,000, however, if
the policy has been taken in the name of senior citizen, deduction shall be allowed to the extent of
`50,000.
Deduction From Gross Total Income 244

Deduction for Preventive Health Check up shall be maximum `5,000 in aggregate for self, spouse,
dependant children and parents.
Hindu Undivided Family can take the policy in the name of any of its members and deduction shall
be allowed in the similar manner.
Payment should be made otherwise than in cash but payment for preventive health check-up can be
made in any manner.
In case of a senior citizen, in general medi-claim policy is not issued hence expenditure can be incurred
on their medical treatment and deduction for such expenditure shall also be allowed but limit shall be the
same as given above.
e.g. (i) Mr. X has taken medi claim policy in his name and paid premium `27,000 by cheque, in this case
deduction allowed shall be `25,000 but if Mr. X is a senior citizen, deduction allowed shall be `27,000
(ii) Mr. X has paid `7,000 for self for preventive health checkup and `7,000 for preventive health checkup
of his father, in this case deduction allowed shall be `5,000
(iii) Mr. X paid premium of medi claim policy by cheque for self, spouse and children `22,000 and for
parents `28,000, in this case deduction allowed shall be `47,000
(iv) Mr. X paid premium of medi claim policy `15,000 in cash, deduction allowed shall be Nil
(v) Mr. X paid premium of medi claim policy by cheque `18,000 in the name of his father who is not
dependant on Mr. X, deduction allowed shall be `18,000
Illustration 8: Mr. X, aged 40 years, paid medical insurance premium of `18,000 by cheque during the
P.Y.2018-19 to insure his health as well as the health of his spouse. He also paid medical insurance premium
of `26,000 by cheque during the year to insure the health of his father, aged 63 years, who is not dependent
on him. He contributed ` 5,000 by cheque to Central Government Health Scheme during the year. He has
incurred `3,000 in cash on preventive health check-up of himself and his spouse and `4,000 by cheque on
preventive health check-up of his father.
Compute the deduction allowable under section 80D for the A.Y.2019-20.
Solution:
Deduction allowable under section 80D for the A.Y.2019-20
Particulars ` `
Actual Maximum
Payment deduction
allowable
A. Premium paid and medical expenditure incurred for self and spouse
(i) Medical insurance premium paid for self and spouse 18,000 18,000
(ii) Contribution to CGHS 5,000 5,000
(iii) Exp. on preventive health check-up of self & spouse 3,000 2,000
26,000 25,000
B. Premium paid and medical expenditure incurred for father, who is a
senior citizen
(i) Mediclaim premium paid for father, who is 63 years of age 26,000 26,000
(ii) Expenditure on preventive health check-up of father 4,000 3,000
30,000 29,000
Total deduction under section 80D (25,000 + 29,000) 54,000

Illustration 9: Mr. X, aged 40 years, paid medical insurance premium of ` 20,000 by cheque during the
P.Y.2018-19 to insure his health as well as the health of his spouse and dependent children. He also paid
medical insurance premium of `21,000 by cheque during the year to insure the health of his father, aged 67
years, who is not dependent on him. He contributed `6,000 by cheque to Central Government Health
Scheme during the year. Compute the deduction allowable under section 80D for the A.Y.2019-20.
Solution:
Deduction allowable under section 80D for the A.Y.2019-20
Particulars `
(i) Medical insurance premium paid for self, spouse and dependent children 20,000
(ii) Contribution to CGHS 6,000
Deduction From Gross Total Income 245

26,000
Deduction allowed 25,000
(iii) Mediclaim premium paid for father, who is 67 years of age 21,000
46,000
Note – The total deduction under (i) and (ii) above should not exceed ` 25,000. In this case, since the total
of (i) and (ii) is exceeding `25,000 (i.e., ` 26,000) hence it is restricted to ` 25,000.
Illustration 10: Mr. Arjun (52 years old) furnishes the following particulars in respect of the following
payments:
S. No. Particulars Amount (`)
1. Premium paid for insuring the health of –
 Self 10,000
 Spouse 8,000
 Dependant son 4,000
 Mother 18,000
2. Paid for Preventive Health Check-up of
 himself 2,000
 spouse 1,500
 mother 4,000
3. Incurred medical expenditure of ` 25,000 and `15,000 for his mother, aged 80 years
and father, aged 85 years. Both mother and father are resident in India.

Compute the deduction available to Mr. Arjun under section 80D for the A.Y. 2019-20.
Solution:
Computation of deduction under section 80D for the A.Y. 2019-20
S. No. Particulars Amount (`)
1. I. In respect of premium paid for insuring the health of -
 Self 10,000
 Spouse 8,000
 Dependant son 4,000
22,000
II. In respect of expenditure on preventive health check-up of -
 Self 2,000
 Spouse 1,500
3,500
Restricted to [`25,000 – ` 22,000, since maximum deduction is `25,000] 3,000
Aggregate of deduction (I+II) under (1) restricted to 25,000
2. I. In respect of payment towards health insurance premium for his mother 18,000
II. In respect of preventive health check-up of his mother [`4,000,
restricted to `2,000, (`5,000 – `3,000), since maximum deduction for 2,000
preventive health check-up under section 80D is `5,000]
III. Medical expenditure for father would only be eligible for deduction
[See Note below] 15,000
35,000
Amount of deduction under (2) 35,000
Total deduction under section 80D [(1) + (2)] 60,000
Note: Irrespective of the fact that the mother of Arjun is a very senior citizen the deduction under section
80D would not available to him in respect of the medical expenditure incurred for his mother, since Mr.
Arjun has taken a health insurance policy for his mother.
Deduction From Gross Total Income 246

MAY – 2015 (4 Marks)


Compute the eligible deduction under Chapter VI-A for the Assessment year 2019-20 of Ms. Roma, who has
a gross total income of `15,00,000 for the assessment year 2019-20 and provide the following informations
about his investments/payments during the financial year 2018-19:
SI. Particulars Amount
No. (` )
1. Life Insurance premium paid (Policy taken on 01.01.2012 and sum assured is 35,000
`1,50,000)
2. Public Provident Fund contribution. 90,000
3. Repayment of Housing loan to Bhartiya Mahila Bank, Bangalore. 20,000
4. Payment to L.I.C. Pension Fund 25,000
5. Mediclaim Policy taken for self, spouse and dependent children, premium paid by 20,000
cheque
6. Medical Insurance premium paid for parents (Senior Citizen), premium paid by cheque 25,000
Solution: ` `
Gross Total Income 15,00,000
Less:
Deduction u/s 80C
L.I.C. Premium paid 30,000
(Paid `35,000, but maximum allowed 20% of `1,50,000)
P.P.F. amount paid 90,000
Repayment of housing loan to Bhartiya Mahila Bank 20,000
Deduction u/s 80 CCC
L.I.C. pension fund 25,000
Total 1,65,000
Maximum deduction allowed 80CCE (1,50,000)
Deduction u/s 80D
Mediclaim for self, spouse and dependent children (20,000)
(assumed amount paid through cheque)
Mediclaim premium for parents (25,000)
(assumed amount paid through cheque)
Total Income 13,05,000
NOV – 2011 (3 Marks)
The gross total income of Mr. X for the Assessment Year 2019-20, was `12,00,000. He has made the
following investments/payments during the financial year 2018-19.
1. L.I.C. Premium paid (Policy value `1,00,000) (Policy taken after 01.04.2012) 25,000
2. P.P.F. amount paid 70,000
3. Repayment of housing Loan to Indian Bank 50,000
4. Payment made to L.I.C. pension fund 25,000
5. Medical insurance premium for self, wife and dependant Children. 18,000
6. Mediclaim premium for parents (aged over 80 years) 30,000
Compute eligible deduction under Chapter VI-A for the Assessment Year 2019-20 and also compute his tax
liability.
Answer: ` `
Gross Total Income 12,00,000
Less:
Deduction u/s 80C
L.I.C. Premium paid 10,000
(Paid `25,000, but maximum allowed 10% of `1,00,000)
P.P.F. amount paid 70,000
Repayment of housing loan to Indian Bank 50,000
Deduction From Gross Total Income 247

Deduction u/s 80 CCC


L.I.C. pension fund 25,000
Total 1,55,000
Maximum deduction allowed 80CCE (1,50,000)
Deduction u/s 80D
Mediclaim for self, wife and dependant children (18,000)
(assumed amount paid through cheque)
Mediclaim premium for parents (30,000)
(assumed amount paid through cheque)
Total Income 10,02,000
Computation of Tax Liability
Tax on `10,02,000 at slab rate 1,13,100
Add: HEC @ 4% 4,524
Tax Liability 1,17,624
Rounded off u/s 288B 1,17,620
JUNE – 2009 (2 Marks)
Mr. X, an individual, made payment of health insurance premium to General Insurance Corporation in an
approved scheme. Premium paid on his health is `10,000 and his spouse’s health is `15,000 during the year
2018-19. He also paid health insurance premium of `25,000 on his father’s health who is a senior citizen and
not dependent on him. The payments have not been made by cash. Compute the amount of deduction under
Chapter VI-A of the Act, available to Mr. X from his gross total income for the assessment year 2019-20.
Answer.
Mr. X will be eligible to claim deduction under section 80D on payment of health insurance premium. The
premium is paid otherwise than by way of cash and hence qualifies for deduction under section 80D.
Therefore, the amount of deduction under section 80D would be –
Particulars Amount (`)
On health insurance premium paid on the health of himself and his spouse
(`10,000 + `15,000 = `25,000) 25,000
On health insurance premium paid on the health of his father, `25,000
(whether dependent or not) 25,000
Total deduction under section 80D 50,000
Question 4 [Imp.]: Deduction in case of expenditure in connection with handicapped dependant
relative.
Answer: Deduction in respect of maintenance including medical treatment of a dependant who is a
person with disability Section 80DD
1. Deduction is allowed only to a resident individual and a resident Hindu Undivided Family.
2. Deduction is allowed if the assessee has incurred any expenditure for the medical treatment, training
and rehabilitation etc. of a dependant disabled person, or has deposited any amount with LIC or any
other insurer for the benefit of such dependant.
3. “Dependant” in the case of an individual, means the spouse, children, parents, brothers and sisters
who are dependant on the individual and in the case of Hindu Undivided Family means any member of
the Hindu Undivided Family who is dependant on such Hindu Undivided Family.
4. Deduction allowed shall be `75,000 irrespective of the expenditure incurred by the assessee and in case
of severe disability, deduction allowed shall be `1,25,000.
5. The assessee should enclose a certificate with the return from prescribed medical authority.
Illustration 11: Mr. X is a resident individual. He deposits a sum of `25,000 with Life Insurance
Corporation every year for the maintenance of his handicapped grandfather. A copy of the certificate from
the medical authority is submitted. Compute the amount of deduction available under section 80DD for the
A.Y. 2019-20.
Solution: Since the amount deposited by Mr. X was for his grandfather, he will not be allowed any
deduction under section 80DD. The deduction is available if the individual assessee incurs any expense for a
dependant disabled relative. Grandfather does not come within the definition of dependant relative.
Illustration 12: What will be the deduction if Mr. X had made this deposit for his dependant father?
Deduction From Gross Total Income 248

Solution: Since the expense was incurred for a dependant disabled relative, Mr. X will be entitled to claim a
deduction of `75,000 under section 80DD, irrespective of the amount deposited. In case his father has severe
disability, the deduction would be `1,25,000.
MAY – 1997 (4 Marks)
In respect of assessment year 2019-20, an author of text-books for schools furnishes the following
particulars and request you to work out his tax liability: ` `
1. Royalty from Printers Ltd. on publication of books 2,20,000
2. Capital gains long term 1,90,000
3. Other Sources:
(a) Interest on Bank fixed Deposits 12,000
(b) Dividend income from Indian company 3,000
(c) Income from units of U.T.I. 5,000 20,000
Deductions: (i) Contributions towards:
(a) LIC Pension Scheme 15,000
(b) LIC Premium 10,000
(ii) Contribution to public provident fund 10,000
(iii) Investment in National Savings Certificates 50,000
(iv) Medical treatment of handicapped dependent relative 20,000
Answer: `
Income under the head Capital Gains (LTCG) 1,90,000
Computation of income under the head Other Sources
Royalty income 2,20,000
Income under the head Other Sources
Interest from bank fixed deposits 12,000
Dividend income {exempt u/s 10(34)} Nil
Interest from units of UTI {exempt u/s 10(35)} Nil
Gross Total Income 4,22,000
Less: Deduction u/s 80C
LIC Premium (10,000)
Contribution in Public provident fund (10,000)
National Saving Certificate (50,000)
Less: Deduction u/s 80CCC (15,000)
Less: Deduction u/s 80DD (75,000)
Total Income 2,62,000
Computation of Tax Liability
Tax on LTCG `12,000 (`1,90,000 – `1,78,000) @ 20% u/s 112 2,400
Tax on `72,000 at slab rate Nil
Less: Rebate u/s 87A (2,400)
Tax Liability Nil
Question 5 [V. Imp.]: Briefly explain the provisions under the Income Tax Act relating to deduction
from gross total income in the case of blind or physically handicapped persons.
Answer:
Deduction in case of handicapped person Section 80U
(1) In computing the total income of an individual, being a resident, who, at any time during the previous
year, is certified by the medical authority to be a person with disability, there shall be allowed a
deduction of a sum of `75,000.
Provided that where such individual is a person with severe disability, the provisions shall have effect
as if for the words “`75,000”, the words “`1,25,000” had been substituted.
(2) The assessee should enclose a certificate with the return from prescribed medical authority.
e.g. (i) Mr. X is suffering from a disability and has income under the head salary `10,00,000 and he has
invested `1,00,000 in NSC, in this case deduction allowed under section 80C shall be `1,00,000 and under
section 80U shall be `75,000
Deduction From Gross Total Income 249

(ii) Mr. X is suffering from a severe disability and has income under the head salary `10,00,000 and he has
invested `2,00,000 in NSC, in this case deduction allowed under section 80C shall be `1,50,000 and under
section 80U shall be `1,25,000.
NOV – 2008 (4 Marks)
Briefly explain provisions of section 80U of the Income-tax Act, 1961, in respect of deduction available on
permanent physical disability.
Solution: Refer Answer given in the Chapter
Question 6 [Imp.]: Write a note on deduction in case of expenditure on the treatment of Specified
Disease.
Answer: Deduction in respect of medical treatment, etc. of specified disease Section 80DDB
1. Deduction is allowed only to a resident individual or resident Hindu Undivided Family.
2. Deduction is allowed if the assessee has incurred any amount for treatment of such disease as are
specified in the rule 11DD.
3. The expenditure can be incurred for himself or a dependant person, and in case of an individual,
such person may be spouse, children, parents, brothers or sisters who are dependant on such
individual and in case of Hindu Undivided Family such person may be any member of the Hindu
Undivided Family who is dependant on the Hindu Undivided Family.
4. Deduction allowed shall be the amount incurred or `40,000 whichever is less and if the amount has
been paid with regard to a Senior Citizen, deduction allowed shall be upto `1,00,000.
5. Deduction allowed shall be reduced by the amount received under medi claim insurance and also
by the amount which has been paid by the employer.
6. The assessee should enclose a certificate with the return from prescribed medical authority.
Example
Mr. X has incurred `1,25,000 on the treatment of a specified disease for himself, in this case deduction
allowed shall be `40,000 but if a claim of `10,000 has been received under medi-claim policy, deduction
allowed shall be `30,000. If Mr. X is a senior citizen, deduction allowed shall be 1,00,000 – 10,000 = 90,000
Question 7 [Imp.]: Write a note on deduction in case of payment of interest on loan taken for Higher
Education.
Answer: Deduction in case of payment of interest on loan taken for pursuing Higher Education
Section 80E
1. Deduction is allowed only to an individual.
2. Deduction is allowed if the assessee has paid interest on loan taken by him from any financial
institutions or any approved charitable institution.
3. The loan should have taken for pursuing higher education which means any course of study pursued
after passing the Senior Secondary Examination or its equivalent.
4. Education can be either of self or spouse or children or any person for whom the assessee is legal
guardian.
5. The entire amount of interest paid by an individual is allowed as deduction.
6. No deduction shall be allowed for repayment of the principal loan amount.
7. Deduction is allowed for a maximum period of 8 years starting from the year in which first payment of
interest was given.
8. Approved charitable institution means the institution notified by the Central Government.
Financial institution means banking company or other financial institution notified by the Government.
9. No deduction is allowed after the period of 8 years.
Example
Mr. X has taken a loan of `2,00,000 from State Bank on 01.10.2009 for pursuing MBBS course & after
becoming a doctor he has given payment of interest of `45,000 on 01.10.2018, in this case deduction
allowed shall be `45,000.
Illustration 13: Mr. X has taken three education loans on April 1st, 2018, the details of which are given
below:
Loan 1 Loan 2 Loan 3
For whose education loan was taken Mr. X Son of Mr. X Daughter of Mr. X
Purpose of loan MBA B. Sc. B.A.
Deduction From Gross Total Income 250

Amount of loan (`) 5,00,000 2,00,000 4,00,000


Annual repayment of loan (`) 1,00,000 40,000 80,000
Annual repayment of interest (`) 20,000 10,000 18,000
Compute the amount deductible under section 80E for the A.Y.2019-20.
Solution:
Deduction under section 80E is available to an individual assessee in respect of any interest paid by him in
the previous year in respect of loan taken for pursuing his higher education or higher education of his spouse
or children. Higher education means any course of study pursued after senior secondary examination.
Therefore, interest repayment in respect of all the above loans would be eligible for deduction.
Deduction under section 80E = `20,000 + `10,000 + `18,000 = `48,000

Question 8 [V. Imp.]: Write a note on deduction in case of donations to the Notified Institutions.
Answer: Deduction in respect of donations to certain Funds, Charitable Institutions, etc. Section 80G
Deduction is allowed to all the assessees if they have given any donation or contribution to any of the below
mentioned institutions or funds and deduction allowed shall be either 100% or 50% of the donation given.
1. The Prime Minister’s National Relief Fund (100%)
2. The Prime Minister’s Armenia Earthquake Relief Fund (100%)
3. The National Foundation for Communal Harmony (100%)
4. The National Defence Fund (100%)
5. The National Children’s Fund (100%)
6. The Africa Fund (100%)
7. A University or any educational institution of national eminence as may be approved by the prescribed
authority in this behalf (100%)
8. The Chief Minister’s Earthquake Relief Fund, Maharashtra (100%)
9. The Andhra Pradesh Chief Minister’s Cyclone Relief Fund, 1996 (100%)
10. Any fund set up by the State Government of Gujarat for providing relief to the victims of earthquake
(100%)
11. The Chief Minister’s Relief Fund (100%)
12. The Lieutenant Governor’s Relief Fund in respect of any Union territory (100%)
13. Zila Saksharta Samiti. (100%)
14. The National Blood Transfusion Council (100%)
15. The State Blood Transfusion Council. (100%)
16. The National Illness Assistance Fund (100%)
17. The Army Central Welfare Fund (100%)
18. The Air Force Central Welfare Fund (100%)
19. The Indian Naval Benevolent Fund (100%)
20. The National Sports Fund (100%)
21. The National Cultural Fund (100%)
22. The Fund for Technology Development and Application (100%)
23. Any fund set up by a State Government to provide medical relief to the poor (100%)
24. The National Trust for Welfare of Persons suffering with Autism, Cerebral Palsy, Mental Retardation
and Multiple Disabilities.(100%)
25. Swachh Bharat Kosh (100%)
26. Clean Ganga Fund(100%)
27. National Fund for Control of Drugs (100%)
28. The Jawaharlal Nehru Memorial Fund (50%)
29. The Indira Gandhi Memorial Trust, (50%)
30. The Rajiv Gandhi Foundation (50%)
31. The Prime Minister’s Drought Relief Fund (50%)

 Deduction in case of donation to other institutions


If the donation has been given to any other institution or fund notified under section 80G, deduction allowed
shall be 50% of the qualifying amount. However, deduction allowed shall be 100% of the qualifying
Deduction From Gross Total Income 251

amount if the donation has been given to Government / local authority/ other notified institution for the
purpose of promoting family planning.
Qualifying amount = 10% of the adjusted gross total income or the donation (except donation to the above
mentioned 31 funds) given, whichever is less.
Adjusted gross total income = Gross Total Income – Long term capital Gains – Short term capital gains u/s
111A – All Deduction under section 80C to 80U except section 80G
The other institutions which may be notified under this section may be charitable organisation or social
organisation or religious organisation or other similar organisation.
No deduction shall be allowed under this section in respect of any donation unless such donation is of a sum
of money i.e. if donation is given in kind, deduction is not allowed.
No deduction shall be allowed under this section in respect of donation of any sum exceeding two
thousand rupees unless such sum is paid by any mode other than cash.
Illustration 14: Mr. X has income from business/profession `6,00,000 and long term capital gain `4,00,000
and short term capital gain u/s 111A `2,00,000 and casual income `1,00,000.
He has paid premium of a mediclaim policy amounting to `20,000 taken in the name of his dependant grand
father who is senior citizen and payment was made by a cheque on 09.01.2019.
He has given premium of Jeevan Suraksha policy `7,000, has donated `12,000 to the National Defence
Fund, `4,000 to Rajiv Gandhi Foundation and `3,00,000 to a charitable institution and `1,00,000 to a social
organization and `4,00,000 to religious organization and all such organization are notified under section
80G. (all the donations was made by cheque)
Compute his total income and tax liability for A.Y. 2019-20.
(b) Presume in the above question the assessee has given donation of `10,000 by cheque for family planning
also to the Government.
Compute his total income and tax liability for the assessment year 2019-20.
(c) Presume in part (b), donation to government for family planning is `2,00,000 by cheque.
Solution 14(a): `
Income under the head Business/Profession 6,00,000.00
Income under the head Capital Gain (LTCG) 4,00,000.00
Income under the head Capital Gain (STCG u/s 111A) 2,00,000.00
Income under the head Other Sources (casual income) 1,00,000.00
Gross Total Income 13,00,000.00
Less: Deduction u/s 80CCC (7,000.00)
Less: Deduction u/s 80G
(i) National Defence Fund (12,000.00)
(ii) Rajiv Gandhi Foundation (2,000.00)
(iii) Charitable Institution/ Social organization/ Religious organization (34,650.00)
Working Note:
Charitable Institution 3,00,000
Social organization 1,00,000
Religious organization 4,00,000
8,00,000

AGTI = GTI – LTCG – STCG u/s 111A – Deduction u/s 80C to 80U (except 80G)
= 13,00,000 – 4,00,000 – 2,00,000 – 7,000
= 6,93,000
Qualifying amount = 10% of AGTI or donation whichever is less
= 69,300 or 8,00,000 whichever is less
= 69,300
50% of the qualifying amount = 34,650
Total Income 12,44,350.00
Computation of Tax Liability
Tax on casual income `1,00,000 @ 30% u/s 115BB 30,000.00
Tax on STCG `2,00,000 @ 15% u/s 111A 30,000.00
Deduction From Gross Total Income 252

Tax on LTCG `4,00,000 @ 20% u/s 112 80,000.00


Tax on normal income `5,44,350 at slab rate 21,370.00
Tax before health & education cess 1,61,370.00
Add: HEC @ 4% 6,454.80
Tax Liability 1,67,824.80
Rounded off u/s 288B 1,67,820.00

Solution 14(b):
Computation of Total Income
Gross Total Income 13,00,000.00
Less: Deduction u/s 80CCC (7,000.00)
Less: Deduction u/s 80G
(i) National Defence Fund (12,000.00)
(ii) Rajiv Gandhi Foundation (2,000.00)
(iii) Other Donations u/s 80G (39,650.00)
Working Note: `
Charitable Institution 3,00,000
Social organization 1,00,000
religious organization 4,00,000
Family planning 10,000
8,10,000
AGTI = GTI – LTCG – STCG u/s 111A – Deduction u/s 80C to 80U (except 80G)
= 13,00,000 – 4,00,000 – 2,00,000 – 7,000
= 6,93,000
Qualifying amount = 10% of AGTI or donation whichever is less
= 69,300 or 8,10,000 whichever is less
= 69,300
Deduction of donation for family planning `10,000
50% of balance amount (i.e. `59,300) `29,650
Total deduction allowed 10,000 + 29,650 = 39,650
Total Income 12,39,350.00
Computation of Tax Liability
Tax on casual income `1,00,000 @ 30% u/s 115BB 30,000.00
Tax on STCG `2,00,000 @ 15% u/s 111A 30,000.00
Tax on LTCG `4,00,000 @ 20% u/s 112 80,000.00
Tax on normal income `5,39,350 at slab rate 20,370.00
Tax before health & education cess 1,60,370.00
Add: HEC @ 4% 6,414.80
Tax Liability 1,66,784.80
Rounded off u/s 288B 1,66,780.00

Solution 14(c): `
Computation of Total Income
Gross Total Income 13,00,000.00
Less: Deduction u/s 80CCC (7,000.00)
Less: Deduction u/s 80G
(i) National Defence Fund (12,000.00)
(ii) Rajiv Gandhi Foundation (2,000.00)
(iii) Other donations u/s 80G (69,300.00)
Working Note: `
Charitable Institution 3,00,000
Social organization 1,00,000
Deduction From Gross Total Income 253

religious organization 4,00,000


Family planning 2,00,000
10,00,000
AGTI = GTI – LTCG – STCG u/s 111A – Deduction u/s 80C to 80U (except 80G)
= 13,00,000 – 4,00,000 – 2,00,000 – 7,000
= 6,93,000
Qualifying amount = 10% of AGTI or donation whichever is less
= 69,300 or 10,00,000 whichever is less
= 69,300
Donation for family planning is `2,00,000 but maximum deduction allowed shall be
`69,300
Total Income 12,09,700.00
Computation of tax liability
Tax on casual income `1,00,000 @ 30% u/s 115BB 30,000.00
Tax on STCG `2,00,000 @ 15% u/s 111A 30,000.00
Tax on LTCG `4,00,000 @ 20% u/s 112 80,000.00
Tax on normal income `5,09,700 at slab rate 14,440.00
Tax before health & education cess 1,54,440.00
Add: HEC @ 4% 6,177.60
Tax Liability 1,60,617.60
Rounded off u/s 288B 1,60,620.00

MAY – 2017
Question 6(a) (ii) (4 Marks)
Mr. Rohan, a resident individual has Gross Total Income of `7,50,000 comprising of Income from Salary
and income from house property for the assessment year 2019-20. He provides the following information:

Paid `70,000 towards premium on life insurance policy of his Handicapped Son (Section 80U disability).
Sum assured `4,00,000 ; and date of issue of policy 1-8-2017.

Deposited `90,000 in tax saver deposit in the name of his major son in State Bank of India.

Contributed by cheque `25,000 to The Clean Ganga Fund, set up by the Central Government.
Compute the Total Income and deduction under Chapter VI-A for the Assessment year 2019-20.
Solution:
Computation of Total Income and deduction under chapter VI-A `
Gross Total Income 7,50,000
Less: Deduction under chapter VI-A
Deduction u/s 80C for LIC Premium (15% of 4,00,000) (60,000)
Deposited in Tax Saver Deposit in the name of major son (Nil)
Deduction u/s 80G – Contribution in Clean Ganga Fund (100%) (25,000)
Total Income 6,65,000
Note: Tax Saver deposits in the name of major son does not qualify for deduction u/s 80C since such
deposits has to be made in the name of assessee himself.

MAY – 2011 (3 Marks)


Mr. X having gross total income of `6,35,000 for the financial year 2018-19 furnishes you the following
information:
(i) Paid `25,000 towards premium on life insurance policy of his married daughter.
(ii) Contributed `10,000 by cheque to Prime Minister’s National Relief Fund.
(iii) Donated `20,000 to a Government recognized institution (by cheque) for scientific research.
Deduction From Gross Total Income 254

Note: Assume that the gross total income of Mr. X does not include any income under the head ‘profits and
gains of business or profession’.
Compute the total income of Mr. X for the assessment year 2019-20.
Answer.
Computation of total income of Mr. X for the A.Y. 2019-20
Particulars ` `
Gross Total Income 6,35,000
Less: Deductions under Chapter VI-A
(i) Premium on life insurance policy of his married daughter - Eligible for 25,000
deduction under section 80C
(ii) Contribution of `10,000 to PM’s National Relief Fund - Eligible for 100% 10,000
deduction under section 80G
(iii) Payment of `20,000 to a Government recognized institution for scientific 20,000
research - Eligible for deduction under section 80GGA (55,000)
Total Income 5,80,000
NOV – 2008 (4 Marks)
Mr. X declares gross total income `4,00,000 for the assessment year 2019-20. The gross total income
includes taxable long term capital gain ` 65,000 and short term capital gain `35,000 which is taxable @
15% under section 111A of the Income-tax Act, 1961. The details of fund investment made during the year
2018-19 are: `
(i) Medical insurance premium paid by cheque –
(a) in the name of Mr. X 4,000
(b) in name of Mrs. X 5,000
(ii) Contribution made to –
(a) Indira Gandhi Memorial Trust by cheque 7,000
(b) Delhi University (declared as an institution of national eminence) by cheque 3,000
(c) Zila Saksharta Samiti by cheque 5,000
(d) An approved charitable institute by cheque 30,000
(e) Government by cheque for the purpose of promoting family planning 10,000
(f) Hanuman Temple in Mohalla by cheque 20,000
Compute the total income of Mr. X chargeable to tax for the Assessment year 2019-20 and also compute his
tax liability.
Answer. (`)
Computation of Total Income of Mr. X for the A.Y. 2019-20
Gross Total Income 4,00,000.00
Less : Deduction
Section 80D
Medical insurance premium paid by cheque
(i) in the name of Mr. Prasad (4,000.00)
(ii) in name of Mrs. Prasad (5,000.00)
Section 80G
Donation to Indira Gandhi Memorial trust @ 50% of ` 7,000 (3,500.00)
Donation to Delhi University @ 100% (3,000.00)
Donation to Zila Saksharta Samiti @ 100% (5,000.00)
Other donations u/s 80G (19,550.00)
Working Note:
Donation to approved Charitable Institute 30,000
Donation to Government for promoting family planning 10,000
40,000
AGTI = GTI – LTCG – STCG u/s 111A – Deduction u/s
80C to 80U (Except 80G)
= 4,00,000 – 65,000 – 35,000 – 9,000
= 2,91,000
Deduction From Gross Total Income 255

Qualifying amount = 10% of AGTI or donation whichever is less


= 29,100 or 40,000
= 29,100
Deduction for family planning `10,000
50% of balance amount (i.e. 19,100)
= 9,550
Total deduction = 10,000 + 9,550 = 19,550
Total Income 3,59,950.00
Computation of Tax Liability
Tax on LTCG `65,000 @ 20% u/s 112 13,000.00
Tax on STCG `35,000 @ 15% u/s 111A 5,250.00
Tax on `2,59,950 at slab rate 497.50
Tax before health & education cess 18,747.50
Add: HEC @ 4% 749.90
Tax Liability 19,497.40
Rounded off u/s 288B 19,500.00

NOV – 2002 (4 Marks)


For the assessment year 2019-20, the gross total income of Mr. X was `4,50,240 which includes long term
capital gain of `45,000 and short term capital gain of `80,000.The gross total income also includes interest
income (fixed deposits) from banks of `12,000.
Mr. X has invested in public provident fund `60,000 and also paid medical insurance premium `11,000 by
cheque. Mr. X also contributed `15,000 by cheque to public charitable trust eligible for deduction u/s 80G.
Compute the total income and tax thereon of Mr. X, who is 70 years old as on 31.03.2019.
Answer:
Computation of Total Income `
Long Term Capital Gain 45,000.00
Short Term Capital Gain 80,000.00
Bank Interest 12,000.00
Other Income 3,13,240.00
Gross Total Income 4,50,240.00
Less: Deduction u/s 80C {Public Provident Fund} (60,000.00)
Less: Deduction u/s 80D (11,000.00)
Less: Deduction u/s 80G (7,500.00)
Working Note:
AGTI = GTI – LTCG – Deductions u/s 80C to 80U (Except 80G)
= 4,50,240 – 60,000 – 45,000 – 11,000
= `3,34,240
Qualifying Amount = 10% of AGTI or Donation given whichever is less
= `33,424 or 15,000
= 15,000
50% of qualifying amount = `7,500
Total Income 3,71,740.00

Computation of Tax Liability


Tax on long term capital gain `45,000 @ 20% u/s 112 9,000.00
Tax on normal income `3,26,740 at slab rate 1,337.00
Tax before health & education cess 10,337.00
Add: HEC @ 4% 413.48
Tax liability 10,750.48
Rounded off u/s 288B 10,750.00
Deduction From Gross Total Income 256

Question 9 [V. Imp.]: Write a note on deduction in case of payment of rent.


Answer: Deduction in case of payment of rent Section 80GG
1. Deduction is allowed only to an individual.
2. He should not be getting any house rent allowance and also he is not being provided with Rent Free
Accommodation by his employer.
3. He should not have any house in his name or in the name of the spouse or in the name of minor child
or in the name of Hindu Undivided Family of which he is a member, at a place where he ordinarily
resides or performs duties of his office or carries on his business or profession.
4. The assessee may have house at any other place but it should not be self occupied i.e. it may be let out or
vacant.
5. He has paid rent for the accommodation taken by him for his residence.
6. Deduction shall be allowed to such individual in case of payment of rent and deduction shall be allowed
to the extent of the least of the following:
(i) Rent paid over 10% of the adjusted gross total income
(ii) `5,000 p.m.
(iii) 25% of the adjusted gross total income
Adjusted Gross Total Income = Gross Total Income – Long term capital gains – Short term capital gains
u/s 111A – All Deduction of section 80C to 80U except section 80GG
 Deduction can be allowed even where the assessee is not an employee i.e. the persons having
business/profession can also avail deduction under section 80GG.
Illustration 15: Mr. X has income under the head Business/Profession `5,00,000 and LTCG of `2,00,000,
STCG u/s 111A `3,00,000 and casual income of `1,00,000.
He is paying rent for a house of `40,000 p.m. He has deposited `30,000 in home loan account scheme of
National Housing Bank.
He has complied with all the condition of section 80GG.
Compute income tax liability for A.Y. 2019-20.
Solution: `
Income under the head Business/Profession 5,00,000
Computation of income under the head Capital Gain
Long Term Capital Gain 2,00,000
Short Term Capital Gain u/s 111A 3,00,000
Income under the head capital gain 5,00,000
Computation of income under the head Other Sources
Casual income 1,00,000
Income under the head Other Sources 1,00,000
Gross Total Income 11,00,000
Less:
Deduction u/s 80C (30,000)
Deduction u/s 80GG (60,000)
Working Note:
Least of the following:
1. `4,80,000 – 57,000 = 4,23,000
2. 60,000
3. 25% x 5,70,000 = 1,42,500
AGTI = GTI – LTCG – STCG 111A – Deduction u/s 80C to 80U
(except 80GG)
= 11,00,000 – 2,00,000 – 3,00,000 – 30,000
= 5,70,000
Total Income 10,10,000
Computation of Tax Liability
Tax on casual income ` 1,00,000 @ 30% 30,000
Deduction From Gross Total Income 257

Tax on LTCG `2,00,000 @ 20% 40,000


Tax on STCG 111A ` 3,00,000 @ 15% 45,000
Tax on `4,10,000 at slab rate 8,000
Tax before health & education cess 1,23,000
Add: HEC @ 4% 4,920
Tax Liability 1,27,920

MAY – 2000 (4 Marks)


Mr. X is a retired Government officer aged 65 years, who derived the following income in respect of
financial year 2018-19. He resides in Cochin: `
Pension 1,95,000
Interest from bank deposits (fixed deposits) 1,52,000
Total income 3,47,000
He has paid `18,000 as premium to effect an insurance on his health and his dependant parents and it was
paid by a cheque. He pays a rent of `3,000 per month in respect of furnished accommodation. What is his
eligibility for deduction under Section 80GG? Compute his total income and tax liability for assessment year
2019-20.
What are the conditions to be satisfied by him to qualify for the deduction?
Answer: `
Pension 1,95,000.00
Income under the head Salary 1,95,000.00
Income under the head Other Sources {Bank Interest} 1,52,000.00
Gross Total Income 3,47,000.00
Less: Deduction under section 80D (18,000.00)
Less: Deduction under section 80TTB (50,000.00)
Less: Deduction under section 80GG (8,100.00)
Working Note:
Least of the following:
1. `60,000
2. 25% x 2,79,000 = `69,750
3. `36,000 – `27,900 = `8,100
(AGTI = `3,47,000 – `18,000 - `50,000 = `2,79,000)
Total Income 2,70,900.00
Computation of Tax Liability
Tax Liability Nil

Conditions to be fulfilled for grant of deduction under section 80GG :


1. The assessee should not be getting any house rent allowance and also he is not being provided with Rent
Free Accommodation by his employer.
2. The assessee should not have any house in his name or in the name of the spouse or in the name of minor
child or in the name of Hindu Undivided Family of which he is a member, at a place where he ordinarily
resides or performs duties of his office or employment or carries on his business or profession.
3. Also he should not have house even at any other place which he has declared to be self occupied.
4. The assessee has paid rent for the accommodation taken by him for his residence.

Question 10: Write a note on deductions under section 80GGA in respect of donations etc to certain
notified institutions.
Answer: Deduction in case of certain donation Section 80GGA
Deduction is allowed to all the assessees except the assessees whose gross total income includes income
which is chargeable under the head “Profits and gains of business or profession”. (because such assesses is
allowed to debit the amount to profit and loss account of business/profession)
Deduction is allowed in case of donation or contributions to any of the below mentioned institutions.
Deduction allowed is equal to the amount of donations.
Deduction From Gross Total Income 258

(i) Donation given to an institution notified under section 35 for scientific research / research in social
science or statistical research.
(ii) Donation given to an institution notified under section 35AC for eligible project i.e. the projects of
social or economic importance like construction of houses for the poor person or taking up drinking
water project or other similar projects.
(iii) Donation given to an institution notified under section 35CCA for rural development including
donation to Rural Development Fund setup by central Government or donation to National
Urban Poverty Eradication Fund.
No deduction shall be allowed under this section in respect of any sum exceeding ten thousand rupees
unless such sum is paid by any mode other than cash.
Example:
(i) Mr. X has donated `2,00,000 by cheque to an institution notified under section 35AC for eligible projects
and Mr. X do not have any business/profession, in this case he will be allowed deduction under section
80GGA for `2,00,000 but if he has business/profession, he will not be allowed deduction under section
80GGA rather he will be allowed to debit the amount to profit and loss account.
(ii) ABC Ltd. has donated `2,00,000 by cheque to an institution notified under section 35CCA for rural
development and company has business/profession, in this case deduction under section 80GGA is not
allowed but company can debit the amount to profit and loss account.

Illustration 16: Mr. X has income under the head salary `6,00,000 and income under the head house
property `7,00,000 and he has submitted information as given below:
(i) Paid premium of life policy `40,000 (sum assured `1,50,000) and policy has taken before 01.04.2012 in
the name of Mr. X
(ii) Paid premium of life policy `40,000 (sum assured `1,50,000) and policy has taken after 01.04.2012 in
the name of Mrs. X
(iii) Paid premium of life policy `40,000 (sum assured `1,50,000) and policy has taken before 01.04.2012 in
the name of father of Mr. X who is dependant on Mr. X.
(iv) Paid premium of life policy `40,000 (sum assured `1,50,000) and policy has taken before 01.04.2012 in
the name of son of Mr. X who is not dependant on Mr. X.
(v) He has donated `1,00,000 by cheque in rural development fund setup by government.
(vi) He has paid premium of Jeevan Suraksha Policy `10,000 by cheque in the name of Mrs. X.
(vii) He has paid `15,000 in cash in connection with preventive health checkup for his father.
(viii) He has donated `60,000 by cheque to a charitable institution notified under section 80G
Compute his tax liability assessment year 2019-20.
Solution: `
Income under the head Salary 6,00,000
Income under the head House Property 7,00,000
Gross Total Income 13,00,000
Less: Deduction u/s 80C
Premium of life policy in name of Mr. X (30,000)
Premium of life policy in name of Mrs. X (15,000)
Premium of life policy in name of Son (30,000)
Less: Deduction u/s 80D (Preventive Health Checkup) (5,000)
Less: Deduction u/s 80GGA (1,00,000)
Less: Deduction u/s 80G
Charitable Institution (30,000)
Working Note:
AGTI = GTI – Deductions u/s 80C to 80U (Except 80G)
= 13,00,000 – 30,000 – 15,000 – 30,000 – 1,00,000 – 5,000
= `11,20,000
Qualifying Amount = 10% of AGTI or Donation given whichever is less
= `1,12,000 or 60,000
= 60,000
Deduction From Gross Total Income 259

50% of qualifying amount = `30,000


Total Income 10,90,000
Computation of Tax Liability
Tax on `10,90,000 at slab rate 1,39,500
Add: HEC @ 4% 5,580
Tax Liability 1,45,080

Question 11: Write a note on deduction in case of donation to the political parties or an Electoral
trust.
Answer: Deduction in respect of contributions given by companies to political parties Section
80GGB
In computing the total income of an assessee, being an Indian company, there shall be deducted any sum
contributed by it, in the previous year to any political party or an electoral trust.
No deduction shall be allowed under this section in respect of any sum contributed by way of cash.
E.g. ABC Ltd. an Indian company has given donation of `10,00,000 by cheque to Bharatiya Janata Party, in
this case deduction allowed shall be `10,00,000
E.g. ABC Ltd. an Indian company has given donation of `9,000 in cash to Bharatiya Janata Party, in this
case deduction shall not be allowed.
Deduction in respect of contributions given by any person to political parties Section 80GGC
In computing the total income of an assessee, there shall be deducted any amount of contribution made by
him, in the previous year, to a political party or an electoral trust.
No deduction shall be allowed under this section in respect of any sum contributed by way of cash.
E.g. Mr. X has given donation of `10,00,000 by cheque to Bharatiya Janata Party, in this case deduction
allowed shall be `10,00,000
E.g. Mr. X has given donation of `9,000 in cash to Bharatiya Janata Party, in this case deduction shall not be
allowed.
MAY – 2012 (4 Marks)
Explain how contributions to political parties are deductible in the hands of corporate and non-corporate
assesses under the Income tax law.
Answer: refer answer given above

Question 12: Write a note on deduction in case of employment by any Person.


Answer: Deduction in case of new employment Section 80JJAA
1. Deduction is allowed to all assessee whose accounts are required to be audited.
2. Deduction shall be allowed equal to 30% of additional employee cost incurred.
3. Deduction is allowed for 3 assessment years including the assessment year in which such employment
is provided.
4. It should be a new business.
5. “Additional employee cost” means total emoluments paid or payable to additional employees employed
during the previous year.
Provided that in the case of an existing business, the additional employee cost shall be nil, if there is no
increase in the number of employees from the total number of employees employed as on the last day of the
preceding year.
6. “Additional employee” means an employee who has been employed during the previous year and whose
employment has the effect of increasing the total number of employees employed by the employer as on
the last day of the preceding year, but does not include,—
(a) an employee whose total emoluments are more than twenty-five thousand rupees per month; or
(b) an employee employed for a period of less than two hundred and forty days during the previous year.

In the case of an assessee who is engaged in the business of manufacturing of apparel or footwear or
leather products, 240 days shall be taken as 150 days.
Deduction From Gross Total Income 260

Where an employee is employed during the previous year for a period of less than two hundred and forty
days or one hundred and fifty days, as the case may be, but is employed for a period of two hundred and
forty days or one hundred and fifty days, as the case may be, in the immediately succeeding year, he shall
be deemed to have been employed in the succeeding year and the provisions of this section shall apply
accordingly.

NOV-2016 (4 Marks)
Mr. Satya is a manufacturer of household goods in a factory located in Navi Mumbai and commenced his
business on 1st April 2018 and he employed 120 new work men during the previous year 2018-19 which
included:
(a) 20 employee whose total emoluments paid @ `30,000 p.m. per employee;
(b) 40 worker employed on 01st April,2018
(c) 35 worker employed on 1st May,2018
(d) 25 worker employed on 5th October,2018
Compute the Deduction under Section 80JJAA, if available to Mr. Satya for Assessment year 2019-20, if
wages are paid to each worker @ `3,000 per month. His profit from the manufacture of goods for
Assessment year 2019-20 is `9.50 lakhs.
The Assessee is liable to Audit his accounts.

Solution:
Mr. Satya is eligible for deduction under section 80JJAA since he is liable to Audit of his accounts in the
previous year 2018-19.
Calculation of Additional employee cost = (`3,000 x 12 x 40) + (`3,000 x 11 x 35) = `25,95,000
Deduction u/s 80JJAA = 30% of `25,95,000 = `7,78,500

Calculation of additional employee


Particulars No. of Employee
Total No. of workmen employed 120
Less: Employee Emoluments paid @ 30,000 p.m. (20)
Less: Employee Employed for less than 240 days (employed on 05.10.2018) (25)
Total No. of Additional Employee ` 75

Question 13. Write a note on deduction in case of royalty income from certain books.
Answer: Deduction in respect of royalty income, etc., of authors of certain books other than text books
Section 80QQB
1. Deduction is allowed only to a resident individual who is an author.
2. He should have income through his copyright in a book which is a work of literary, artistic or
scientific nature but such should not be text-books for schools/colleges etc. and also it should not be
any help book or guide etc. or any newspaper or magazine etc.
3. Deduction allowed shall be equal to the amount of royalty income or `3,00,000 whichever is less.
4. Royalty received by the author in excess of 15% of the value of such books sold during the previous
year shall be ignored.
e.g. Mr. X is an author of a book of literary nature and print price is `200 and total copies sold are 3000
and he claims that he is getting royalty @ 50% of print price, in this case deduction allowed shall be
200 x 15% x 3000 = 90,000
5. In respect of any income earned from any source outside India, so much of the income shall be taken into
account for the purpose of this section as is brought into India by the assessee in convertible foreign
exchange within a period of six months from the end of the previous year in which such income is earned
or within such further period as the competent authority may allow in this behalf.
Illustration 17: Mrs. X is author of one book of scientific nature and its print price is `500 and total copies
sold are 2000 and she has received royalty @ 50%.
Deduction From Gross Total Income 261

She has taken a loan from State Bank in 2010 for pursuing bachelor’s degree in Engineering and she has
given repayment of principal amount `80,000 and interest `20,000 to State Bank. (payment of interest was
given for the first time in financial year 2016-17)
She has paid tuition fee of her son for whole time education ` 3,000 in India
Compute Income Tax liability A.Y. 2019-20.
Solution: `
Income under the head Other Sources
500 x 50% x 2000 5,00,000.00
Gross Total Income 5,00,000.00
Less: Deduction u/s 80C
Tuition Fee (3,000.00)
Less: Deduction u/s 80E
Payment of Interest (20,000.00)
Less: Deduction u/s 80QQB
500 x 15% x 2000 (1,50,000.00)
Total Income 3,27,000.00
Computation of tax liability
Tax on `3,27,000 at slab rate 3,850.00
Less: Rebate u/s 87A (2,500.00)
Tax before health & education cess 1,350.00
Add: HEC @ 4% 54.00
Tax Liability 1,404.00
Rounded off u/s 288B 1,400.00
NOV – 2001 (3 Marks)
Mr. X, a writer and a professional furnishes the following particulars for the previous year ended
31.03.2019: `
(a) Royalty on books (eligible for deduction u/s 80QQB) 42,000
(b) Expenditure on books 8,000
(c) Income from profession 3,80,000
(d) Deposited in public provident fund (15.03.2019) 70,000
You are required to compute
(i) Taxable income,
(ii) Tax payable for assessment year 2019-20.
Answer:
Computation of total income and tax payable by Mr. X `
Income from business/profession
Income from profession 3,80,000
Income under the head Business/Profession 3,80,000
Income under the head Other Sources
Royalty on books 42,000
Less: Expenses (8,000)
Income under the head Other Sources 34,000
Gross Total Income 4,14,000
Less: Deduction u/s 80C (70,000)
Less: Deduction u/s 80QQB (34,000)
Total Income 3,10,000

Computation of Tax Payable


Tax on `3,10,000 at slab rate 3,000
Less: Rebate u/s 87A (2,500)
Tax before health & education cess 500
Add: HEC @ 4% 20
Tax Payable 520
Deduction From Gross Total Income 262

Question 14: Write a note on deduction in case of Royalty on Patents.


Answer: Deduction in respect of royalty on patents Section 80RRB
1. Deduction is allowed only to resident individual.
2. His gross total income should include royalty in respect of a patent.
3. Deduction allowed shall be equal to the amount of royalty or `3,00,000 whichever is less.
4. In respect of any income earned from any source outside India, so much of the income, shall be taken
into account for the purpose of this section as is brought into India by the assessee in convertible foreign
exchange within a period of six months from the end of the previous year in which such income is earned
or within such further period as the competent authority may allow in this behalf.

Question 15: Write a note on deduction in respect of interest on deposits in savings account.
Answer: Deduction in respect of interest on deposits in savings account Section 80TTA
1. Deduction is allowed only to an individual or HUF. (Other than those covered in 80TTB)
2. Deduction is allowed is the assessee has interest income on saving bank accounts with any bank/ Post
Office.
3. No deduction is allowed from interest on time deposit/ fixed deposit.
4. Deduction is allowed to the extent of `10,000.
E.g. Mr. X has interest income `8,000 from savings bank account with State Bank and interest income of
`13,000 from fixed deposit with State Bank, deduction allowed under section 80TTA shall be `8,000.

As per section 10(15), Interest on Post Office Savings Bank Account to the extent of `3,500 per year shall
be exempt from income tax and in the case of joint account, exemption shall be allowed upto `7,000 per
year.

Example: Mr. X has Income under the head salary `7,00,000 and interest on post office savings bank
account `7,000 and interest on savings bank account with State Bank `9,000, in this case tax liability of Mr.
X shall be
`
Income under the head Salary 7,00,000
Income under the head Other sources
Interest on Post office Saving Bank Account 7,000
Less: Exemption u/s 10(15) (3,500) 3,500
Interest on Saving Bank Account with SBI 9,000
Income under the head other sources 12,500

Gross Total Income 7,12,500


Less: Deduction u/s 80TTA (10,000)
Total Income 7,02,500
Computation of Tax Liability
Tax on ` 7,02,500 at slab rate 53,000
Add: HEC @ 4% 2,120
Tax Liability 55,120

Question 16: Write a note on deduction in respect of interest on deposits in savings account.
Answer: Deduction in respect of interest on deposits in savings account Section 80TTB
Deduction shall be allowed only to a senior citizen with regard to interest income from banks/cooperative
bank/ cooperative society/post office and further it may be in connection with time deposits/saving bank
account or any other deposits.
Deduction shall be allowed upto such income but maximum ` 50,000.
(Deduction 80TTA not allowed)
Deduction From Gross Total Income 263

Illustration 18: Mr. X, aged 62 years, earned professional income (computed) of `5,50,000 during the year
ended 31.03.2019. He has earned interest of `14,500 on the saving bank account with State Bank of India
during the year. Compute the total income of Mr. X for the assessment year 2019-20 from the following
particulars:
(i) Life insurance premium paid to Birla Sun life Insurance in cash amounting to `25,000 for insurance of
life of his dependent parents. The insurance policy was taken on 15.07.2018 and the sum assured on life of
his dependent parents is ` 1,25,000.
(ii) Life insurance premium of ` 25,000 paid for the insurance of life of his major son who is not dependent
on him. The sum assured on life of his son is `1,75,000 and the life insurance policy was taken on
18.04.2011.
(iii) Life insurance premium paid by cheque of ` 22,500 for insurance of his life. The insurance policy was
taken on 08.09.2018 and the sum assured is ` 2,00,000.
(iv) Premium of ` 16,000 paid by cheque for health insurance of self and his wife (`8,000 for self and
`8,000 for spouse).
(v) `1,500 paid in cash for his health check-up and ` 4,500 paid in cheque for health checkup for his parents.
(vi) Paid interest of ` 6,500 on loan taken from bank for MBA course pursued by his daughter.
(vii) A sum of ` 15,000 donated in cash to an institution approved for purpose of section 80G for promoting
family planning.
(viii) Contribution ` 10,500 made in cheque to an electoral trust.
Solution: Computation of total income of Mr. X for the Assessment Year 2019-20
Particulars `
Professional Income (computed) 5,50,000
Interest on saving bank deposit 14,500
Gross Total Income 5,64,500
Less: Deduction under Chapter VIA
Under section 80C
Life insurance premium paid for life insurance of:
- major son (25,000)
- self ` 22,500 restricted to 10% of ` 2,00,000 (20,000)
Under section 80D
Premium paid for health insurance of self and wife by cheque (16,000)
Payment made for health check-up: - Self (1,500)
Parents ` 4,500 (but maximum amount PHC is `5,000) (3,500)
Under section 80E
For payment of interest on loan taken from bank for MBA course of his daughter (6,500)
Under section 80GGC
Contribution to electoral trust (10,500)
Under section 80TTB
Interest on savings bank account (14,500)
Total Income 4,67,000
Deduction From Gross Total Income 264

PRACTICE PROBLEMS
TOTAL PROBLEMS 17
Problem 1.
Mr. X has taken a loan of `10,00,000 from S.B.I @ 10 % p.a. on 01.07.2014 for construction of one
residential house which was completed on 01.07.2016. It was let out @ ` 80,000 p.m. w.e.f 01.04.2018 and
Mr. X has paid Municipal tax of `20,000 though the amount due is `30,000.
He has repaid Principal amount of `70,000 on 01.07.2018.
He has Agricultural income of ` 3,00,000 and unadjusted loss of house property of P.Y. 2009-10 `10,000
and P.Y. 2010-11 `21,000.
He has invested `10,000 in NSC and `5,000 in Public Provident Fund and `5,000 in Post Office 5 Year
Time Deposit.
Compute his Income Tax Liability for the A.Y. 2019-20.
Answer: Tax Liability: `34,790

Problem 2.
Mr. X has taken a loan of `12,00,000 @ 10 % p.a. on 01.07.2014 and the house completed on 01.05.2018. It
was let out @ ` 30,000 p.m. w.e.f 01.08.2018 and the loan was repaid in annual installment of `40,000
starting from 01.01.2016.
Mr. X has STCG 111A `10,00,000.
He has paid premium of life policy `40,000 on 12.12.2018 and sum assured is `1,00,000.
He has paid premium of Jeevan Suraksha Policy `20,000.
Compute his Total Income and Tax Liability for the A.Y. 2019-20.
Answer: Total Income: `9,74,000; Tax Liability: `1,12,940

Problem 3.
Mr. X has income from business `410000 and he has received income of `3,80,000 from subletting also.
He has paid premium of medi-claim policy of `11,000. It was paid by cheque and the policy was taken on
02.01.2019 in the name of his father.
Compute his total income and tax liability for assessment year 2019-20.
Answer: Total Income: `7,79,000; Tax Liability: `71,030

Problem 4.
Mr. X has income from business `7,00,000.
Mr. X has incurred `65,000 on the treatment of his dependent brother who is suffering from a disease
notified under Rule 11DD and he has received claim under medi-claim policy `35,000.
Compute his income and tax liability for assessment year 2019-20.
Answer: Total Income: `6,95,000; Tax Liability: `53,560

(b) Presume assessee incurred `65,000 on the treatment of his independent brother.
Answer: Total Income: `7,00,000; Tax Liability: `54,600

Problem 5.
Mrs. X has let out one residential house property @ `1,00,000 p.m. and she has paid municipal tax of
`1,00,000.
She has taken a Medi-claim policy on 17.07.2018 in the name of Mr. X and paid premium of `18,000 by
cheque
She has also taken a Medi-claim policy on 15.10.2018 in the name of her Father in law who is aged 66 years
Deduction From Gross Total Income 265

and paid premium of `16,000 by cheque.


She has incurred `21,000 on the treatment of her brother who is dependent on her and suffering from severe
disability.
She has purchased N.S.C. in P.Y. 2016-17 and there is accrued interest of `30,000 and also there is accrued
interest of PPF `10,000
She has taken Jeevan Suraksha Policy in the name of Mr. X and paid premium of `19,000.
She has taken a loan in 2012-13 from SBI for the education of his son who is studying in B.com (Hons) in
SRCC and she had paid principal amount of `60,000 and interest `10,000 in P.Y.2018-19.
She has Agricultural Income `1,00,000
Compute her Income Tax Liability for the A.Y.2019-20.
Answer: Tax Liability: `52,940

Problem 6.
Mr. X has incomes asunder: `
1. Rent from letting out one house property 3,00,000
2. Long term capital gains 2,00,000
He has donated `5,000 by cheque to MCD which is notified under section 80G and has donated `2,000 by
cheque to National Children’s Fund and `2,000 by cheque to the Government for promotion of family
planning norms.
He has invested `6,000 in NSC. He is aged about 67 years.
Compute his total income and tax liability for the assessment year 2019-20.
Answer: Total Income: `3,97,500; Tax Liability: `20,280

Problem 7.
Mr. X is a Practicing Chartered Accountant and he started his practice from 01.04.2018 and he has income
from profession `8,00,000.
He has LTCG of `3,00,000, STCG 111A of `1,00,000, casual income `2,00,000.
Investment and donations are as given below:-
 NSC `10,000.
 Medi-claim premium (by cheque) of `15,000.
 Prime Minister’s National Relief Fund `10,000. (Paid by cheque)
 Rajiv Gandhi Foundation `8,000. (Paid by cheque)
 Donation to Birla Temple (Notified u/s 80G) `1,60,000. (Paid by cheque)
 Charitable institution (Notified u/s 80G) `40,000. (Paid by cheque)
 Social organization (Notified u/s 80G) `20,000. (Paid by cheque)
 Municipal Corporation of Delhi (notified under section 80G) `10,000. (Paid by cheque)
Compute income tax liability for A.Y. 2019-20.
Answer: Tax Liability: `1,97,550

(b) Presume in the above question the assessee has given donation to the Government also for family
planning is ` 20,000.
Answer: Tax Liability: `1,95,470

(c) Presume in the above question the assessee has given donation to the Government also for family
planning is ` 3,00,000.
Answer: Tax Liability: `1,87,410

Problem 8.
Mr. X has short term capital gain of `6 lakhs and he has donated `20,000 by cheque to a charitable
institution which is notified under section 80G and he has spent `25,000 on the treatment of his handicapped
dependant brother.
Compute his total income and tax liability for the assessment year 2019-20.
Deduction From Gross Total Income 266

Answer: Total Income: `5,15,000; Tax Liability `16,120


Problem 9.
Mr. Ram Kumar has incomes asunder:
1. He has received dividends from an Indian company of `11,000.
2. He has income from Business/Profession `1,22,000.
3. He has long term capital gains of `2,00,000.
4. He has donated `10,000 by cheque to MCD for family planning and has donated `3,000 by cheque to a
charitable institution notified under section 80G.
Compute his total income and tax liability for the assessment year 2019-20.
Answer: Total Income: `3,10,900; Tax Liability: `10,070
Problem 10.
Mr. X has incomes asunder:
1. Short term capital gains on sale of a capital asset `5,00,000.
2. Mr. X has donated `7,000 by cheque to the Prime Minister’s National Relief Fund and `20,000 by
cheque to Birla temple which is notified under section 80G.
Compute his total income and tax liability for the assessment year 2019-20.
Answer: Total Income: `4,83,000; Tax Liability: `12,120

Problem 11.
Mr. X has incomes asunder: `
1. Income from Business/Profession 1,00,000
2. He has long term capital gains 2,50,000
3. He has income from other sources 1,10,000
4. He has donated `10,000 by cheque to National Defence Fund
5. He has donated ` 5,000 by cheque to charitable institution notified under section 80G.
Compute his total income and tax liability for assessment year 2019-20.
Answer: Total Income: `4,47,500; Tax Liability: `41,080

Problem 12.
Mr. X has incomes asunder:
Long Term Capital Gains: `1,00,000
Short Term Capital Gains: `2,55,000
Casual income: `10,000
Donations given to charitable institutions notified under section 80G `45,000 paid by cheque and donation
to MCD for family planning `3,000 paid by cheque.
Compute his total income and tax liability for the assessment year 2019-20.
Answer: Total Income: `3,50,250; Tax Liability: `21,890
Problem 13.
Mr. X is engaged in the business of manufacturing chemicals and has income under the head
business/profession of `5,00,000 and has paid rent of `10,000 p.m. for taking a house on rent because he did
not have any house in his name or in the name of his spouse or minor child or the Hindu Undivided Family
of which he is a member. He has invested `75,000 in NSC which were taken in the name of his spouse.
Compute his total income and tax liability for assessment year 2019-20.
Answer: Total Income: `3,65,000; Tax Liability: `5,980
Problem 14.
During the previous year 2018-19, Mr. X has income under the head house property `4,00,000. He has
donated `12,000 by cheque to a notified institution for the purpose of scientific research.
Compute his total income and tax liability for the assessment year 2019-20.
Answer: Total Income: `3,88,000; Tax Liability: `7,180

Problem 15.
Deduction From Gross Total Income 267

For the assessment year 2019-20, Mr. X submits the following information:
Income from business 9,800
Property income House I House II
` `
Fair Rent 75,000 82,000
Rent Received/Receivable 78,000 85,000
Municipal Valuation 76,000 75,000
Municipal Taxes (due but outstanding) 13,000 14,000
Repairs 3,500 47,000
Insurance 2,000 3,000
Land Revenue (Paid) 2,500 4,000
Ground Rent (due but outstanding) 1,600 6,000
Interest on capital borrowed by mortgaging house I
(Funds are used for construction of house II) 14,000 ———
Nature of Occupation Let out for Residence Let out for Business
Date of completion of construction 30.04.2014 07.04.2016
Assume that standard rent is less than rent actually received.
Mr. X has brought forward losses of house property asunder `
Assessment year 2010-11 1,00,000
Assessment year 2014-15 10,100
Other incomes of Mr. X
1. Vacant site lease rent 4,12,000
2. Rent from house property at Chennai 3,000 p.m.
This house was constructed by taking a loan of ` 2,00,000 @ 10% from State Bank of India but it was
repaid on 01.10.2018 by taking a loan of `2,00,000 from Punjab National Bank on 01.10.2018 @ 9.5%
p.a.
3. He has also received `3,000 during the year from Calcutta University for acting as an examiner and
`1,500 from Delhi University also, for acting as an examiner.
4. He has received from Life Insurance Corporation of India `1,20,000 being the maturity amount of life
insurance policy.
5. He has received a reward of `5,000 from Central Government and the reward is notified under section
10(17A).
He has invested `1,000 in the notified bonds of NABARD eligible for deduction under section 80C, but
the investment is out of past savings and has invested `1,000 in master equity plan of Unit Trust of India.
He is eligible for deductions under section 80D to 80U amounting to `1,005.
Compute his income and also tax liability for assessment year 2019-20.
Answer = Total Income: `3,71,000; Tax Liability: `6,290

Problem 16.
Mrs. X has income under the head Business/Profession `6,00,000.
She has spent `30,000 on the treatment and education of her sister who is a disabled person as per section
80U and she is dependent on Mrs. X.
She has paid premium of medi-claim policy `18,000 by cheque and policy is taken on 15.11.2018 in the
name of her father who is not dependent on Mrs. X.
She is holder of one patent right and has received royalty of `3,20,000
She has paid rent of `30,000 p.m. and she has complied with all the conditions of section 80GG
You are required to compute her Income tax liability for the A.Y. 2019-20.
Answer: Tax Liability: `11,280

Problem 17.
Mr. X has received royalty of `8,00,000 in connection with a patent right registered in his name.
Other informations are as given below:
1. He has donated ` 30,000 to a political party by cheque.
Deduction From Gross Total Income 268

2. He donated `10,000 by cheque to Delhi University notified under section 80G


3. He donated `10,000 by cheque to Government for the purpose of promoting family planning.
4. He paid premium of medi-claim policy `6,000 by cheque in the name of his major married independent
son.
5. LIC premium paid `25,000 on 15.01.2019 (Policy value `1,00,000)
6. Repayment of housing loan to Indian Bank `50,000
7. Payment made to LIC pension fund notified under section 80CCC `20,000
Compute income tax liability for A.Y 2019-20.
Answer: Tax Liability: `6,240
Deduction From Gross Total Income 269

SOLUTIONS
TO
PRACTICE PROBLEMS
Solution 1:
Computation of income under the head House Property
Gross Annual Value 9,60,000.00
Less: Municipal Tax (20,000.00)
Net Annual Value 9,40,000.00
Less: 30% of NAV u/s 24(a) (2,82,000.00)
Less: Interest on capital borrowed u/s 24(b) (1,29,750.00)
Working Note:
Prior period interest
From 01.07.2014 to 31.03.2016
= (10,00,000 x 10% x 9/12) + (10,00,000 x 10% x 1)
= `75,000 + `1,00,000 = `1,75,000
Installment = `1,75,000/5 = `35,000
Current period interest
From 01.04.2018 to 31.03.2019
= (10,00,000 x 10% x 3/12) + (9,30,000 x 10% x 9/12)
= `25,000 + `69,750 = `94,750
Total interest on capital borrowed
= `35,000 + ` 94,750 = `1,29,750
Income under the head House Property 5,28,250.00
Less: Brought forward Loss of P.Y.2010-11 (21,000.00)
Income under the head House Property 5,07,250.00
Gross Total Income 5,07,250.00
Less: Deduction u/s 80C
Repayment of Housing loan (70,000.00)
NSC (10,000.00)
PPF (5,000.00)
Post Office Time Deposit (5,000.00)
Total Income 4,17,250.00
Agricultural Income 3,00,000.00
Computation of Tax Liability
Step 1 Tax on (4,17,250 + 3,00,000) at slab rates 55,950.00
Step 2 Tax on (2,50,000 + 3,00,000) at slab rates (22,500.00)
Deduct Tax at Step 2 from Step 1 33,450.00
Add: HEC @ 4% 1,338.00
Tax Liability 34,788.00
Deduction From Gross Total Income 270

Rounded off u/s 288B 34,790.00

Solution 2: `
Computation of income under the head House Property
Gross Annual Value 2,40,000
Less: Municipal Tax NIL
Net Annual Value 2,40,000
Less: 30% of NAV u/s 24(a) (72,000)
Less: Interest on capital borrowed u/s 24(b) (1,94,000)
Working Note:
Prior period interest
From 01.07.2014 to 31.03.2018
= (12,00,000 x 10% x 6/12) + (12,00,000 x 10% x 1)+ (11,60,000 x 10%
x 1)+ (11,20,000 x 10% x 1)+ (10,80,000 x 10% x 3/12)
= `60,000 + `1,20,000+ `1,16,000+ `1,12,000+ `27,000 = `4,35,000
Installment = `4,35,000/5 = `87,000
Current period interest
From 01.04.2018 to 31.03.2019
= (10,80,000 x 10% x 9/12) + (10,40,000 x 10% x 3/12)
= `81,000 + `26,000 = `1,07,000
Total interest on capital borrowed
= `87,000 + ` 1,07,000 = `1,94,000
Loss under the head House Property (26,000)
Income under the head Capital Gains (STCG u/s 111A) 10,00,000
Gross Total Income 9,74,000
Less: Deduction u/s 80C to 80U NIL
Total Income 9,74,000
Computation of Tax Liability
Tax on STCG ` 7,24,000 (9,74,000-2,50,000) u/s 111A @ 15 % 1,08,600
Add: HEC @ 4% 4,344
Tax Liability 1,12,944
Rounded off u/s 288B 1,12,940
Note: Deduction under section 80C to 80U is not allowed from STCG u/s 111A.

Solution 3: `
Income under the head Business/Profession 4,10,000
Income under the head Other Sources
Income from subletting 3,80,000
Income under the head Other Sources 3,80,000
Gross Total Income 7,90,000
Less: Deduction u/s 80D (11,000)
Total Income 7,79,000
Computation of Tax Liability
Tax on `7,79,000 at slab rate 68,300.00
Add: HEC @ 4% 2,732.00
Tax Liability 71,032.00
Rounded off u/s 288B 71,030.00

Solution 4: `
Income under the head Business/Profession 7,00,000
Gross Total Income 7,00,000
Less: Deduction u/s 80DDB (40,000 – 35,000) (5,000)
Total Income 6,95,000
Deduction From Gross Total Income 271

Computation of Tax Liability


Tax on `6,95,000 at slab rate 51,500
Add: HEC @ 4% 2,060
Tax Liability 53,560

Solution 4(b): `
Income under the head Business/Profession 7,00,000
Gross Total Income 7,00,000
Less: Deduction u/s 80DDB Nil
Total Income 7,00,000
Computation of Tax Liability
Tax on `7,00,000 at slab rate 52,500
Add: HEC @ 4% 2,100
Tax Liability 54,600
Note: Deduction under section 80DDB is not allowed in case assessee incurred expenditure on treatment of
his independent brother.

Solution 5: `
Computation of income under the head House Property
Gross Annual Value 12,00,000
Less: Municipal Tax (1,00,000)
Net Annual Value 11,00,000
Less: 30% of NAV u/s 24(a) (3,30,000)
Less: Interest on capital borrowed u/s 24(b) NIL
Income under the head House Property 7,70,000
Income under the head other sources 30,000
Gross Total Income 8,00,000
Less: Deductions
Deduction u/s 80C for NSC (30,000)
Deduction u/s 80CCC for Jeevan Suraksha Policy NIL
Deduction u/s 80D for Mediclaim Policy (18,000)
Deduction u/s 80DD for Severe Disability (1,25,000)
Deduction u/s 80E for Education Loan Interest (10,000)
Total Income 6,17,000
Agricultural Income 1,00,000
Computation of Tax Liability
Step 1 Tax on (6,17,000 + 1,00,000) at slab rates 55,900
Step 2 Tax on (2,50,000 + 1,00,000) at slab rates (5,000)
Deduct Tax at Step 2 from Step 1 50,900
Tax before HEC 50,900
Add: HEC @ 4% 2,036
Tax Liability 52,936
Rounded off u/s 288B 52,940

Solution 6: `
Computation of income under the head House Property
Gross Annual Value 3,00,000
Less: Municipal taxes Nil
Net Annual Value 3,00,000
Less: 30% of NAV u/s 24(a) (90,000)
Less: Interest on capital borrowed u/s 24(b) Nil
Income under the head House Property 2,10,000
Deduction From Gross Total Income 272

Computation of Total Income


Income under the head House Property 2,10,000
Income under the head Capital Gains (LTCG) 2,00,000
Gross Total Income 4,10,000
Less: Deduction u/s 80C {NSC} (6,000)
Less: Deduction u/s 80G
(i) National Children Fund {100% of `2,000} (2,000)
(ii) Other Donations u/s 80G (4,500)
Working Note: `
MCD 5,000
Family planning 2,000
7,000
AGTI = GTI – LTCG – STCG u/s 111A – 80C to 80U (Except 80G)
= 4,10,000 – 2,00,000 – 6,000 = 2,04,000
Qualifying amount = 10% of AGTI or donation whichever is less
= 20,400 or 7,000
= 7,000
Deduction = 50% of `5,000 + `2,000
= `4,500
Total Income 3,97,500
Computation of Tax Liability
Tax on LTCG `97,500 (2,00,000 – 1,02,500) @ 20% u/s 112 19,500
Tax on `1,97,500 at slab rate Nil
Tax before health & education cess 19,500
Add: HEC @ 4% 780
Tax Liability 20,280

Solution 7 (a): `
Income under the head business / profession 8,00,000.00
Income under the head Other Sources
Casual income 2,00,000.00
Income under the head other sources 2,00,000.00
Income under the head Capital Gain
Long term capital gain 3,00,000.00
Short term capital gain 111A 1,00,000.00
Income under the head capital gain 4,00,000.00
Gross Total Income 14,00,000.00
Less: Deductions
Deduction u/s 80C for NSC (10,000.00)
Deduction u/s 80D for Mediclaim policy (15,000.00)
Deduction u/s 80G
Prime Minister National Relief Fund (10,000.00)
Rajiv Gandhi Foundation (50% of `8,000) (4,000.00)
Donations (48,750.00)
Working Note:
AGTI = GTI – LTCG – STCG u/s 111A – Deduction u/s 80C to 80U
(except 80G)
= 14,00,000 – 3,00,000 – 1,00,000 – 25,000
= 9,75,000
Qualifying amount = 10% of AGTI or donation whichever is less
= 97,500 or 2,30,000 whichever is less
= 97,500
Deduction From Gross Total Income 273

50% of qualifying amount = 48,750


Total Income 13,12,250.00
Computation of Tax Liability
Tax on casual income `2,00,000 @ 30% u/s 115BB 60,000.00
Tax on LTCG `3,00,000 @ 20% 60,000.00
Tax on STCG 111A `1,00,000 @ 15% 15,000.00
Tax on normal income `7,12,250 at slab rate 54,950.00
Tax before health & education cess 1,89,950.00
Add: HEC @ 4% 7,598.00
Tax Liability 1,97,548.00
Rounded off u/s 288B 1,97,550.00

Solution 7(b): `
Gross Total Income 14,00,000.00
Less: Deductions
Deduction u/s 80C for NSC (10,000.00)
Deduction u/s 80D for Mediclaim policy (15,000.00)
Deduction u/s 80G
Prime minister national relief fund (10,000.00)
Rajiv Gandhi Foundation (4,000.00)
Donations (58,750.00)
Working Note:
AGTI = GTI – LTCG – STCG u/s 111A – Deduction u/s 80C to 80U
(except 80G)
= 14,00,000 – 3,00,000 – 1,00,000 – 25,000
= 9,75,000
Qualifying amount = 10% of AGTI or donation whichever is less
= 97,500 or 2,50,000 whichever is less
= 97,500
50% of qualifying amount = 77,500 x 50% + 20,000
= 58,750
Total Income 13,02,250.00
Computation of Tax Liability
Tax on casual income `2,00,000 @ 30% u/s 115BB 60,000.00
Tax on LTCG `3,00,000 @ 20% 60,000.00
Tax on STCG 111A `1,00,000 @ 15% 15,000.00
Tax on normal income `7,02,250 at slab rate 52,950.00
Tax before health & education cess 1,87,950.00
Add: HEC @ 4% 7,518.00
Tax Liability 1,95,468.00
Rounded off u/s 288B 1,95,470.00
Solution 7(c): `
Gross Total Income 14,00,000.00
Less: Deductions
Deduction u/s 80C for NSC (10,000.00)
Deduction u/s 80D for Mediclaim policy (15,000.00)
Deduction u/s 80G
Prime minister national relief fund (10,000.00)
Rajiv Gandhi Foundation (4,000.00)
Donations (97,500.00)
Working Note:
AGTI = GTI – LTCG – STCG u/s 111A – Deduction u/s 80C to 80U
Deduction From Gross Total Income 274

(except 80G)
= 14,00,000 – 3,00,000 – 1,00,000 – 25,000
= 9,75,000
Qualifying amount = 10% of AGTI or donation whichever is less
= 97,500 or 5,30,000 whichever is less
= 97,500
100% deduction is allowed 97,500
Total Income 12,63,500.00
Computation of Tax Liability
Tax on casual income `2,00,000 @ 30% u/s 115BB 60,000.00
Tax on LTCG ` 3,00,000 @ 20% 60,000.00
Tax on STCG 111A `1,00,000 @ 15% 15,000.00
Tax on normal income `6,63,500 at slab rate 45,200.00
Tax before health & education cess 1,80,200.00
Add: HEC @ 4% 7,208.00
Tax Liability 1,87,408.00
Rounded off u/s 288B 1,87,410.00

Solution 8: `
Income under the head Capital Gains (STCG) 6,00,000
Gross Total Income 6,00,000
Less: Deduction u/s 80DD (75,000)
Less: Deduction u/s 80G (10,000)
Working Note:
AGTI = GTI – LTCG – STCG u/s 111A – 80C to 80U (Except 80G)
= 6,00,000 – 75,000 = 5,25,000
Qualifying amount = 10% of AGTI or donation, whichever is less
= 52,500 or 20,000
= 20,000
Deduction = 50% of `20,000
= `10,000
Total Income 5,15,000
Computation of Tax Liability
Tax on `5,15,000 at slab rate 15,500
Add: HEC @ 4% 620
Tax Liability 16,120

Solution 9: `
Income under the head Other Sources
Dividend from Indian company {exempt u/s 10(34)} Nil
Income under the head Other Sources Nil
Income under the head Business/Profession 1,22,000.00
Income under the head Capital Gains {LTCG} 2,00,000.00
Gross Total Income 3,22,000.00
Less: Deduction u/s 80G (11,100.00)
Working Note: `
MCD for family planning 10,000
Charitable institution 3,000
13,000
Adjusted GTI = GTI – LTCG – STCG u/s 111A – 80C to 80U (Except 80G)
= `3,22,000 – `2,00,000 = `1,22,000
Qualifying amount = 12,200 or 13,000 whichever is less
Deduction From Gross Total Income 275

= `12,200
Deduction = 50% of `2,200 + `10,000 = `11,100
Total Income 3,10,900.00
Computation of Tax Liability
Tax on Long term capital gain `60,900 (2,00,000 – 1,39,100) @ 20% u/s 112 12,180.00
Tax on `1,10,900 at slab rate Nil
Less: Rebate u/s 87A (2,500.00)
Tax before health & education cess 9,680.00
Add: HEC @ 4% 387.20
Tax Liability 10,067.20
Rounded off u/s 288B 10,070.00

Solution 10: `
Income under the head Capital Gains {STCG} 5,00,000
Gross Total Income 5,00,000
Less: Deduction u/s 80G
(i) Prime Minister’s National Relief Fund (7,000)
(ii) Other donations (10,000)
Working Note:
AGTI = GTI – LTCG – STCG u/s 111A – Deduction u/s 80C to 80U (except 80G)
= 5,00,000
Qualifying amount = 10% of AGTI or donation whichever is less
= 50,000 or 20,000 whichever is less
= 20,000
50 % of the qualifying amount (i.e. 10,000)
Total Income 4,83,000
Computation of Tax Liability
Tax on `4,83,000 at slab rate 11,650.00
Add: HEC @ 4% 466.00
Tax Liability 12,116.00
Rounded off u/s 288B 12,120.00

Solution 11: `
Income under the head Business/Profession
Income under the head Business/Profession 1,00,000.00
Income under the head Other Sources
Income under the head Other Sources 1,10,000.00
Income under the head Capital gains
Long term capital gains 2,50,000.00
Income under the head Capital Gains 2,50,000.00
Gross Total Income 4,60,000.00
Less: Deduction u/s 80G
(i) National Defence Fund (10,000.00)
(ii) Charitable institutions (2,500.00)
Working Note:
AGTI = GTI – LTCG – STCG u/s 111A – Deduction u/s 80C to 80U (except 80G)
= 4,60,000 – 2,50,000 = 2,10,000
Qualifying amount = 10% of AGTI or donation whichever is less
= 21,000 or 5,000
= 5,000
50% of the qualifying amount = 2,500
Total Income 4,47,500.00
Deduction From Gross Total Income 276

Computation of Tax Liability


Tax on long term capital gains `1,97,500 (`2,50,000 – `52,500) @ 20% u/s 112 39,500.00
Tax on normal income `1,97,500 at slab rate Nil
Tax before health & education cess 39,500.00
Add: HEC @ 4% 1,580.00
Tax Liability 41,080.00

Solution 12: `
Computation of income under the head Capital Gains
Short term capital gains 2,55,000.00
Long term capital gains 1,00,000.00
Income under the head Capital Gains 3,55,000.00
Computation of income under the head Other Sources
Casual income 10,000.00
Income under the head Other Sources 10,000.00
Gross Total Income 3,65,000.00
Less: Deduction u/s 80G (14,750.00)
Working Note: `
Charitable institutions 45,000
Family planning 3,000
48,000
AGTI = GTI – LTCG – STCG u/s 111A – Deduction u/s 80C to 80U (except 80G)
= 3,65,000 – 1,00,000 = 2,65,000
Qualifying amount = 10% of AGTI or donation whichever is less
= 26,500 or 48,000
= 26,500
Deduction = 50% of 23,500 + 3,000
= 11,750 + 3,000 = 14,750
Total Income 3,50,250.00
Computation of Tax Liability
Tax on LTCG `90,250 (`1,00,000 – 9,750) @ 20% u/s 112 18,050.00
Tax on casual income `10,000 @ 30% u/s 115BB 3,000.00
Tax on normal income `2,40,250 at slab rate Nil
Tax before health & education cess 21,050.00
Add: HEC @ 4% 842.00
Tax Liability 21,892.00
Rounded off u/s 288B 21,890.00

Solution 13. `
Income under the head Business/Profession 5,00,000
Gross Total Income 5,00,000
Less: Deduction u/s 80C (75,000)
Less: Deduction u/s 80GG (60,000)
Working Note:
Least of the following:
(i) `1,20,000 – 10% of `4,25,000 = `77,500
(ii) `60,000
(iii) 25% of `4,25,000 = `1,06,250
(AGTI = `5,00,000 – `75,000 = `4,25,000)
Total Income 3,65,000
Computation of Tax Liability
Tax on `3,65,000 at slab rate 5,750.00
Deduction From Gross Total Income 277

Add: HEC @ 4% 230.00


Tax Liability 5,980.00

Solution 14: `
Income under the head House Property 4,00,000
Gross Total Income 4,00,000
Less: Deductions u/s 80GGA (12,000)
Total Income 3,88,000
Computation of Tax Liability
Tax on `3,88,000 at slab rate 6,900
Add: HEC @ 4% 276
Tax Liability 7,176
Rounded off u/s 288B 7,180

Solution 15: ` `
Computation of income under the head House Property
HOUSE I
Gross Annual Value 78,000
Working Note: `
(a) Fair Rent 75,000
(b) Municipal valuation 76,000
(c) Higher of (a) or (b) 76,000
(d) Expected Rent 76,000
(e) Rent Received or Receivable 78,000
GAV = Higher of (d) or (e) 78,000
Less: Municipal taxes Nil
Net Annual Value 78,000
Less: 30% of NAV u/s 24(a) (23,400)
Less: Interest on capital borrowed u/s 24(b) Nil
Income under the head House Property 54,600
HOUSE II
Gross Annual Value 85,000
Working Note: `
(a) Fair Rent 82,000
(b) Municipal valuation 75,000
(c) Higher of (a) or (b) 82,000
(d) Expected Rent 82,000
(e) Rent Received or Receivable 85,000
GAV = Higher of (d) or (e) 85,000
Less: Municipal Taxes Nil
Net Annual Value 85,000
Less: 30% of NAV u/s 24(a) (25,500)
Less: Interest on capital borrowed u/s 24(b) (14,000)
Income under the head House Property 45,500
HOUSE AT CHENNAI
Gross Annual Value (3,000 x 12) 36,000
Less: Municipal taxes Nil
Net Annual Value 36,000
Less: 30% of NAV u/s 24(a) (10,800)
Less: Interest on capital borrowed u/s 24(b) (19,500)
Working Note:
Current period interest
Deduction From Gross Total Income 278

From 01.04.2018 to 30.09.2018


= 2,00,000 x 10% x 6/12 = `10,000
From 01.10.2018 to 31.03.2019
= 2,00,000 x 9.5% x 6/12 = ` 9,500
Total interest = 10,000 + 9,500 = 19,500
Income under the head House Property 5,700
Brought forward house property loss of assessment year 2014-15 10,100
Income under the head house property after adjusting losses 95,700
Income under the head Other Sources
Vacant site lease rent 4,12,000
Remunerations from Calcutta University 3,000
Remuneration from Delhi University 1,500
Income under the head Other Sources 4,16,500
Income under the head Business/Profession 9,800
Gross Total Income 5,22,000
Less: Deduction u/s 80C (1,50,000)
Investment in bonds of NABARD 1,000
Investment in master equity plan of UTI 1,000
Repayment of housing loan 2,00,000
(Whether deduction u/s 80C on repayment of the loan by
taking a fresh loan is allowed or not is not clear in the act)
(But maximum upto `1,50,000)
Less: Deductions u/s 80D to 80U (1,005)
Total Income (rounded off u/s 288A) 3,71,000
Computation of Tax Liability
Tax on `3,71,000 at slab rate 6,050.00
Add: HEC @ 4% 242.00
Tax Liability 6,292.00
Rounded off u/s 288B 6,290.00
Explanations
1. Payments received from LIC on maturity of LIC policy is exempt under section 10(10D)
2. Investment under section 80C is allowed even from past savings and out of incomes exempt from tax.
3. Any award/reward of Central Government or State Government notified under section 10(17A) shall be
exempt from income tax.

Solution 16: `
Computation of Total Income
Income under the head Business/Profession 6,00,000.00
Income under the head Other Sources (Patent right) 3,20,000.00
Gross Total Income 9,20,000.00
Less: Deduction u/s 80D (18,000.00)
Less: Deduction u/s 80DD (75,000.00)
Less: Deduction u/s 80RRB (3,00,000.00)
Less: Deduction u/s 80GG (60,000.00)
Working Note:
Least of the following:
1. `60,000
2. 25% x 5,27,000 = `1,31,750
3. `3,60,000 – `52,700 = `3,07,300
(AGTI = `9,20,000 – 18,000 – 75,000 – 3,00,000 = `5,27,000)
Total Income 4,67,000.00
Computation of Tax Liability
Deduction From Gross Total Income 279

Tax on `4,67,000 at slab rate 10,850.00


Add: HEC @ 4% 434.00
Tax Liability 11,284.00
Rounded off u/s 288B 11,280.00

Solution 17: `
Income under the head Other Sources
Royalty received in connection with a patent right 8,00,000.00
Gross Total Income 8,00,000.00
Less: Deduction u/s 80C
LIC premium (allowed 10% of sum assured) (10,000.00)
Repayment of housing loan to Indian Bank (50,000.00)
Less: Deduction u/s 80CCC
LIC Pension Fund (20,000.00)
Less: Deduction u/s 80D
Premium of medi-claim policy by cheque in the
name of his major married independent son. Nil
Less: Deduction u/s 80G
Donation to Delhi University (10,000.00)
Family planning (10,000.00)
Working Note:
Donation to Government for promoting family planning 10,000
AGTI = GTI – LTCG – STCG u/s 111A – Deduction u/s 80C to 80U (Except
section 80G)
= 8,00,000 – 10,000 – 50,000 – 20,000 – 30,000 – 3,00,000
= 3,90,000
Qualifying amount = 10% of AGTI or donation whichever is less
= 39,000 or 10,000
100% of qualifying amount = `10,000
Less: Deduction u/s 80GGC
Donation to a political party (30,000.00)
Less: Deduction u/s 80RRB (3,00,000.00)
Total Income 3,70,000.00
Computation of Tax Liability
Tax on `3,70,000 at slab rate 6,000.00
Add: HEC @ 4% 240.00
Tax Liability 6,240.00
Deduction From Gross Total Income 280

Question 1 [V. Imp.]: Explain exemption in case of Units established in Special Economic Zone.
Section 10AA
Answer:
Units established in Special Economic Zone
1. Exemption shall be allowed to all the assessees, may be individual, firm, company etc. provided the
assessee has its unit in Special Economic Zone.
2. Quantum of exemption:
Exemption shall be allowed to the units in the Special Economic Zone for a continuous period of 15 years in
the manner given below:
For first 5 Assessment Years 100% of export profits
For next 5 Assessment Years 50% of export profits
For next 5 Assessment Years 50% of export profit provided such profits have been credited
to the Special Economic Zone Re-investment Reserve
Account.
Export profits means Profits of Business x Export Turnover
Total Turnover
e.g. ABC Ltd. has one unit in SEZ and total turnover is `1000 lakhs and profits `400 lakhs and export
turnover `800 lakhs, in this case export profits shall be 400 / 1000 x 800 = 320 lakhs
The amount credited to the Special Economic Zone Reinvestment Reserve Account should be utilised for
acquiring a new plant and machinery within a period of 3 years. The period of 3 years shall be determined
from the end of the previous year in which the reserve was created e.g. If amount has been transferred in
reserve account in the previous year 2018-19, amount should be utilized for purchasing plant and machinery
upto 31.03.2022.
The particulars of the new plant and machinery is to be given to the Assessing Officer alongwith return of
income in the prescribed form.
3. If the amount credited to the Special Economic Zone Reinvestment Reserve Account is not utilised within
3 years, it will be taxable in the 4th year.
If the amount is misutilised within the period of 3 years, it will be taxable in the year in which it was
misutilised.
4. The assessee should furnish in the prescribed form a report of Chartered Accountant certifying that the
deduction has been correctly claimed.
MAY – 2016 (4 Marks)
Mr. Suresh has set up an undertaking in SEZ (Unit A) and another undertaking in DTA (Unit B) in the
financial year 2013-14.In the Previous year 2018-19, total turnover of the unit A is `180 lacs and total
turnover of Unit B is `120 lacs. Export Turnover of Unit A for the year is `150 lacs and Profit for the unit A
is `60 lacs.
Calculate the deduction available, if any, to Mr. Suresh under section 10AA of the Income-tax Act, 1961, for
the Assessment Year 2019-20, If the manufacturing started in Unit A in the Financial year 2013-14.
Solution:
As per Section 10AA,in case of a unit in SEZ, Deduction shall be allowed to the extent of 100% of Export
Profits in the first five years and 50% of export profits in the next five years. In the given case Unit was
setup in Financial year 2013-14 hence it is sixth year in Previous year 2018-19 and Deduction shall be
allowed to the extent of 50% of export profit and shall be as given below:

= Profit of the business of Unit in SEZ x Export Turnover of Unit in SEZ


Total Turnover of Unit in SEZ
= `60 lacs x 150 Lacs
180 Lacs
= `50lacs

Deduction allowed = 50% of `50 lacs


= `25lacs
Taxable amount of Unit A (`60 lacs – `25 lacs) = `35 lacs
Deduction From Gross Total Income 281

MAY – 2015 (4 Marks)


Rudra Ltd. has one unit at Special Economic Zone (SEZ) and other unit at Domestic Tariff Area (DTA), the
company provides the following details for the previous year 2018-19.
Particulars Rudra Ltd. Unit in DTA
(`) (`)
Total Sales 6,00,00,000 2,00,00,000
Export Sales 4,60,00,000 1,60,00,000
Net Profit 80,00,000 20,00,000
Calculate the eligible deduction under section 10AA of the Income-tax Act, 1961, for the Assessment Year
2019-20, in the following situations:
(i) If both the units were set up and start manufacturing from 22.05.2012.
(ii) If both the units were set up and start manufacturing from 14.05.2016.
Solution:
(i) 50% of the profit derived from export of articles or things or services is eligible for deduction under
section 10AA, F.Y.2018-19 falls in the next five year period commencing from the year of manufacture or
production of articles or things or provision of services by the Unit in SEZ. As per section 10AA, the profit
derived from export of articles or things or services shall be
= Profit of the business of Unit in SEZ x Export Turnover of Unit in SEZ
Total Turnover of Unit in SEZ
= 50% of ` 60 lakhs x 300 Lakhs
400 Lakhs
= 50% x `45 lakhs
= `22.5 lakhs
(ii) 100% of the profit derived from export of articles or things or services is eligible for deduction under
section 10AA, F.Y.2018-19 falls in the first five year period commencing from the year of manufacture or
production of articles or things or provision of services by the Unit in SEZ. As per section 10AA, the profit
derived from export of articles or things or services shall be
= Profit of the business of Unit in SEZ x Export Turnover of Unit in SEZ
Total Turnover of Unit in SEZ
= 100% of `60 lakhs x 300 Lakhs
400 Lakhs
= 100% x `45 lakhs
= `45 lakhs
NOV – 2013 (4 Marks)
Mr. X is running two industrial undertakings, one in a SEZ (Unit A) and another in a Domestic Tariff Area
(Unit B). The brief details for the year ended 31.03.2019 are as under:
Particulars Amount (` in lacs)
Unit A Unit B
Domestic turnover 10 100
Export turnover 120 Nil
Gross Profit 20 10
Less: Expenses and depreciation (07) (05)
Profits derived from the units 13 05
The brought forward business loss pertaining to assessment year 2016-17 for Unit B is `3.2 lacs. Briefly
compute the business income of the assessee.
Answer.
Computation of business income of Mr. X
Particulars ` (in lacs)
Total profit derived from Units A & B (` 13 lacs + ` 5 lacs) 18.0
13  120 (12.0)
Less: Exemption under section 10AA ( )
130
6.0
Deduction From Gross Total Income 282

Less: Set-off of brought forward business loss as per section 72 (3.2)


2.8
Note - 100% of the profit derived from export of articles or things or services is eligible for exemption
under section 10AA, assuming that F.Y.2018-19 falls within the first five year period commencing from the
year of manufacture or production of articles or things or provision of services by Unit A in SEZ.
MAY – 2011 (5 Marks)
ABC Ltd. is running two industrial undertakings, one in a SEZ (Unit S) and another in a normal area (Unit
N). The brief summarized details for the year ended 31.03.2019 are as under:
(` in lacs)
S N
Domestic turnover 10 100
Export turnover 120 Nil
Gross profit 20 10
Less: Expenses and depreciation (7) (6)
Profits derived from the unit 13 4
The brought forward business loss pertaining to Unit N is `2 lacs. Briefly compute the business income of
the assessee.
Answer. Computation of business income of ABC Ltd.
Particulars ` in lacs
Total profit derived from Units S & N (`13 lacs + `4 lacs) 17
13  120 (12)
Less: Exemption under section 10AA ( ) 5
130
Less: Brought forward business loss (2)
3
Note – 100% of the profit derived from export of articles or things or from services is eligible for exemption
under section 10AA, assuming that F.Y. 2018-19 falls within the first five year period commencing from the
year of manufacture or production of articles or things or provision of services by the Unit in SEZ.
MAY – 2011 (4 Marks)
Y Co. Ltd. Furnishes you the following information for the year ended 31.03.2019: `
Total turnover of Unit A located in Special Economic Zone 100 lakhs
Profit of the business of Unit A 30 lakhs
Export turnover of Unit A 50 lakhs
Total turnover of Unit B located in Domestic Tariff Area (DTA) 200 lakhs
Profit of the business of Unit B 20 lakhs
Compute deduction under section 10AA for the assessment year 2019-20.
Answer.
100% of the profit derived from export of articles or things or services is eligible for deduction under section
10AA, assuming that F.Y.2018-19 falls within the first five year period commencing from the year of
manufacture or production of articles or things or provision of services by the Unit in SEZ. As per section
10AA, the profit derived from export of articles or things or services shall be
= Profit of the business of Unit A x Export Turnover of Unit A
Total Turnover of Unit A
= ` 30 lakhs x 50
100
= `15 lakhs.
Agricultural Income 283

AGRICULTURAL INCOME
PARTICULARS SECTIONS
Definition of agricultural income 2(1A)
 Rent or revenue derived from agricultural land 2(1A)(a)
 Income derived from agricultural land by agricultural operations 2(1A)(b)
 Income of a farm building  2(1A)(c)
Exemption of agricultural income  10(1)
Income which is partially agricultural and partially from business Rule 7
Income from growing and manufacturing of rubber Rule 7A
Income from growing and manufacturing of coffee Rule 7B
Income from growing and manufacturing of tea Rule 8

Question 1. [V. Imp.] Explain meaning of agricultural income.


Answer: Meaning of Agricultural Income Section 2(1A)
The term Agricultural Income is defined in three parts under Income Tax Act under section 2(1A) (a), 2(1A)
(b), 2(1A) (c) as given below:
Income from leasing out of agricultural land Section 2(1A) (a)
If any person has given any agricultural land on rent, rent so received shall be considered to be agricultural
income and shall be exempt from income tax e.g. Mr. X has ten acres of agricultural land in India which is
given on lease at a rent of ` 2,00,000. It will be considered to be agricultural income.
If rent is received in kind, still it will be considered to be agricultural income e.g. Mr. X has leased out ten
acres of agricultural land and has received wheat crop worth `2,00,000. In this case, `2,00,000 shall be
considered to be his agricultural income.
If rent to be received has not been received in time and accordingly interest has been received, such interest
shall not be considered to be agricultural income, rather it is his income under the head other sources.
If the agricultural land is situated outside India, income from agricultural land is taxable as income from
other sources.
(One acre is equal to an area of 4,840 square yards (0.405 hectare))
Income from Agricultural Operations Section 2(1A)(b)
If any person is engaged in agricultural activities, income derived from such agricultural operations shall be
considered to be agricultural income.
Dividends received by a shareholder from the company having agricultural income
If any shareholder has received dividend from a company having income from agricultural activities, such
income shall not be considered to be agricultural income rather it will be considered to be dividend income.
However, if the company paying dividend is a domestic company, dividend upto `10,00,000 shall be
exempt under section 10(34) and the company has to pay additional income tax under section 115-O @
15% plus surcharge @ 12% plus HEC @ 4%. E.g. ABC Ltd. an Indian company has agricultural income of
`500 lakhs and company has distributed dividend of `50 lakh and one of the shareholder Mr. X has received
dividend of `5 lakh, in this case tax treatment shall be:
Tax liability of ABC Ltd. Shall be nil as per section 10(1), however company has to pay additional income
tax as given below:
50,00,000 / 82.528 x 17.472 = 10,58,549.83 Rounded off u/s 288B 10,58,550
Dividend received by Mr. X shall be exempt from income tax under section 10(34)
If any foreign company is doing agriculture, its agricultural income in India shall also be exempt and if the
company has paid dividend, it will be taxable in the hands of the shareholder and the company shall be
exempt from additional income tax e.g. If in the above case it is a foreign company, its tax liability shall be
nil also additional income tax shall be nil and tax liability of shareholder shall be as given below:
Tax on `5,00,000 at slab rate 12,500
Add: HEC @ 4% 500
Agricultural Income 284

Tax Liability 13,000


Illustration 1: ABC Ltd. an Indian company has agricultural income `350 lakhs and company has
distributed dividend of `60 lakhs to its shareholders and one of the shareholder Mr. X has received dividend
of `4,00,000. Compute tax liability and additional income tax liability of the company and tax liability of
shareholder.
Solution:
Tax liability of ABC Ltd. Shall be nil as per section 10(1), however company has to pay additional income
tax as given below:
60,00,000 / 82.528 x 17.472 = 12,70,259.79 Rounded off u/s 288B 12,70,260
Dividend received by Mr. X shall be exempt from income tax under section 10(34)
(b) Presume it is foreign company.
Solution:
It is a foreign company, its tax liability shall be nil also additional income tax shall be nil and tax liability of
shareholder shall be as given below:
Tax on `4,00,000 at slab rate 7,500
Add: HEC @ 4% 300
Tax Liability 7,800
(c) Presume it is Indian company and income is from business and not from agriculture.
Solution:
Tax liability and additional tax liability of the company shall be as given below: `
Profit before tax 350,00,000.00
Income tax on `350,00,000 @ 30% 105,00,000.00
Add: Surcharge @ 7% 7,35,000.00
Add: HEC @ 4% 4,49,400.00
Income tax liability 116,84,400.00
Dividend 60,00,000.00
Additional Income Tax
(60,00,000 / 82.528% x 17.452%) 12,70,259.79
Rounded off u/s 288B 12,70,260.00
Tax liability of the shareholder shall be nil.
(d) Presume it is foreign company and income is from business and not from agriculture
Solution:
Tax liability and additional tax liability of the company shall be as given below: `
Profit before tax 350,00,000.00
Income tax on `350,00,000 @ 40% 140,00,000.00
Add: Surcharge @ 2% 2,80,000.00
Add: HEC @ 4% 5,71,200.00
Income tax liability 148,51,200.00
Additional income tax of the foreign company shall be nil
Tax liability of the shareholder shall be as given below:
Dividend from foreign company
Tax on `4,00,000 at slab rate 7,500
Add: HEC @ 4% 300
Tax Liability 7,800

Payments received by a partner from the partnership firm


If any partnership firm has agricultural income, it will be exempt from income tax and if partnership firm
has paid any salary or interest to the partners, it will be considered to be agricultural income to the partners
as decided in R.M. Chidambaram Pillai v CIT (SC)
If any partner has received any share out of profits of partnership firm, it will be exempt under section
10(2A) and it do not matter whether partnership firm has agricultural income or non-agricultural income.
If partnership firm has non-agricultural income, salary or interest received by a partner from the partnership
firm shall be considered to be their income under the head business/profession as per section 28 and shall be
Agricultural Income 285

taxable in the hands of partner e.g. XY partnership firm has two partners Mr. X and Mr. Y and profit sharing
ratio is 1:1 and the firm has agricultural income `300 lakhs without debiting salary or interest to the
partners. The firm has paid salary of `8 lakh to each of the partner and interest of `4 lakh to each of the
partner. Mr. X has income under the head house property `5 lakh and Mr. Y has income under the head
house property `7 lakh. Compute tax liability of the firm and also that of partners.
Solution:
Since partnership firm has agricultural income, it is exempt from income tax under section 10(1).
Tax liability of Mr. X shall be
Income under the head House Property 5,00,000
Agricultural income (8,00,000 + 4,00,000) 12,00,000
Partial integration
5,00,000 + 12,00,000 = 17,00,000 at slab rate 3,22,500
2,50,000 + 12,00,000 = 14,50,000 at slab rate 2,47,500
(3,22,500 – 2,47,500) 75,000
Tax before health & education cess 75,000
Add: HEC @ 4% 3,000
Tax Liability 78,000
Tax liability of Mr. Y shall be
Income under the head House Property 7,00,000
Agricultural income (8,00,000 + 4,00,000) 12,00,000
Partial integration
7,00,000 + 12,00,000 = 19,00,000 at slab rate 3,82,500
2,50,000 + 12,00,000 = 14,50,000 at slab rate 2,47,500
(3,82,500 – 2,47,500) 1,35,000
Add: HEC @ 4% 5,400
Tax Liability 1,40,400
Share received out of profits is exempt under section 10(2A).
Presume in the above case partnership firm has income from business and not agricultural income.
Solution:
Tax Liability of partnership firm shall be as given below:
Profits before debiting salary and interest 300,00,000
Less: Salary and Interest 24,00,000
Income under the head Business/Profession 276,00,000
Gross Total Income/Total Income 276,00,000
Tax Liability 276,00,000 x 30% 82,80,000
Add: Surcharge @ 12% 9,93,600
Tax before health & education cess 92,73,600
Add: HEC @ 4% 3,70,944
Tax Liability 96,44,544
Rounded off u/s 288B 96,44,540
Tax Liability of Mr. X
Income under the head Business/Profession (salary + interest) 12,00,000
Income under the head House Property 5,00,000
Gross Total Income/Total Income 17,00,000
Tax on `17,00,000 at slab rate 3,22,500
Add: HEC @ 4% 12,900
Tax Liability 3,35,400
Tax Liability of Mr. Y
Income under the head Business/Profession (salary + interest) 12,00,000
Income under the head House Property 7,00,000
Gross Total Income/Total Income 19,00,000
Tax on `19,00,000 at slab rate 3,82,500
Add: HEC @ 4% 15,300
Agricultural Income 286

Tax Liability 3,97,800


Meaning of Agriculture: The term agriculture and agricultural purposes has not been defined under Income
Tax Act, accordingly its meaning has been explained in Raja Benoy Kumar Sahas Roy v CIT (SC). If any
person has performed the following two operations, it will be called agriculture.
1. Basic Operations:
In order to constitute agriculture, there must be basic operations like ploughing of land, sowing of seeds,
planting and similar kind of operations on the land.
2. Subsequent Operations:
After carrying out basic operations, there must be subsequent operations like weeding, digging the soil
around the growth, watering of the plant at regular intervals, using pesticides and insecticides to
protect the crop and it will also include pruning, cutting, harvesting etc.
(Pruning means to trim (a tree, shrub, or bush) by cutting away dead or overgrown branches or stems,
especially to encourage growth.)
If there are basic and subsequent operations, it will be considered to be agricultural income even if what is
produced is not food grains, example:
(i) If a person is growing betel, coffee, tea, spices etc through basic and subsequent operations, it will be
agricultural income.
(ii) If a person is growing commercial crops like cotton, flax, jute, indigo etc. through basic and
subsequent operations, it will be considered to be agricultural income.
(iii)If a person is growing trees like Sal, Seesam, Sangwan etc for obtaining timber, it will be considered to
be agricultural income, provided there are basic and subsequent operations.
Income which is partially agricultural and partially from business Rule 7
If any person is engaged in growing as well as manufacturing activity, in such cases it will be presumed that
he has transferred his agricultural produce to his industrial undertaking at the market price and expenses on
agriculture shall be deducted from such amount and balance shall be agricultural income. While computing
income of business, such market price is allowed to be deducted as cost of raw material. E.g. Mr. X is
engaged in growing of sugarcane and also has a sugar factory. He has incurred expenses of `3,00,000 in
connection with growing of sugarcane crop. Entire sugarcane crop was transferred to the industrial unit
when market price of sugarcane was ` 10,00,000. In this case, agricultural income of Mr. X shall be `
7,00,000. While computing income of sugar factory, ` 10,00,000 shall be debited to profit and loss account
as the cost of raw material.
Example
Mr. X grows sugarcane and uses the same for the purpose of manufacturing sugar in his factory.
50% of sugarcane produce is sold for `10 lacs, and the cost of cultivation of such sugarcane is `3 lacs.
The cost of cultivation of the balance sugarcane (50%) is 3 lacs and the market value of the same is `10 lacs.
After incurring `1.5 lacs in the manufacturing process on the balance sugarcane, the sugar was sold for `25
lacs.
Compute Mr. X’s business income and agricultural income. Compute his Tax Liability.
Solution:
Agricultural income = Actual sale of sugarcane + Market value of sugarcane transferred to the
manufacturing unit – Cost of cultivation
= [`10 lacs + `10 lacs] – [`3 lacs + `3 lacs]
= `20 lacs – `6 lacs
= `14 lacs
Business income = Sales – Market value of 50% of sugarcane produce – Manufacturing expenses
= `25 lacs – 10 lacs – 1.5 lacs
= 13.5 lacs
Computation of tax liability
Step 1: Tax on (`14,00,000 + ` 13,50,000 = ` 27,50,000) 6,37,500
Step 2: Tax on (` 2,50,000 + ` 14,00,000) = ` 16,50,000) (3,07,500)
Step 3: ` 6,37,500 – ` 3,07,500 3,30,000
Tax before health & education cess 3,30,000
Add : HEC @ 4% 13,200
Agricultural Income 287

Tax Liability 3,43,200

Computation of income in case of growing and manufacturing of Rubber Rule 7A


If any person is engaged in growing and manufacturing of rubber, income shall be computed combined for
agriculture as well as business and 35% of such income shall be business income and balance shall be
agricultural income e.g. If income from growing + manufacturing is `100 lakhs, income from business shall
be `35 lakhs and income from agriculture shall be `65 lakhs.
Computation of income from the growing and manufacturing of Coffee Rule 7B
If any person is engaged in growing and manufacturing of coffee, income shall be computed combined for
agriculture as well as business and 40% of such income shall be business income and balance shall be
agricultural income e.g. If income from growing + manufacturing is `100 lakhs, income from business shall
be `40 lakhs and income from agriculture shall be `60 lakhs.
If any person is engaged in growing and curing of coffee, 25% of such income shall be business income and
balance shall be agricultural income.
Computation of income in case of persons Growing and Manufacturing Tea Rule 8
If any person is engaged in growing and manufacturing of tea, income shall be computed combined for
agriculture as well as business and 40% of such income shall be business income and balance shall be
agricultural income e.g. If income from growing + manufacturing is `100 lakhs, income from business shall
be `40 lakhs and income from agriculture shall be `60 lakhs.
Marketing operations /Marketing process
Sometimes a cultivator may not get the ready market to sell the crop in the form in which it was grown,
rather some essential processes are required to be carried out to make the crop fit for sale. In such cases, the
income shall continue to be agricultural income.
Example
Threshing is done in case of wheat crop to render it fit for sale, similarly, tobacco leaves are dried to make
them fit for sale. In all such cases, it will continue to be agricultural income.
 Income derived from animal husbandry, fisheries, poultry farming, dairy farming etc. shall not
be considered to be agricultural income.
 Income derived from saplings or seedlings growing in a nursery shall be considered to be
agricultural income.
 Income from sale of agricultural land shall not be considered to be agricultural income rather
it will be considered to be capital gain.
 Rent received for letting out agricultural land for a movie shooting shall not be considered to
be agricultural income.
Illustration 2: Mr. X, a resident, has provided the following particulars of his income for the P.Y.2018-19.
i. Income from salary (computed) `2,40,000
ii. Income from house property (computed) `2,00,000
iii. Agricultural income from a land in Jaipur `1,80,000
iv. Expenses incurred for earning agricultural income `1,20,000
Compute his tax liability assuming his age is -
(a) 45 years
(b) 70 years
Solution: Computation of total income of Mr. X for the A.Y.2019-20
(a) Computation of tax liability (age 45 years)
Particulars `
Income from salary 2,40,000
Income from house property 2,00,000
Gross Total Income 4,40,000
Less: Deductions under Chapter VI-A Nil
Total Income 4,40,000
Agricultural income (1,80,000 – 1,20,000) = 60,000
Computation of tax liability
Step 1: Tax on (`4,40,000 + ` 60,000 = ` 5,00,000) 12,500
Agricultural Income 288

Step 2: Tax on (` 2,50,000 + ` 60,000) = ` 3,10,000) (3,000)


Step 3: ` 12,500 – ` 3,000 9,500
Tax before health & education cess 9,500
Add : HEC @ 4% 380
Tax Liability 9,880
(b) Computation of tax liability (age 70 years)
Step 1: Tax on (` 4,40,000 + ` 60,000 = ` 5,00,000) 10,000
Step 2: Tax on (` 3,00,000 + ` 60,000 = ` 3,60,000) (3,000)
Step 3: ` 10,000 – ` 3,000 7,000
Tax before health & education cess 7,000
Add : HEC @ 4% 280
Tax Liability 7,280
Illustration 3: Mr. X grows sugarcane and uses the same for the purpose of manufacturing sugar in his
factory. 30% of sugarcane produce is sold for ` 10 lacs, and the cost of cultivation of such sugarcane is ` 5
lacs. The cost of cultivation of the balance sugarcane (70%) is `14 lacs and the market value of the same is
`22 lacs. After incurring ` 1.5 lacs in the manufacturing process on the balance sugarcane, the sugar was
sold for ` 25 lacs. Compute Mr. X’s business income and agricultural income.
Solution:
Income from sale of sugarcane gives rise to agricultural income and from sale of sugar gives rise to business
income.
Business income = Sales – Market value of 70% of sugarcane produce – Manufacturing expenses
= `25 lacs – `22 lacs - `1.5 lacs = `1.5 lacs.
Agricultural income = Market value of sugarcane produce – Cost of cultivation
= [`10 lacs + `22 lacs] – [`5 lacs + `14 lacs]
= `32 lacs – `19 lacs
= `13 lacs.
Illustration 4: Mr. X is engaged in growing and manufacturing of rubber. These are then sold in the market
for `30 lacs. The cost of growing rubber plants is `10 lacs and that of manufacturing rubber is `8 lacs.
Compute his total income.
Solution:
The total income of Mr. X comprises of agricultural income and business income.
Total profits from the sale of rubber = `30 lacs – `10 lacs – `8 lacs = `12 lacs.
Agricultural income = 65% of `12 lacs. = `7.8 lacs
Business income = 35% of `12 lacs. = `4.2 lacs
Illustration 5: Mr. X has estates in rubber, tea and coffee. He derives income from them. He has a nursery
wherein he grows and sells the plants. For the previous year ending 31.03.2019, he furnishes the following
particulars of his income from estates and sale of plants. You are requested to compute the taxable income
and tax liability for the assessment year 2019-20: `
(i) Growing and manufacturing of rubber 5,00,000
(ii) Sale of coffee grown and cured 3,50,000
(iii) Growing and manufacturing of tea 7,00,000
(iv) Sale of plants from nursery 1,00,000
He has long term capital gain on the sale of agricultural land in Delhi `3,13,500. He has received rent of
`7,000 p.m. by letting out one farm house near Delhi and he has incurred `20,000 on the repairs of the farm
house. He has not paid municipal taxes for the last ten years in connection with farm house and MCD has
issued him a notice for selling of farm house, hence he has paid municipal tax of `90,000.
Solution:
Agricultural Income Business Income
(a) Income from growing and manufacturing of Rubber {Rule 7A}
[Agricultural income 65% and business income 35%] 3,25,000 1,75,000
(b) Income from Coffee grown and cured {Rule 7B}
[Agricultural income 75% and business income 25%] 2,62,500 87,500
(c) Income from growing and manufacturing of Tea {Rule 8}
Agricultural Income 289

[Agricultural income 60% and business income 40%] 4,20,000 2,80,000


(d) Income from growing and selling of plants 1,00,000 ----------
Total 11,07,500 5,42,500
Computation of Income under the head House Property `
Gross Annual Value (7,000 x 12) 84,000.00
Less: Municipal Taxes (90,000.00)
Net Annual Value (6,000.00)
Less: 30% of NAV u/s 24(a) Nil
Less: Interest on capital borrowed u/s 24(b) Nil
Loss under the head House Property (6,000.00)
Option I
Loss under the head house property set off from long term capital gains
Long term capital gains 3,13,500.00
Loss under the head House Property (6,000.00)
Long term capital gains after adjusting loss from house property 3,07,500.00
Income under the head Business/Profession 5,42,500.00
Gross Total Income 8,50,000.00
Less: Deduction under section 80C to 80U Nil
Total Income 8,50,000.00
Computation of Tax Liability
Tax on long term capital gain `3,07,500 @ 20% u/s 112 61,500.00
Normal income `5,42,500
Tax on (5,42,500 + 11,07,500) 3,07,500.00
Tax on(2,50,000 + 11,07,500) (2,19,750.00)
Tax on Normal Income (3,07,500 – 2,19,750) 87,750.00
Tax before health & education cess 1,49,250.00
Add: HEC @ 4% 5,970.00
Tax Liability 1,55,220.00
Option II
Loss under the head house property set off from business income
Income under the head Business/Profession 5,42,500.00
Loss under the head House Property (6,000.00)
Income under the head Business/Profession 5,36,500.00
Income under the head Capital gain (LTCG) 3,13,500.00
Gross Total Income 8,50,000.00
Less: Deduction under section 80C to 80U Nil
Total Income 8,50,000.00

Computation of Tax Liability


Tax on long term capital gain `3,13,500 @ 20% u/s 112 62,700.00
Normal income `5,36,500
Tax on (5,36,500 + 11,07,500) 3,05,700.00
Tax on (2,50,000 + 11,07,500) (2,19,750.00)
Tax on Normal Income (3,05,700 – 2,19,750) 85,950.00
Tax before health & education cess 1,48,650.00
Add: HEC @ 4% 5,946.00
Tax Liability 1,54,596.00
Rounded off u/s 288B 1,54,600.00
Hence Option II is better.
Income from a Farm Building Section 2(1A)(c)
If any building is in the agricultural field or is very near to the agricultural field and it is being used for
storing agricultural produce or for storing agricultural implements or it is being used as dwelling unit by the
farmer himself, such building is called farm building and its income shall be computed as per provisions
Agricultural Income 290

given under the head house property and income shall be considered to be agricultural income.
Such building must be in the rural area. If it is in the urban area, it should be constructed on the land which
has been classified as agricultural land.
Land in rural area shall be considered to be urban land in the following cases:
1. If rural area is within the distance of 2 kms from the limits of urban area having population more
than 10000 but not exceeding 100000
2. If rural area is within the distance of 6 kms from the limits of urban area having population more
than 100000 but not exceeding 1000000
3. If rural area is within the distance of 8 kms from the limits of urban area having population more
than 1000000.
A farm house shall not be considered to be farm building and income of farm house shall be taxable.
Self Reading
Judicial Decisions
B. Gupta Private Ltd. v CIT, (HC)
Compensation received from an insurance company on account of damage caused to the crops is
agricultural income.
Venkataswamy Naidu v CIT, (SC)
Income from butter and cheese making is not agricultural income.
Sri Ranga Vilas Ginning & Oil Mills v. CIT, (HC)
Income from supplying surplus water to other agriculturists is not agricultural income.
New Ambadi Estates Ltd. v CIT, (SC)
Harvest crops on purchased land is not agricultural income.
K. Lakshmansa & Co. v CIT, (SC)
If the assessee was growing mulberry leaves, feeding them to silkworms and obtaining silk cocoons,
income from sale of silk cocoons would not be agricultural income.
Illustration 6: Mr. X is employed in MP Agricultural University and getting basic pay `20,000 p.m. He
claims that it is his agricultural income. Discuss.
Solution: Income from an agricultural university cannot be considered to be agricultural income rather it is
his income under the head salary.
Illustration 7: Mr. X has sold his agricultural land in Delhi and there are long term capital gains of
`10,00,000. Mr. X claims it to be his agricultural income. Discuss.
Solution: Income from sale of agricultural land cannot be considered to be agricultural income and
accordingly it is chargeable to tax under the head capital gains.
Illustration 8: Mr. X holds shares in ABC Ltd., an Indian Company, which is engaged in agricultural
operations. He has received dividends of `1,20,000 from ABC Ltd. and claims that it is his agricultural
income. Discuss.
Solution: Dividend from a company which is engaged in agricultural operations cannot be considered to be
agricultural income rather it is dividend income of the recipient. However, dividend income is exempt from
tax under section 10(34).
Agricultural Income 291

PRACTICE PROBLEMS
TOTAL PROBLEMS 6
Problem 1.
Mr. X (non-resident, aged 68 years) has incomes as given below:
(i) Income under the head Salary `3,00,000
(ii) Income under the head House Property `1,20,000
(iii) Income from long term capital gains `50,000
(iv) Casual income `30,000
(v) Agricultural income `60,000
(vi) Deductions under section 80D to 80U `1,40,000
(vii) He has invested `40,000 in Kisan Vikas Patra, `20,000 in equity shares of infrastructure
development companies.
Compute his total income and tax liability for the assessment year 2019-20.
Answer: Total Income: `3,40,000; Tax Liability: `20,280

Problem 2.
Mrs. X (aged 58 years) has income and losses as given below:
(i) Income from growing and manufacturing of Rubber `3,00,000
(ii) Income from growing and curing coffee `2,00,000
(iii) Income under the head Salary `2,40,000
(iv) Loss under the head House Property `1,00,000
(v) Income from short term capital gains `40,000
(vi) Income from long term capital gains `50,000
(vii) Casual income `60,000
Compute her total income and tax liability for the assessment year 2019-20.
Answer: Total Income: `4,45,000; Tax Liability: `46,800

Problem 3.
Mrs. X (resident but not ordinarily resident) have incomes as given below:
(i) Income from growing and manufacturing of Tea in India `10,00,000
(ii) Income from house property situated outside India `3,50,000, received outside India.
(iii) Income from agriculture in Nepal `1,50,000, received in India
(iv) Income from business in Paris and received in Paris ` 1,00,000
Compute her total income and tax liability for the assessment year 2019-20.
Answer: Total Income: `5,50,000; Tax Liability: `78,000

Problem 4.
Mr. X (resident but not ordinarily resident) have incomes and losses as given below:
(i) Loss from house I in India `80,000
(ii) Income from house II in India `1,00,000
(iii) Carried forward loss assessment year 2008-09 from house III in India `50,000
(iv) Income under the head Business/Profession in India `2,20,000
(v) Royalty received in the UK for use of formula in U.K. `30,000
(vi) Long term capital gains in India `1,00,000
(vii) Income from agriculture in Indonesia but received in India and subsequently invested it in
Indonesia `50,000
(viii) Income from agriculture in India `2,00,000
Compute his total income and tax liability for the assessment year 2019-20.
Answer: Total Income: `3,90,000; Tax Liability: `22,880
Agricultural Income 292

Problem 5.
A partnership firm XY has agricultural income `2,00,000, income under the head business/profession
`1,00,000 and long term capital gains `10,000.
Compute its tax liability for the assessment year 2019-20.
Answer: Tax Liability: `33,280

Problem 6.
A partnership firm Z & Co. has agricultural income `20,00,000 and its partner Mr. Z has received `5,00,000
being his share in the profits of partnership. Mr. Z has income under the head house property `2,75,000.
Compute tax liability of the partnership firm and also that of Mr. Z.
Answer: Tax Liability: Partnership Firm: Nil; Mr. Z: Nil
Agricultural Income 293

SOLUTIONS
TO
PRACTICE PROBLEMS
Solution 1: `
Income under the head Salary 3,00,000
Income under the head House Property 1,20,000
Income under the head Capital Gains (LTCG) 50,000
Income under the head Other Sources (Casual Income) 30,000
Gross Total Income 5,00,000
Less: Deduction u/s 80C (20,000)
Less: Deduction u/s 80D to 80U (1,40,000)
Total Income 3,40,000
Agricultural income 60,000
Computation of Tax Liability
Tax on casual income `30,000 @ 30% u/s 115BB 9,000
Tax on Long term capital gain `50,000 @ 20% u/s 112 10,000
Normal income `2,60,000
Tax on (2,60,000 + 60,000) at slab rate 3,500
Tax on (2,50,000 + 60,000) at slab rate (3,000)
Tax on normal income (3,500 – 3,000) 500
Tax before health & education cess 19,500
Add: HEC @ 4% 780
Tax Liability 20,280
Note: 1. Benefit of the slab rate for senior citizen is not available to non-resident assessee.
2. Rebate u/s 87A is not allowed to non-resident.

Solution 2:
Agricultural Income Business Income
Income from growing and manufacturing of Rubber {Rule 7A}
Agricultural income 65% and business income 35% 1,95,000 1,05,000
Income from Coffee grown and cured {Rule 7B}
Agricultural income 75% and business income 25% 1,50,000 50,000
Total 3,45,000 1,55,000
Option I
House property loss can be set off from normal income `
Income under the head Salary 2,40,000
Loss under the head House Property (1,00,000)
Income under the head salary after adjusting house property loss 1,40,000
Income under the head Business/Profession 1,55,000
Income under the head Capital Gains
Short term capital gains 40,000
Long term capital gains 50,000
Income under the head Other Sources (Casual Income) 60,000
Gross Total Income 4,45,000
Less: Deductions u/s 80C to 80U Nil
Agricultural Income 294

Total Income 4,45,000


Agricultural income 3,45,000
Computation of Tax Liability
Tax on casual income `60,000 @ 30% u/s 115BB 18,000
Tax on Long term capital gain `50,000 @ 20% u/s 112 10,000
Normal income `3,35,000
Tax on (3,35,000 + 3,45,000) at slab rate 48,500
Tax on (2,50,000 + 3,45,000) at slab rate (31,500)
Tax on normal income (48,500 – 31,500) 17,000
Tax before health & education cess 45,000
Add: HEC @ 4% 1,800
Tax Liability 46,800
Option II
House property loss can be set off from LTCG `
Income under the head Capital Gains (LTCG) 50,000
Loss under the head House Property 50,000
Income under the head Capital Gains (LTCG) after adjusting house property loss Nil
Income under the head Capital Gains (STCG) 40,000
Income under the head Salary 2,40,000
Loss under the head House Property 50,000
Income under the head salary after adjusting house property loss 1,90,000
Income under the head Business/Profession 1,55,000
Income under the head Other Sources (Casual income) 60,000
Gross Total Income 4,45,000
Less: Deduction u/s 80C to 80U Nil
Total Income 4,45,000
Agricultural Income 3,45,000
Computation of Tax Liability
Tax on casual income `60,000 @ 30% u/s 115BB 18,000
Normal income `3,85,000
Tax on (3,85,000 + 3,45,000) at slab rate 58,500
Tax on (2,50,000 + 3,45,000) at slab rate (31,500)
Tax on normal income (58,500 – 31,500) 27,000
Tax before health & education cess 45,000
Add: HEC @ 4% 1,800
Tax Liability 46,800
Tax liability is same in both the options. Therefore, house property loss can be set off either from income of
normal business or from income of long term capital gain.

Solution 3:
Agricultural Income Business Income
Income from growing and manufacturing of Tea {Rule 8}
Agricultural income 60% and business income 40% 6,00,000 4,00,000
Total 6,00,000 4,00,000
`
Income under the head business/Profession 4,00,000

Income from agriculture in Nepal but received in India 1,50,000


Income under the head Other Sources 1,50,000
Gross Total Income 5,50,000
Less: Deductions u/s 80C to 80U Nil
Total Income 5,50,000
Agricultural income 6,00,000
Agricultural Income 295

Computation of Tax Liability


Tax on (5,50,000 + 6,00,000) at slab rate 1,57,500
Tax on (2,50,000 + 6,00,000) at slab rate (82,500)
Tax on normal income (1,57,500 – 82,500) 75,000
Add: HEC @ 4% 3,000
Tax Liability 78,000

Solution 4: `
Income from House II 1,00,000
Loss from House I (80,000)
Income under the head House Property 20,000
Income under the head Business/Profession 2,20,000
Income under the head Capital Gains (LTCG) 1,00,000
Income under the head Other Sources
{Income from agriculture in Indonesia, received in India} 50,000
Gross Total Income 3,90,000
Less: Deduction u/s 80C to 80U Nil
Total Income 3,90,000
Agricultural Income 2,00,000
Computation of Tax Liability
Tax on Long term capital gain `1,00,000 @ 20% u/s 112 20,000
Normal income `2,90,000
Tax on (2,90,000 + 2,00,000) at slab rate 12,000
Tax on (2,50,000 + 2,00,000) at slab rate (10,000)
Tax on normal income (12,000 – 10,000) 2,000
Tax before health & education cess 22,000
Add: HEC @ 4% 880
Tax Liability 22,880

Solution 5: `
Income under the head Business/Profession 1,00,000
Income under the head Capital Gains (LTCG) 10,000
Gross Total Income 1,10,000
Less: Deduction u/s 80C to 80U Nil
Total Income 1,10,000
Agricultural income 2,00,000
Computation of Tax Liability
Tax on `1,00,000 @ 30% 30,000
Tax on Long term capital gain `10,000 @ 20% u/s 112 2,000
Tax before health & education cess 32,000
Add: HEC @ 4% 1,280
Tax Liability 33,280
Note: Partial integration is not applicable in case of a partnership firm or a company.

Solution 6: `
Computation of Tax Liability of Partnership firm
Agricultural income 20,00,000
Tax liability Nil
Computation of Tax Liability of Mr. Z
Share of profit from partnership firm {exempt u/s 10(2A)} Nil
Income under the head House Property 2,75,000
Gross Total Income 2,75,000
Less: Deduction u/s 80C to 80U Nil
Agricultural Income 296

Total Income 2,75,000


Tax on `2,75,000 at slab rate 1,250
Less: Rebate u/s 87A (1,250)
Tax Liability Nil
Agricultural Income 297

EXAMINATION QUESTIONS
MAY – 2017
Question 2(a) (ii) (4 Marks)
Discuss with brief reasons, whether rent received for letting out agricultural land for a movie shooting and
amounts received from sale of seedlings in a nursery adjacent to the agricultural lands owned by an assessee
can be regarded as agricultural income, as per the provisions of the provisions of the Income tax Act,1961.
Answer:
Rent received from letting out agricultural land for a movie shooting: As per section 2(1A) Agricultural
income means, any rent or revenue derived from land which is situated in India and is used for agricultural
purposes.
In the present case, rent is being derived from letting out of agricultural land for a movie shoot, which is not
an agricultural purpose. Hence, Rent received from letting out agricultural land for a movie shooting is not
Agricultural income
Amount received from sale of seedlings in a nursery: As per Section 2(1A), Income derived from sapling
or seedling grown in nursery is deemed to be agricultural Income.
Therefore, Amount received from sale of seedlings in a nursery adjacent to the agricultural lands is
Agricultural income.
NOV – 2016
Question 5(a) (4 Marks)
Mr. Kamal grows paddy and uses the same for the purpose of manufacturing of rice in his own Rice Mill.
The cost of cultivation of 40% of paddy produce is `7,00,000 which is sold for `15,00,000; and the cost of
cultivation of balance 60% of paddy is `12,00,000 and the market value of such paddy is `24,00,000. To
manufacture the rice, he incurred `2,00,000 in the manufacturing process on the balance (60%) paddy.
The rice was sold for `30,00,000.
Compute the Business income and Agriculture Income of Mr. Kamal.
Solution:
As per Rule 7 of Income Tax Rules 1962, if any person is growing agricultural produce and is using it in his
own factory to manufacture a product, in such cases it will be presumed that the assessee has sold his
agricultural produce to his manufacturing unit at the market price and income shall be computed
accordingly. In the given case, computation of income shall be as given below:

Agricultural income = Actual sale of paddy + Market value of paddy transferred to the
manufacturing unit – Cost of cultivation
= [`15 lacs + `24 lacs] – [`7 lacs + `12 lacs]
= `39 lacs – `19 lacs
= `20 lacs

Business income = Sales – Market value of 60% of paddy produce – Manufacturing expenses
= `30 lacs – 24 lacs – 2 lacs
= 4 lacs
NOV – 2011
Question 4 (2 Marks)
Mr. X, a 60 years old individual, is engaged in the business of roasting and grinding of coffee, derives
income `10 lacs during the financial year 2018-19. Compute the tax payable by him assuming he has not
earned any other income during the financial year 2018-19.
Answer:
Computation of income from the growing and manufacturing of Coffee Rule 7B
Income derived from the sale of coffee grown and cured by the seller in India shall be computed as if it were
income derived from business, and 25% of such income shall be deemed to be income liable to tax.
Agricultural Income 298

Income derived from the sale of coffee grown and manufactured by the seller in India, with or without
mixing of chicory or other flavouring ingredients, shall be computed as if it were income derived from
business, and 40% of such income shall be deemed to be income liable to tax.
In the given case, assessee is not engaged in growing of coffee hence entire income is business income and
tax liability shall be as given below: `
Total income 10,00,000
Tax + HEC 1,14,400
MAY – 2011
Question 1 (2 Marks)
Mr. X earned `5,00,000 from sale of Coffee grown and cured by him. He claims the entire income as
agricultural income, hence exempt from tax. Is he correct?
Answer.
Mr. X is not correct in claiming the entire income as agricultural income. As per rule 7B, in the case of
income derived from the sale of coffee grown and cured by the seller in India, 25% of such income is
taxable as business income under the head ‘Profits and gains from business or profession’ and the balance
(i.e. 75%) is agricultural income. Hence, only `3,75,000 (75% of `5,00,000) being agricultural income is
exempt from tax.
JUNE – 2009
Question 1 (2 Marks)
Whether the income derived from saplings or seedlings grown in a nursery is taxable under the Income-tax
Act, 1961?
Answer .
As per Explanation 3 to section 2(1A) of the Act, income derived from saplings or seedlings grown in a
nursery shall be deemed to be agricultural income and exempt from tax, whether or not the basic operations
were carried out on land.
NOV – 2004
Question 1 (3 Marks)
Mr. X has estates in Rubber, Tea and Coffee. He derives income from them. He has also a nursery wherein
he grows plants and sells. For the previous year ending 31.03.2019, he furnishes the following particulars of
his sources of income from estates and sale of Plants.
You are requested to compute the taxable income and tax liability for the assessment year 2019-20. `
(i) Manufacture of rubber 5,00,000
(ii) Manufacture of coffee grown and cured 3,50,000
(iii) Manufacture of tea 7,00,000
(iv) Sale of plants from nursery 1,00,000
Answer:
Agricultural Business
Income Income
Income from growing and manufacturing of Rubber {Rule 7A}
Agricultural income 65% and business income 35% 3,25,000 1,75,000
Income from Coffee grown and cured {Rule 7B}
Agricultural income 75% and business income 25% 2,62,500 87,500
Income from growing and manufacturing of Tea {Rule 8}
Agricultural income 60% and business income 40% 4,20,000 2,80,000
Income from growing and selling of plants 1,00,000 xxxxx
Total 11,07,500 5,42,500
Computation of Tax Liability
Normal income `5,42,500
Tax on (5,42,500 + 11,07,500) 3,07,500.00
Tax on (2,50,000 + 11,07,500) (2,19,750.00)
Agricultural Income 299

Tax before education cess (3,07,500 – 2,19,750) 87,750.00


Add: HEC @ 4% 3,510.00
Tax Liability 91,260.00

MAY – 1998
Question 2 (3 Marks)
From the following information, compute taxable income and tax liability of Mrs. X for the assessment year
2019-20.
`
Income from business – letting cycles on hire 2,40,000
Fixed deposit interest received from companies on deposits made of sale proceeds of land 18,000
Dividends from an Indian company having rubber plantations 6,000
Salary received as a partner from a firm growing and manufacturing tea 40,000
Sale of agricultural produce 1,75,000
Payment of government tax on agricultural lands 6,000
Expenses on power, irrigation cess and farm labour 10,000
Purchase of seeds 1,000
Tractor hire charges (for agricultural operations) 2,500
Answer:
Computation of income from agriculture
Salary from firm growing and manufacturing tea 24,000
40,000 x 60% (as per decision in R.M. Chidambaram Pillai v CIT)
Sale of agricultural produce 1,75,000
Less : Government tax (6,000)
Power, Irrigation cess etc (10,000)
Purchase of seeds (1,000)
Tractor hire charges (2,500)
Agricultural income 1,79,500
Computation of Non agricultural income :
Income from Business:
Cycle hire charges 2,40,000
Salary from firm (non –agricultural part – 40,000 x 40%) 16,000
Other sources:
Dividends from Plantation company – exempt u/s.10(34) Nil
Interest on fixed deposit with companies: 18,000
Non-Agricultural Income 2,74,000
Computation of Tax Liability
Tax on (2,74,000 + 1,79,500) at slab rate 10,175
Tax on (2,50,000 + 1,79,500) at slab rate (8,975)
Tax liability (10,175 – 8,975) 1,200
Less: Rebate u/s 87A (1,200)
Tax Liability Nil
Clubbing of Income 300

CLUBBING OF INCOME
(INCOME OF OTHER PERSONS INCLUDED
IN ASSESSEE’S TOTAL INCOME)

SECTION 60 TO 65
PARTICULARS SECTIONS
Transfer of income without transfer of assets 60
Revocable transfer of assets 61
Transfer irrevocable for a specified period 62
Transfer and revocable transfer 63
Income from assets transferred to the spouse 64(1)
Income from assets transferred to son’s wife 64(1)
Income from assets transferred to any person for the benefit of the spouse of the 64(1)
transferor/ son’s wife of the transferor
Remuneration of a spouse from a concern in which the other spouse has substantial 64(1)
interest
Clubbing of income of a minor child 64(1A)
Income from self acquired property converted to joint family property 64(2)
Liability of person in respect of income included in the income of another person 65
(Deleted from syllabus)

In general a person has to pay tax only on his own income but sometimes incomes of other persons is added
to his income to charge tax from him, it is called ‘clubbing of income’. Clubbing provision are applicable to
check tax evasion.
Clubbing provision are applicable in the following cases: -
1.Transfer of income without transferring the asset Section 60
If any person has transferred any income without transferring the asset, in such cases clubbing provision
shall be applicable.
Example
Mr. X has two deposits of `10 lakhs each and interest income of each deposit is `1.5 lakhs. He has
transferred income of one of the deposit to his brother Mr. Y. In this case, clubbing provision shall be
applicable and income shall be taxable in the hands of Mr. X.

2. Transfer of asset through revocable transfer Section 61


If any person has transferred any asset through revocable transfer, income from that asset shall be clubbed in
the income of transferor.
Example
Mr. X has transferred a deposit of `10 lakhs to his friend Mr. Y with the condition that the deposit can be
taken back by him at any time. In this case, clubbing provision shall be applicable.

3. Transfer of an asset through irrevocable transfer Section 62, 63


If any person has transferred any asset through irrevocable transfer, in this case clubbing provision shall not
apply. However, if the transferor has any right to interfere with the asset in any manner or has any right to
derive any benefit from the asset, clubbing provision shall be applicable.
All income arising to any person by virtue of any such transfer shall be chargeable to income-tax as the
Clubbing of Income 301

income of the transferor as and when the power to revoke the transfer arises.
If any person has transferred any asset through a transfer which is not revocable during the life time of the
beneficiary, in this case clubbing provision shall not apply.
Example
Mr. X has transferred one asset to Mr. Y with the condition that the asset shall be retained by Mr. Y as long
as he is alive and after that the asset shall be taken back by Mr. X. In this case, clubbing provision shall not
apply.
Meaning of revocable transfer
A transfer shall be deemed to be revocable if it contains any provision for the re-transfer directly or
indirectly of the whole or any part of the income or assets to the transferor, or it, in any way, gives the
transferor a right to re-assume power directly or indirectly over the whole or any part of the income or
assets.

4. TRANSFER OF ASSETS TO SPOUSE SECTION 64(1)[V. IMP.]


(i) If any person has transferred any asset, other than a house property to his or her spouse directly or
indirectly without adequate consideration, in such cases, income of the asset shall be clubbed in the
income of transferor.
(ii) If the asset is transferred for adequate consideration, clubbing provisions are not applicable. Similarly if
the asset is transferred under an agreement to live apart, clubbing provision shall not apply.
Example
Mr. X has transferred one deposit to his wife Mrs. X by charging full consideration of `10,00,000. In this
case, interest income shall not be clubbed in the income of Mr. X.
(iii) If there is inadequate consideration, clubbing provisions shall be applicable only with regard to the
income relating to that part of the consideration which is considered to be inadequate.
Example
Mr. X has transferred one deposit of `10,00,000 for a consideration of `7,00,000 and there is interest
income of `1,00,000 from the said deposit, in this case income of `30,000 shall be clubbed.
(iv) In order to apply clubbing provision relationship of husband and wife must exist on the date of transfer
of the asset and also on the date of accrual of income otherwise clubbing provision shall not be apply, as
decided in Philip Johan Plasket Thomas v. CIT (SC).
Example
Mr. X has transferred certain assets on 01.01.2019 to his would be wife. He got married on 10.01.2019, in
this case clubbing provision shall not apply.
(v) If any person has transferred the asset to the spouse and there is accretion to the asset, income from such
accretion shall not be clubbed, as decided in case of M. P. Birla (HC).e.g. Mr. X gifted certain shares to
Mrs. X and Mrs. X has received bonus shares, dividend from bonus shares shall not be clubbed and also
capital gains on sale of bonus shares shall not be clubbed.

Similarly if any asset has been transferred to spouse, income from the asset shall be clubbed but if same
income is invested further, income from such income shall not be clubbed e.g. Mr. X has gifted one fixed
deposit to Mrs. X, interest income from such fixed deposit shall be clubbed but if interest income is
invested further, any fresh income from such income shall not be clubbed.
Illustration 1: Mr. X transferred 2,000 debentures of `100 each of Wild Fox Ltd. to Mrs. X on 03.04.2018
without consideration. The company paid interest of `30,000 in September, 2018 which was deposited by
Mrs. X with Kartar Finance Co. in October, 2018. Kartar Finance Co. paid interest of `3,000 upto March,
2019. How would both the interest income be charged to tax in assessment year 2019-20?
Solution:
As per section 64(1), income arising from assets transferred without adequate consideration by an individual
to his spouse is liable to be clubbed in the hands of the individual, but if there is any further income from
such income, it will not be clubbed.
Therefore, `30,000, being the interest on debentures received by Mrs. X in September, 2018 will be clubbed
with the income of Mr. X, since he had transferred the debentures of the company without consideration to
her.
Clubbing of Income 302

However, the interest of `3,000 upto March 2019 earned by Mrs. X on the interest of the debentures
deposited by her with Kartar Finance Company shall be taxable in her individual capacity and will not be
clubbed with the income of Mr. X.
(vi) Where the asset transferred directly or indirectly by an individual to the spouse has been invested by the
transferee in any business, the income arising out of the business to the transferee in any previous year
shall be clubbed in the income of transferor but for this purpose capital as on first day of relevant
previous year shall be taken into consideration.
Example
(a) Mr. X has gifted `5,00,000 to Mrs. X on 01.04.2018 and She invested it in the proprietary business on
the same date and there were profits of `2,00,000. In this case, entire income of `2,00,000 shall be
clubbed in the income of Mr. X.
(b) Mrs. X has one business on 01.04.2018 with capital of `5 lakh and Mr. X has gifted `5,00,000 to Mrs. X
on 01.04.2018 and She invested it in the proprietary business on the same date and there were profits of
`2,00,000. In this case, income of `1,00,000 shall be clubbed in the income of Mr. X.
(c) Mrs. X has one business on 01.04.2018 with capital of `5 lakh and Mr. X has gifted `5,00,000 to Mrs. X
on 20.04.2018 and She invested it in the proprietary business on the same date and there were profits of
`2,00,000. In this case, income from business shall not be clubbed in the income of Mr. X because
amount was transferred in business after first day of previous year .
Illustration 2: A proprietary business was started by Smt. X in the year 2016. As on 01.04.2018 her capital
in business was `4,00,000. Her husband gifted `3,00,000, on 01.04.2018, which Smt. X invested in her
business on the same date. Smt. X earned profits from her proprietary business for the
Financial year 2018-19 `2,00,000
Financial year 2019-20 `2,40,000
Financial year 2020-21 `2,80,000
Financial year 2021-22 `3,00,000
Amount of profit was further invested in the business.
Compute amount to be clubbed in the income of Mr. X in each of the year.
Solution:
Amount to be clubbed in various years shall be as given below:
(i) Previous Year 2018-19: amount to be clubbed shall be as given below:
2,00,000 / 7,00,000 x 3,00,000 = 85,714.29
(ii) Previous Year 2019-20: amount to be clubbed shall be
2,40,000 / 9,00,000 x 3,00,000 = 80,000
(iii) Previous Year 2020-21: amount to be clubbed shall be
2,80,000 / 11,40,000 x 3,00,000 = 73,684.21
(iv) Previous Year 2021-22: amount to be clubbed shall be
3,00,000 / 14,20,000 x 3,00,000 = 63,380.28
(b) Presume amount was gifted on 10.04.2018.
Solution:
Amount to be clubbed in various years shall be as given below:
(i) Previous Year 2018-19: amount to be clubbed shall be Nil because amount was not invested in business
on the first day of the previous year
(ii) Previous Year 2019-20: amount to be clubbed shall be
2,40,000 / 9,00,000 x 3,00,000 = 80,000
(iii) Previous Year 2020-21: amount to be clubbed shall be
2,80,000 / 11,40,000 x 3,00,000 = 73,684.21
(iv) Previous Year 2021-22: amount to be clubbed shall be
3,00,000 / 14,20,000 x 3,00,000 = 63,380.28

(vii) If any person has transferred the asset to the spouse and the spouse has invested it in some partnership
firm as capital contribution or otherwise, in this case interest received from the partnership firm shall be
clubbed in the income of the transferor and capital as on first day of relevant previous year shall be taken
into consideration.
Clubbing of Income 303

If any salary has been received from partnership firm, it will not be clubbed.
If any share has been received from the profits of partnership firm, such shares shall be exempt under
section 10(2A).

(viii). If any person has transferred any asset to the spouse and spouse has further transferred this asset, in
this case, capital gain shall be considered to be the income of the transferor.

(ix) Cross-transfers are also covered


The Supreme Court, in case of Keshavji Morarji, observed that clubbing provisions shall be applicable in
case of cross transfers also e.g. A making gift of ` 50,000 to the wife of his brother B for the purchase of a
house by her and a simultaneous gift by B to A’s minor son of shares in a foreign company worth ` 50,000
owned by him, in the case, the income arising to Mrs. B from the house property should be included in the
total income of B and the dividend from shares transferred to A’s minor son would be taxable in the hands
of A.

(x) If there is indirect transfer, clubbing provisions shall be applicable in that case also e.g. Mr. X gifted
certain cash/ asset to his major son and son gifted the same asset to mother, in this case it will be considered
transfer and income shall be clubbed in the income of Mr. X.

(xi) If any person has given loan to the spouse, income from such loan shall not be clubbed.

5. Transfer of the asset to the son’s wife Section 64(1)


If any person has transferred the asset to the son’s wife, in this case, clubbing provision shall apply in the
similar manner as in the case of transfer of the assets to the spouse.
Such clubbing provisions are applicable from 01.06.1973.

6. Transfer of assets to any other person Section 64(1)


If any person has transferred the asset to any other person, clubbing provision shall not be applicable, but if
the transferor has any right to receive any benefit from the asset or the benefit shall be received by the
spouse of the transferor or by the son’s wife of the transferor, in that case, clubbing provision shall be
applicable.

7. Salary/commission/fee etc. from a concern in which the spouse has substantial interest Section
64(1)
(i) If any person is getting salary, commission, fee or any other remuneration whether in cash or in kind
from a concern in which his or her spouse has substantial interest and further salary etc. is being received
without any technical or professional qualification, in such case, salary etc. so received shall be
clubbed in the income of the spouse having substantial interest. However clubbing shall not be applicable
in relation to any income arising to the spouse where the spouse possesses technical or professional
qualifications and the income is solely attributable to the application of his or her technical or professional
knowledge and experience.
If the spouse has substantial interest along with his relative, even in that case clubbing provisions are
applicable.
Example
Mr. X is holding 11% shares of ABC Ltd. and his father is holding 10% shares in ABC Ltd. and his wife
Mrs. X is employed in ABC Ltd. without any technical or professional qualification, in this case, salary
income of Mrs. X shall be clubbed in the income of Mr. X.
(ii)Technical and professional qualification shall include not only degree or membership but also any
experience or expertise or any natural talent also, as decided in Batta Kalyani v. CIT, (HC).
(iii) As per section 2(41), Relative, means the husband, wife, brother or sister or any lineal ascendant or
descendant.
(iv) As per section 2(32), Substantial Interest means having 20% or more of the equity shares in a
company or having 20% of more of the shares in profits in any other concern.
Clubbing of Income 304

8. Asset held by Minor Child Section 64(1A) [V. IMP.]


(i) If any income accrues or arises to a minor child, such income shall be clubbed in the income of mother or
father whosoever has higher income before taking into consideration the income to be clubbed.
(ii) If the marriage of mother, father doesn’t subsist, in that case, income shall be clubbed in the income
of mother or father whosoever maintains the minor child.
(iii) Minor child for this purpose shall include even an adopted child and also step child, however, it will not
include the minor child suffering from a disability mentioned under section 80U. e.g. Minor son of
Mr. X has interest income of `2,00,000 and the minor child is suffering from a disability, in this case,
clubbing provisions shall not be applicable.
(iv) If any minor child has income through
(i) Manual labour or
(ii) has income through activity involving application of his skill, talent or specialized
knowledge and experience,
in this case, clubbing provision shall not apply, rather it will be considered to be the income of minor
child and his tax liability shall be computed separately but the return shall be filed by his father as his
guardian.
(v) If the income of minor child is to be clubbed, exemption shall be allowed under section 10(32) upto
`1,500 per annum per child. E.g. Minor son of Mr. X has interest income from bank fixed deposit
`35,000, in this case amount to be clubbed shall be 35,000 – 1,500 = 33,500
If any minor child has casual income and such income is to be clubbed, in that case exemption of `1,500
under section 10(32) shall be allowed or not is controversial.
(vi) If any person has transferred any asset to minor married daughter, clubbing provision shall applicable in
that case also e.g. Minor married daughter of Mr. X has interest income of `1,00,000 from bank deposit,
in this case income shall be clubbed in the minor of mother or father whosoever has higher income and
exemption under section 10(32) shall be allowed.
(vii) If any minor child has income from manual labour or through activity involving application of his skill,
talent or specialized knowledge and experience, such income shall not be clubbed but if such income has
been invested further, any new income shall be clubbed in the income of mother or father.
Example
Minor son of Mr. X is a child actor. He has income of `5,00,000 from stage acting, this income will not be
clubbed but if this amount was invested by him in a bank as fixed deposit, interest received by him shall be
clubbed.

9. Transfer of the asset by the member of Hindu Undivided Family to the Hindu undivided family
Section 64(2)(Conversion of self acquired property into common property of HUF)
If any member of HUF has gifted any asset to the HUF, income from such asset shall be clubbed in the
income of such member but if partition has been taken place, in that case clubbing provision shall not be
applicable however income from that part of asset which has been received by the spouse of such person,
shall be clubbed in the income of such member.
 Income for the purpose of clubbing includes loss also.
Illustration 3: Mr. X is an employee of X Ltd. and he has 25% shares of that company. His salary is
`50,000 p.m. Mrs. X is working as a computer software programmer in X Ltd. at a salary of `30,000 p.m.
She is, however, not qualified for the job. Compute the gross total income of Mr. X and Mrs. X for the
A.Y.2019-20, assuming that they do not have any other income.
Solution:
Mr. X is an employee of X Ltd and has 25% shares of X Ltd i.e. a substantial interest in the company. His
wife is working in the same company without any professional qualifications for the same. Thus, by virtue
of the clubbing provisions of the Act, the salary received by Mrs. X from X Ltd. will be clubbed in the hands
of Mr. X.
Computation of Gross Total Income of Mr. X
Particulars `
Salary received by Mr. X (` 50,000 x 12) 6,00,000
3,60,000
Clubbing of Income 305

Salary received by Mrs. X (` 30,000 x 12) 9,60,000


Gross total income
The gross total income of Mrs. X is nil.
Illustration 4: Will your answer be different if Mrs. A was qualified for the job?
Solution:
If Mrs. A possesses professional qualifications for the job, then the clubbing provisions shall not be
applicable.
Gross total income of Mr. X = Salary received by Mr. A [` 50,000 × 12 ] = ` 6,00,000
Gross total income of Mrs. X = Salary received by Mrs. A [` 30,000×12] = ` 3,60,000
Illustration 5: Mr. X is an employee of Y Ltd. and has substantial interest in the company. His salary is
`20,000 p.m. Mrs. X is also working in Y Ltd. at a salary of `12,000 p.m. without any qualifications. Mr. X
also receives `30,000 as interest on securities. Mrs. X owns a house property which she has let out. Rent
received from tenants is `6000 p.m. Compute the gross total income of Mr. X and Mrs. X for the A.Y.2019-
20.
Solution:
Since Mrs. X is not professionally qualified for the job, the clubbing provisions shall be applicable.
Computation of Gross Total Income of Mr. X
Particulars `
Income from Salary
Salary received by Mr. X (` 20,000 × 12) 2,40,000
Salary received by Mrs. X (` 12,000 × 12) 1,44,000
Income from other sources 3,84,000
Interest on securities 30,000
4,14,000
Computation of Gross Total Income of Mrs. X
Particulars ` `
Income from Salary Nil
[clubbed in the hands of Mr. X]
Income from house property
Gross Annual Value [` 6,000 × 12] 72,000
Less: Municipal taxes paid -
Net Annual Value (NAV) 72,000
Less: Deductions under section 24
- 30% of NAV i.e., 30% of ` 72,000 (21,600)
- Interest on loan - 50,400

Gross Total Income 50,400


Illustration 6: Mr. X has three minor children – two twin daughters and one son. Income of the twin
daughters is `2,000 p.a. each and that of the son is `1,200 p.a. Compute the income, in respect of minor
children, to be clubbed in the hands of Mr. X.
Solution:
Taxable income, in respect of minor children, in the hands of Mr. X is
Particulars ` `
Twin minor daughter [` 2,000 × 2] 4,000
Less: Exempt under section 10 (32) [` 1,500 × 2] (3,000) 1,000

Minor son 1,200


Less: Exempt under section 10 (32) (1,200) Nil
Income to be clubbed in the hands of Mr. X 1,000
Illustration 7: Mr. X, a mentally retarded minor, has a total income of `1,20,000 for the assessment year
2019-20. The total income of his father Mr. Y and of his mother Mrs. Y for the relevant assessment year is
`2,40,000 and `1,80,000 respectively. Discuss the treatment to be accorded to the total income of Mr. X for
the relevant assessment year.
Clubbing of Income 306

Solution: Section 64(1A) provides that all income accruing or arising to a minor child has to be included in
the income of that parent, whose total income is greater. However, the income of a minor child suffering
from any disability of the nature specified in section 80U shall not be included in the income of the parents
but shall be assessed in the hands of the child. Thus, the total income of Mr. X has to be assessed in his
hands and cannot be included in the total income of either his father or his mother.
Illustration 8: Mr. X gifts `1 lakh to his wife Mrs. X on April 1, 2018 which she invests in a firm on
interest rate of 14% per annum. On January 1, 2019, Mrs. X withdraws the money and gift it to her son’s
wife. She claims that interest which has accrued to the daughter-in-law, from January 1, 2019 to March 31,
2019 on investment made by her is not assessable in her hands but in the hands of Mr. X. Is this correct?
What would be the position, if Mrs. X has gifted the money to minor grandson, instead of the daughter-in-
law?
Solution: Section 64(1) provides that in computing the total income of any individual, there shall be clubbed
all such income as arises directly or indirectly to the son’s wife, of such individual, from assets transferred
directly or indirectly to the son’s wife by such individual otherwise than for adequate consideration.
There is an indirect transfer by Mr. X to the daughter-in-law and therefore, the interest income shall be
clubbed with income of Mr. X.
If Mrs. X had gifted the money to her minor grandson, then the interest income arising to the minor shall be
clubbed under section 64(1A) in the total income of that parent (son/daughter-in-law) whose total income
(before including such income) is higher.
Illustration 9: Mr. X, entered into the following transactions during the previous year 2018-19:
(a) Mr. X had a fixed deposit of ` 8,00,000 with State Bank of India. He instructed the bank to credit the
interest on the deposit @ 9% from 01.04.2018 to 31.03.2019 to the savings bank account of Ms. Y, his
niece, to help her in her higher education.
(b) Mr. X holds 51% share in a partnership firm. Mrs. X (wife of Mr. X) received a remuneration of `45,000
from the firm for writing its books of accounts. Mrs. X, being a fashion designer, does not possess any
qualification or training in the accountancy field.
(c) Mr. X gifted a flat to Mrs. X on April 1, 2018. During the previous year 2018-19, she received rent of
`8,500 p.m. from letting out of the flat.
(d) Mr. X gifted ` 4,00,000 to his minor son who invested the same in a business and he derived income of
`40,000 from the investment.
(e) Mr. X’s minor daughter derived an income of ` 25,000 from participation in music shows.
During the year, Mr. X got a monthly pension of ` 18,000. He had no other income. Mrs. X received salary
of ` 25,000 per month from a part time job as a fashion designer.
Discuss the tax implications of each transaction and compute the total income of Mr. X and Mrs. X.
Solution: `
Computation of Total Income of Mr. X
(a) Interest income received by Miss. Y shall be clubbed in the income of Mr. X
as per section 60 (8,00,000 x 9%) 72,000
(b) Remuneration of `45,000 received by Mrs. X shall be clubbed in the income of Mr. X
as per section 64(1) 45,000
(c) Income from House Property gifted to Mrs. X shall be taxable in the hands of Mr. X
because as per section 27 Mr. X is the deemed owner 71,400
(Rent received (i.e. ` 1,02,000) is taken as Gross Annual Value. Deduction @ 30% of
Net Annual Value is allowed u/s 24. The net income from house property would be
`71,400 (i.e. ` 1,02,000- `30,600 being 30% of NAV)
(d) Income of minor child shall be clubbed in the income of Mr. X as per section 64(1A)
because Mr. X has higher income (40,000 – 1,500) 38,500
(e) Income of minor daughter from music show shall not be clubbed Nil
Pension income of Mr. X (` 18,000×12) 2,16,000
Total Income 4,42,900
Salary income of Mrs. X 3,00,000
Clubbing of Income 307

EXAMINATION QUESTIONS
NOV – 2017
Question 6(b). (5 Marks)
Kamal gifted `10 lakhs to his wife, Sulochana on her birthday on, 1st January, 2018.
Sulochana lent `5,00,000 out of the gifted amount to Krishna on 1st April, 2018 for six months on which
she received interest of `50,000. The said sum of `50,000 was invested in shares of a listed company on 15th
October, 2018, which were sold for `75000 on 30th December, 2018. Securities transactions tax was paid on
such sale. The balance amount of gift was invested as capital by Sulochana in a business. She suffered loss
of `15,000 in the business in financial Year 2018-19.
In whose hands the above income and loss shall be included in Assessment Year 2019-20? Support your
answer with brief reasons.

Answer:
As per section 64(1), If any person has transferred any asset to his or her spouse without adequate
consideration in such case Income shall be clubbed in the income of the transferor, hence Interest income of
` 50,000 shall be clubbed in the income of Mr. Kamal.

If asset received by the spouse has been invested in the proprietor business, income from the business shall
be clubbed in the income of transferor and if there is any loss, it will also be clubbed. In the given case there
is a loss of ` 15,000 from business, such loss shall be clubbed in the income of Mr. Kamal.

If any person has transferred the asset to the spouse, income from the asset shall be clubbed but if same
income is invested further, any subsequent income shall not be clubbed as decided in the case of M.P.
BIRLA. In the given case, Mrs. Sulochana has invested interest income in the shares and there was capital
gain on the sale of shares, such capital gain shall not be clubbed rather it will be taxable in the hands of Mrs.
Sulochana.

MAY – 2015
Question 6(a)(ii). (4 Marks)
Mr. Ramesh gifted a sum of `5 lacs to his brother's minor son on 16.04.2018. On 18.04.2018, his brother
gifted debentures worth `6 lacs to Mrs. Ramesh. Son of Mr. Ramesh' brother invested the amount in fixed
deposit with Bank of India @ 9% p.a. interest and Mrs. Ramesh received interest of `45,000 on debentures
received by her.
Discuss the implications under the provisions of the Income- tax Act, 1961.
Answer: The Supreme Court, in case of Keshavji Morarji, observed that if two transactions are inter-
connected and are parts of the same transaction in such a way that it can be said that the circuitous method
was adopted as a device to evade tax, the implication of clubbing provisions would be attracted.
In the given case, the income arising to Mrs. Ramesh from debentures should be included in the total income
of Mr. Ramesh and the interest from fixed deposit transferred to minor son of Ramesh brother would be
taxable in the hands of brother of Mr. Ramesh.

NOV – 2014
Question 6(a)(ii) (4 Marks)
Mr. X has four minor children consisting of three daughters and one son. The annual income of all the
children for the Assessment Year 2019-20 were as follows:
First daughter (including Scholarship received `5,000) 10,000
Second Daughter 8,500
Third Daughter (Suffering from disability specified U/s 80U) 4,500
Son 40,000
Clubbing of Income 308

Mr. X gifted `2,00,000 to his minor Son who invested the same in the business and derived income of
`20,000 which is included above.
Compute the amount of Income earned by Minor Children to be clubbed in the hands of Mr. X.
Solution:
Computation of Amount of Income of minor children to be clubbed in the income of Mr. X
`
(i) Income of First Daughter 10,000
Less: Scholarship received exempt u/s 10(16) (assumed received for education) (5,000)
Less: Exempt u/s 10(32) (1,500)
3,500
(ii) Income of Second Daughter 8,500
Less: Exempt u/s 10(32) (1,500)
7,000
(iii) Income of Third Daughter who is suffering from disability shall not be clubbed
(iv) Income of Son 40,000
Less: Exempt u/s 10(32) (1,500)
38,500
Total Income to be clubbed (3,500 + 7,000 + 38,500) 49,000
MAY – 2013
Question 5(a) (4 Marks)
Mr. X is an employee of Larsen Limited and has substantial interest in the company. His salary is `25,000
p.m. Mrs. X also is working in that company at a salary of `10,000 p.m. without any professional
qualification.
Mr. X also receives ` 30,000 as income from securities, Mrs. X owns a house property which she has let out.
Rent received from such house property is ` 12,000 p.m.
Mr. & Mrs. X have three minor children-two twin daughters and one son. Income of the twin daughters is
`2,000 p.a. each and that of his son is ` 1,200 p.a. Compute the income of Mr. & Mrs. X.

Solution: Computation of Total Income of Mr. X and Mrs. X for the A.Y. 2019-20
Particulars Mr. X Mrs. X
(`) (`)
Income from Salaries
3,00,000
Salary income of Mr. X (` 25,000 × 12)
1,20,000
Salary income of Mrs. X (` 10,000 × 12) (Note 1)
Income from House Property -
Rent received (` 12,000×12) 1,44,000
Less: Deduction under section 24 @ 30% (43,200) 1,00,800
Income from other sources
Income from securities 30,000
Income before including income of minor children under section 64(1A) 4,50,000 1,00,800
(Note 2)
Income of twin daughters ` 4,000 -
(` 2,000 per child x 2)
Less: Exempt u/s 10(32) (`1,500 x 2) ( ` 3,000)
1,000
Income of the minor son ` 1,200
Less: Exempt u/s 10(32) (`1,200) -
Total Income 4,51,000 1,00,800
Clubbing of Income 309

Notes:
(1) As per section 64(1), the salary of ` 10,000 p.m. received by Mrs. X from the company has to be
included in the total income of Mr. X, as Mrs. X does not possess any technical or professional qualification
for earning such income and Mr. X has substantial interest in the company.
(2) As per section 64(1A), the income of a minor child is to be included in the total income of the parent
whose total income (excluding the income of minor child to be so clubbed) is greater. Further, as per section
10(32), income of a minor child which is includible in the income of the parent shall be exempt to the extent
of ` 1,500 per child.
NOV – 2012
Question No. 2(a) (3 Marks)
Mr. X is the Karta of an HUF, whose members derive income as given below: `
(i) Income from Mr. X’s Business 4,50,000
(ii) Mrs. X’s Salary as fashion designer
(Mrs. X is working as fashion designer in the business of Mr. X) 7,60,000
(iii) Minor son D (interest on fixed deposits with a bank which were gifted to him by his uncle) 10,000
(iv) Minor daughter P’s earning from sports 95,000
(v) D’s winning from lottery (gross) 1,95,000
Discuss the tax implications in the hands of Mr. and Mrs. X.
Answer:
Computation of income of Mr. X and Mrs. X
Mr. X Mrs. X
Income from X’s Business 4,50,000 -------
Salary as fashion designer ------ 7,60,000
Bank Interest to Minor Son D (10,000-1,500) `1,500 exempt u/s 10(32) ------ 8,500
Income of Minor Daughter from Sports (since she is earning income from her ------- ------
(own talent, sports, income is not to be clubbed)
Lottery income to minor son D ------- 1,95,000
TOTAL 4,50,000 9,63,500
Note: Whether exemption under section 10(32) shall be allowed from casual income or not is controversial.

MAY – 2012
Question 2 (3 Marks)
Mr. X has four children consisting 2 daughters and 2 sons. The annual income of 2 daughters were `9,000
and `4,500 and of sons were `6,200 and `4,300 respectively. The daughter who has income of `4,500 was
suffering from a disability specified under section 80U. Compute the amount of income earned by minor
children to be clubbed in hands of Mr. X.
Answer:
Computation of Amount of Income of minor children to be clubbed in the income of Mr. X
`
(i) Income of First Daughter 9,000
Less: Exempt u/s 10(32) (1,500)
7,500
(ii) Income of Second Daughter who is suffering from disability shall not be clubbed
(iii) Income of First Son 6,200
Less: Exempt u/s 10(32) (1,500)
4,700
(iv) Income of Second Son 4,300
Less: Exempt u/s 10(32) (1,500)
2,800
Total Income to be clubbed (7,500 + 4,700 + 2,800) 15,000
Clubbing of Income 310

Question 5 (5 Marks)
During the previous year 2018-19 the following transactions occurred in respect of Mr. X.
(a) Mr. X had a fixed deposit of ` 5,00,000 in Bank of India. He instructed the bank to credit the interest
on the deposit @ 9% from 01.04.2018 to 31.03.2019 to the savings bank account of Mr. B, son of his
brother, to help him in his education.
(b) Mr. X holds 75% share in a partnership firm. Mrs. X received a commission of `25,000 from the
firm for promoting the sales of the firm. Mrs. X possesses no technical or professional qualification.
(c) Mr. X gifted a flat to Mrs. X on April 1, 2018. During the previous year the flat had income under
the head House Property `52,000 to Mrs. X.
(d) Mr. X gifted `2,00,000 to his minor son who invested the same in a business and he got a share
income of ` 20,000 from the investment.
(e) Mr. X’s minor son derived an income of `20,000 through a business activity involving application of
his skill and talent.
During the year Mr. X got a monthly pension of `10,000. He had no other income. Mrs. X received
salary of ` 20,000 per month from a part time job.
Discuss the tax implications of each transaction and compute the total income of Mr. X, Mrs. X and
their minor child.
Answer:
(a) As per Section 60 of the Income Tax Act, if any person has transferred any income without
transferring the asset in such case clubbing provision shall be applicable.
In the given case, Mr. X transferred interest on fixed deposit to Mr. B (son of his brother) without
transferring the fixed deposit, such income shall be clubbed in the hands of Mr. X as per section 60.
Amount to be clubbed = `5,00,000 x 9% = `45,000
(b) As per Section 64(1) of the Income Tax Act, if any person is getting salary, commission etc. from a
concern in which his or her spouse has substantial interest and further salary etc. is received without
any professional or technical qualification, in such case, salary etc. so received shall be clubbed in
the income of the spouse having substantial interest.
In the given case Mr. X is having substantial interest in the partnership firm and Mrs. X received a
commission of `25,000 from the firm for promoting the sales of the firm without any technical or
professional qualification. So the commission shall be clubbed in the hands of Mr. X
(c) As per section 27, An individual who transfers otherwise than for adequate consideration any house
property to his or her spouse, not being a transfer in connection with an agreement to live apart shall
be deemed to be the owner of the house property so transferred.
In the given case Mr. X transfers flat to Mrs. A without adequate consideration on April 1, 2018.
So Mr. X shall be deemed to be the owner of the house property and income `52,000 shall be
considered as income of Mr. X.
(d) As per section 64(1A), if any income accrues or arises to a minor child, such income shall be clubbed
in the income of mother or father whosoever has higher income before taking in to consideration the
income to be clubbed. So in the given case, income of ` 20,000 shall be clubbed in the income of
mother or father whosoever has higher income before taking in to consideration the income to be
clubbed. Amount to be clubbed = 20,000 – 1500 = `18,500
(e) As per section 64(1A), if any minor child has income from manual labour or through activity
involving application of his skill, talent or specialized knowledge and experience, such income shall
not be clubbed but if such income has been invested further, any new income shall be clubbed in the
income of mother or father.
In the given case clubbing provision is not applicable as Mr. X’s minor son derived an income of
`20,000 through a business activity involving application of his skill and talent.
Computation of Total Income of Mr. X `
Income under the head Salary
Pension (10,000 x 12) 1,20,000.00
Less: Standard Deduction u/s 16(ia) (40,000.00)
Income under the head Salary 80,000.00
Clubbing of Income 311

Income under the head House property


(Since asset is transferred to wife hence deeming provision shall apply)
Income under the head house property 52,000.00
Income under the head other sources
Commission given to Mrs. X 25,000.00
(Since Mr. X is having substantial interest in firm)
Interest on Fixed deposit transferred to Mr. B 45,000.00
Income from investment made by Minor son 18,500.00
Income under the head other sources 88,500.00
Gross Total Income 2,20,500.00
Less: Deduction u/s 80C to 80U Nil
Total Income 2,20,500.00
Computation of Total Income of Mrs. X
Gross Salary 2,40,000.00
Less: Standard Deduction u/s 16(ia) (40,000.00)
Income under the head salary 2,00,000.00
Gross Total Income 2,00,000.00
Less: Deduction u/s 80C to 80U Nil
Total Income 2,00,000.00
Computation of Total Income of Minor Child
Income from Business 20,000.00
Gross Total Income 20,000.00
Less: Deduction u/s 80C to 80U Nil
Total Income 20,000.00
NOV – 2011
Question 1 (4 Marks)
Mr. X (age 67 years) gifted a building owned by him to his son’s wife Mrs. X on 01.10.2017. The building
fetched a rental income of `10,000 per month throughout the year. Municipal tax for the first half-year of
`5,000 was paid in June 2018 and the municipal tax for the second half-year was not paid till 30.09.2019.
Incomes of Mr. X and Mrs. X other than income from house property are given below:
Name Business income Capital gain Other sources
Mr. X `1,00,000 `50,000 (long-term) `1,50,000

Mrs. X `(75,000) `2,00,000 (short-term) `50,000


Note: Capital gain does not relate to gain from shares and securities.
Compute the total income of Mr. X and Mrs. X taking into account income from property given above and
also compute their income-tax liability for the assessment year 2019-20.
Answer:
Computation of total income and income tax liability of Mr. X `
Income under the head Business Profession 1,00,000.00
Income under the head Capital gains
LTCG 50,000.00
Income under the head House property
(Since asset is transferred to son's wife hence clubbing shall be done)
Gross Annual Value 1,20,000.00
Less: Municipal Taxes (5,000.00)
Net Annual Value 1,15,000.00
Less: 30% of NAV u/s 24(a) (34,500.00)
Less: Interest on capital borrowed u/s 24(b) Nil
Income under the head house property 80,500.00
Income under the head other sources 1,50,000.00
Gross Total Income 3,80,500.00
Less: Deduction u/s 80C to 80U Nil
Clubbing of Income 312

Total Income 3,80,500.00


Computation of Tax Liability
Tax on `3,30,500 at slab rate 1,525.00
Tax on capital gains @ 20% 10,000.00
Tax before health & education cess 11,525.00
Add: HEC @ 4% 461.00
Tax Liability 11,986.00
Rounded off u/s 288B 11,990.00
Computation of total income and tax liability of Mrs. X
Income from house property Nil
Loss from business profession (75,000.00)
Income under the head Other Sources 50,000.00
Income under the head Capital Gains (STCG) 2,00,000.00
Gross Total Income 1,75,000.00
Less: Deduction u/s 80C to 80U Nil
Total Income 1,75,000.00
Tax Liability Nil
NOV – 2011
Question 1 (3 Marks)
Mr. X started a proprietary business on 01.04.2017 with a capital of `5,00,000. He incurred a loss of
`2,00,000 during the year 2017-18. To overcome the financial position, Mrs. X, a software Engineer gave a
gift of `5,00,000 on 01.04.2018, which was immediately invested in the business by Mr. X. He earned a
profit of `4,00,000 during the year 2018-19. Compute the amount to be clubbed in the hands of Mrs. X for
the Assessment Year 2019-2020. If Mrs. X gave the said amount as loan, what would be the amount to be
clubbed?
Answer:
Computation of amount to be clubbed `
Capital of Mr. X as on 01.04.2018
Opening Capital 5,00,000
Less: Loss from the business during 2017-18 (2,00,000)
Capital as on 01.04.2018 3,00,000

Amount to be clubbed in the income of Mrs. X


= 4,00,000 /8,00,000 x 5,00,000 2,50,000
In case it has been given as loan, then no clubbing shall be done.
MAY – 2011
Question 1 (5 Marks)
Mr. X bought 200 listed shares on 19.04.2017 @ `2,000 per share.
He gifted these shares to Mrs. X on 21.03.2018.
On 01.04.2018, bonus shares were allotted in the ratio of 1:1.
All these shares were sold by Mrs. X as under:
Date of sale Manner of sale No. of shares Net sales value (`)
10.04.2018 Private sale, to her friend Mrs. Y (Market value 100 original shares 1,70,000
on this date was `2,50,000)
21.05.2018 Sold in recognized stock exchange, STT paid 100 original shares 4,20,000 (Taxable
LTCG – 2,20,000)
21.07.2018 Private sale to an outsider All bonus shares 2,50,000

Briefly state the income-tax consequences in respect of the sale of the shares by Mrs. X, showing clearly the
person in whose hands the same is chargeable, the quantum and the head of income in respect of the above
transactions. Detailed computation of total income is NOT required. Also Discuss tax consequences in the
hands of Mrs.Y.
Clubbing of Income 313

Answer. Where an asset has been transferred by an individual to his spouse otherwise than for adequate
consideration, the income arising from the sale of the said asset by the spouse will be clubbed in the hands
of the individual.
Where there is any accretion to the asset transferred, income arising to the transferee from such accretion
will not be clubbed. Hence, the profit from sale of bonus shares allotted to Mrs. X will be chargeable to tax
in the hands of Mrs. X.
Therefore, the capital gains arising from the sale of the original shares has to be included in the hands of Mr.
X, and the capital gains arising from the sale of bonus shares would be taxable in the hands of Mrs. X.
Income/loss to be clubbed in the hands of Mr. X
`
(i) 100 Original shares sold on 10.04.2018
Sale consideration 1,70,000.00
Less: Cost of acquisition of 100 shares (` 2,000 x 100) (2,00,000.00)
Short term capital loss to be included in the hands of Mr. X (30,000.00)
(ii) 100 Original shares sold on 21.05.2018
100 shares sold on 21.05.2018 in a recognized stock exchange, STT paid. Long- 1,20,000
term capital gains on sale of such shares is taxable in excess of ` 1,00,000 so
taxable value shall be (2,20,000-1,00,000)
Income taxable in the hands of Mrs. X Short-term capital gains (on sale of 200 bonus shares)
Bonus shares `
Sale consideration 2,50,000
Less: Cost of acquisition of bonus shares Nil
Short-term capital gains 2,50,000
Taxability in the hands of Mrs. Y under the head “Income from other sources”
Mrs. Y has received shares from her friend for `1,70,000 but market value is `2,50,000 hence `80,000 is
taxable as gift under the head other sources as per section 56.
NOV – 2010
Question 1 (3 Marks)
Mrs. X transferred her immovable property to ABC Co. Ltd. subject to a condition that out of the rental
income, a sum of `36,000 per annum shall be utilized for the benefit of her son’s wife.
Mrs. X claims that the amount of `36,000 (utilized by her son’s wife) should not be included in her total
income as she no longer owned the property.
State with reasons whether the contention of Mrs. X is valid in law?

Answer.
The clubbing provisions are attracted in case of transfer of any asset, directly or indirectly, otherwise than
for adequate consideration, to any person to the extent to which the income from such asset is for the
immediate or deferred benefit of son’s wife. Such income shall be included in computing the total income of
the transferor-individual. Therefore, income of `36,000 meant for the benefit of daughter-in-law is
chargeable to tax in the hands of transferor i.e., Smt. X in this case. The contention of Smt. X is, hence, not
valid in law.
MAY – 2010
Question 3 (3 Marks)
Mr. X commenced a proprietary business in the year 2013. His capital as on 01.04.2017 was `6,00,000.
On 10.04.2017 his wife gifted `2,00,000 which he invested in the business on the same date.
Mr. X earned profit from his proprietary business as given below:
Previous year 2017-18 = Profit `3,00,000
Previous year 2018-19 = Profit `4,40,000
During the Previous Year 2018-19, he sold a vacant site which resulted in chargeable long-term capital gain
of `5,00,000 (computed). The vacant site was sold on 20.12.2018.
Compute the total income and tax liability of Mr. X.
Answer.
Clubbing of Income 314

Computation of business income of Mr. X `


Capital as on 01.04.2017 6,00,000
Add: Gift from wife (10.04.2017) 2,00,000
8,00,000
Add: Profit for the year ended 31.03.2018 3,00,000
Capital as on 01.04.2018 11,00,000
Profit for the year ended 31.03.2019 4,40,000
Amount of income to be clubbed in the hands of Mrs. X under section 64(1)
`4,40,000 /11,00,000 x 2,00,000 80,000
Income assessable in the hands of Mr. X (4,40,000 – 80,000) 3,60,000
Total Income of Mr. X
Income from Business 3,60,000
Long term capital gain 5,00,000
Gross Total Income 8,60,000
Less: Deduction u/s 80C to 80U Nil
Total Income 8,60,000
Tax liability on normal income of ` 3,60,000 5,500
On LTCG @ 20% on ` 5,00,000 1,00,000
1,05,500
Add: HEC @ 4% 4,220
Tax Liability 1,09,720
Question 5 (4 Marks)
In whose hands the income from an asset is chargeable to tax in the case of transfer which is not revocable
during the life time of the beneficiary/transferee?
Answer.
As per section 61, all income arising to any person by virtue of a revocable transfer of assets is to be
included in the total income of the transferor.
As per section 62 the clubbing provisions are not attracted, if there is a transfer of asset which is not
revocable during the life time of the transferee, the income from the transferred asset is not includable in the
total income of the transferor provided the transferor derives no direct or indirect benefit from such income.
MAY – 2008
Question 1 (2 Marks)
Mr. X has transferred through a duly registered document the income arising from a godown, to his son,
without transferring the godown. In whose hands will the rental income from godown be charged?
Answer.
Section 60 expressly states that where there is transfer of income from an asset without transfer of the asset
itself, such income shall be included in the total income of the transferor. Hence, the rental income derived
from the godown shall be charged in the hands of Mr. X.
Question 4 (4 Marks)
Mr. X and Mrs. X furnish the following information:
`
(i) Salary income of Mrs. X 4,60,000
(ii) Income of minor son ‘B’ who suffers from disability specified in Section 80U 1,08,000
(iii) Income of minor daughter ‘C' from singing 86,000
(iv) Income from profession of Mr. X 7,50,000
(v) Cash gift received by 'C' on 02.10.2018 from friend of Mrs. X 48,000
(vi) Income of minor married daughter ‘A’ from company deposit 30,000
Compute the total income of Mr. X and Mrs. X for the assessment year 2019-20.
Answer: Computation of Total Income of Mr. X and Mrs. X for the A.Y. 2019-20
Particulars Mr. X Mrs. X
` `
Salaries 4,60,000
Profits and gains of business or profession 7,50,000
Clubbing of Income 315

Income from other sources


Income by way of interest from company
deposit earned by minor daughter A
[Note (iv)] 30,000
Less: Exemption under section 10(32) (1,500) 28,500
Gross Total Income 7,78,500 4,60,000
Less: Deduction u/s 80C to 80U Nil Nil
Total Income 7,78,500 4,60,000
Notes:
(i) The income of a minor child suffering from any disability of the nature specified in section 80U shall not
be included in the hands of the parents.
(ii) The income derived by the minor from manual work or from any activity involving exercise of his skill,
talent or specialized knowledge or experience will not be included in the income of his parent.
(iii) Under section 56, cash gifts received from any person/persons exceeding ` 50,000 during the year in
aggregate are taxable. Since the cash gift in this case does not exceed ` 50,000 the same is not taxable.
(iv) The clubbing provisions are attracted even in respect of income of minor married daughter. The income
of the minor will be included in the income of that parent whose total income is greater. Hence, income of
minor married daughter ‘A’ from company deposit shall be clubbed in the hands of the Mr. X and
exemption under section 10(32) of ` 1,500 per child shall be allowed in respect of such income.
NOV – 2007
Question 4 (3 Marks)
Mr. X has four minor children consisting 2 daughters and 2 sons. The annual income of 2 daughters was
`7,500 and ` 5,000 and of sons was ` 5,500 and ` 1,250 respectively. The daughter who was having income
of ` 5,000 was suffering from a disability specified under section 80U. Work out the amount of income
earned by minor children to be clubbed in the hands of Mr. X.
Answer.
Income earned by minor children to be clubbed with the income of Mr. X
Income of first daughter to be clubbed (7,500 – 1,500 u/s 10(32)) 6,000
Income of second daughter suffering from disability shall not be clubbed Nil
Income of first son to be clubbed (5,500 – 1,500 u/s 10(32)) 4,000
Income of second son to be clubbed (1,250 – 1,250 u/s 10(32)) Nil
Total Income to be clubbed as per section 64(1A) 10,000
The income of daughter suffering from disability specified under section 80U is not to be clubbed with the
income of Mr. X.
NOV – 2005
Question 1 (3 Marks)
Compute the total income of Mr. & Mrs. X from the following information: `
(a) Salary Income (computed) of Mrs. X 2,30,000
(b) Income from profession of Mr. X 3,90,000
(c) Income of minor son A from company deposit 15,000
(d) Income of minor daughter B from special talent 32,000
(e) Interest from bank received by B on fixed deposit made out of her special talent 3,000
(f) Gift received by B on 30.09.2018 from friend of Mrs. X 2,500
Answer: `
Since income of Mr. X is higher, income shall be clubbed in the income of Mr. X
Interest income (minor son A) (15,000 – 1,500) 13,500
Income of minor daughter B from special talent shall not be clubbed Nil
Interest income of minor daughter B (3,000 – 1,500) 1,500
Total income of Mr. X (3,90,000 + 13,500 + 1,500) 4,05,000
Total income of Mrs. X shall be 2,30,000
Clubbing of Income 316

NOV – 2004
Question 1 (3 Marks)
A proprietary business was started by Smt. X in the year 2016. As on 01.04.2017 her capital in business was
`3,00,000. Her husband gifted `2,00,000, on 10.04.2017, which amount Smt. X invested in her business on
the same date. Smt. X earned profits from her proprietary business for the financial years 2017-18,
`1,50,000 and financial year 2018-19 `3,90,000.
Compute the income, to be clubbed in the hands of X’s husband for the assessment year 2018-19 with
reasons.
Answer: `
Capital as on 01.04.2018 (3,00,000 + 2,00,000 + 1,50,000) 6,50,000
Total profits 3,90,000
Amount to be clubbed (3,90,000/6,50,000 x 2,00,000) 1,20,000
Income relating to the amount gifted shall be clubbed and not any subsequent income from the income so
clubbed.
NOV – 1999
Question 1 (4 Marks)
Mr. X is the Karta of a HUF, whose members derive income as given below: `
(i) Income of Mr. X from own business 5,00,000
(ii) Income of Mrs. X under the head Salary 8,00,000
(iii) Minor son Deepak (earning interest on fixed deposits
with bank, which were gifted to him by his grandfather) 15,000
(iv) Deepak got winnings from lottery (gross) 2,00,000
(v) Minor daughter Priya gave a dance performance and received remuneration 1,00,000
Explain how the above will be taxed.
Answer: `
Since Mrs. X has higher income, income of minor children shall be clubbed in the income of Mrs. X and
incomes to be clubbed shall be as given below:
Interest income + Income of lottery of minor son – Deepak (15,000 + 2,00,000) – 1,500 2,13,500
Income of minor daughter Priya shall not be clubbed Nil
Note 1: Income of Priya arises out of an activity involving application of her talent and is therefore not to be
included in the taxable income of her parents [Section 64(1A)].
Note 2: Whether exemption under section 10(32) shall be allowed from casual income or not is
controversial.
NOV – 1998
Question 3 (3 Marks)
Mr. X is a trader. Particulars of his income and those of the members of his family are given below. These
incomes relate to the previous year ended 31st March, 2019: `
(i) Income from business—Mr. X’s 9,00,000
(ii) Salary derived from an educational institution by Mrs. X.
She is the principal of the institution 5,00,000
(iii) Interest on company deposits derived by master Deep Singh
(minor son). These deposits were made in the name of master Deep Singh
by his father’s father about 6 years ago (Gross) 12,000
(iv) Receipts from sale of paintings and drawings made by minor
Dipali Singh (minor daughter and noted child artist) 60,000
(v) Income by way of lottery earnings by Master Dipindar Singh (minor son) 6,000
Discuss whether the above will form part of the assessable income of any individual and also compute the
assessable income of Mr. X.
Answer: Since income of Mr. X is higher, income shall be clubbed in the income of Mr. X and such
incomes shall be
Interest income of master Deep Singh (12,000 – 1,500) 10,500
Income of minor daughter Dipali Singh shall not be clubbed Nil
Clubbing of Income 317

Lottery income of master Dipindar Singh (6,000 – 1,500) 4,500


Total income of Mr. X shall be (9,00,000 + 10,500 + 4,500) 9,15,000
Note: Whether exemption under section 10(32) shall be allowed from casual income or not is controversial.
Income Under The Head Other Sources 318

INCOME UNDER THE HEAD


OTHER SOURCES
SECTION 56 TO 59
PARTICULARS SECTIONS
Income from other sources 56
Deductions while computing income under the head other sources 57
Amounts not deductible 58
Deemed income chargeable to tax 59
Deemed dividend 2(22)
Method of accounting 145

Question 1: What are the incomes taxable under the head Other Sources.
Answer: Incomes taxable under the head Other Sources Section 56
If any income cannot be taxed under first 4 heads, such income shall be taxable under the head other sources
and such income may be
1. Interest income
2. Dividend income
3. Casual income
4. Gift
5. Family pension
6. Payment received under keyman insurance policy to a person who is not an employee
7. Income from owning and maintaining of race horses
8. Forfeiture of advance money
9. Income from undisclosed sources.
10. Any other income which is not taxable under first four heads.
Question 2 [Imp.]: Discuss the deductions allowable under section 57 of the Income Tax Act, 1961, in
respect of Income from Other Sources.
Answer:
Deductions allowable under Section 57
While computing income under the head other sources, expenses incurred in connection with earning of
such income shall be allowed to be deducted.
Amounts not deductible Section 58
While computing income, any personal expense shall not be allowed to be deducted and also in case of
capital expenditure only depreciation shall be allowed.
As per section 58(4), no deduction in respect of any expenditure or allowance shall be allowed in
computing the income by way of any winnings from lotteries, crossword puzzles, races including horse
races, card games and other games of any sort or from gambling or betting of any form or nature,
whatsoever.
e.g. Mr. X purchased lottery tickets of `10,000 and he had a winning of `1,00,000, in this case, his income
shall be considered to be `1,00,000 and expenditure of `10,000 shall not be allowed.

Profits chargeable to tax Section 59


If the assessee has claimed any expenditure while computing income and subsequently he has recovered the
same amount, the amount so recovered shall be considered to be income of the year in which amount has
been recovered.
Income Under The Head Other Sources 319

Question 3: Write a note on Family Pension.


Answer: Family Pension
Regular payments given by the employer to the employee after retirement is called pension and it is taxable
under the head Salary. After the death of the employee, employer may pay some pension to the family
member of the employee and it is called family pension. It is taxable under the head Other Sources but as
per section 57 deduction is allowed equal to 1/3rd of such pension but maximum `15,000 E.g. Mrs. X is
getting family pension of `5,000 p.m., in this case taxable amount shall be (5,000 x 12)minus 1/3 of `60,000
or `15,000 whichever is less i.e. taxable amount shall be `45,000
Illustration 1: Mrs. X is getting family pension of `7,000 p.m. She also has dividend income from domestic
company of `7,00,000. She has long term capital gain of `3,89,000.
Compute her tax liability for assessment year 2019-20.
Solution: ` `
Family Pension 84,000
(7,000 x 12)
Less: Deduction u/s 57 (15,000) 69,000
1/3 of `84,000 or `15,000 whichever is less
Dividend income (exempt u/s 10(34)) Nil
Income under the head Other Sources 69,000
Income under the head Capital Gains
Long term capital gain 3,89,000
Gross Total Income 4,58,000
Less: Deduction u/s 80C to 80U Nil
Total Income 4,58,000
Computation of Tax Liability
Tax on `2,08,000 (`3,89,000 – `1,81,000) @ 20% u/s 112 41,600
Tax on `69,000 at slab rate Nil
Tax before health & education cess 41,600
Add: HEC @ 4% 1,664
Tax Liability 43,264
Rounded off u/s 288B 43,260
Question 4: Write a note on taxability of interest received on late payment of compensation from the
government.
Answer:
As per section 145A, interest received for late payment of compensation from the Government or other
similar agency in connection with compulsory acquisition of land or building shall be taxable in the year in
which it has been received and it will be taxable under the head other sources however, deduction shall be
allowed @ 50% of such interest. e.g. Government has acquired one land of Mr. X in Noida in 2012 and
payment was given by the Government in the year 2018-19 and has also paid interest of `1,00,000, in this
case, taxable amount shall be `1,00,000 – `50,000 = `50,000.
Illustration 2: Interest on enhanced compensation received by Mr. X during the previous year 2018-19 is
`5,00,000. Out of this interest, ` 1,50,000 relates to the previous year 2015-16, `1,65,000 relates to previous
year 2016-17 and `1,85,000 relates to previous year 2017-18. Discuss the tax implication, if any, of such
interest income for A.Y.2019-20.
Solution:
The entire interest of ` 5,00,000 would be taxable in the year of receipt, namely, P.Y.2018-19.
Particulars `
Interest on enhanced compensation taxable u/s 56 5,00,000
Less: Deduction under section 57 @ 50% (2,50,000)
Interest chargeable under the head “Income from other sources” 2,50,000
Income Under The Head Other Sources 320

NOV – 2013 (4 Marks)


State with brief reasoning whether the following receipts are chargeable to income-tax or are exempt (if
chargeable, the amount taxable is to be mentioned) for the assessment year 2019-20:
Nature of receipt Amount (`)
(i) Interest on enhanced compensation received on 12.03.2019 for acquisition of urban 96,000
land, of which 40% relates to the earlier year.
(ii) Rent received for letting out agricultural land for a movie shooting. 72,000
Computation is NOT required.
Solution:
(i) Yes, it is chargeable to tax.
As per section 145A, interest received by the assessee on enhanced compensation shall be deemed to be the
income of the year in which it is received, irrespective of the method of accounting followed by the assessee.
Interest of ` 96,000 on enhanced compensation is chargeable to tax in the year of receipt i.e. P.Y.2018-19
under section 56 after providing deduction of 50% under section 57. Therefore, ` 48,000 is chargeable to tax
under the head “Income from other sources”.
(ii) Yes, it is chargeable to tax.
Agricultural income is exempt from tax as per section 10(1). Agricultural income means, inter alia, any rent
or revenue derived from land which is situated in India and is used for agricultural purposes. In the present
case, rent is being derived from letting out of agricultural land for a movie shoot, which is not an agricultural
purpose. In effect, the land is not being put to use for agricultural purposes. Therefore, ` 72,000, being rent
received from letting out of agricultural land for movie shooting, is not exempt under section 10(1). The
same is chargeable to tax under the head “Income from other sources”.
NOV – 2011 (2 Marks)
On 10.10.2018, Mr. X (a bank employee) received `5,00,000 towards interest on enhanced compensation
from State Government in respect of compulsory acquisition of his land effected during the financial year
2014-15.
Out of this interest, `1,50,000 relates to the financial year 2015-16; `1,65,000 to the financial year 2016-17;
and `1,85,000 to the financial year 2017-18. He incurred `50,000 by way of legal expenses to receive the
interest on such enhanced compensation.
How much of interest on enhanced compensation would be chargeable to tax for the assessment year 2019-
20?
Answer:
Computation of amount chargeable to tax `
Interest on enhanced compensation received 5,00,000.00
Less: Deduction u/s 57 (2,50,000.00)
(5,00,000 x 50/100)
Amount chargeable to tax 2,50,000.00
Question 5 [V. Imp.]: Write a note on taxability of Dividend Income.
Answer: Dividend Income Section 56
Dividend income from the domestic company shall be exempt from tax in the hands of the shareholder as
per section 10(34). (however dividends from a foreign company shall continue to be taxed in the hands of
the shareholder.
As per section 115BBDA, Dividend received by All Assessee except a domestic company or a fund or
institution or trust or any university, exceeding `10 lakh shall be taxable @ 10%. No further deduction or
expenditure shall be allowed from such income.
As per section 115O the domestic company has to pay additional income tax @ 15% + surcharge @ 12% +
HEC @ 4%.
The effective rate shall be 15%
Add: Surcharge @ 12% 1.8%
Total 16.8%
Add: HEC @ 4% 0.672%
Total 17.472%
Income Under The Head Other Sources 321

Example
ABC Ltd. is a domestic company and has total income `80,00,000. It has declared the dividends of
`10,00,000 and one of the shareholders Mr. X gets dividends of `25,000. In this case, tax liability of the
company and Mr. X shall be: `
Total Income 80,00,000.00
Income tax @ 30% 24,00,000.00
Add: HEC @ 4% 96,000.00
Tax Liability 24,96,000.00
Additional Income Tax
(10,00,000 / 82.528% x 17.472%) 2,11,709.97
Rounded off 288B 2,11,710.00
Tax liability of Mr. X shall be nil.
The company has to pay surcharge on additional income tax in every case even if total income is less than
`100,00,000
Alternative Calculation:
Amount of Dividend 10,00,000.00
Additional Income Tax
(10,00,000 / 85% x 15%) 1,76,470.59
Add: Surcharge @ 12% 21,176.47
Tax plus Surcharge 1,97,647.06
Add: HEC @ 4% 7,905.88
Total Amount of AIT 2,05,552.94
Rounded off u/s 288B 2,05,550.00

Illustration 3: ABC Ltd. has current profits of `150 lakhs and the company has distributed dividends of `55
lakhs. Compute income tax liability of the company and that of shareholder.
Calculate additional income tax payable by the company also.
Solution: `
Profit before tax 150,00,000.00
Income tax on `150,00,000 @ 30% 45,00,000.00
Surcharge @ 7% 3,15,000.00
HEC @ 4% 1,92,600.00
Income tax liability 50,07,600.00
Dividend 55,00,000.00
55,00,000 + (55,00,000 / 82.528% x 17.472%) 66,64,404.81
Additional income tax
66,64,404.81 x 17.472% 11,64,404.81
Rounded off u/s 288B 11,64,400.00

Dividend chargeable to tax


If any person has received any dividend from a foreign company such dividends shall be chargeable to tax in
the hands of the shareholder.
MEANING OF DIVIDEND SECTION 2(22)
The term dividend has a very limited meaning under Companies Act but it has a very wide meaning under
Income Tax Act and is called deemed dividend and it is divided into 5 parts:
 Distribution in cash or as assets Section 2(22)(a)
If any company has distributed any amount to its shareholders either in cash or in kind, it will be considered
to be dividend but only to the extent of accumulated profits.
Example
ABC Ltd. has share capital of ` 35 lakhs. The company has general reserve of ` 25 lakhs and has distributed
dividends. One of the shareholders Mr. X has received dividend of ` 27,000 and is holding 2% of the shares.
In this case, entire amount of `27,000 received by him shall be considered to be dividend.
Income Under The Head Other Sources 322

 Issue of bonus shares etc. Section 2(22)(b)


If any company has issued bonus shares to the equity shareholders, it will not be considered to be dividend
but if the bonus shares have been issued to the preference shareholders, it will be considered to be dividend
but to the extent of accumulated profits whether capitalised or not. Further, market value of the bonus shares
shall be taken into consideration.
Example
Mr. X is holding 100 preference share in ABC Ltd. The company has issued him 100 bonus shares and their
market value is `1,200. In this case, it will be considered to be dividend but only to the extent of
accumulated profits whether capitalized or not.
 Distribution on liquidation Section 2(22)(c)
If any company has distributed any amount to its shareholders in connection with its liquidation, it will be
considered to be dividend but only to the extent of accumulated profits and any excess over it shall be
considered to be full value of consideration as per section 46 and capital gains shall be computed
accordingly.
Example
ABC Ltd. has 1,00,000 equity shares of `10 each and the company goes into liquidation on 31.07.2018 and
company has net distributable amount of `60 lakhs after discharging all the liabilities including income tax
and additional income tax and it includes accumulated profits of `20 lakhs and the entire amount was
distributed among the shareholders and Mr. X is holding 10,000 equity shares which were purchased by
him on 01.03.2018 for ` 1,10,000, in this case, tax treatment shall be as given below:
`
Net Distributable Amount 60,00,000
Share of Mr. X (10%) 6,00,000
Share of Mr. X out of accumulated profits which is considered dividend u/s 2(22)(c) (2,00,000)
Balance to be considered full value of consideration 4,00,000
Less: Cost of acquisition of shares (1,10,000)
Short term Capital Gain 2,90,000
Dividend u/s 2(22)(c) shall be exempt u/s 10(34)
Tax liability on ` 2,90,000 at slab rate 2,000
Less: Rebate u/s 87A (2,000)
Tax Liability Nil
MAY – 2008 (5 Marks)
Mr. X purchased 10,000 equity shares of ABC Co. Pvt. Ltd. on 28.02.2018 for ` 1,20,000. The company
was wound up on 31.07.2018. The following is the summarized financial position of the company as on
31.07.2018:
Liability ` Assets `
1,00,000 Equity shares 10,00,000 Land 42,00,000
General reserve 40,00,000 Cash at bank 10,50,000
Provision for taxation 2,50,000
52,50,000 52,50,000
The tax liability (towards dividend distribution tax) was ascertained at `3,00,000. The remaining assets were
distributed to the shareholders in the proportion of their shareholding. The market value of land as on
31.07.2018 is `100,00,000.
The land received above was sold by Mr. X on 28.02.2019 for ` 15,00,000.
Discuss the tax consequences in the hands of the company and Mr. X

Answer.
In the hands of the company:
As per section 46, if any company is in liquidation and it has sold its assets to its shareholders, no capital
gains shall be computed in the hands of the company hence in the given case there is no capital gains for
sale of land to shareholder.
In the hands of Mr. X (shareholder)
Income Under The Head Other Sources 323

Particulars Amount (`)


Mr. X holds 1/10th of the shareholding of the company
Market value of land received (1/10 x 100,00,000) 10,00,000
Cash at bank [1/10th of ` (10,50,000 – ` 3,00,000)] 75,000
10,75,000
Less: Deemed Dividend u/s 2(22)(c) 1/10th of (`40,00,000-` 50,000) (3,95,000)
Consideration for computing Capital Gain 6,80,000
Cost of acquisition of Shares 1,20,000
Short term capital gains 5,60,000
Mr. X has received the land from the company for `10 lakh but it has been sold by him for `15 lakh, in this
case capital gains shall be computed in the manner given below:
Particulars `
Sale consideration 15,00,000
Less : Fair market value of the land on the date of distribution (10,00,000)
Short term capital gain 5,00,000
Dividend u/s 2(22)(c) ` 3,95,000 will be exempt under section 10(34).
 Distribution on reduction of share capital Section 2(22)(d)
Any distribution to its shareholders by a company on the reduction of its capital, to the extent to which the
company possesses accumulated profits e.g. Mr. X is holding 100 shares in ABC Ltd. of `10 each and
company has paid `5 per share in connection with reduction of share capital, in this case amount so received
shall be considered to be dividend but only to the extent of accumulated profits including capitalized profits.
Example
Mr. X is holding 1000 shares of ABC Ltd. of `10 each and company has reduced its share capital and has
refunded `5 per share to the shareholders, the amount so received by the shareholders shall be considered to
be dividend to the extent of accumulated profit.
 Loan and advance by a closely held company Section 2(22)(e)
If any closely held company (also called company in which public are not substantially interested) has given
any loan or advance to an equity shareholder who is holding not less than 10% of the voting power of the
company, in such cases such loan or advance shall be considered to be dividend in the hands of such
shareholder but only to the extent of accumulated profits excluding capitalized profits (however AIT shall be
paid by the company) e.g. ABC Pvt. Ltd. a closely held company has general reserves of `7,00,000 and
current profits of `2,00,000. The company has given a loan of `3,00,000 to one such shareholder Mr. X. in
this case, it will be considered to be dividend in the hands of Mr. X. (however AIT shall be paid by the
company) If loan given by the company is `10,00,000, the amount of dividend shall be `9,00,000.

If the loan or advance has been given to any concern in which such a shareholder has substantial interest,
such loan or advance shall also be considered to be dividend in the hands of such concern but only to the
extent of accumulated profits excluding capitalized profits (however AIT shall be paid by the company).
Example
(i) Mr. X is the beneficial owner of 10% equity shares in ABC Pvt. Ltd. and the company has general
reserve of `10,00,000 and has given a loan of `6,00,000 to a partnership firm XY in which Mr. X is holding
20% shares. In this case, the loan so given shall be considered to be dividend in the hands of partnership
firm (however AIT shall be paid by the company).
(ii) Mr. X is a shareholder in a Company A as well as Company B. He has 10% shareholding in Company A
and 20% shareholding in Company B. The accumulated profits of Company A = `10 lakh. A loan of `12
lakh is given by Company A to Company B.
This loan up to the extent of accumulated profits of ` 10 lakh is treated as dividend and is taxable in the
hands of Company B (however AIT shall be paid by the company).
If the loan or advance has been given to any person on behalf of such a shareholder, it will also be
considered to be dividend.
If any such company has the business of lending as substantial part of its business, in such cases the above
provisions shall not apply e.g. ABC Pvt. Ltd. is a closely held company and is engaged in banking business
(lending of money), in this case section 2(22)(e) is not applicable for ABC Pvt. Ltd.
Income Under The Head Other Sources 324

‘Concern’ means a Hindu Undivided Family, or a firm or an association of persons or a body of individuals
or a company.

If Dividend is covered u/s 2(22)(e), rate of AIT shall be 30% instead of 15%

MAY – 2011 (2 Marks)


Mr. X holding 28% of equity shares in a company took a loan of `5,00,000 from the same company. On the
date of granting the loan, the company had accumulated profit of `4,00,000. The company is engaged in
some manufacturing activity.
(i) Is the amount of loan taxable as deemed dividend in the hands of Mr. X, if the company is a
company in which the public are substantially interested?
(ii) What would be your answer, if the lending company is a private limited company (i.e.) a
company in which the public are not substantially interested?
Answer:
Any payment by a company, other than a company in which the public are substantially interested, of any
sum by way of advance or loan to an equity shareholder, being a person who is the beneficial owner of
shares holding not less than 10% of the voting power, is deemed as dividend under section 2(22)(e), to the
extent the company possesses accumulated profits.
(i) The provisions of section 2(22)(e), however, will not apply where the loan is given by a company in
which public are substantially interested. In such a case, the loan would not be taxable as deemed dividend.
(ii) However, if the loan is taken from a private company (i.e. a company in which the public are not
substantially interested), which is a manufacturing company and not a company where lending of money is a
substantial part of the business of the company, then, the provisions of section 2(22)(e) would be attracted,
since Mr. X holds more than 10% of the equity shares in the company.
The amount chargeable as deemed dividend cannot, however, exceed the accumulated profits held by the
company on the date of giving the loan. Therefore, the amount taxable as deemed dividend would be limited
to the accumulated profit i.e., `4,00,000 and not the amount of loan which is `5,00,000 (however AIT shall
be paid by the company).

Question 6 [V. Imp.]: Write a note on taxability of Casual Income.


Answer: Casual Income Section 56
Under section 2(24)(ix), casual income shall include card games, cross word puzzles, betting, races
including horse races, any game show on electronic media or any other gambling.
While computing income from casual income, as per section 58(4) no expenditure or allowance or
deductions shall be allowed and accordingly the gross receipt itself shall be considered to be income.
Example
Mr. X purchased one lottery ticket of `10,000 and there was a winning of `1,20,000. In this case, his tax
liability shall be `37,440.
 Since no expenditure etc. is allowed hence there cannot be any loss from casual income, accordingly
set off and carry forward is not applicable.
 Casual income shall be taxed at flat rate of 30%.

Question 7. Write a note on taxability of income from Owning and Maintaining of Race Horses.
Answer: Income from Owning and Maintaining of Race Horses Section 56
If any person has income from owning and maintaining of race horses, such income shall be taxable under
the head other sources and income shall be computed in the normal manner and will be taxed at the
normal rates.
As per Section 74A, If any person has any loss from the activities of owning and maintaining race horse,
such loss is not allowed to be set off from any income under any head. However, if the assessee has any
other business of owning and maintaining race horses, loss of one such business can be set off from the
income of other such business.
If the loss can not be set off, it will be allowed to be carried forward, but such carry forward is allowed for a
maximum period of four years and brought forward loss can be set off only from the income of owning and
Income Under The Head Other Sources 325

maintaining race horses.


Income from owning and maintaining of any other animal
If the assessee is engaged in the business of owning and maintaining any other animal, his income shall be
computed under the head business/profession because section 56 includes only income from owning and
maintaining race horses. E.g. Mr. X has income from owning and maintaining of race camels, in this case
income shall be taxable under the head business/profession.
E.g. (i) Mr. X has loss of `5,00,000 from owning and maintaining of race horses and income under the head
house property `5,00,000, in this case loss is not allowed to be setoff, however its carry forward is allowed
for a period of 4 years.
(ii) Mr. X has loss of `2,00,000 from house property and income from owning and maintaining of race
horses `2,00,000, in this case loss is allowed to be setoff.
(iii) Mr. X has loss of `5,00,000 from business/profession and income from owning and maintaining of race
horses `5,00,000, in this case loss is allowed to be setoff.
(iv) Mr. X has loss of `5,00,000 from owning and maintaining of race horses and income under the head
capital gains `5,00,000, in this case loss is not allowed to be setoff, however its carry forward is allowed for
a period of 4 years.
Question 8 [Imp.]: Write a note on taxability of interest income.
Answer: Taxability of interest income Section 56
Any Interest income shall be taxable under the head Other Sources however some of the interest incomes
shall be exempt from income tax under section 10(15) and are as given below:
1. Interest on Capital Investment Bonds issued by the Government.
2. Interest on Relief Bonds issued by RBI.
3. Interest on Post Office Savings Bank Account to the extent of `3,500 per year and in the case of joint
account, exemption shall be allowed upto `7,000 per year.
4. Any other interest income notified under section 10(15).
Question 9: Explain taxability of Interest or dividend income from UTI or Mutual Funds.
Answer: Interest or dividend income from UTI or Mutual Funds Section 10(35)
If any person has received any interest or dividend from the UTI or Mutual Fund notified under section
10(23D), such income is exempt from income tax.
If UTI or Mutual Funds have distributed any interest or dividend, as per section 115R, UTI, Mutual Funds,
have to pay additional income tax (Corporate dividend tax) at the rate of 25% plus surcharge @ 12% plus
HEC @ 4%, if the amount is distributed to individual or Hindu Undivided Family.
If amount is distributed to any other person, rate shall be 30% plus surcharge @ 12% plus HEC @ 4%.

Question 10: Explain taxability of income from letting out of building alongwith furniture, fixtures
etc.
Answer: If any person has let out any building alongwith plant and machinery and furniture, fixtures etc.
and it is not a case of composite rent and also income is not taxable under the head business/profession, in
such cases income shall be taxable under the head Other sources and while computing income all expenses
incurred shall be allowed to be deducted e.g. Mr. X has one factory building along with machines and
furniture in Bombay which has been let out @ `50,000 p.m. Repair charges of the building is `7,000 and
that of furniture fixtures are `4,000, insurance premium paid `3,000 and depreciation is `27,000, in this case
income shall be computed in the manner given below:
Solution: `
Gross Rent (50,000 x 12) 6,00,000
Less: Repair of building (7,000)
Less: Repair of Furniture and fixtures (4,000)
Less: Insurance premium (3,000)
Less: Depreciation (27,000)
Income under the head Other Sources 5,59,000
Income Under The Head Other Sources 326

Question 11: Write a note on Books of Accounts.


Answer: Books of Accounts Section 145
A person is not required to maintain any books of accounts under the head salary or house property or
capital gains and income has to be computed as per the procedure given in the relevant head.
Books of accounts are required under the head Business/Profession and under the head Other Sources. An
assessee has the option to maintain books of accounts either on the basis of mercantile system of accounting
or on cash basis. Any system of accounting once adopted has to be followed consistently, however it can be
changed with the permission of Assessing Officer.
Example
Mr. X has deposited `10,00,000 in ABC Ltd. @ 10% p.a. and interest income is due on yearly basis on 31st
March every year. Interest income which was due on 31.03.2019 was received on 01.04.2019. In this case, if
the assessee is maintaining books of account on the basis of mercantile system of accounting, income is
taxable in previous year 2018-19, and if the books are maintained on cash basis, income is taxable in the
previous year 2019-20.
Question 12: Write a note on payment under keyman insurance policy.
Answer: Payment under Keyman Insurance Policy
Sometimes employer may take a life policy in the name of any of his employees who are considered to be
very important for business or profession and such policy is called keyman insurance policy and premium is
paid by employer and employer is allowed to debit it to profit and loss account and amount received on
maturity shall be considered to be income of employer as per section 28.
If any payment has been received by the employee, it will be considered to be income under the head salary.
Similarly a policy may be taken in the name of any other person who is considered to be very important for
the business of the employer, such policy is also called keyman insurance policy. If payment has been
received by such other person, it will be considered to be his income under the head other sources as per
section 56.
Question 13: Write a note on forfeiture of advance money.
Answer: Forfeiture of advance money
If any person has entered into an agreement to sell any capital asset and some advance money was received
but the buyer refused to purchase the capital asset and advance money was forfeited, in such cases the
amount so forfeited shall be considered to be income under the head Other Sources. e.g. Mr. X has entered
into agreement to sell a house property for `50 lakh to Mr. Y and advance money of `5,00,000 was received
but Mr. Y refused to purchase the property and advance money was forfeited, in this case `5,00,000 shall be
considered to be income of Mr. X under the head Other Sources.
Question 14 [Imp.]: Write short note on Set Off and Carry Forward of loss arising under the head
“Income from Other Sources”.
Answer: Set Off and Carry Forward of Losses under the head “Income from Other Sources”
As per section 70, if there is loss in one source under the head other sources, such loss can be set off from
income of any other source under the same head.
However, as per section 58(4), no deduction or set off shall be allowed from the income by way of any
winnings from lotteries, crossword puzzles, races including horse races, card games, and other games of any
sort or from gambling or betting of any form whatsoever.
As per section 71, if the loss can not be set off under the same head, it can be set off from the incomes of
other heads.
If the loss can not be set off even from the incomes of other heads, its carry forward is not allowed.
e.g. (i) Mr. X has loss under the head other sources `2,00,000 and income under the head other sources
`5,00,000, in this case loss is allowed to be setoff.
(ii) Mr. X has loss under the head other sources `2,00,000 and income under the head house property
`5,00,000, in this case loss is allowed to be setoff.
(iii) Mr. X has loss under the head other sources `2,00,000 and income from owning and maintaining of race
horses `5,00,000, in this case loss is allowed to be setoff.
(iv) Mr. X has loss under the head other sources `2,00,000 but do not have income under any other head, in
this case carry forward of loss is not allowed.
Illustration 4: Mr. X has taken a loan of `1,00,000 @ 10%. The amount was invested by him in the
Income Under The Head Other Sources 327

securities of one company. During the year he has received gross interest of `18,000 and has paid collection
charges to the bank `500. He has paid interest `10,000 on the loan taken by him for investment and has long
term capital gain of `2,80,000.
Compute his tax liability for assessment year 2019-20.
Solution: `
Gross Interest 18,000
Less:
(i) Bank Charges u/s 57 (500)
(ii) Interest paid for borrowing the amount u/s 57 (10,000)
Income under the head Other Sources 7,500
Income under the head Capital Gains (LTCG) 2,80,000
Total Income 2,87,500
Computation of Tax Liability
Tax on `37,500 (`2,80,000 – `2,42,500) @ 20% u/s 112 7,500
Tax on `7,500 at slab rate Nil
Less: Rebate u/s 87A (2,500)
Tax before health & education cess 5,000
Add: HEC @ 4% 200
Tax Liability 5,200
Illustration 5 (From RTP): Mr. X reports the following transactions to you:
(i) Received cash gifts on the occasion of his marriage on 15.06.2018 of ` 1,08,000. It includes gift of
`28,000 received from non-relatives.’
(ii) On 15.08.2018, being his birthday, he received a gift by means of cheque from his mother's maternal
aunt for an amount of ` 49,000.
(iii) On 25.12.2018 he acquired a vacant site from his friend for ` 1,50,000. The State stamp valuation
authority fixed the value of site at ` 2,25,000 for stamp duty purpose.
(iv) He bought 200 equity shares of a listed company from another friend for ` 75,000.
The value of shares in the stock exchange on the date of purchase was ` 1,75,000.
(v) A cell phone worth ` 21,000 is gifted by his friend on 16.8.2018
Determine the amount chargeable to tax in the hands of Mr. X for the Assessment Year 2019-20.
Your answer should be supported with reasons.
Solution:
Computation of amount chargeable to tax in hands of Mr. X for A.Y. 2019-20
Particulars `
(i) Cash gift of ` 1,08,000 received on the occasion of his marriage is not taxable since gifts Nil
received by an individual on the occasion of marriage are excluded under section 56, even
if the same are from non-relatives.
(ii) Even though mother’s maternal aunt does not fall within the definition of “relative” under Nil
section 56, gift of ` 49,000 received from her by cheque is not chargeable to tax since the
aggregate sum of money received by Mr. X without consideration from non-relatives
(other than on the occasion of marriage) during the previous year 2018-19 does not exceed
` 50,000.
(iii) Purchase of land for inadequate consideration on 25.12.2018 would attract the provisions 75,000
of section 56. Where any immovable property is received for a consideration which is less
than the stamp duty value of the property by an amount exceeding `50,000, the difference
between the stamp duty value and consideration is chargeable to tax in the hands of the
individual. Therefore, in the given case `75,000 is taxable in the hands of Mr. X.
(Difference amount is more than `50,000 and more than 5% of the consideration)
(iv) Since shares are included in the definition of “property” and difference between the 1,00,000
purchase price and fair market value of shares is ` 1,00,000 (` 1,75,000 – ` 75,000) i.e. it
exceeds ` 50,000, the difference would be taxable under section 56.
(v) Cell phone is not included in the definition of “property” as per section 56. Hence, it is not Nil
taxable.
Income Under The Head Other Sources 328

Amount chargeable to tax 1,75,000


Question 15: Write a note on income of closely held company by issue of Shares.
Answer: As per Section 56(2) (viib), if any closely held company has issued shares at a value exceeding
the market value, in such cases excess over market value shall be considered to be income of closely held
company under the head other sources. E.g. ABC Private Limited is a closely held company and market
value of its shares is `20 per share but company has issued share at a rate of `100 per share, in this case `80
shall be considered to be income of the company.

Question 16: Write a note on Bond Washing Transactions.


Answer: Bond Washing Transactions Section 94
If any person has transferred any security in the name of any other person sometimes before the due date and
has reacquired it sometimes after the due date in order to evade tax, it will be considered to be a bond
washing transaction and income shall be considered to be of the person who has manipulated in this manner.
Example
Mr. Yuvraj Arora has purchased security of `10,00,000 in ABC Ltd. on 01.04.2018 @ 10% and interest is
due on half yearly basis i.e. on 30th Sept and 31st March of every year. If Mr. Yuvraj Arora has transferred
this security just before the due date in the name of any other person through a fictitious sale transaction and
has re-transferred it in his name after the due date through a fictitious purchase transaction so that he can
evade tax, it will be called bond washing transaction and in such cases interest income is taxable in the
hands of Mr. Yuvraj Arora.
[This practice is generally adopted by high-income class assessees to evade the tax by transferring securities
to low income class assessee on the eve of due date of payment of interest.]

Question 17: Write a note on Dividend Stripping.


Answer: Dividend Stripping Section 94
As per section 94, if any person buys any securities or units within a period of three months prior to the
record date and such person sells such securities within a period of three months after such date, or such
units within a period of nine months after such date and the dividend or income on such securities or units
received or receivable by such person is exempted, then, the loss, if any, arising therefrom shall not be
allowed to be setoff or carried forward but if such loss is more than the amount of income, in that case
excess over such income shall be allowed to be setoff or carried forward.
Example: Mr. X purchased equity shares of `1,00,000 of ABC limited on 01.11.2018 and company declared
dividend of `10,000 and record date is 31.12.2018. Mr. X sold the shares on 10.11.2018 to Mr. Y for
`1,08,000 and Mr. Y received dividend of `10,000 from the company on 31.12.2018. He sold the shares on
10.01.2019 for `1,01,000, in this case loss of `7,000 shall not be allowed to be setoff or carried forward.
Income Under The Head Other Sources 329

PRACTICE PROBLEMS
TOTAL PROBLEMS 6
Problem 1.
Mr. X has income from business of owning and maintaining race camels `60,000, loss from owning and
maintaining race horses `7,000 and income from horse races `7,000. He has brought forward business loss
of `7,000 of the assessment year 2006-07 and brought forward business loss of `7,000 of the assessment
year 2014-15.
Compute his tax liability for the assessment year 2019-20.
Answer = Total Income: `60,000; Tax Liability: Nil; Carry forward loss from owning and maintaining race
horses: `7,000

Problem 2.
Mr. X has income from owning and maintaining of race horses ` 4 lakhs and loss from horse races ` 10
lakhs.
Determine his tax liability for the assessment year 2019-20.
Answer = Tax Liability: `7,800

Problem 3.
Mr. X has loss from owning and maintaining of race horses `4 lakhs and income from owning and
maintaining of race camels `4 lakhs.
Determine his tax liability for the assessment year 2019-20.
Answer = Tax `7,800; Carry forward loss from owning and maintaining of race horses: `4,00,000

Problem 4.
Find the tax liability of Mrs. X (age 40 years), a resident individual, for the assessment year 2019-20. From
the following particulars of her incomes and spending for the previous year ending March 31st, 2019.
`
Income from house property (Computed) 90,000
Dividend from UTI 35,000
Family pension (gross) 90,000
Interest on bank FD (gross) 14,000
Dividend from foreign company 36,000
Gift received from her sister 26,000
Winnings from lotteries (gross) 70,000
Long-term capital gain 1,20,000
Payment for purchase of National Savings Certificates 35,000
Answer = Tax Liability: `32,240

Problem 5.
Mr. X has submitted information given below.
i) Income from owning and maintaining of race horse `2,00,000.
ii) Income from owning and maintaining of race camels `1,00,000.
iii) He had winning of `1,60,000 from horse race on 01.12.2018 and winning from camel race
`1,80,000 on 07.12.2018.
iv) He purchased lottery tickets of `10,000 on 01.02.2019 and had winning of `2,00,000 on
12.02.2019.
v) He has received Royalty of book of literary nature @ 50% of print price of ` 600 and total copies
sold are 2,000
vi) He has paid advance tax as given below:
Upto 15.06.2018 ` 20,000
Upto 15.09.2018 ` 35,000
Income Under The Head Other Sources 330

Upto 15.12.2018 ` 80,000


Upto 15.03.2019 ` 1,30,000
Balance was paid on 10.06.2019
Compute tax liability for the A.Y 2019-20 and interest under section 234A, 234B and 234C.
Answer = Tax Liability: `2,27,240; Interest under section 234A: Nil; 234B: `2,916; 234C: `2,280
Problem 6.
ABC Ltd. has 1,00,000 equity shares of `10 each and Mr. X purchased 10,000 equity shares on 01.01.2018
of `10 each and the company goes into liquidation on 31.07.2018 and company has net distributable amount
of `60 lakhs after discharging all the liabilities including additional income tax and it includes accumulated
profits of ` 20 lakhs and the entire amount was distributed among the shareholders.
Minor son of Mr. X has interest income of `2 lakhs from one bank deposit which was gifted to him by his
grand father.
Mrs. X has one business and income from business is `1 lakh entire capital was gifted by Mr. X.
Mr. X is growing flowers and has income of ` 7 lakh from sale of flowers.
Compute his tax liability for Assessment Year 2019-20.
Answer = Tax Liability: `1,03,530
Income Under The Head Other Sources 331

SOLUTIONS
TO
PRACTICE PROBLEMS
Solution 1: `
Income under the head Business/Profession 60,000
Less: Brought forward business loss (7,000)
Income under the head Business/Profession 53,000
Income under the head Other Sources (horse races) 7,000
Gross Total Income 60,000
Less: Deductions u/s 80C to 80U Nil
Total Income 60,000
Computation of tax liability
Tax on `7,000 @ 30% 2,100
Tax on `53,000 at slab rate Nil
Less: Rebate u/s 87A (2,100)
Tax Liability Nil
Carry forward loss from owning and maintaining race horses 7,000

Solution 2: `
Income under the head Other Sources 4,00,000
Gross Total Income 4,00,000
Less: Deductions u/s 80C to 80U Nil
Total Income 4,00,000
Tax on `4,00,000 at slab rate 7,500
Add: HEC @ 4% 300
Tax Liability 7,800
Note: Loss from casual income has no tax treatment and hence it is dead loss.

Solution 3: `
Income under the head Business/Profession 4,00,000
Gross Total Income 4,00,000
Less: Deductions u/s 80C to 80U Nil
Total Income 4,00,000
Tax on `4,00,000 at slab rate 7,500
Add: HEC @ 4% 300
Tax Liability 7,800
Carry forward loss from owning and maintaining race horses 4,00,000

Solution 4: `
Income from House Property 90,000
Computation of income under the head Other Sources
Dividend from UTI {exempt u/s 10(35)} Nil
Pension 75,000
Working Note:
Received = `90,000
Income Under The Head Other Sources 332

Exempt = 1/3 of pension or `15,000,whichever is less


Taxable = `75,000
Interest on bank FD 14,000
Dividend from foreign company 36,000
Winning from lottery 70,000
Income under the head Other Sources 1,95,000
Income under the head Capital Gains (LTCG) 1,20,000
Gross Total Income 4,05,000
Less: Deduction u/s 80C (35,000)
Total Income 3,70,000
Computation of Tax Liability
Tax on Long term capital gain `50,000 (1,20,000 – 70,000) @ 20% u/s 112 10,000
Tax on casual income `70,000 @ 30% u/s 115BB 21,000
Tax on `1,80,000 at slab rate Nil
Tax before health & education cess 31,000
Add: HEC @ 4% 1,240
Tax Liability 32,240

Solution 5:
Computation of Total Income for the A.Y 2019-20 `
Income under head Other Source
Income from owning and maintaining race horse 2,00,000
Income from Royalty 6,00,000
Income from winning horse race (casual income) 1,60,000
Income from winning camel race (casual income) 1,80,000
Income from lottery income (casual income) 2,00,000
Income under head Other Sources 13,40,000
Income under head Business/Profession
Income from owning and maintaining race camel 1,00,000
Gross Total Income 14,40,000
Less: Deduction u/s 80QQB (WN 1) (1,80,000)
Total Income 12,60,000
Computation of Tax Liability
Tax on `7,20,000 at slab rate 56,500
Tax on casual income i.e. `5,40,000 @ 30% 1,62,000
Tax before health & education cess 2,18,500
Add: HEC @ 4% 8,740
Tax Liability 2,27,240
Tax Liability excluding amount of casual income
Tax on ` 7,20,000 at slab rate 56,500
Add: HEC @ 4% 2,260
Total 58,760
Tax Liability including amount of casual income upto 15.12.2018
Tax on ` 7,20,000 at slab rate 56,500
Tax on casual income i.e. ` 3,40,000 @ 30% 1,02,000
Tax before health & education cess 1,58,500
Add: HEC @ 4% 6,340
Total 1,64,840
Interest u/s 234A
Nil
Interest u/s 234B
2,27,240 – 1,30,000= 97,240 = 97,200 x 1% x 3 2,916
Income Under The Head Other Sources 333

Interest u/s 234C


Date Advance tax paid Amount payable Shortfall
upto 15.06.2018 20,000 8,814 (58,760 x 15%) NIL
Interest = NIL

upto 15.09.2018 35,000 26,442 (58,760 x 45%) NIL


Interest = NIL

upto 15.12.2018 80,000 1,23,360 (1,64,840 x 75%) 43,630


Rounded off 119A = 43,600
Interest = 43,600 x 1% x 3 = 1,308

upto 15.03.2019 1,30,000 2,27,240 97,240


Rounded off 119A = 97,200
Interest = 97,200 x 1% x 1 = 972

Interest u/s 234C 2,280


Total Interest Payable 5,196
Rounded off u/s 288B 5,200

Working Notes:
1. Maximum deduction allowed u/s 80QQB
15% of print price i.e. ` 600 x 15% x 2,000= `1,80,000.
Solution 6: `
Net Distributable Amount 60,00,000
Share of Mr. X (10%) 6,00,000
Share of Mr. X out of accumulated profits which is considered dividend u/s 2(22)(c) (2,00,000)
Balance to be considered full value of consideration 4,00,000
Less: Cost of acquisition of shares (1,00,000)
Short term Capital Gain 3,00,000
Dividend u/s 2(22)(c) shall be exempt u/s 10(34)
Computation of Total Income of Mr. X for A.Y. 2019-20
Income under the head Capital Gains
Short Term Capital Gains 3,00,000
Income under the head Other Sources
Interest Income of Minor Son clubbed u/s 64(1A) 2,00,000
Less: Exempt u/s 10(32) 1,500 1,98,500
Income under the head Other Sources 1,98,500
Income under the head Business Profession
Income of Mrs. X clubbed u/s 64(1) 1,00,000
Gross Total Income 5,98,500
Less: Deduction u/s 80C to 80U Nil
Total Income 5,98,500
Agricultural Income 7,00,000
Computation of Tax Liability
Normal income 5,98,500
Step 1. Tax on (5,98,500 + 7,00,000) at slab rate 2,02,050.00
Step 2. Tax on (`2,50,000 + 7,00,000) at slab rates (1,02,500.00)
Step 3. Deduct Tax at Step 2 from Tax at Step 1 99,550.00
Add: HEC @ 4% 3,982.00
Tax Liability 1,03,532.00
Rounded off u/s 288B 1,03,530.00
Income Under The Head Other Sources 334

EXAMINATION QUESTIONS
NOV – 2016
Question 3(a) (4 Marks)
Mr. Rakesh has 15% share holding in RSL (P) Ltd and has also 50% share in Rakesh & Sons, a partnership
firm.
The accumulated profit of RSL (P) Ltd. is 20 Lakh. Rakesh & Sons had taken a loan of `25 Lakh, from RSL
(P) Ltd. Explain, whether the above loan is treated as dividend, as per the provision of Income Tax Act,
1961.
Solution:
As per Section 2(22)(e), If the loan or advance has been given to any concern in which shareholder has
substantial interest, such loan or advance shall be considered to be dividend in the hands of such concern but
only to the extent of accumulated profits excluding capitalized profits. (however AIT shall be paid by the
company).
In this case dividend in the hands of the shareholder is nil and in hands of the firm are `20 lakhs. (however
AIT shall be paid by the company).

Question 4(a) (2 Marks)


State with reasons whether the following receipts are taxable or not under the provisions of Income-tax Act,
1961?
Mr. Suman received an advance of `3 lakhs on 06-06-2018 to transfer his residential house property. Since
the transfer was not effected during the previous year due to failure in negotiations, he deducted the advance
money forfeited from the cost of acquisition of the property.
Answer:
As per section 56, if any person has entered into an agreement to sell any capital asset and some advance
money was received but the buyer refused to purchase the capital asset and advance money was forfeited, in
such cases the amount so forfeited shall be considered to be income under the head Other Sources. So
Forfeiture of advance money shall be considered as income from other source and shall not be deducted
from cost of acquisition of the property.

MAY – 2016
Question 4(a) (2 x 2 = 4 Marks)
Discuss the taxability or otherwise in the hands of the recipients, as per the provisions of the Income-tax
Act, 1961:
(i) ABC Private Limited, a closely held company, issued 10,000 share at `130 per share. (The face
value of the share is `100 per share and the fair market value of the share is `120 per share).
(ii) Mr. A received an advance of `50,000 on 01.09.2018 against the sale of his house. However, due to
non-payment of instalment in time, the contract has cancelled and the amount of `50,000 was
forfeited.
Answer:
(i) Taxable: As per section 56(2)(viib), If any closely held company receives any consideration for issue of
shares that exceeds the face value of shares then the aggregate consideration received as exceeds the fair
market value of shares is considered as income under the other sources. In the given case, ABC Private
Ltd. issued shares at a price which exceeds the face value of shares. So the taxable amount shall be
`1,00,000 (10,000 shares x (`130-`120))
(ii) Taxable: If any person has entered into an agreement to sell any capital asset and some advance money
was received but the buyer refused to purchase the capital asset and advance money was forfeited, in such
cases the amount so forfeited shall be considered to be income under the head Other Sources. In the given
case, Mr. A forfeited `50,000 against sale of his house shall be considered as income under the head other
sources.
Income Under The Head Other Sources 335

Question 7(a)(iii) (2 Marks)


Discuss with reason, whether the following transactions are true or false, as per the provisions of Income
Tax Act, 1961:
Dividend received (on which no Dividend Distribution Tax has been paid) by a dealer in shares or one
engaged in buying/selling of shares, is chargeable under the head “Income from other sources”. (Discussion
must be on the head of income).
Answer:
True: Dividend received by a dealer of shares is chargeable under the head “Income from Other Sources”.

NOV – 2010
Question 7 (4 Marks)
State under which heads the following incomes are taxable:
(i) Rental income in case of dealer in property
(ii) Dividend on shares in case of a dealer in shares
(iii) Salary by a partner from his partnership firm
(iv) Rental income of machinery
(v) Winnings from lotteries by a person having the same as business activity
(vi) Salaries payable to a Member of Parliament
(vii) Receipts without consideration
Answer.
Particulars Head of Income
(i) Rental income in case of dealer in property Income from house property
(ii) Dividend on shares in case of a dealer in shares Income from other sources
(iii) Salary by partner from his partnership firm Profit and gains of business or
profession
(iv) Rental income of machinery (See Note below) Income from other sources/ Profits and
gains of business or profession
(v) Winnings from lotteries by a person having the same as Income from other sources
business activity
(vi) Salaries payable to a Member of Parliament Income from other sources
(vii) Receipts without consideration Income from other sources
Note: As per section 56, rental income of machinery would be chargeable to tax under the head “Income
from Other Sources”, if the same is not chargeable to income-tax under the head “Profits and gains of
business or profession”.
Deduction Of Tax At Source 336

DEDUCTION OF TAX AT SOURCE


SECTION 190 TO 206AA
 Section 190 – Deduction at source and advance payment
 Section 191 – Direct payment
 Section 192 – Deduction of tax from salary income
 Section 192A – Deduction of tax at source in case of payment from recognized provident fund
 Section 193 – Deduction of tax from interest on securities
 Section 194A – Deduction of tax from interest other than “interest on securities”
 Section 194B – Deduction of tax from winnings from lottery, crossword puzzle, card game, etc.
 Section 194BB – Deduction of tax from winnings from horse races
 Section 194C – Deduction of tax from payment to contractors (V. Imp.)
 Section 194D – Deduction of tax from insurance commission
 Section 194DA – Payment in respect of life insurance policy
 Section 194E – Payments to non-resident sportsmen or sports associations
 Section 194G – Deduction of tax from commission, etc. on sale of lottery tickets
 Section 194H – Commission or brokerage
 Section 194-I – Deduction of tax from rent (V. Imp.)
 Section 194-IA – Payment for purchase of immovable property
 Section 194-IB – Payment of rent by certain individuals or HUF.
 Section 194-IC – Payment under specified agreement.
 Section 194J – Deduction of tax from fees for professional or technical services
 Section 194LA – Payment of compensation on acquisition of certain immovable property
 Section 195 – Payments to non-residents
 Section 197 – Certificate for deduction at lower rate (V. Imp.)
 Section 197A – Declaration for not Deducting Tax at Source
 Section 200 – Duty of person deducting tax (V. Imp.)
 Section 201 – Consequences of failure to deduct or pay (V. Imp.)
 Section 203 – Certificate for tax deducted
 Section 203A – Tax deduction account number
 Section 203AA – Furnishing of annual statement of tax deducted
 Section 206AA – Requirements to furnish Permanent Account Number
 Section 206C – Collection of Tax Source
Deduction Of Tax At Source 337

Deduction at Source and Advance Payment Section 190


Every person shall be required to pay tax through TDS and advance tax even if exact tax is to be computed
in the assessment year.
Direct Payment Section 191
If tax is not to be deducted at source with regard to any income, assessee shall pay advance tax. Similarly if
tax was to be deducted at source but it has not been deducted at source, in such cases also the assessee is
required to pay advance tax.

Question 1: Write a note on Deduction of Tax at Source with regard to Salary Income.
Answer: Deduction of Tax at Source with regard to Salary Income Section 192
1. Every person (including individual and HUF even if limit prescribed under section 44AB has not
exceeded in the preceding year) making payment of salary income to resident or non-resident shall deduct
tax at source and for this purpose the employer shall estimate tax liability of the employee and tax so
estimated shall be deducted in 12 monthly equal installments. While estimating tax liability, deduction under
section 80C to 80U shall be allowed. It can be shown in the manner given below:
Mr. X is employed in ABC Ltd. and salary is `50,000 p.m. and he has invested `50,000 in NSC. In this case,
tax to be deducted at source at the time of payment of salary shall be:
`
Gross Salary (50,000 x 12) 6,00,000.00
Less: Standard Deduction u/s 16(ia) (40,000.00)
Income under the head Salary 5,60,000.00
Gross Total Income 5,60,000.00
Less: Deduction u/s 80C {NSC} (50,000.00)
Total Income 5,10,000.00
Tax on `5,10,000 at slab rate 14,500.00
Add: HEC @ 4% 580.00
Tax Liability 15,080.00
Monthly installment shall be 15,080 / 12 1,256.67
If employer has deducted tax at source for the month of April and May and salary was increased to `70,000
p.m. w.e.f. 01.06.2018, tax to be deducted in subsequent installments shall be
Gross Salary (50,000 x 2) + (70,000 x 10) 8,00,000.00
Less: Standard Deduction u/s 16(ia) (40,000.00)
Income under the head Salary 7,60,000.00
Gross Total Income 7,60,000.00
Less: Deduction u/s 80C {NSC} (50,000.00)
Total Income 7,10,000.00
Tax at slab rate including HEC 56,680.00
Tax deducted at source in April and May (1,256.67 x 2) (2,513.34)
Balance amount of tax 54,166.66
Tax to be deducted in subsequent installments (54,166.66 / 10) 5,416.67

2. If any person is working with two or more employers, in that case he should submit the particulars of his
salary income from all the employers to one of the employer who will deduct tax at source taking into
consideration income from all employers. (Information has to be given in Form 12B)

Example
Mr. X is working with two employer A Ltd. and B Ltd. and is getting basic pay of `15,000 p.m. from each
of the employer. In this case, he must inform one of the employer regarding his salary income from other
employer and such employer shall deduct tax at source taking into consideration income from other
employer.

3. If any employee has income/loss under any other head, the employee shall be allowed even to report
such incomes/loss to the employer and the employer shall take it into consideration. The employee shall be
Deduction Of Tax At Source 338

required to give proof.

Question 2: Write a note on deduction of tax at source in case of payment from recognized provident
fund.
Answer: Deduction of tax at source in case of payment from recognized provident fund Section 192A
The person responsible for making payment of recognized provident fund to any person shall deduct tax
at source if the amount to be paid is taxable and tax shall be deducted at source @ 10% provided the amount
paid or payable during a particular year is `50,000 or more.

Payments received from recognised provident fund shall be exempt from income tax if the employee has
complied with any of the conditions given below:
(i) If the employee has rendered continuous service for a period of 5 years or more, or
(ii) If he has not rendered such continuous service, the service has been terminated by reason of the
employee’s ill-health, or by the contraction or discontinuance of the employer’s business or other cause
beyond the control of the employee, or
(iii) If the employee obtains employment with any other employer and the provident fund has been
transferred to such employer and the total service with the former employer and the current employer is
of 5 years or more.
If the employee has not complied with even a single condition, in that case amount received by him shall be
taxable but only the amount which was exempt earlier.
(To be discussed under the head Salary)

Question 3: Write a note on TDS in case of interest on securities.


Answer: TDS in case of interest on securities Section 193
Every person responsible for making payment of interest on securities to any resident shall deduct tax at
source @ 10% provided the amount being paid or payable during a particular year to a particular person is
exceeding `5,000.
No tax shall be deducted at source in the following cases:
1. Any interest payable on any security of the Central Government or a State Government.
2. Any interest being paid to Bank/LIC or other notified financial organizations.
3. Interest payable by a company in connection with security held in dematerialised form.
“Interest on securities” as per section 2(28B), interest on securities means interest on bond / debenture etc.
issued by Government / local authority / company or statutory corporation etc.
Example
(i) ABC Ltd. has to pay interest of `2,00,000 to Mr. X in connection with listed debentures, amount of TDS
shall be ` 20,000.
(ii) ABC Ltd. has to pay interest of `12,00,000 to Mr. X in connection with listed debentures, amount of
TDS shall be `1,20,000
(iii) ABC Ltd. has to pay interest of `5,000 to Mr. X, no tax shall be deducted at source in this case.
Illustration 1: Mr. X has invested some amount in ABC Ltd. and the company has paid him interest of
`2,70,000 after deducting tax at source @ 10%. The cheque was collected by the bank and the bank charges
were 1%. He has also received dividend from domestic company of `65,000 and bank charges are 1%.
Compute his tax liability and tax refund for assessment year 2019-20.
Solution: `
Gross interest (2,70,000 x 100 /90) 3,00,000.00
Less: bank charges u/s 57 (1% of 2,70,000) (2,700.00)
Income under the head Other Sources 2,97,300.00
Total Income 2,97,300.00
Computation of Tax Liability
Tax on `2,97,300 at slab rate 2,365.00
Less: Rebate u/s 87A (2,365.00)
Tax Liability Nil
Less: TDS (30,000.00)
Deduction Of Tax At Source 339

Refund 30,000.00

Illustration 2: Mr. X has borrowed `1,00,000 from the market. The amount was invested in some company
and the assessee has received a cheque for ` 36,000 (after TDS @ 10%) being the amount of interest and
assessee has paid interest of ` 11,000.
The cheque was given for collection to a bank and the bank has deducted collection charges of 2%.
Mr. X has income under the head house property ` 2,50,000.
Compute his tax liability / tax payable for assessment year 2019-20.
Solution: `
Income under the head House Property 2,50,000.00
Income under the head other sources 28,280.00
(36,000 x 100 / 90) -11,000-720}
(40,000 – 11,000 – 720)
Gross Total Income 2,78,280.00
Less: Deduction u/s 80C to 80U Nil
Total Income 2,78,280.00
Computation of Tax Liability
Tax on `2,78,280 at slab rate 1,414.00
Less: Rebate u/s 87A (1,414.00)
Tax Liability Nil
Less: TDS (4,000.00)
Tax Refund 4,000.00
Question 4: Write a note on TDS in case of Interest other than “Interest on Securities”.
Answer: TDS in case of Interest other than “Interest on Securities” Section 194A
Every person making payment of interest other than interest on securities to any resident shall deduct tax at
source @ 10% provided the amount being paid or payable during a particular year to a particular person is
exceeding `5,000 but if payment is being made by bank or post office, tax shall be deducted only if interest
being paid or payable is exceeding `10,000, however if the payee is senior citizen, `10,000 shall be taken as
`50,000 .
Further TDS shall be only on time deposit including recurring deposit. Limit of `10,000 (`50,000 for senior
citizen) shall be per branch of the bank but if the bank has core banking solution, limit shall be per bank and
not per branch.
An Individual or Hindu Undivided Family shall be required to deduct tax at source only if the turnover of
business or profession has exceeded the limit mentioned under section 44AB during the financial year
immediately preceding the relevant year.
Example
(i) Punjab National Bank has to pay interest of `1,00,000 to Mr. X. In this case, amount of TDS shall be
`10,000.
(ii) Punjab National Bank has to pay interest of `10,00,000 to Mr. X. In this case, amount of TDS shall be
`1,00,000.
(iii) Punjab National Bank has to pay interest of `1,00,000 to X Ltd. In this case, amount of TDS shall be
`10,000.
(iv) Punjab National Bank has to pay interest of `1,00,000 to an Hindu Undivided Family. In this case,
amount of TDS shall be `10,000.
(v) Punjab National Bank has to pay interest of `1,000 to a Hindu Undivided Family. In this case, amount of
TDS shall be Nil.
(vi) Punjab National Bank has to pay interest of `9,900 to Mr. X. In this case, amount of TDS shall be Nil.
No tax shall be deducted at source in the following cases:
(1) Interest paid by a firm to a partner of the firm;
(2) Any interest being paid to Bank/LIC or other notified financial organizations
(3) Interest paid by a co-operative society (other than a co-operative bank) to their member;
(4) Interest on income tax refund or wealth tax refund etc.
(5) Income paid in relation to a Zero Coupon Bond.
Deduction Of Tax At Source 340

(6) Interest paid in respect of deposits under any scheme notified by the government.
"Zero Coupon Bond" Section 2(48)
means a bond which are issued by the specified companies and which are issued for minimum 10 years and
maximum 20 years and in respect of which no payment and benefit is received before maturity or
redemption from such specified company and further such bonds shall be notified by the Central
Government.
Additional amount received on redemption shall be considered to be capital gain.
Illustration 3: Mrs. Nupur Sharma is getting a family pension of `7,000 p.m. She has also received interest
on fixed deposit of `36,000 after deducting tax at source of `4,000. The bank has deducted collection
charges @ 1.5%.
Compute her tax liability for assessment year 2019-20.
Solution: ` `
Family Pension 84,000
(7,000 x 12)
Less: Deduction u/s 57 (15,000) 69,000
1/3 of `84,000 or `15,000 whichever is less
Interest 40,000
Less: Bank Charges [36,000 @ 1.5 %] (540) 39,460
Income under the head Other Sources 1,08,460
Tax Liability Nil
Illustration 4 (From RTP): Examine the implications of tax deduction at source under section 194A in the
cases mentioned hereunder, based on the provisions of the Income-tax Act, 1961.
(i) On 01.10.2018, Mr. Mohit made a six-month fixed deposit of ` 12 lakh @ 8% p.a. with Theta Co-
operative Bank. The fixed deposit matures on 31.3.2019.
(ii) Mr. Harish made fixed deposits carrying interest @10% p.a. with the following branches of Omega
Bank, a bank which has adopted CBS.
Branch Amount (`) Date of deposit Date of Maturity
Adyar 60,000 01.06.2018 31.03.2019
Anna Nagar 80,000 01.07.2018 31.03.2019
Nungambakkam 75,000 01.08.2018 31.03.2019
(iii) On 01.04.2018, Ms. Meena started a 1 year recurring deposit of ` 20,000 per month @ 10% p.a. with
Gamma Bank. The recurring deposit matures on 31.3.2019. Gamma bank pays interest of `13,000.
Solution:
(i) Theta Co-operative Bank has to deduct tax at source@10% on the interest of ` 48,000 (8% × ` 12 lakh ×
½) under section 194A.
(ii) Since Omega Bank has adopted CBS, the aggregate interest credited/paid by all branches has to be
considered, and if the same exceeds ` 10,000, tax is deductible under section 194A. Omega Bank has to
deduct tax at source @10% under section 194A, since the aggregate interest on fixed deposit with the three
branches of the bank is ` 16,000, which exceeds the threshold limit of ` 10,000.
Branch Amount of Rate of Period in Amount of
deposit (`) Interest months Interest (`)
Adyar 60,000 10% 10 5,000
Anna Nagar 80,000 10% 9 6,000
Nungambakkam 75,000 10% 8 5,000
Total 16,000
(iii) Tax has to be deducted @ 10% under section 194A by Gamma Bank on the interest of `13,000 on
recurring deposit on 31.3.2019 to Ms. Meena, since –
(1) ―recurring deposit has been included in the definition of “time deposit”; and
(2) such interest exceeds the threshold limit of ` 10,000.
Illustration 5 (From RTP): Examine the TDS implications under section 194A in the cases mentioned
hereunder –
(i) On 1.10.2018, Mr. Harish made a six-month fixed deposit of ` 10 lakh @ 9% p.a. with ABC Co-
operative Bank. The fixed deposit matures on 31.3.2019.
Deduction Of Tax At Source 341

(ii) On 01.06.2018, Mr. Ganesh made three nine month fixed deposits of ` 1 lakh each carrying interest @
9% with Dwarka Branch, Janakpuri Branch and Rohini Branches of XYZ Bank, a bank which has adopted
CBS. The fixed deposits mature on 28.2.2019.
(iii) On 01.04.2018, Mr. Rajesh started a 1 year recurring deposit of ` 20,000 per month @ 8% p.a. with
PQR Bank. The recurring deposit matures on 31.03.2019. PQR bank pays interest of `10,400.
Answer:
(i) ABC Co-operative Bank has to deduct tax at source @10% on the interest of ` 45,000 (9% × ` 10 lakh ×
½) under section 194A. The tax deductible at source under section 194A from such interest is, therefore,
`4,500.
(ii) XYZ Bank has to deduct tax at source@10% under section 194A, since the aggregate interest on fixed
deposit with the three branches of the bank is ` 20,250 [1,00,000 × 3 × 9% × 9/12], which exceeds the
threshold limit of ` 10,000. Since XYZ Bank has adopted CBS, the aggregate interest credited/paid by all
branches has to be considered. Since the aggregate interest of ` 20,250 exceeds the threshold limit of `
10,000, tax has to be deducted @ 10% under section 194A.
(iii) Tax has to be deducted under section 194A @ 10% by PQR Bank on the interest of `10,400 on
recurring deposit on 31.03.2019 to Mr. Rajesh, since –
(1) “recurring deposit” has been included in the definition of “time deposit”; and
(2) such interest exceeds the threshold limit of ` 10,000.
Question 5: Write a note on TDS in case of Winnings from Lottery or Crossword Puzzle etc.
Answer: TDS in case of Winnings from Lottery or Crossword Puzzle etc. Section 194B
Every person (including individual and HUF even if limit prescribed under section 44AB has not exceeded
in the preceding year) responsible for paying to any resident or non-resident, any income by way of
winnings from any lottery or crossword puzzle or card game and other game of any sort shall deduct
tax at source @ 30% provided the amount being paid or payable is exceeding `10,000 e.g. If ABC Ltd. has
to pay `7,000 being winning of a lottery, no tax shall be deducted at source but if amount being paid is `10
lakh, tax to be deducted at source shall be `10,00,000 x 30% = `3,00,000
If any such winning is in kind, winning shall be released only after collecting the amount of tax e.g. Mr. X
has won a motor car valued `5,00,000, in this case the organizer should collect tax of `1,50,000 and only
after that motor car shall be released.
Question 6: Write a note on TDS in case of Winnings from Horse Race.
Answer: TDS in case of Winnings from Horse Race Section 194BB
Every person (including individual and HUF even if limit prescribed under section 44AB has not exceeded
in the preceding year), shall be required to deduct tax at source @ 30% in case of payment of winning from
horse races but tax shall be deducted at source only if amount paid or payable during a particular year to a
particular person is exceeding `10,000.

Example
ABC Ltd. has to pay winnings of horse race `3,00,000 to Mr. X, amount of TDS shall be `90,000 but if the
amount to be paid is `2,000, amount of TDS shall be Nil.
Illustration 6: Mr. X purchased 20 lottery tickets of ` 250 each with a winning of `2.80 lakhs (after TDS
@ 30%). He has also received interest of `72,000 after deducting tax at source @ 10% and the cheque was
collected by bank and service charges @ 2% was taken by the bank. He has income from subletting of house
property `9,000 p.m. He has received family pension of ` 4,000 p.m.
Compute his tax liability and also tax payable/refund for assessment year 2019-20.
Solution: ` `
Income from lottery (2,80,000 /70% x 100%) 4,00,000.00
Interest 80,000
Less: Bank charges u/s 57 (2% of `72,000) (1,440) 78,560.00
Sub-letting of house property (9,000 x 12) 1,08,000.00
Family pension 48,000
Less: 1/3 of income or `15,000 whichever is less (15,000) 33,000.00
Income under the head Other Sources 6,19,560.00
Deduction Of Tax At Source 342

Computation of Tax Liability


Tax on casual income `4,00,000 @ 30% u/s 115BB 1,20,000.00
Tax on `2,19,560 at slab rate Nil
Tax before health & education cess 1,20,000.00
Add: HEC @ 4% 4,800.00
Tax Liability 1,24,800.00
Less: TDS (1,20,000 + 8,000) (1,28,000.00)
Tax Refund 3,200.00
Note: Interest received is a interest other than interest on deposit in a saving bank account with bank, co-
operative society, post office i.e. not eligible for deduction under section 80TTA.
Question 7: Write a note on TDS in case of Payments to Contractors.
Answer: TDS in case of Payments to Contractors Section 194C
1. Every person responsible for making payment to a resident contractor in connection with any work shall
deduct tax at source @ 2% and in case of payment to individual or Hindu Undivided Family, the rate of
TDS shall be 1%. Tax shall be deducted at source only if the amount being paid is exceeding `30,000 or the
amount paid or payable during a particular financial year to a particular person exceeds `1,00,000.
Example
ABC Ltd. makes the following payments to Mr. X, a contractor, for contract work during the P.Y.2018-19 –
` 15,000 on 01.05.2018
` 25,000 on 01.08.2018
` 30,000 on 01.12.2018
On 01.03.2019, a payment of ` 48,000 is due to Mr. X on account of a contract work.
Discuss whether ABC Ltd. is liable to deduct tax at source under section 194C from payments made to Mr.
X.
Solution:
In this case, the individual contract payments made to Mr. X does not exceed ` 30,000. However, since the
aggregate amount paid to Mr. X during the P.Y.2018-19 exceeds ` 1,00,000 (on account of the last payment
of `48,000, due on 01.03.2019, taking the total from ` 70,000 to ` 1,18,000), the TDS provisions under
section 194C would get attracted. Tax has to be deducted @ 1% on the entire amount of 1,18,000 from the
last payment of ` 48,000 and the balance of ` 46,820 (i.e. ` 48,000 – ` 1,180) has to be paid to Mr. X.
Example
(i) If DDA has to pay a sum of `5,00,000 to Mr. X in connection with a particular contract, amount of TDS
shall be `5,000.
(ii) If in the above case amount is to be paid to X Ltd. An Indian company, amount of TDS shall be `10,000.

2. An Individual or Hindu Undivided Family shall be required to deduct tax at source only if the turnover of
business or profession has exceeded the limit mentioned under section 44AB during the financial year
immediately preceding the relevant year.
Example
If Mr. X is engaged in a business and turnover of business is `41,00,000 in the previous year 2017-18 and he
has to pay `1,10,000 to Mr. Y in the previous year 2018-19 in connection with a contract, amount of TDS
shall be Nil but if his turnover in previous year 2017-18 was `110,00,000, amount of TDS shall be `1,100
but if payment is to given to Y Ltd., amount of TDS shall be `2,200.
3. No individual or HUF shall deduct tax at source if the amount is paid for personal purpose of such
individual or HUF.
Example
If in the above case, Mr. X has to pay `1,10,000 to Mr. Y in connection with a contract which is for personal
purpose of Mr. X, TDS shall be Nil.
4. Contract for this purpose shall include every type of contract e.g. Advertising contract/Broadcasting and
telecasting contract / Carriage of passenger by any mode of transport / Catering contract / Contract for
construction / Contract for courier services / Contract of maintenance of plant and machinery etc.
5. If any person making payment for purchase of goods, no tax shall be deducted at source but if such person
has supplied raw material etc. and contract is only for labour etc., tax shall be deducted at source e.g. ABC
Deduction Of Tax At Source 343

Ltd. purchased furniture for `10 lakh and no tax shall be deducted at source but if ABC Ltd. has supplied
raw material like wood etc. and has paid `2 lakh for labour charges for making the furniture, tax shall be
deducted at source.
Example
ABC Ltd. has given orders to Mr. X to stitch uniform for their employees and Mr. X purchased material
from the market and has stitched uniform for ABC Ltd. and has charged `7,00,000, in this case amount of
TDS shall be nil but if material is supplied by ABC Ltd. and Mr. X has charged `1,10,000 as labour charge,
tax shall be deducted at source @ 1% i.e. `1,100.
6. No tax shall be deducted at source in case of payment to a contractor in connection with transportation
of goods where such contractor do not own more than 10 goods carriages at any time during the year and
also submitted a declaration in this regard and has also furnished permanent account number.
Example
ABC Ltd. has paid `5,00,000 to Mr. X for transportation of goods and Mr. X do not have more than 10
goods carriages and he has furnished a declaration in this regard and has submitted permanent account
number, in this case no tax shall be deducted at source but if PAN has not been provided, tax shall be
deducted at source @ 20%.
Question 8: Write a note on TDS in case of payment of Insurance Commission.
Answer: TDS in case of payment of Insurance Commission Section 194D
Every person responsible for making payment for insurance commission to a resident insurance agent shall
deduct tax at source @ 5% provided the amount paid or payable during a particular year to a particular agent
is exceeding `15,000 e.g. If LIC has to pay commission of `5,00,000 to one of the agent Mr. X, amount of
TDS shall be `5,00,000 x 5% = 25,000
Question 9: write a note on Payment on maturity of life insurance policy.
Answer: Payment on maturity of life insurance policy Section 194DA
In general payment on maturity of Life policy is exempt from income tax under section 10(10D) however
sometimes the amount is taxable (if premium paid has exceeded the prescribed percentage (i.e. 10% / 15% /
20%)) and in that case tax has to deducted at source @ 1% provided the amount paid or payable to any
resident during a particular financial year is `1,00,000 or more.
Example
Examine the applicability of the provisions for tax deduction at source under section 194DA in the following
cases -
(i) Mr. X, a resident, is due to receive ` 4.50 lakhs on 31.03.2019, towards maturity proceeds of LIC policy
taken on 01.4.2012, for which the sum assured is ` 4 lakhs and the annual premium is ` 1,25,000.
(ii) Mr. Y, a resident, is due to receive ` 2.20 lakhs on 31.03.2019 on LIC policy taken on 01.04.2010, for
which the sum assured is ` 2 lakhs and the annual premium is `35,000.
(iii) Mr. Z, a resident, is due to receive `95,000 on 01.10.2018 towards maturity proceeds of LIC policy
taken on 01.10.2010 for which the sum assured is ` 90,000 and the annual premium was ` 19,000.
Answer
(i) Since the annual premium exceeds 10% of sum assured in respect of a policy taken on 01.04.2012, the
maturity proceeds of ` 4.50 lakhs are not exempt under section 10(10D) in the hands of Mr. X. Therefore,
tax is required to be deducted @ 1% under section 194DA on the maturity proceeds of ` 4.50 lakhs payable
to Mr. X.
(ii) Since the annual premium is less than 20% of sum assured in respect of a policy taken before
01.04.2012, the sum of `2.20 lakhs due to Mr. Y would be exempt under section 10(10D) in his hands.
Hence, no tax is required to be deducted at source under section 194DA on such sum payable to Mr. Y.
(iii) Even though the annual premium exceeds 20% of sum assured in respect of a policy taken before
01.04.2012, and consequently, the maturity proceeds of `95,000 would not be exempt under section
10(10D) in the hands of Mr. Z, the tax deduction provisions under section 194DA are not attracted since the
maturity proceeds are less than ` 1 lakh.
Question 10: Write a note on Payments to non-resident sportsmen or sports association.
Answer: Payments to non-resident sportsmen or sports association Section 194E
Every person (including individual and HUF even if limit prescribed under section 44AB has not exceeded
in the preceding year) responsible for making payment to a non-resident sportman / athlete / entertainer shall
Deduction Of Tax At Source 344

deduct tax at source @ 20% plus HEC. Further such non-resident should not be a citizen of India. The
incomes should be such as are covered under section 115BBA i.e. the incomes listed below:
(a) participation in any game or sport in India or
(b) advertisement; or
(c) Contribution of articles relating to any game or sport in India in newspapers, magazines or journals.
Example: Madona, a non-resident, received ` 40 lakh for her stage shows in India from Optimistic Ltd., an
event management company in India, on 26.12.2018, in this case, payments made to a non-resident
entertainer, shall be subject to tax deduction @ 20% under the provisions of section 194E plus HEC @ 4%.
Tax deductible under section 194E = ` 40 lakh × 20.8% = ` 8,32,000
Question 11: Write a note on TDS in case of Commission, etc., on the Sale of Lottery Tickets.
Answer: TDS in case of Commission, etc., on the Sale of Lottery Tickets Section 194G
Every person (including individual and HUF even if limit prescribed under section 44AB has not exceeded
in the preceding year) making payment of commission for sale of lottery tickets to any person resident or
non-resident, shall deduct tax at source @ 5% provided the amount paid or payable to a particular person
during a particular year is exceeding `15,000.
Question 12: Write a note on TDS on payment of Commission or Brokerage.
Answer: TDS on payment of Commission or Brokerage Section 194H
Every person making payment of any commission or brokerage to a resident shall, deduct income-tax at
the rate of 5%, provided amount paid or payable during a particular year to a particular person is exceeding
`15,000.
An Individual or Hindu Undivided Family shall be required to deduct tax at source only if the turnover of
business or profession has exceeded the limit mentioned under section 44AB during the financial year
immediately preceding the relevant year.
Example
Manoj trading limited rendered services in relation to sale of mustard oil to Ashish oils limited and
commission charged is `7,00,000, in this case, tax to be deducted at source by Ashish oils limited shall be
7,00,000 x 5% = `35,000 and amount payable to Manoj trading limited shall be
Commission 7,00,000
Less: TDS(7,00,000 x 5%) (35,000)
Amount Payable 6,65,000
Question 13: Write a note on TDS in case of payment of rent.
Answer: TDS in case of payment of rent Section 194-I
Every person making payment of rent to a resident shall deduct tax at source provided the amount paid or
payable during a particular year is exceeding `1,80,000. Tax shall be deducted at source @ 2% if rent is for
plant and machinery but @ 10% if rent is for land/building / furniture / fixture etc.
An Individual or Hindu Undivided Family shall be required to deduct tax at source only if the turnover of
business or profession has exceeded the limit mentioned under section 44AB during the financial year
immediately preceding the relevant year.
Illustration 7:
(i) Amount of TDS in the following cases shall be:
Person receiving the payment Mr. A Mr. B Mr. C Mr. D
Person making the payment Individual (not Individual Partnership firm Company
exceeded the limit (exceeded the (Indian
of 44AB in P.Y. limit of 44AB in company)
2017-18) P.Y. 2017-18)
Rent for house property `2,00,000 `2,00,000 `2,00,000 `2,00,000
Amount of TDS Nil `20,000 `20,000 `20,000
Rent for plant and machinery `2,00,000 `2,00,000 `2,00,000 `2,00,000
Amount of TDS Nil `4,000 `4,000 `4,000
Rent for house property `1,00,000 `1,00,000 `1,00,000 `1,00,000
Amount of TDS Nil Nil Nil Nil
Rent for plant and machinery `1,00,000 `1,00,000 `1,00,000 `1,00,000
Amount of TDS Nil Nil Nil Nil
Deduction Of Tax At Source 345

Example 1.
XYZ Ltd. raised an invoice of `3,00,000/- to ABC Limited for renting of commercial building. The above
figure includes `50,000/- of parking charges. The bill is raised on 30th June, 2018 and ABC Limited made
the payment on the same date.
Compute the Amount of TDS required to be deducted by ABC Limited and the due date of deposit of TDS
amount and last date of filing of quarterly Statement?
Solution:
Rent 3,00,000
Less: TDS(3,00,000 x 10%) (30,000)
Amount payable 2,70,000
th
Last date of deposit = 7 July, 2018.
Last date of filing of quarterly Statement = 31st July, 2018.
Example 2
ABC limited has let out one commercial building to Idea cellular limited at Gurgaon and rent charged is
`2,50,000 per month, in this case, tax to be deducted at source by Idea cellular limited shall be as given
below:
Solution:
Rent (2,50,000 x 12) 30,00,000
Less: TDS(30,00,000 x 10%) (3,00,000)
Amount payable 27,00,000
Question 14: Write a note on TDS in case of Payment for purchase of immovable property.
Answer: TDS in case of Payment for purchase of immovable property Section 194-IA
1. Every person (including individual and HUF even if limit prescribed under section 44AB has not
exceeded in the preceding year) making payment to a resident for purchase of immovable property of `50
lakhs or more shall deduct tax at source @ 1% of such sum.
2. No tax shall be deducted at source in case of payment for purchase of agricultural land which is
situated in the rural area.
E.g. Mr. X has purchased one building for `65 lakhs, in this case amount of TDS shall be 65,00,000 x 1% =
`65,000 but if building was purchased for `47 lakhs, amount of TDS shall be nil.
3. The person deducting tax at source shall not be required to obtain Tax Deduction Account Number as per
section 203A.
Example: Mr. X sold his house property in Chennai for a consideration of `75 lakh to Mr. Y on 31.01.2019,
in this case, Mr. Y is required to deduct tax at source under section 194-IA @ 1% of `75 lakh and tax
deductible under section 194-IA shall be ` 75 lakh × 1% = ` 75,000
Question 15: Write a note on TDS in case of Payment of Rent by Certain Individual and HUF.
Answer: TDS in case of Payment of Rent by Certain Individual and HUF Section 194-IB
(1) Any person, being an individual or a Hindu undivided family not liable to audit in the preceding
financial year, responsible for paying to a resident any income by way of rent exceeding `50,000 for a
month or part of a month during the previous year, shall deduct tax @ 5%.
(2) Tax shall be deducted at the time of making payment of rent for the last month of the previous year or
the last month of tenancy whichever is earlier.
(3) No requirement to take tax deduction account number.
(4) If the person receiving payment of rent has not submitted PAN, tax shall be deducted @ 20% but
maximum rent payable for the last month.
Example: Mr. X has taken a house on rent `60,000 p.m. (he is not liable to audit in P.Y. 2018-19), in this
case he will be required to deduct tax at source @ 5% but tax is to be deducted in the last month instead of
every month. While paying rent of ` 60,000 for March 2020 he should deduct tax at source ` 7,20,000 x 5%
= 36,000 but if per receiving payment has not submitted PAN, amount of TDS shall be 7,20,000 x 20% =
1,44,000 but maximum ` 60,000.
Question 16: Write a note on TDS in case of Payment under specified agreement.
Answer: TDS in case of Payment under specified agreement Section 194-IC
Notwithstanding anything contained in section 194-IA, any person responsible for paying to a resident any
sum by way of consideration, not being consideration in kind, under the agreement referred to in section 45
Deduction Of Tax At Source 346

(5A), shall deduct tax @ 10%.


Question 17: Write a note on TDS in case of Fees for Professional or Technical Services.
Answer: TDS in case of Fees for Professional or Technical Services Section 194J
1. Every person, who is responsible for paying to a resident any sum by way of –
(i) fees for Professional services
(ii) fees for Technical services
(iii) any Remuneration or fees or commission to a director of a company (in case salary is being paid to a
director, tax shall be deducted at source under section 192).
(iv) Royalty
(v) Non-compete fee referred to in section 28
shall deduct tax at source at the rate of 10%.
An Individual or Hindu Undivided Family shall be required to deduct tax at source only if the turnover of
business or profession has exceeded the limit mentioned under section 44AB during the financial year
immediately preceding the relevant year.

TDS shall be deducted @ 2% (instead of 10%) in case of a payee engaged only in the business of operation
of a call centre.

2. No tax shall be deducted at source where the amount paid or payable during the year do not exceed
`30,000. (limit of `30,000 is applicable separately for each of the above payments). There is no such limit in
case of payment to a director i.e. tax has to be deducted at source in case of payment to a director
irrespective of the amount to be paid.

Illustration 8: XYZ Ltd. makes a payment of `28,000 to Mr. X on 02.08.2018 towards fees for professional
services and another payment of `25,000 to him on the same date towards fees for technical services.
Discuss whether TDS provisions under section 194J are attracted.
Solution:
TDS provisions under section 194J would not get attracted, since the limit of `30,000 is applicable for fees
for professional services and fees for technical services, separately. It is assumed that there is no other
payment to Mr. X towards fees for professional services and fees for technical services during the P.Y.2018-
19.
Example: X Ltd. paid retainership fees of `25,000 to its Director, Mr. Ram Sharma, on 30.01.2019, as per
section 194J, the company shall be liable to deduct tax at source @ 10% on any remuneration or fees or
commission paid to a director, on which the tax is not deductible under section 192. The limit of `30,000
under section 194J is not applicable on any remuneration or fees or commission payable to director of
a company.
Tax deductible under section 194J = ` 25,000 x 10% = ` 2,500

3. “Professional services” means services rendered by a person in the course of carrying on legal, medical,
engineering or architectural profession or the profession of accountancy or technical consultancy or interior
decoration or advertising or such other profession as is notified by the Board for the purposes of section
44AA or of this section.

Example
(i) If ABC Ltd. has to pay a sum of `2,00,000 to an architect, amount of TDS shall be `20,000.
(ii) If ABC Ltd. has to pay `10,00,000 to a Chartered Accountant, amount of TDS shall be `1,00,000.
(iii) If Mr. X has to pay `50,000 to an advocate, amount of TDS shall be Nil and if Turnover of Mr. X was
exceeded the prescribed limit u/s 44AB during 2017-18, amount of TDS shall be `5,000.
(iv) If Z Ltd. has to pay `15,000 in connection with technical services, amount of TDS shall be Nil.

4. If individual or HUF is making payment for professional services and it is for personal purpose, no tax
shall be deducted at source.
Deduction Of Tax At Source 347

Question 18: Write a note on TDS in case of Payment of Compensation on Acquisition of certain
Immovable Property.
Answer: TDS in case of Payment of Compensation on Acquisition of certain Immovable Property
Section 194LA
If any land or building has been acquired by the government or other similar agency, tax shall be deducted at
source @ 10% provided the amount paid or payable to any resident is exceeding `2,50,000. No tax shall be
deducted at source if the payment relates to acquisition of agricultural land.
No deduction shall be made under this section where such payment is made in respect of any award or
agreement which has been exempted from levy of income-tax under section 96 of the Right to Fair
Compensation and Transparency in Land Acquisition, Rehabilitation and Resettlement Act, 2013.

Example: If `3,00,000 is to be paid to Mr. X on 05.05.2018 by State Government on compulsory


acquisition of his urban land, amount of TDS shall be 3,00,000 x 10% = 30,000.

Question 19: Write a note on TDS in case of Payment to Non-Resident or Foreign Company.
Answer: TDS in case of Payment to Non-Resident or Foreign Company Section 195
Every person making any payment to a non-resident or to a foreign company shall deduct tax at source at the
prescribed rate.

Question 20: Write a note on TDS in case of payment by individual or Hindu Undivided Family.
Answer: TDS in case of payment by individual or Hindu Undivided Family
An Individual or Hindu Undivided Family shall be required to deduct tax at source only if the turnover of
business or profession has exceeded the limit mentioned under section 44AB during the financial year
immediately preceding the relevant year.
The above provisions are applicable for TDS under section 194A, 194C, 194H, 194I, 194J.
The above provisions are not applicable for TDS under other sections like 192, 192A, 193,194B, 194BB,
194D, 194DA, 194E, 194G, 194-IA,194-IB, 194LA, 195.

Question 21: Write a note on Deduction of tax at Lower Rate.


Answer: Deduction of tax at Lower Rate Section 197
If on income of any person, income-tax is required to be deducted at the time of payment under section 192,
193,194A, 194C, 194D, 194G, 194H, 194I, 194J, 194LA and 195 and the Assessing Officer is satisfied that
the total income of the recipient justifies the deduction of income-tax at any lower rates or no deduction of
income-tax, as the case may be, the Assessing Officer shall, on an application made by the assessee in this
behalf, give to him such certificate as may be appropriate.
Application should be given in Form No. 13.
(Section 197 is not applicable in case of TDS under section 192A, 194B, 194BB, 194DA, 194E, 194-IA)

Question 22: Write a note on self declaration for not deducting tax at source.
Answer: Self declaration for not deducting tax at source Section 197A
As per section 197A, if any individual or Hindu Undivided Family has interest income not exceeding the
exemption limit and also his tax liability is nil, such individual or HUF can furnish a declaration in Form
No. 15G to the person making payment of interest and in that case no tax shall be deducted at source. A
senior citizen can give a declaration in Form No 15H if his tax liability is nil.
Similar provision shall be applicable in case of section 192A, 194D, 194DA & 194I.

Question 23: Write a note on time period for depositing tax deducted at source.
Answer: Time period for depositing tax deducted at source Section 200 / Rule 30
As per Rule 30, the payment is to be made in general within 7 days from the last day of the month in
which the deduction is made.
If the tax has been deducted in the month of March, tax should be deposited on or before 30th April.
In certain cases, Assessing Officer may permit the payments on quarterly basis.
Deduction Of Tax At Source 348

Question 24: Write a note on filing of quarterly statement of TDS.


Answer: Filing of quarterly statement of TDS Section 200 / Rule 31A
Every person deducting tax at source has to submit quarterly statement containing details of the tax deducted
at source. The statement should be submitted latest by 31st of the month succeeding the relevant quarter but
statement for the quarter ending March can be submitted upto 31st May. e.g. Statement for quarter ending
March, 2019 can be submitted upto 31st May, 2019.

Question 25: Write a note on Consequences of Failure to Deduct or Pay.


Answer: Consequences of Failure to Deduct or Pay Section 201
1. If any person has failed to deduct tax at source, interest shall be charged @ 1% p.m. or part of a
month for the period of delay. E.g. ABC Ltd. has made one payment on 03.01.2019 but tax was deducted at
source on 20.01.2019, in this case interest shall be charged @ 1% for one month.
2. If person has deducted tax at source but tax was not deposited within the time allowed under section
200, interest shall be charged @ 1.5% p.m. or part of a month from date of deducting tax at source upto
the date of depositing the amount
Example
Assessee deduct TDS on 10.10.2018 but pays TDS on 31.12.2018, Interest under section 201 shall be
charged from 10.10.2018 to 31.12.2018 @ 1.5% per month i.e., for 3 months.
If in the above case assesse has not deducted tax at source on 10.10.2018 rather assesse deducted TDS on
31.12.2018 and assesse pays TDS on 17.01.2019, interest shall be charged in the manner given below:
(i) Interest under section 201 shall be charged for 3 months @ 1% for the period 10.10.2018 to 31.12.2018.
(ii) Interest under section 201 shall be charged @ 1.5% per month for one month from 31.12.2018 to
17.01.2019.
3. Assessee shall also be considered to be assessee in default and penalty may be imposed equal to the
amount which he has failed to deduct or pay but in following two situations he will not be considered to be
assessee in default
1. If there were sufficient reasons for not deducting tax at source
2. Payment was made to a resident and such resident has shown the amount in his income and has paid tax
and also return has been filed and it has been confirmed by a Chartered Accountant.
In this case assessee shall pay interest from the date when tax was to be deducted upto the date of filing the
return.
Example
ABC Ltd. paid certain amount on 05.01.2019 to Mr. X and tax was not deducted at source but Mr. X himself
has paid his tax and return was filed on 31.07.2019, in this case interest shall be charged @ 1% p.m. for a
period of 7 months i.e. from 05.01.2019 to 31.07.2019.
Illustration 9: An amount of `40,000 was paid to Mr. X on 01.07.2018 towards fees for professional
services without deduction of tax at source. Subsequently, another payment of `50,000 was due to Mr. X on
28.02.2019, from which tax @ 10% (amounting to `9,000) on the entire amount of ` 90,000 was deducted.
However, this tax of ` 9,000 was deposited only on 22.06.2019.
Compute the interest chargeable under section 201.
Solution:
Interest under section 201 would be computed as follows –
Particulars `
1% on tax deductible but not deducted i.e., 1% on ` 4,000 for 8 months 320
(01.07.2018 to 28.02.2019)
1½% on tax deducted but not deposited i.e. 1½% on ` 9,000 for 4 months 540
(28.02.2019 to 22.06.2019) 860
Question 26: Write a note on Certificate for Tax Deducted.
Answer: Certificate for Tax Deducted Section 203/ Rule 31
TDS Certificate
Every person deducting tax at source shall issue a certificate to the person with regard to whom tax has been
Deduction Of Tax At Source 349

deducted at source. In case of payment of salary, certificate shall be issued in Form No. 16 and in other
cases it will be in Form No. 16A.
The certificate in Form No. 16 should be given upto 31st May of the succeeding year in case of an employee
and it will be an annual certificate.
In other cases certificate in Form No. 16A should be issued on quarterly basis and it should be issued within
15 days from the last date of submitting the quarterly statement under section 200.

Question 27: Write a note on Tax Deduction Account Number (TAN).


Answer: Tax Deduction Account Number (TAN) Section 203A
Every person, deducting tax at source shall apply for allotment of tax deduction account number and
application has to be given in Form No.49B within one month from the end of the month in which tax
was deducted for the first time.

Question 28: Write a note on Furnishing of Statement of Tax Deducted.


Answer: Furnishing of Statement of Tax Deducted Section 203AA / Rule 31AB
Income Tax Authorities shall prepare annual statement for every person with regard to whom tax has been
deducted at source and such statement shall be prepared in the Form No. 26AS on the basis of quarterly
return filed by the person deducting tax at source. Such statement shall be sent to every person upto 31st July
of assessment year. Tax credit can be claimed on the basis of such statement. In case of any discrepancy,
such person shall report it to the Income Tax Department.

Question 29: Write a note on Requirement to Furnish Permanent Account Number.


Answer: Requirement to Furnish Permanent Account Number Section 206AA
Every person on whose behalf, tax is being deducted at source shall submit his PAN to the person deducting
tax at source otherwise rate of TDS shall be the actual rate or 20% whichever is higher. The person
deducting tax at source has to mention such PAN in the quarterly statement.
TDS shall be on the amount excluding tax
As per Circular No. 23/2017, Dated 19-7-2017, wherever in terms of the agreement/contract between the
payer and the payee, the tax component comprised in the amount payable to a resident is indicated
separately, tax shall be deducted at source on the amount paid/payable without including such tax
component.
Illustration 10: Mrs. X has received incomes as given below during the previous year 2018-19:
1. Interest on savings bank account with State Bank `50,000 (gross).
2. Interest from Government securities `1,00,000 on 01.01.2019 (collection charge paid to the bank @
1.5%).
3. Interest from ABC Ltd on non listed debentures `3,60,000 (after TDS) on 01.03.2019 (collection
charge paid to the bank `30).
4. Interest credited to post office savings bank account during the year ` 10,000.
5. Interest credited to public provident fund during the year ` 15,000.
6. Interest received from XYZ Ltd on listed debentures ` 1,35,000 (Net).
(Collection charge `30) The amount was invested by taking a loan of `15,00,000 @ 12% p.a.
7. Mrs. X received rent of house property ` 72,000 per month after TDS.
8. Winnings from a lottery `70,000 (after TDS)
Compute her tax liability and also tax payable for the assessment year 2019-20.
Solution: `
Income under the head other sources
Gross interest from State Bank of India 50,000.00
Interest from Government securities
{`1,00,000 – `1,500} 98,500.00
Interest from ABC Ltd 3,99,970.00
{(`3,60,000 / 90 x 100) – `30}
Interest on P.O.S.B (10,000 – 3,500) 6,500.00
Deduction Of Tax At Source 350

Interest on PPF (exempt u/s 10(15)) Nil


Interest from XYZ Ltd. (30,030.00)
{Gross interest = `1,35,000 / 90 x 100 = 1,50,000
Less: Collection charges = (`30)
Less: Interest paid on loan = (`1,80,000)}
Winning from lottery 1,00,000.00
(70,000 / 70 x 100)
Income under the head Other Sources 6,24,940.00
Income under the head House Property
Gross Annual Value (72,000/90% x 12) 9,60,000.00
Less: Municipal Tax (Nil)
Net Annual Value 9,60,000.00
Less: 30% of NAV u/s 24(a) (2,88,000.00)
Less: Interest on capital borrowed u/s 24(b) (Nil)
Income from house property 6,72,000.00

Gross Total Income 12,96,940.00


Less: Deduction u/s 80TTA (10,000.00)
Total Income 12,86,940.00
Computation of tax liability
Tax on winning from lottery `1,00,000 @ 30% u/s 115BB 30,000.00
Tax on `11,86,940 at slab rate 1,68,582.00
Tax before health & education cess 1,98,582.00
Add: HEC @ 4% 7,943.28
Tax Liability 2,06,525.28
Less: TDS u/s 193 on Non Listed Debentures (40,000.00)
Less: TDS u/s 193 on Listed Debentures (15,000.00)
Less: TDS u/s 194B on winning from lottery (30,000.00)
Less: TDS u/s 194I (96,000.00)
Tax Payable 25,525.28
Rounded off u/s 288B 25,530.00

Illustration 11: Mr. X has let out one House property and rent received is `90,000 p.m. after TDS. He paid
Municipal Tax `1,00,000 and Interest u/s 24 (b) is `2,00,000. He has received ` 9,00,000 in connection
with professional services after TDS. The Assessee made the payment of tax on 10.05.2019. Compute Total
Income and Tax Payable and also Compute Interest u/s 234A, 234B & 234C.
Solution:
Computation of income under the head House Property `
Gross Annual Value (90,000/90% x 12) 12,00,000.00
Less: Municipal Tax (1,00,000.00)
Net Annual Value 11,00,000.00
Less: 30% of NAV u/s 24(a) (3,30,000.00)
Less: Interest on capital borrowed u/s 24(b) (2,00,000.00)
Income from house property 5,70,000.00
Income under the head Business/Profession (9,00,000/90%) 10,00,000.00
Gross Total Income 15,70,000.00
Less: Deduction u/s 80C to 80U Nil
Total Income 15,70,000.00

Computation of Tax Liability


Tax on ` 15,70,000 at slab rate 2,83,500.00
Add: HEC @ 4% 11,340.00
Tax Liability 2,94,840.00
Deduction Of Tax At Source 351

Less: TDS u/s 194I (1,20,000.00)


Less: TDS u/s 194J (1,00,000.00)
Tax Payable 74,840.00
Interest under section 234C shall be computed in the manner given below:
Tax Payable Tax Paid Shortfall
15.06.2018 11,226 (74,840 x 15%) Nil 11,226
Rounded off Rule 119A = 11,200
Interest u/s 234C = 11,200 x 1% x 3 = 336
15.09.2018 33,678 (74,840 x 45%) Nil 33,678
Rounded off Rule 119A = 33,600
Interest u/s 234C = 33,600 x 1% x 3 = 1,008
15.12.2018 56,130 (74,840 x 75%) Nil 56,130
Rounded off Rule 119A = 56,100
Interest u/s 234C = 56,100 x 1% x 3 = 1,683
15.03.2019 74,840 (74,840 x 100%) Nil 74,840
Rounded off Rule 119A = 74,800
Interest u/s 234C = 74,800 x 1% x 1 = 748

Total interest payable u/s 234C 3,775

Interest under section 234B shall be computed from 01.04.2019 to 10.05.2019 and is as given below:
Tax Liability – TDS shall be considered to be tax payable i.e. 74,840
74,840 = 74,800 x 1% x 2 = 1,496
(Rounded off Rule 119A = 74,800)
Total interest payable (3,775 + 1,496) 5,271
Rounded off u/s 288B 5,270

Question 30: Write a note on TCS under the Income- tax Act, 1961.
Answer:
As per section 206C, tax collection at source means the person receiving payment for selling of alcoholic
liquor shall collect tax at source from the buyer and also in some other cases, tax has to be collected at
source and is given below:
(i) Alcoholic Liquor for human consumption One per cent
(ii) Timber obtained under a forest lease Two and one-half per cent
(iii) Scrap One per cent
(iv) Parking lot Two per cent
(v) Toll plaza Two per cent
(vi) Mining Two per cent
(vii) Minerals, being coal or lignite or iron ore One per cent
(viii) Motor vehicle, value exceeding ten lakh rupees One per cent
Deduction Of Tax At Source 352

EXAMINATION QUESTIONS
MAY – 2017
Question 7 (a) (4 Marks)
Pallavi Bank Ltd., has paid interest of `9,000 to Mr. A, a resident Indian, from its Chennai branch and
`8,000 from Bangalore branch. If there is no core banking services in the bank, is tax required to be
deducted at source from such interest payments made on 31-3-2019? Will your answer be different if there
is core banking service present in the bank? Also, explain the provisions of the Income-tax Act, 1961 in this
regard.
Answer:
As per section 194A, Every person making payment of interest other than interest on securities to any
resident shall deduct tax at source @ 10% provided the amount being paid or payable during a particular
year to a particular person is exceeding `5,000 but if payment is being made by bank or post office, tax shall
be deducted only if interest being paid or payable is exceeding `10,000. Further TDS shall be only on time
deposit including recurring deposit. Limit of `10,000 shall be per branch of the bank but if the bank has core
banking solution, limit shall be per bank and not per branch.
In the first case there is no core banking services in the bank then the limit shall be per branch and amount
has not exceeded `10,000 per branch hence no TDS shall be required to be deducted.
In the Second case, there is a core banking services and limit has exceeded including both the branch hence
TDS shall be required to be deducted @ 10% on `17,000.

Question 7 (c) (4 Marks)


Mr. Sachal , a resident individual aged 54, furnishes income details as under:
(i) Wholesale Cloth business, whose turnover is `150 lakhs, for which accounts are audited u/s 44AB.
Income from such business `8,10,000.
(ii) Income from other sources `2,70,000.
(iii) Tax deducted at source `25,000.
(iv) Advance tax paid `1,03,000 on 14-3-2019 .
Return of income will be filed on 11-12-2019. The assesse is willing to pay the requisite self-assessment tax.
Calculate the interest payable under section 234C and 234B of the income-tax Act, 1961. Assume that the
return of income would be processed on the same day of filing of return.
Solution: As per section 234B, if advance tax paid is less than 90% of actual tax liability, assessee shall be
required to pay interest @ 1% per month or part of a month from 1st April of assessment year upto the date
of payment. If advance tax paid is 90% or more of actual tax liability, no interest is payable.
`
Income from Business 8,10,000
Income from Other source 2,70,000
Gross Total Income 10,80,000
Less: Deduction u/s 80C to 80U Nil
Total Income 10,80,000

Computation of Tax Payable


Tax on 10,80,000 at slab rate 1,36,500
Add: HEC @ 4% 5,460
Tax Liability 1,41,960
Less: TDS (25,000)
Tax Payable before adjustment of advance tax 1,16,960
Deduction Of Tax At Source 353

Less: Advance Tax (1,03,000)


Tax Payable 13,960
Calculation of Interest u/s 234C
Interest under section 234C shall be computed in the manner given below:
Tax Payable Tax Paid Shortfall
15.06.2018 17,544 (1,16,960 x 15%) Nil 17,544
Rounded off Rule 119A = 17,500
Interest u/s 234C = 17,500 x 1% x 3 = 525

15.09.2018 52,632 (1,16,960 x 45%) Nil 52,632


Rounded off Rule 119A = 52,600
Interest u/s 234C = 52,600 x 1% x 3 = 1,578
15.12.2018 87,720 (1,16,960 x 75%) Nil 87,720
Interest u/s 234C = 87,700 x 1% x 3 = 2,631
15.03.2019 1,16,960 (1,16,960 x 100%) 1,03,000 13,960
Interest u/s 234C = 13,900 x 1% x 1 = 139

Total interest payable u/s 234C 4,873

Calculation of Interest u/s 234B (01-04-2019 to 11-12-2019)


Since Advance Tax paid i.e. `1,03,000 is less than 90% of the actual tax payable i.e. `1,15,600, hence
Interest u/s 234B shall be payable and Interest shall be –
13,900 x 1% x 9 = 1,251

Total Interest Payable (4,873 +1,251) 6,124

NOV – 2016
Question 4(a) (2 Marks)
State with reasons whether TDS is applicable or not on the following payment under the provisions of
Income-tax Act, 1961?
TDS is not applicable in respect of payment of `1,00,000 to Mr. Pandey a resident, being interest on
recurring deposit with SBI.
Answer:
False: As per Section 194A, every person making payment of interest other than interest on securities to any
resident shall deduct tax at source @ 10% provided the amount being paid or payable during a particular
year to a particular person is exceeding `10,000. Further TDS shall be only on time deposit including
recurring deposit. Hence TDS shall be applicable in the given case.

Question 7(a) (4 Marks)


(i) Discuss the provisions, relating to the premature withdrawal from Employees Provident Fund, under
section 192A, for A.Y. 2019-20.
Answer: Refer Answer given in the book
(4 Marks)
(ii) Mr. Barun provides you the following information and requests you to determine the Advance Tax
liability with due dates for the financial year 2018-19.
Estimated tax liability for the financial year 2018-19 `65,000
Tax deducted at source for this year ` 5,000
Answer: As per Section 208, if tax payable is ` 10,000 or more, in that case assessee shall be required to
pay advance tax. Tax payable means total tax liability less tax deducted at source hence in the given case
advance tax has to be paid with regard to ` 60,000 (65,000 – 5,000)
Deduction Of Tax At Source 354

As per section 211, all assessee has to pay advance tax in the manner given below:
Due date of installment Amount payable
Upto 15th June of P.Y. `9,000 (15% of `60,000)
Upto 15th September of P.Y. `27,000 (45% of `60,000)
Upto 15th December of P.Y. `45,000 (75% of `60,000)
Upto 15th March of P.Y. `60,000

MAY – 2016
Question 7(a)(i) (4 Marks)
Ashwin a resident Individual carrying on business, furnishes you the following information:
Total turnover for the financial year `
2017-18 120,00,000
2018-19 98,00,000
State whether tax deduction at source provisions are attracted for the under - mentioned expenses incurred
during the financial year 2018-19:
Particulars `
Commission paid to Babloo 18,500
Payment to Vijay for repair of office building 23,000
Payment of fees for Technical Services, to Vivek 35,000
All payments are made to residents.
If Tax has to be deducted at source, state the amount of tax to be deducted at source.
Answer.
As the turnover of Mr. Ashwin for F.Y.2017-18, i.e. `120 lakh, has exceeded the monetary limit prescribed
under section 44AB, he has to comply with the tax deduction provisions during the financial year 2018-19.
Commission paid to Babloo – Tax has to be deducted under section 194-H as the commission exceeds
`15,000.
Tax shall be deducted at source u/s 194H and shall be = 18,500 x 5% = `925

Contract payment of `23,000 to Mr. Vijay


TDS provisions under section 194C would not be attracted if the amount paid to a contractor does not
exceed `30,000 in a single payment or `1,00,000 in the aggregate during the financial year. Therefore, TDS
provisions under section 194C are not attracted in this case.

Payment of fees for Technical Services, to Vivek


Tax shall be deducted at source under section 194J @ 10% because the total amount payable is exceeding
`30,000 and amount of TDS shall be = 35,000 x 10% = `3,500

NOV – 2015
Question 7(a). (4 Marks)
What are the consequences of failure to deduct or pay the tax under section 201 of the Income Tax Act,
1961?
Refer Answer given in the Chapter
MAY – 2015
Question 7(a)(ii). (4 Marks)
Mr. Madan sold his house property in Surat as well as his rural agricultural land for a consideration of `65
lakhs and `20 lakhs respectively, to Mr. Raman on 01.10.2018. He has purchased the house property for `40
lakhs and the land for `15 lakhs, in the year 2013. There was no difference in the stamp valuation. You are
required to determine TDS implications, if any, assuming both persons are resident Indians.
Deduction Of Tax At Source 355

Solution:
As per Section 194-IA every person making payment to a resident for purchase of immovable property of
`50 lakhs or more shall deduct tax at source @ 1% of such sum. No tax shall be deducted at source in case
of payment for purchase of agricultural land which is situated in the rural area.
In the given case Mr. Raman is required to deduct tax at source @ 1% of `65 lakh = ` 65,000.
The person deducting tax at source shall not be required to obtain Tax deduction account number as per
section 203A.

Question 7(a)(iii). (4 Marks)


What is the difference between TDS and TCS under the Income- tax Act, 1961?
Answer: Refer Answer given in the Chapter

NOV – 2014
Question 7(a) (4 Marks)
State in brief the applicability of tax deduction at source provisions, the rate and amount of tax deduction in
the following cases for the financial year 2018-19
(1) Payment of `27,000 made to a South African cricketer, by an Indian newspaper agency on 02.07.2018
for contribution of articles in relation to the sport of cricket.
(2) Rent of `1,70,000 paid by a partnership firm for use of plant and machinery.
(3) Winning from horse race `1,50,000.
(4) `2,00,000 paid to Mr. X, a resident individual on 22.02.2019 by the State of Uttar Pradesh on
compulsory acquisition of his urban land.
Solution:
(1) TDS shall be deducted u/s 194E @ 20%+HEC. TDS shall be 20.8% of `27,000 = `5,616 .
(2) As per Section 194-I TDS is not required to be deducted as the Rent amount does not exceeds `1,80,000.
(3) TDS shall be deducted under section 194BB @ 30% as the amount exceeds `10,000. TDS shall be 30%
of `1,50,000 = `45,000.
(4) As per Section 194LA No TDS is deductible by State of Uttar Pradesh as the amount paid does not
exceeds `2,50,000.
MAY – 2014
Question 7(A). (2 x 2 = 4 Marks)
What are the provisions relating to tax deduction at source in respect of:
(A) ABC and Co. Ltd. Paid `19,000 to one of its Directors as sitting fees on 01.01.2019.
(B) Mr. X sold his House to Mr. Y on 01.02.2019 for `60 lacs?
Solution:
(A) Section 194J provides for deduction of tax at source @10% from any sum paid by way of any
remuneration or fees, by whatever name called, to a resident director, which is not in the nature of salary on
which tax is deductible under section 192. The threshold limit of ` 30,000 upto which the
provisions of tax deduction at source are not attracted in respect of every other payment covered under
section 194J is, however, not applicable in respect of sum paid to a director. Therefore, tax @ 10% has to be
deducted at source under section 194J in respect of the sum of ` 19,000 paid by ABC Ltd. to its director.
(B) Section 194-IA, tax shall be deducted at source if the amount paid or payable is `50,00,000 or more
hence in the given case tax deducted at source amounting to `60,00,000 x 1% = 60,000.

NOV – 2012
Question No. 7(a) (4 Marks)
(i) Mr. X doing textiles business furnishes you the following information:
Total turnover for the financial year:
`
2017-18 201,00,000
2018-19 95,00,000
Deduction Of Tax At Source 356

State whether the provisions of tax deduction at source are attracted for the following expenses incurred
during the financial year 2018-19;
Interest paid to Indian Bank on Term Loan 92,800
Advertisement expenses to Mr. X (two individual expense of `24,000 and `34,000) 58,000
Factory rent paid to C 1,85,000
Brokerage paid to B, a sub-broker 16,000
Answer.
As the turnover of Mr. X for F.Y.2017-18, i.e. ` 201 lakh, has exceeded the limit prescribed under section
44AB, he has to comply with the tax deduction provisions during the financial year 2018-19.
(i) TDS under section 194A is not attracted in respect of interest paid to a banking company.
(ii) Advertisement =TDS on `34,000 @ 1% = `340 (Sec 194C is applicable if single payment exceeds
`30,000)
(iii) Tax has to be deducted under section 194-I as the rent amount exceeds `1,80,000.
TDS @ 10% on ` 1,85,000 = `18,500
(iv) Tax has to be deducted under section 194-H as the commission exceeds `15,000.
TDS @ 5% on ` 16,000 = `800
NOV – 2011
Question 7 (4 Marks)
State the applicability of TDS provisions and TDS amount in the following cases:
(a) Rent paid for hire of machinery by ABC Ltd. to Mr. X `2,10,000.
(b) Fee paid to Dr. Y by X (HUF) `35,000 for surgery performed to a member of the family.
Answer.
(a) Since the rent paid for hire of machinery by ABC Ltd. to Mr. X exceeds `1,80,000, the provisions of
section 194-I for deduction of tax at source are attracted.
The rate applicable for deduction of tax at source under section 194-I on rent paid for hire of plant and
machinery is 2% assuming that Mr. X had furnished his permanent account number to ABC Ltd.
Therefore, the amount of tax to be deducted at source: = `2,10,000 x 2% = `4,200
(b) As per the provisions of section 194J, a Hindu Undivided Family is required to deduct tax at source on
fees paid for professional services only if the turnover of business or profession has exceeded the limit
mentioned under section 44AB during the financial year immediately preceding the relevant year.
However, if such payment made for professional services is exclusively for the personal purpose of any
member of Hindu Undivided Family, then, the liability to deduct tax is not attracted. Therefore, in the given
case, even if X (HUF) turnover has exceeded the limit prescribed u/s 44AB in the immediately preceding
financial year, the liability to deduct tax at source is not attracted.
NOV – 2011
Question 6 (6 Marks)
Compute amount of tax to be deducted at source on the following payments made by M/s ABC Ltd. during
the financial year 2018-19 as per the provisions of the Income-Tax Act, 1961.
Sr. No. Date Nature of Payment
(i) 01.10.2018 Payment of `2,00,000 to Mr. “X” a transporter who is having PAN
and who do not have more than 10 goods carriages.
(ii) 01.11.2018 Payment of fee for technical services of `45,000 to Mr. X who is
having PAN.
(iii) 30.12.2018 Payment of `25,000 to M/s X Ltd. for repair of building.
(iv) 01.01.2019 Payment of `2,00,000 made to Mr. Y for purchase of diaries made
according to specifications of M/s ABC Ltd. However, no material
was supplied for such diaries to Mr. Y by M/s ABC Ltd.
(v) 01.01.2019 Payment made `2,20,000 to Mr. Z for compulsory acquisition of his
house as per Law of the State Government.
(vi) 01.02.2019 Payment of commission of 25,000 to Mr. A.
Answer.
(i) No tax shall be deducted at source in case of payment to a transporter who has submitted his PAN.
Deduction Of Tax At Source 357

(ii) Tax shall be deducted at source under section 194J @ 10% because the total amount payable is
exceeding `30,000 and amount of TDS shall be = 45,000 x 10% = `4,500
(iii) It is covered under section 194C but payment is not exceeding `30,000 hence no tax shall be deducted
at source.
(iv) Tax shall not be deducted at source in case of purchase of goods.
(v) Since payment is not exceeding `2,50,000 hence no tax shall be deducted at source under section 194LA.
(vi) Tax shall be deducted at source u/s 194H and shall be = 25,000 x 5% = `1,250

MAY – 2011
Question 5 (2 Marks)
List any 5 instances where the tax deductible at source in terms of section 194A will not apply.
Answer.
The provisions of section 194A will not apply in the following cases where the –
(1) Interest is paid by a firm to a partner of the firm;
(2) Any interest being paid to Bank/LIC or other notified financial organizations
(3) Interest paid by a co-operative society (other than cooperative bonus) to their member;
(4) Interest on income tax refund or wealth tax refund etc.
(5) Income paid in relation to a Zero Coupon Bond.
Question 7 (4 Marks)
Mr. X doing manufacture and wholesale trade furnishes you the following information:
Total turnover for the financial year `
2017-18 215,00,000
2018-19 105,00,000
State whether tax deduction at source provisions are attracted for the below said expenses incurred during
the financial year 2018-19: `
Interest paid to UCO Bank 41,000
Contract payment to Mr. Y (2 contracts of `12,000 each) 24,000
Shop rent paid (one payee) 1,90,000
Commission paid to Mr. Z 18,000
Answer.
As the turnover of Mr. X for F.Y.2017-18, i.e. `215 lakh, has exceeded the monetary limit prescribed under
section 44AB, he has to comply with the tax deduction provisions during the financial year 2018-19, subject
to however the exemptions provided for under the relevant sections for applicability of TDS provisions.
Interest paid to UCO Bank
TDS under section 194A is not attracted in respect of interest paid to a banking company.
Contract payment of `24,000 to Mr. Y for 2 contracts of `12,000 each
TDS provisions under section 194C would not be attracted if the amount paid to a contractor does not
exceed ` 30,000 in a single payment or `1,00,000 in the aggregate during the financial year. Therefore, TDS
provisions under section 194C are not attracted in this case.
Shop Rent paid to one payee – Tax has to be deducted under section 194-I as the rental payment exceeds
`1,80,000.
Commission paid to Balu – Tax has to be deducted under section 194-H as the commission exceeds
`15,000.

NOV – 2010
Question 4 (4 Marks)
Explain the consequences of not deducting tax and paying to Govt. account under section 201 of the Income
Tax Act, 1961.
Refer Answer given in the Chapter
Deduction Of Tax At Source 358

MAY – 2010
Question 4 (2 Marks)
State the concessions granted to transport operators in the context of deduction of tax at sources under
section 194-C.
Answer. No deduction is required to be made from any sum credited or paid or likely to be credited or paid
during the previous year to the account of a contractor, during the course of the business of plying, hiring or
leasing goods carriages, if the contractor furnishes his permanent account number (PAN) to the person
paying or crediting such sum and submitted a declaration that he don’t own more than 10 goods carriages at
any time during the year.

MAY – 2010
Question 4 (4 Marks)
State with reasons, whether tax deduction at source provisions are applicable to the following transactions
and if so, the rate of tax deduction:
(i) X & Co. (Firm) engaged in wholesale business assigned a contract for construction of its godown
building to Mr. X, a contractor. It paid `25,00,000 to Mr. X as contract payment.
(ii) Y & Co. engaged in real estate business conducted a lucky dip and gave Maruti car to a prize winner.
(iii) An Insurance Company paid `45,000 as Insurance Commission to its agent Mr. Y.
(iv) AB Ltd. allowed a discount of `50,000 to XY & Co. (a firm) on prompt (immediate) payment towards
supply of automobile parts.
Answer.
(i) Section 194C provides for deduction of tax at source from the payment made to resident contractors and
sub-contractors. Therefore, tax is deductible at source under section 194C for the contract payments made
for the construction of godown building. The rate of TDS under section 194C on payments made to
contractors who are individuals or HUF shall be @ 1%. Hence, X & Co. (firm) must deduct tax at source on
the contract payments made to Mr. X.
(ii) In respect of lucky dip conducted by Y & Co., the provisions of Section 194B would apply. As per
Section 194B, winning from lottery or crossword puzzle or card game or other game of any sort exceeding
`10,000 payable by any person to any other person, subject to tax deduction at the rate of 30%. Since the
value of prize i.e. Maruti car would exceed `10,000 tax is deductible at source @ 30%. As the winning is in
kind, the winner must deposit 30% of the prize value to Y & Co. for remitting the same as tax. Only after
such deduction / recovery, the Maruti car is to be delivered to the prize winner.
(iii) As per section 194D, any person paying insurance commission in excess of `15,000 to any resident
person is liable to deduct tax at the rate of 5% in case of all assesses. Therefore, the insurance company must
deduct tax at source @ 5% in respect of the insurance commission paid to Mr. Y.
(iv) Discount allowed to a customer for prompt payment is not covered by any of the tax deduction at source
provisions of the Income tax Act, 1961. Therefore, AB Ltd. need not deduct any tax at source since no
payment was involved in allowing discount to its customer viz. (namely) XY & Co.

NOV – 2009
Question 2 (6 Marks)
st
From the following particulars of Mr. X for the previous year ended 31 March, 2019 compute total income
and tax liability for assessment year 2019-20.
`
(i) Directors Fee from a company 3,00,000
(ii) Interest on saving bank deposits 23,000
(iii) Income from undisclosed source 12,000
(iv) Winnings from Lotteries (Net) 35,000
(v) Royalty on a book written by him 2,09,000
(vi) Lectures in Seminars 5,000
(vii) Interest on loan given to a relative 7,000
(viii) Interest on debentures of a company (listed in a Recognised Stock Exchange) net of 36,000
Deduction Of Tax At Source 359

TDS
(ix) Interest on Post Office Savings Bank Account 500
(x) Interest on Government Securities 2,200
(xi) Interest on Monthly Income Scheme of Post office 33,000
He paid `10,000 for typing the manuscript of book written by him.
Answer.
Computation of income of Mr. X chargeable under the head “Income from other sources” for the
A.Y. 2019-20
Particulars `
1. Directors’ fees 3,00,000.00
2. Interest on bank deposit 23,000.00
3. Income from undisclosed source 12,000.00
4. Royalty on books written (2,09,000 – 10,000) 1,99,000.00
5. Lectures in seminars 5,000.00
6. Interest on loan given to a relative 7,000.00
7. Interest on listed debentures
36,000/ 90% x 100% 40,000.00
8. Interest on Post Office Savings Bank [exempt under section 10(15)] -
9. Interest on Government securities 2,200.00
10. Interest on Post Office Monthly Income Scheme 33,000.00
11. Winnings from lotteries
35,000 / 70% x 100% 50,000.00
Income from Other Sources 6,71,200.00
Gross Total Income 6,71,200.00
Less: Deduction u/s 80TTA (10,000.00)
Total Income 6,61,200.00
Computation of Tax Liability
Tax on casual income `50,000 @ 30% u/s 115BB 15,000.00
Tax on `6,11,200 at slab rate 34,740.00
Tax before health & education cess 49,740.00
Add: HEC @ 4% 1,989.60
Tax Liability 51,729.60
Rounded off u/s 288B 51,730.00
Question 4 (4 Marks)
Explain the consequences of not deducting tax and paying to Govt. account under section 201 of the Income
Tax Act, 1961.
Refer Answer given in the Chapter
NOV – 2009
Question 1 (2 Marks)
State with reasons, whether the following statements are true or false having regard to the provisions of the
Income-tax Act, 1961, for the assessment year 2019-20:
An individual having gross receipts of `110 lacs during the financial year 2017-18 is not required to deduct
tax at source under section 194C of the Income-tax Act, 1961, on payment made to contractors during the
financial year 2018-19.
Answer.
False: An individual whose total sales/gross receipts/turnover from the business or profession exceeds the
monetary limits specified in section 44AB during the financial year immediately preceding the financial year
in which such sum is credited or paid to the account of the contractor shall be required to deduct tax at
source.
Deduction Of Tax At Source 360

Question 3 (4 Marks)
Mrs. X, a landlord, derived income from rent from letting a house property to M/s ABC Corporation Ltd. of
`1,00,000 per month. Calculate the deduction of tax at source (TDS) to be made by M/s ABC Corporation
Ltd. on payment made to Mrs. X and narrate related formalities in relation to TDS.
Answer.
(1) Tax deducted at source under section 194-I would be required to be made on the amount of rent i.e. on
`12,00,000.
(2) TDS shall be applicable @ 10%.
(3) Hence, in the given case, TDS under section 194-I @ 10% on `12,00,000 comes to `1,20,000 i.e.
`10,000 will be deducted every month and cheque for net amount of (`1,00,000 – `10,000 = `90,000) will
be issued by M/s. ABC Corporation Ltd. to Mrs. X.
(4) Tax deducted will be deposited within one week from the last day of the month in which deduction is
made but if the tax has been deducted in the month of March, tax should be deposited on or before 30th
April.
(5) Form No. 16-A has to be issued by M/s. ABC Corporation Ltd. to Mrs. X within 15 days from the last
date of submitting the quarterly return under section 200.
(6) Assessee should file quarterly statement upto 31st of the month succeeding the relevant quarter but for
the last quarter ending March, return can be submitted upto 31st May.

JUNE – 2009
Question 1 (2 Marks)
When will tax not required to be deducted at source on interest payable to a resident on any bond or security
issued by a company though the aggregate amount of interest exceeds `5,000, the basic exemption limit
under section 193 of the Act?
Answer. As per section 193 of the Act, no tax is required to be deducted at source on any interest payable to
a resident on any bond or security issued by a company, where the following conditions are satisfied –
(i) where such security is in dematerialised form and
(ii) is listed on a recognised stock exchange in India.
MAY – 2008
Question 1 (2 Marks)
Mrs. X has made payments of ` 5 lacs to a contractor (for business purposes) during the last two quarters of
the year ended 31.03.2019. Her turnover for the year ended 31.03.2018 was `215 lacs. Is there any
obligation to deduct tax at source?
Answer. In the case of an individual, the provisions of section 194C shall apply, where the turnover from
business has exceeded the prescribed limit u/s 44AB during the financial year immediately preceding the
financial year in which such payment is made and payment is made for a purpose other than the personal
purpose. In the given case, since the turnover of Mrs. X has exceeded the prescribed limit for the year ended
31st March 2018 and the payment is relating to business activities, she shall be liable to deduct tax at source
in respect of payment made to a contractor at the applicable rate.

NOV – 2007
Question 5 (4 Marks)
Briefly explain the provisions of section 197 in respect of obtaining certificate for deduction of tax at a
lower rate.
Refer Answer given in the Chapter
MAY – 2007
Question 5 (4 Marks)
What are the consequences of failure to deduct tax at source or pay the tax deducted at source to the credit of
Central Government?
Answer. Refer to Answer given in NOV – 2010 Question No.4
Deduction Of Tax At Source 361

T.D.S./TCS TAX CHALLAN

CHALLAN NO. Tax Applicable(Tick One)*


ITNS Assessment Year

281 TAX DEDUCTED/COLLECTED AT SOURCE FROM

(0020) Company (0021) Non-Company


-
Deductees Deductees

Tax Deduction Account No.(T.A.N.)

Full Name

Complete Address with City & State

Tel. No. Pin


Type of Payment
(Tick One) FOR USE IN
TDS/TCS Payable by Taxpayer (200) RECEIVING BANK
TDS/TCS Regular Assessment (Raised by I.T. Deptt.) (400)
DETAILS OF PAYMENTS Amount (in Rs. Only) Debit to A/c / Cheque credited on
Income Tax
_
Surcharge
Health & education cess D D M M Y Y
Interest
Penalty SPACE FOR BANK SEAL
Total
Total (in words)
CRORES LACS THOUSANDS HUNDREDS TENS UNITS

Paid in Cash/Debit to A/c /Cheque No. Dated


Drawn on
(Name of the Bank and Branch)
Date:
Signature of person making payment Rs.
Taxpayers Counterfoil (To be filled by the taxpayer) SPACE FOR BANK SEAL
TAN
Received From

(Name)
Cash/Debit to A/c / Cheque No. For Rs.
Rs. (in words)
drawn on
(Name of the Bank and Branch)
Company/Non-Company Deductees
on account of Tax Deducted at Source(TDS)/Tax Collected at
Source(TCS)
from___________(Fill up Code)
(Strike out whichever is not applicable)
For the Assessment Year - Rs.
Set Off and Carry Forward Of Losses 362

SET OFF AND CARRY FORWARD OF


LOSSES
SECTION 70 TO 80
PARTICULARS SECTIONS
Set off of loss from one source against income from another source under the same 70
head of income
Set off of loss from one head against income from another head 71
Carry forward and set off of loss from house property 71B
Carry forward and set off of business losses 72
Losses in speculation business 73
Losses under the head “Capital gains” 74
Losses from certain specified sources under the head “Income from other sources” 74A
Question 1 [V. Imp.]: Write a note on Set Off and Carry Forward of Losses under the head House
Property.
Answer: As per section 70, if any person has loss from any house property, such loss can be set off from
income of any other house property and it is called inter-source adjustment or intra-head adjustment. E.g.
Mr. X has two houses: there is loss of `34,000 from one house and income of `80,000 from the other house,
in this case, loss of one source (house) can be set off from income of the other source (house).
As per section 71, unadjusted loss can be set off from incomes of other heads but as per section 58(4), such
loss can not be set off from casual income and it is called inter-head adjustment. E.g. Mr. X has loss from
house property `2,00,000 and income from business/profession `5,00,000, in this case, loss is allowed to be
set off but if he has any casual income, loss can not be set off from casual income.
Where in respect of any assessment year, the net result of the computation under the head “Income from
house property” is a loss and the assessee has income assessable under any other head of income, the
assessee shall not be entitled to set off such loss, to the extent the amount of the loss exceeds `2,00,000,
against income under the other head.’.
As per section 71B, unadjusted loss is allowed to be carried forward to the subsequent years but for a
maximum period of 8 years starting from the year subsequent to the year in which the loss was incurred and
in the subsequent years, loss can be set off only from income under the head house property. E.g. Mr. X has
incurred loss under the head house property in the previous year 2018-19/assessment year 2019-20 and it
could not be set off in the same year, it can be carried forward upto previous year 2026-27/assessment year
2027-28 (as shown below)
Year 1 Previous year 2019-20 Assessment Year 2020-21
Year 2 Previous year 2020-21 Assessment Year 2021-22
Year 3 Previous year 2021-22 Assessment Year 2022-23
Year 4 Previous year 2022-23 Assessment Year 2023-24
Year 5 Previous year 2023-24 Assessment Year 2024-25
Year 6 Previous year 2024-25 Assessment Year 2025-26
Year 7 Previous year 2025-26 Assessment Year 2026-27
Year 8 Previous year 2026-27 Assessment Year 2027-28
E.g. Mr. X has loss under the head house property of the previous year 2010-11/assessment year 2011-12
`5,00,000 and income under the head house property `5,00,000 in previous year 2018-19/assessment year
2019-20, in this case, loss shall be allowed to be set off because it will be allowed to be carried forward upto
a period of 8 years starting from previous year 2011-12/assessment year 2012-13 and is as shown below:
Year 1 Previous year 2011-12 Assessment Year 2012-13
Year 2 Previous year 2012-13 Assessment Year 2013-14
Set Off and Carry Forward Of Losses 363

Year 3 Previous year 2013-14 Assessment Year 2014-15


Year 4 Previous year 2014-15 Assessment Year 2015-16
Year 5 Previous year 2015-16 Assessment Year 2016-17
Year 6 Previous year 2016-17 Assessment Year 2017-18
Year 7 Previous year 2017-18 Assessment Year 2018-19
Year 8 Previous year 2018-19 Assessment Year 2019-20

If the loss can be set off, it has to be set off compulsorily i.e. it is not voluntary. E.g. Mr. X has loss under
the head house property `2,00,000 in previous year 2018-19/assessment year 2019-20 and income under the
head business/profession `2,50,000 in the same year, in this case loss has to be set off.
Question 2 [Imp.]: Write a note on Set Off and Carry Forward of Losses under the head Business/
Profession.
Answer: As per section 70, if any person has loss from any business/profession, such loss can be set off
from income of any other business/profession and it is called inter-source adjustment or intra-head
adjustment. E.g. Mr. X has two business: there is loss of `5,00,000 from one business and income of
`10,00,000 from the other business, in this case, loss of one source (business) can be set off from income of
the other source (business).
As per section 71, unadjusted loss can be set off from incomes of other heads except salary but as per
section 58(4), such loss can not be set off from casual income and it is called inter-head adjustment. E.g. Mr.
X has loss from business/profession `3,00,000 and income from house property `5,00,000, in this case, loss
is allowed to be set off but if he has any casual income, loss can not be set off from casual income. Similarly
it can not be set off from income under the head salary.
As per section 72, unadjusted loss is allowed to be carried forward to the subsequent years but for a
maximum period of 8 years starting from the year subsequent to the year in which the loss was incurred and
in the subsequent years, loss can be set off only from income under the head business/profession. E.g. Mr. X
has incurred loss under the head business/profession in the previous year 2018-19/assessment year 2019-20
and it could not be set off in the same year, it can be carried forward upto previous year 2026-27/assessment
year 2027-28 (as shown below)
Year 1 Previous year 2019-20 Assessment Year 2020-21
Year 2 Previous year 2020-21 Assessment Year 2021-22
Year 3 Previous year 2021-22 Assessment Year 2022-23
Year 4 Previous year 2022-23 Assessment Year 2023-24
Year 5 Previous year 2023-24 Assessment Year 2024-25
Year 6 Previous year 2024-25 Assessment Year 2025-26
Year 7 Previous year 2025-26 Assessment Year 2026-27
Year 8 Previous year 2026-27 Assessment Year 2027-28

Loss of the current year shall be adjusted first and only after that brought forward loss can be adjusted. E.g.
Mr. X has loss under the head house property `2,00,000 and income from business/profession `2,00,000 in
previous year 2018-19 and also brought forward loss of business/profession `2,00,000 of previous year
2010-11, in this case, loss of house property shall be adjusted from income of business/profession and loss
of previous year 2010-11 shall lapse.
Illustration 1: Mr. X has loss under the head House Property `5,00,000 and income under the head Salary
`3,00,000 and income under the head Business/Profession `10,00,000 and LTCG `6,00,000 and Casual
income `2,00,000 and unadjusted loss of Business/Profession `4,00,000 of Assessment Year 2014-15 and
loss of Business/Profession `3,00,000 of Assessment Year 2015-16.
Compute his tax liability for assessment year 2019-20.
Solution: `
The assessee has the option to adjust loss of house property from normal income or from LTCG.
Option I
Loss of house property is set off from normal income
Income under the head Salary 3,00,000.00
Less: loss of house property (2,00,000.00)
Set Off and Carry Forward Of Losses 364

Income under the head Salary 1,00,000.00


Income under the head Business/Profession 10,00,000.00
Less: Brought forward business loss of A.Y. 2014-15 (4,00,000.00)
Less: Brought forward business loss of A.Y. 2015-16 (3,00,000.00)
Income under the head Business/Profession 3,00,000.00
Long term capital gain 6,00,000.00
Casual Income 2,00,000.00
Gross Total Income 12,00,000.00
Less: Deduction u/s 80C to 80U Nil
Total Income 12,00,000.00
Computation of Tax Liability
Tax on LTCG `6,00,000 @ 20% 1,20,000.00
Tax on Casual income `2,00,000 @ 30% 60,000.00
Tax on Normal income `4,00,000 at slab rate 7,500.00
Tax before health & education cess 1,87,500.00
Add: HEC @ 4% 7,500.00
Tax Liability 1,95,000.00
Option II
Loss of house property is set off from LTCG
Income under the head Salary 3,00,000.00
Income under the head Business/Profession 10,00,000.00
Less: Brought forward business loss of A.Y. 2014-15 (4,00,000.00)
Less: Brought forward business loss of A.Y. 2015-16 (3,00,000.00)
Income under the head Business/Profession 3,00,000.00
Long term capital gain 6,00,000.00
Less: loss of house property (2,00,000.00)
Long term capital gain 4,00,000.00
Casual Income 2,00,000.00
Gross Total Income 12,00,000.00
Less: Deduction u/s 80C to 80U Nil
Total Income 12,00,000.00
Computation of Tax Liability
Tax on LTCG `4,00,000 @ 20% 80,000.00
Tax on Casual income `2,00,000 @ 30% 60,000.00
Tax on Normal income `6,00,000 at slab rate 32,500.00
Tax before health & education cess 1,72,500.00
Add: HEC @ 4% 6,900.00
Tax Liability 1,79,400.00
Option II is better.
Illustration 2: Mrs. X has income and losses as given below:
1. Income under the head Salary ` 5,00,000
2. Loss under the head House Property `10,00,000
3. Income under the head Business/Profession `12,00,000
4. Income from STCG ` 2,00,000
5. Income from STCG u/s 111A `10,00,000
6. Casual Income ` 3,00,000
Deduction under section 80C to 80U ` 1,00,000
Brought forward Business/Profession loss for
- Previous year 2009-10 ` 3,00,000
- Previous year 2011-12 ` 6,00,000
- Previous year 2012-13 ` 3,00,000
Compute tax liability of Mrs. X for Assessment Year 2019-20.
Set Off and Carry Forward Of Losses 365

Solution: `
Option I
Loss of house property is set off from normal income
Income under the head Salary 5,00,000.00
Less: Loss of house property (2,00,000.00)
Income under the head Salary 3,00,000.00
Income under the head Business/Profession 12,00,000.00
Less: Brought forward business/profession loss P.Y. 2011-12 (6,00,000.00)
Less: Brought forward business/profession loss P.Y. 2012-13 (3,00,000.00)
Income under the head Business/Profession 3,00,000.00
Short term capital gain 2,00,000.00
Short term capital gain u/s 111A 10,00,000.00
Casual Income 3,00,000.00
Gross Total Income 21,00,000.00
Less: Deduction u/s 80C to 80U (1,00,000.00)
Total Income 20,00,000.00
Computation of Tax Liability
Tax on STCG u/s 111A `10,00,000 @ 15% 1,50,000.00
Tax on Casual income `3,00,000 @ 30% 90,000.00
Tax on Normal income `7,00,000 at slab rate 52,500.00
Tax before health & education cess 2,92,500.00
Add: HEC @ 4% 11,700.00
Tax Liability 3,04,200.00
Option II
Loss of house property is set off from STCG u/s 111A
Income under the head Salary 5,00,000.00
Income under the head Business/Profession 12,00,000.00
Less: Brought forward business/profession loss P.Y. 2011-12 (6,00,000.00)
Less: Brought forward business/profession loss P.Y. 2012-13 (3,00,000.00)
Income under the head Business/Profession 3,00,000.00
Short term capital gain 2,00,000.00
Short term capital gain u/s 111A 10,00,000.00
Less: loss of house property (2,00,000.00)
Short term capital gain u/s 111A 8,00,000.00
Casual Income 3,00,000.00
Gross Total Income 21,00,000.00
Less: Deduction u/s 80C to 80U (1,00,000.00)
Total Income 20,00,000.00
Computation of Tax Liability
Tax on STCG u/s 111A `8,00,000 @ 15% 1,20,000.00
Tax on Casual income `3,00,000 @ 30% 90,000.00
Tax on Normal income `9,00,000 at slab rate 92,500.00
Tax before health & education cess 3,02,500.00
Add: HEC @ 4% 12,100.00
Tax Liability 3,14,600.00
Option I is better.

Question 3 [V. Imp.]: Write a note on Unabsorbed Depreciation.


Answer: Unabsorbed Depreciation Section 32(2)
An assessee having business or profession shall debit all expenditures of business/profession before debiting
depreciation i.e. depreciation shall be debited at the end. If there is a loss by debiting other expenditure, it
will be called loss under the head business/profession. Depreciation shall be debited only if income is
available under the head business/profession and the depreciation which can not be debited shall be called
Set Off and Carry Forward Of Losses 366

unabsorbed depreciation and it will be allowed to be adjusted from any income under any head except casual
income.
If it can not be adjusted in the same year, its carry forward is allowed for unlimited period and in the
subsequent years, it can be set off from any income under any head except casual income.
If any assessee has brought forward business loss as well as depreciation, business loss shall be adjusted first
and depreciation afterwards.
E.g. Mr. X has income under the head business/profession `10,00,000 after debiting all expenditures except
depreciation of `13,00,000, in this case, depreciation of only `10,00,000 can be debited to the profit and loss
account and balance `3,00,000 shall be called unabsorbed depreciation and it can be set off from any income
under any head except casual income and even in the subsequent years, it can be set off from any income
under any head except casual income and such carry forward is allowed for unlimited periods.
Question 4 [V. Imp.]: Write a note on Set Off and Carry Forward of loss from Speculative Business.
Answer: As per section 28, income from speculative business shall be taxable under the head
business/profession and such income shall be computed in the normal manner and shall be taxable at the
normal rate i.e. it will not be considered to be causal income.
Meaning of speculative business Section 43(5)
"Speculative Transaction" means a transaction in which a contract for the purchase or sale of any
commodity, including stocks and shares, is settled otherwise than by the actual delivery or transfer of the
commodity or scrips i.e. the transactions are set off against each other instead of actually materializing them
e.g. Mr. X entered into a contact for purchase of one plot from Mr. A and same plot was sold by him to Mr.
Y at a higher rate and he has directed Mr. Y to pay the amount directly to Mr. A and surplus amount to Mr.
X and he directed Mr. A to transfer the plot directly in the name of Mr. Y, it will be called speculative
transaction but if Mr. X has transferred the plot in his name and after that plot was transferred in the name of
Mr. Y, it will be called normal business.
Set Off and Carry forward of loss from Speculative Business Section 73
If any assessee has loss from speculative business, such loss can not be set off from any income under any
head however, if the assessee has two or more similar business, loss of one such business can be set off from
the income of other such business.
Unadjusted loss is allowed to be carried forward but for a maximum period of 4 years starting from the year
subsequent to the year in which the loss was incurred. Even in the subsequent years, loss can be set off only
from income of speculative business.
Loss under the head house property, loss from normal business, unabsorbed depreciation, loss under the
head other sources can be set off from the income of speculative business.
e.g. Mr. X has loss from speculative business `5,00,000 and income from normal business `5,00,000, in this
case, loss is not allowed to be set off however its carry forward is allowed but for a maximum period of 4
years and in the subsequent years its can be set off only from income of speculation business.
e.g. Mr. X has loss from house property `5,00,000 and income from speculative business `5,00,000, in this
case, loss can be set off from income of speculative business.
e.g. Mr. X has loss of speculative business `5,00,000 and income from some other speculative business
`5,00,000, in this case, loss can be set off from income of such speculative business.
Question 5 [V. Imp.]: Write a note on Set Off and Carry Forward of losses under the head Capital
Gains.
Answer: Set off and Carry forward of Loss under the head Capital Gain Section 70, 71 and 74
Set off of loss from one source against income from another source under the same head of income
Section 70
If any person has short term loss, it can be set off either from short term or from long term gain but if any
person has long term loss, it can be set off only from long term gains i.e. set off from short term is not
allowed.
Example
Mr. X has long term loss of `5,00,000 and short term gain of `5,00,000, in this case, long term loss can not
be set off from short term gains and his tax liability shall be `13,000 but if he has long term gain of
`5,00,000 and short term loss of `5,00,000, in this case, set off is allowed and his tax liability shall be nil.
e.g. If Mr. X has short term loss under section 111A and has long term gain, such loss can be set off from
Set Off and Carry Forward Of Losses 367

long term gain and also it can be set off from short term gain under section 111A or from normal short term
gain.
Set off of loss from one head against income from another Section 71
Where in respect of any assessment year, the net result of the computation under the head “Capital gains” is
a loss and the assessee has income assessable under any other head of income, the assessee shall not be
entitled to have such loss set off against income under the other head.
Example
Mr. X has short term loss `5,00,000 and business income of `5,00,000, his tax liability shall be `13,000
Carry forward of losses under the head “Capital Gains” Section 74
If any assessee has short term loss or long term loss which could not be set off, such losses shall be allowed
to be carried forward but for a maximum period of 8 years starting from the year next to the year in which
the loss was incurred.
Brought forward short term loss can be set off either from short term or from long term gain but brought
forward long term loss can be set off only from long term gain and not from short term gain.
Question 6: Write a note on Set Off and Carry Forward of Losses under the head “Income from
Other Sources”.
Answer: Set off and Carry forward of Losses under the head “Income from Other Sources”
As per section 70, if the assessee has loss under the head other sources, such loss is allowed to be set off
from any income under the same head except casual income.
As per section 71, unadjusted loss can be set off even from incomes of other heads.
Carry forward of the loss is not allowed.
Set Off and Carry Forward of losses in connection with Owning and Maintaining of Race Horses
Section 74A
If any assessee has loss from the business of owning and maintaining race horses, such loss can not be set
off from any income under any head but if the assessee has two or more business of owning and maintaining
race horses, loss of one such business can be set off from the income of other such business.
Unadjusted loss is allowed to be carried forward but for a maximum period of 4 years starting from the year
subsequent to the year in which the loss was incurred and even in the subsequent year, the loss can be set off
only from income of business of owning and maintaining race horses.
If the assessee has business of owning and maintaining any other race animal, income shall be taxable under
the head business/profession and loss will have the same treatment as in case of loss under the head
business/profession. E.g. If Mr. X has loss from business of owning and maintaining race camels, the loss
will have the same treatment as the loss under the head business/profession.
Additional points:
1. Set off and carry forward is mandatory not voluntary. e.g. Mr. X has loss under the head house property
`50,000 and income under the head business/profession `50,000 during the assessment year 2019-20, in this
case, loss of house property has to be set off from business income of assessment year 2019-20 and Mr. X is
not allowed to carry forward the loss of house property.
2. Any loss has to be set off first within the same head and after that under some other heads and after that
carry forward is allowed.
3. Loss of current year shall be set off first and only after that brought forward losses can be adjusted, eg.
Mr. X has income from one house ` 10,00,000 and loss from other house ` 10,00,000 in P.Y. 2018-19 and
also unadjusted loss of ` 10,00,000 under the head house property of P.Y. 2010-11, in this case loss of
current year is to be adjusted first.
Question 7: Explain setoff and carry forward of loss from agriculture
Answer: Loss from Agriculture
Loss from Agriculture cannot be set off from incomes of other heads. Similarly loss of other heads cannot be
set off from Agriculture Income.
Loss from agriculture can be set off only from agricultural income and carry forward is allowed for 8 years
and in subsequent years also it can be set off from agriculture income.
Set Off and Carry Forward Of Losses 368

Illustration 4: Mr. A submits the following particulars pertaining to the A.Y.2019-20:


Particulars `
Income under the head salary 4,00,000
Loss from self-occupied property (-) 70,000
Business loss (-) 1,00,000
Bank interest (FD) received 80,000
Compute the taxable income of Mr. A for the A.Y.2019-20.
Solution: Computation of taxable income of Mr. A for the A.Y. 2019-20
Particulars Amount
(`)
Income under the head Salary 4,00,000
Less: Loss from house property (70,000)
3,30,000
Income under the head Other Sources
Income from other sources (interest on fixed deposit with bank) 80,000
Less: Loss from business (1,00,000)
Business loss to be carried forward (20,000)
Gross total income 3,30,000
Less: Deduction under chapter VIA Nil
Taxable income 3,30,000
Illustration 5: Mr. B, a resident individual, furnishes the following particulars for the P.Y. 2018-19:
Particulars `
Income under the head Salary 45,000
Loss from house property (24,000)
Loss from business – non-speculative (22,000)
Loss from speculative business (4,000)
Short-term loss (25,000)
Long-term capital gains 19,000
What is the total income chargeable to tax for the A.Y. 2019-20?
Solution: The total income chargeable to tax for the A.Y. 2019-20 is calculated as under:
Particulars Amount `
Income under the head salaries 45,000
Less: Loss from house property (24,000)
21,000
Profits and gains of business and profession
Business loss to be carried forward [Note 1] (22,000)
Speculative loss to be carried forward [Note 2] (4,000)
Capital Gains
Long term capital gain 19,000
Less: Short term capital loss (25,000)
Short term capital loss to be carried forward [Note 3] (6,000)
Gross Total Income 21,000
Note 1: Business loss cannot be set-off against salary income. Therefore, loss of ` 22,000 from the non-
speculative business cannot be set off against the income from salaries. Hence, such loss has to be carried
forward to the next year for set-off against business profits, if any.
Note 2: Loss of ` 4,000 from the speculative business can be set off only against the income from the
speculative business. Hence, such loss has to be carried forward.
Note 3: Short term capital loss can be set off against both short term capital gain and long term capital gain.
Therefore, short term capital loss of ` 25,000 can be set-off against long-term capital gains to the extent of
`19,000. The balance short term capital loss of ` 6,000 cannot be set-off against any other income and has to
be carried forward to the next year for set-off against capital gains, if any.
Set Off and Carry Forward Of Losses 369

Illustration 6: During the P.Y. 2018-19, Mr. C has the following income and the brought forward losses:
Particulars `
Short term capital gains on sale of shares 1,50,000
Long term capital loss of A.Y. 2017-18 (96,000)
Short term capital loss of A.Y. 2018-19 (37,000)
Long term capital gain 75,000
What is the capital gain taxable in the hands of Mr. C for the A.Y.2019-20?

Solution: The capital gains taxable are as under:


Particulars ` `
Short term capital gains on sale of shares 1,50,000
Less: Brought forward short term capital loss of the A.Y.2018-19 (37,000) 1,13,000
Long term capital gain 75,000
Less: Brought forward long term capital loss of A.Y.2017-18 [See Note below] (75,000) Nil

Taxable short-term capital gains 1,13,000

Note: Long-term capital loss cannot be set off against short-term capital gain. Hence, the unadjusted long
term capital loss of A.Y. 2017-18 of ` 21,000 (i.e. `96,000 – `75,000) has to be carried forward to the next
year to be set-off against long-term capital gains of that year.

Illustration 7: Mr. D has the following income for the P.Y. 2018-19 –
Particulars `
Income from the activity of owning and maintaining the race horses 75,000
Income from textile business 85,000
Brought forward textile business loss 50,000
Brought forward loss from the activity of owning and maintaining the race horses (relating to 96,000
A.Y.2016-17)
What is the taxable income in the hands of Mr. D for the A.Y. 2019-20?
Solution: The taxable income is calculated as under:
Particulars ` `
Income from the activity of owning and maintaining race horses 75,000
Less: Brought forward loss from the activity of owning and maintaining race horses (96,000)
Loss from the activity of owning and maintaining race horses to be carried forward to (21,000)
A.Y.2020-21
Income from textile business 85,000
Less: Brought forward business loss from textile business. (50,000) 35,000
Taxable business income 35,000
Note: Loss from the activity of owning and maintaining race horses cannot be set-off against any other
source/head of income.

Illustration 8: Mr. E has furnished his details for the A.Y.2019-20 as under:
Particulars `
Income from salaries 1,50,000
Income from speculation business 60,000
Loss from non-speculation business (40,000)
Short term capital gain 80,000
Long term capital loss of A.Y.2017-18 (30,000)
Winning from lotteries 20,000
What is the taxable income of Mr. E for the A.Y. 2019-20?
Set Off and Carry Forward Of Losses 370

Solution: Computation of taxable income of Mr. E for the A.Y.2019-20


Particulars ` `
Income from salaries 1,50,000
Income from speculation business 60,000
Less: Loss from non-speculation business (40,000) 20,000
Short-term capital gain 80,000
Winning from lotteries 20,000
Taxable income 2,70,000
Note: Long term capital loss can be set off only against long term capital gain. Therefore, long term capital
loss of ` 30,000 has to be carried forward to the next assessment year.

Illustration 9: Compute the gross total income of Mr. F for the A.Y.2019-20 from the information given
below –
Particulars `
Net income from house property 1,25,000
Income from business (before provided for depreciation) 1,35,000
Short term capital gains on sales of shares 56,000
Long term capital loss from sale of property (brought forward from A.Y. 2018-19) (90,000)
Income from tea business 1,20,000
Dividend from Indian companies carrying on agricultural operations 80,000
Current year depreciation 26,000
Brought forward business loss (loss incurred six years ago) (45,000)

Solution:
The gross total income of Mr. F for the A.Y. 2019-20 is calculated as under:
Particulars ` `
Income from house property 1,25,000
Income from business
Profits before depreciation 1,35,000
Less: Current year depreciation (26,000)
Less: Brought forward business loss (45,000)
64,000
48,000

Income from tea business (40% is business income) 56,000 1,12,000


Income from the capital gains Nil
Short term capital gains
Long term capital loss from property (cannot be set off) 56,000
Gross Total Income 2,93,000

Note: Dividend from Indian companies is exempt from tax. 60% of the income from tea business is treated
as agricultural income and therefore, exempt from tax.
Set Off and Carry Forward Of Losses 371

EXAMINATION QUESTIONS
MAY – 2017
Question 6(a)(i) (4 Marks)
Mr. Shyam, a resident of Chandigarh, provides the following information for the financial year 2018-19:

Particulars `
Income from textile business 4,60,000
Income from speculation business 25,000
Loss from gambling 12,000
Loss on maintenance of race horse 15,000
Eligible current year depreciation of textile business not adjusted in the income given 5,000
above.
Unabsorbed depreciation of Assessment year 2018-19 brought forward 10,000
Speculation business loss of Assessment year 2018-19 30,000

Compute the Gross total Income of Mr. Shyam for the assessment year 2019-20 and any other item of
expense or loss eligible for carry forward.
Solution:
Computation of Gross Total Income `
Income from Textile Business 4,60,000
Less: Current year depreciation (5,000)
Less: Unabsorbed depreciation (10,000)
Income From Textile Business 4,45,000

Income from speculation business 25,000


Less: Brought forward speculation loss (Section 73) (25,000)
Income from Speculation business Nil

Gross Total Income 4,45,000

Note:
(1) As per Section 73, Unadjusted Brought Forward Speculation loss of A.Y. 2018-19 shall be carried
forward of `5,000.
(2) Loss from Gambling shall not be treated as loss and have no treatment.
(3) Loss on maintenance of race horse shall be allowed to be set off from income of maintenance of race
horse only and unadjusted loss of ` 15,000 shall be carried forward for 4 years as per section 74A.

NOV – 2015
Question 4(a). (4 Marks)
Mr. X provides the following details for the previous year ending 31.03.2019.
(i) Salary from XYZ Ltd. `50,000 per month
(ii) Interest on FD with SBI for the Financial Year 2018-19 `72,000 (Net of TDS)
(iii) Determined long term capital loss of AY 2017-18 `96,000
(iv) Long term Capital gain `75,000
(v) Loss of minor son `90,000 computed in accordance with the provisions of Income Tax Act. Mr. X
transferred his own house to his minor son without adequate consideration few years back and minor son let
it out and suffered loss.
(vi) Loss of his wife’s business `(2,00,000)
She carried business with funds which Mr. X gifted to her.
You are required to compute taxable income of Mr. X for the AY 2019-20.
Set Off and Carry Forward Of Losses 372

Solution 4(a):
Computation of taxable income of Mr. X for the A.Y. 2019-20 `
Income under the head Salary 6,00,000.00
Less: Loss under the head house property (Loss of minor son) (90,000.00)
Income under the head Salary 5,10,000.00
Income under the head capital Gains
Long Term Capital Gain 75,000.00
Less: Loss from Business of his wife (75,000.00)
Income under the head capital Gains Nil
Income under the head other sources
Interest Income from Fixed Deposit 80,000.00
Less: Loss from Business of his wife (80,000.00)
Income under the head other sources Nil
(Balance amount of `45,000 to be carried forward)
Gross Total Income 5,10,000.00
Less: Deduction from 80C to 80U Nil
Total Income 5,10,000.00
Note: 1. Mr. X shall be the deemed owner of the house property transferred to minor son hence it will be
considered to be loss of Mr. X.
2. Loss from business of Mrs. X shall also be clubbed
3. Brought Forward Long term capital loss of AY 2017-18 to be carried forward `96,000.
MAY – 2014
Question 6(A). (4 Marks)
Mr. X, a resident individual, furnishes the following particulars of his income and other details for the
previous year 2018-19. `
(1) Income from Salary 15,000
(2) Income from Business 66,000
(3) Long term capital gain on sale of Land 10,800
(4) Loss on maintenance of Race Horses 15,000
(5) Loss from Gambling 9,100
The other details of unabsorbed depreciation and brought forward losses pertaining to Assessment Year
2018-19 are as follows: `
(1) Unabsorbed depreciation 11,000
(2) Loss from Speculative business 22,000
(3) Short term capital loss 9,800
Compute the Gross total income of Mr. X for the Assessment Year 2019-20 and the amount of loss, if any,
that can be carried forward, or not.
Solution:
Computation of Gross Total Income of Mr. X for the A.Y. 2019-20
Particulars `
Income under the head Salary 15,000
Profits and gains of business or profession 66,000
Less : Unabsorbed depreciation brought forward from A.Y.2018-19 (11,000)
55,000
Capital gains
Long term capital gain on sale of land 10,800
Less : Brought forward short term capital loss (9,800)
1,000
Gross Total Income 71,000
Amount of loss to be carried forward to A.Y.2019-20
Particulars `
(1) Loss from speculative business shall be carried forward as per section 73 22,000
(2) Loss on maintenance of race horses shall be carried forward as per section 74A 15,000
Set Off and Carry Forward Of Losses 373

(3) Loss from gambling can neither be set-off nor be carried forward
Working Note:
Short-term capital loss can be set-off against both short-term capital gains and long-term capital gains as per
section 74
NOV – 2012
Question 5(a) (4 Marks)
Mr. X an assessee aged 61 years gives the following information for the previous year 31.03.2019:
Sr. Particulars `
a. Loss from profession 1,05,000
b. Capital loss on the sale of property-short term 55,000
c. Capital gains on sale of shares-long term 2,05,000
d. Loss in respect of self occupied property 15,000
e. Loss in respect of let out property 30,000
f. Share of loss from firm 1,60,000
g. Income from card games 55,000
h. Winnings from lotteries 1,00,000
i. Loss from horse races in Mumbai 40,000
j. Medical insurance premium paid by cheque 18,000
Compute the total income of Mr. X for the assessment year 2019-20.
Solution: `
Income under the head Capital Gains
Long term capital Gain 2,05,000
Less: Short term capital loss on sale of property (55,000)
Less: Loss from profession (1,05,000)
Less: Loss from House Property (45,000)
Income under the head Capital Gains Nil
Income under the head Other Sources
Winning from lottery 1,00,000
Income from card game 55,000
Income under the head Other Sources 1,55,000
Gross Total Income 1,55,000
Less: Deduction u/s 80D (Deductions are not allowed from casual incomes) Nil
Total Income 1,55,000
Working Notes:
1. Share of loss from firm is not allowed to be set off by the partner.
2. Loss from races can neither be set off nor be carried forward.
NOV – 2011
Question 1 (5 Marks)
Mr. X furnishes the following details for year ended 31.03.2019. `
Short term capital gain 1,40,000
Loss from speculative business (60,000)
Long term capital gain on sale of land 30,000
Long term capital loss on sale of shares 1,00,000
(securities transaction tax not paid)
Income from business of textile (after allowing current year depreciation) 50,000
Income from activity of owning and maintaining race horses 15,000
Income from salary 1,00,000
Loss from house property (40,000)
Following are the carry forward losses:
(i) Losses from activity of owning and maintaining race horses-pertaining to A.Y. 2016-17 `25,000.
(ii) Carry forward loss from business of textile `60,000- Loss pertains to A.Y. 2011-12.
Compute gross total income of Mr. X for the assessment year 2019-20.
Set Off and Carry Forward Of Losses 374

Also state the eligible carry forward losses for the assessment Year 2019-20.
Answer:
Calculation of Gross Total Income of Mr. X for the assessment year 2019-20 `
Income under the head Salary
Salary 1,00,000
Less: Loss from house property (40,000)
Income under the head Salary after set off 60,000
Income under the head Business/Profession
Income from Business of textile 50,000
Less: Loss Carried forward from textile business (A.Y. 2011-12) (50,000)
Balance loss of `10,000 shall lapse
Income under the head Capital Gains
Short Term Capital Gains 1,40,000
Long Term Capital Gains 30,000
Less: Long term loss (30,000)
(Balance of loss of 70,000 shall be carried forward)
Income under the head Other Sources
Income from owning and maintaining race horses 15,000
Less: Loss carried forward to be adjusted (A.Y. 2016-17) (15,000)
(Balance brought forward loss of `10,000 to be carried forward to next year)
Gross Total Income 2,00,000
Loss from speculative business of A.Y. 2019-20 `60,000 to be carried forward for 4 years starting from
assessment year 2020-21.
MAY – 2011
Question 4 (4 Marks)
The following are the details relating to Mr. X, a resident Indian, aged 57, relating to the year ended
31.03.2019: (`)
Income from salaries 2,20,000
Loss from house property (1,90,000)
Loss from cloth business (2,40,000)
Income from speculation business 30,000
Long-term capital gains from sale of urban land 2,50,000
Long-term capital loss from sale of listed shares in recognized stock exchange (STT paid) 90,000
Loss from card games (32,000)
Income from betting 45,000
Life insurance premium paid 1,20,000
Compute the total income and show the items eligible for carry forward.
Answer.
Computation of total income of Mr. X for the A.Y. 2019-20
Particulars ` `
Salaries
Income from salaries 2,20,000
Less: Loss from house property (1,90,000) 30,000
Profits and gains of business or profession
Income from speculation business 30,000
Less: Loss from cloth business set off (30,000) Nil
Capital gains
Long-term capital gains from sale of urban land 2,50,000
Less: Loss on sale of listed shares (90,000)
Less: Loss from cloth business set off (1,60,000) Nil
Income from other sources
Income from betting 45,000
Gross total income 75,000
Set Off and Carry Forward Of Losses 375

Less: Deduction under section 80C (life insurance premium paid) (30,000)
Total income 45,000
Losses to be carried forward
(1) Loss from cloth business (2,40,000-30,000-1,60,000) 50,000
Notes
(i) Business loss cannot be set off against salary income.
(ii) Loss from card games can neither be set off against any other income, nor can it be carried forward.
(iii) Income from betting is chargeable at a flat rate of 30% under section 115BB and no expenditure or
allowance can be allowed as deduction from such income, nor can any loss be set-off against such income.
Question 4 (4 Marks)
Mr. X furnishes the following information for the year ended 31.03.2019: `
Income from business (1,35,000)
Income from house property (15,000)
Lottery winning (Gross) 3,00,000
Speculation business income 1,00,000
Income by way of salary (computed) 60,000
Long term capital gain 70,000
Compute his total income, tax liability and advance tax obligations.
Solution: Computation of total income of Mr. X for the year ended 31.03.2019
Option 1: Loss of House property is set off from Normal Income

Particulars ` `
Salaries 60,000
Less: Loss from house property (15,000)
Net Salary (after set off of loss from house property) 45,000
Profits and gains of business or profession
Speculation business income 1,00,000
Less: Business loss set-off (1,35,000)
Net business loss to be set-off against long-term capital gain (35,000)
Capital Gains
Long term capital gain 70,000
Less: Business loss set-off (35,000)
Long term capital gain after set off of business loss 35,000
Income from other sources
Lottery winnings (Gross) 3,00,000
Total Income 3,80,000
Computation of tax liability
Particulars `
On Normal Income Nil (`45,000 - `45,000) at slab rate Nil
On LTCG Nil (`35,000 - `35,000) u/s 112 @ 20% Nil
On lottery winnings of `3,00,000 @ 30% 90,000
Add: HEC @ 4% 3,600
Total tax liability 93,600

The assessee need not pay advance tax since the total income (excluding lottery income) liable to tax is
below the basic exemption limit. Further, in respect of lottery income, tax would have been deducted at
source @ 30% under section 194B. Since the tax liability is less than `10,000, advance tax liability is not
attracted.

Option 2: Loss of House property is set off from LTCG


Particulars ` `
Salaries 60,000
Profits and gains of business or profession
Set Off and Carry Forward Of Losses 376

Speculation business income 1,00,000


Less: Business loss set-off (1,35,000)
Net business loss to be set-off against long-term capital gain (35,000)
Capital Gains
Long term capital gain 70,000
Less: Business loss set-off (35,000)
Less: Loss from House Property (15,000)
Long term capital gain after set off of loss 20,000
Income from other sources
Lottery winnings (Gross) 3,00,000
Total Income 3,80,000

Computation of tax liability


Particulars `
On Normal Income Nil (`60,000 - `60,000) at slab rate Nil
On LTCG Nil (`20,000 - `20,000) u/s 112 @ 20% Nil
On lottery winnings of `3,00,000 @ 30% 90,000
Add: HEC @ 4% 3,600
Total tax liability 93,600
The assessee need not pay advance tax since the total income (excluding lottery income) liable to tax is
below the basic exemption limit. Further, in respect of lottery income, tax would have been deducted at
source @ 30% under section 194B. Since the tax liability is less than `10,000, advance tax liability is not
attracted.
Since Tax Liability in both the options is same hence assessee can take any option.

NOV – 2010
Question 1 (5 Marks)
Mr. X submits the following details of his income for the assessment year 2019-20.
Income from salary 3,00,000.00
Loss from let out house property 40,000.00
Income from sugar business 50,000.00
Loss from iron ore business b/f (discontinued in 2012-13) 1,20,000.00
Short term capital loss 60,000.00
Long term capital gain 40,000.00
Dividend 5,000.00
Income received from lottery winning (Gross) 50,000.00
Winning in card games (Gross) 6,000.00
Agricultural income 20,000.00
Long term capital gain from shares (STT paid) 10,000.00
Short term capital loss under section 111A 10,000.00
Bank interest on fixed deposit 5,000.00
Calculate gross total income and losses to be carried forward.
Answer. Computation of gross total income of Mr. X for the A.Y. 2019-20
Particulars ` `
Salaries
Income from salary 3,00,000
Less: Loss from house property set-off against salary income (40,000) 2,60,000
as per section 71
Profits and gains of business or profession
Income from sugar business 50,000
Less: Brought forward loss from iron-ore business set-off as per (50,000) Nil
section 72
Set Off and Carry Forward Of Losses 377

Balance business loss of `70,000 of P.Y.2012-13 carried


forward to A.Y.2020-21
Capital gains
Long term capital gain 40,000
Less: Short term capital loss set-off (40,000) Nil
Balance short-term capital loss of `20,000 to be carried forward
Short-term capital loss of `10,000 under section 111A to be carried forward
Income from other sources
Winnings from lottery 50,000
Winnings from card games 6,000
Bank interest on fixed deposit 5,000 61,000
Gross Total Income 3,21,000
Losses to be carried forward to A.Y. 2020-21
Loss of iron-ore business 70,000
Short term capital loss 30,000
Notes:
1. The following income are exempt under section 10 –
(i) Dividend income [Exempt under section 10(34)], assuming that dividend is received from a domestic
company.
(ii) Agricultural income [Exempt under section 10(1)]
(iii) Long-term capital gains on which STT is paid [Exempt upto `1,00,000 under section 112A]
2. It is presumed that loss from iron-ore business relates to P.Y.2012-13, the year in which the business was
discontinued.
Question 1 (5 Marks)
Determine the total income of Mr. X from the following information for the Assessment Year 2019-20:
`
(i) Interest received on enhanced compensation (It relates to transfer of land in the financial 4,00,000
year 2013-14. Out of the above `65,000 relates to financial year 2018-19 and the balance
relate to preceding years)
(ii) Business loss relating to discontinued business of the assessment year 2013-14 brought 1,50,000
forward and eligible for set off.
(iii) Current year business income (i.e. financial year 2018-19) (Computed) 1,10,000
Answer.
Computation of total income of Mr. X for A.Y. 2019-20
Particulars Amount Amount
(`) (`)
Profits and gains of business or profession
Current year business income 1,10,000
Less: Brought forward business loss of discontinued business
`1,50,000 set-off to the extent of current year business
income as per section 72 (1,10,000) Nil
Income from other sources
Interest on enhanced compensation taxable on receipt basis
under section 56(2) 4,00,000
Less: Deduction under section 57 @ 50% (2,00,000) 2,00,000
Total Income 2,00,000
The unabsorbed business loss of `40,000 (`1,50,000 – `1,10,000) of A.Y. 2013-14 relating to discontinued
business will be carried forward for set-off against income from any business in the next year i.e. A.Y. 2020-
21.

MAY – 2010
Question 4 (6 Marks)
Mr. X furnishes you the following details for the year ended 31.03.2019:
Set Off and Carry Forward Of Losses 378

Income (loss) from house property `


House – 1 36,000
House – 2 Self occupied (20,000)
House – 3 60,000
Profits and gains from Business or Profession
Textile Business 2,00,000
Automobile Business (3,00,000)
Speculation Business 2,00,000
Capital Gains
Long-term capital gain from sale of shares (STT paid) 1,50,000
Long-term capital gain from sale of vacant site 2,00,000
Short-term capital loss from sale of building 1,00,000
(Note: Assume that the figures given above are computed and arrived at after considering eligible
deductions).
Other sources:
Gift from a Friend (non-relative) on 05.06.2018 60,000
Gift from Maternal Uncle on 25.02.2019 1,00,000
Gift from Grandfather’s Younger Brother on 10.02.2019 1,00,000
Compute the total income of Mr. X for the Assessment Year 2019-20.
Answer. Computation of total income of Mr. X for the Assessment year 2019-20
` `
Income (loss) House property
House –I 36,000
House-2 –Self occupied (20,000)
House-3 60,000
Income from House Property 76,000
Profits and gains of business and profession
Textile business 2,00,000
Automobile business (3,00,000)
Speculation business 2,00,000
Income from business or profession representing speculation
business profit (after set off of loss of automobile business) 1,00,000
Capital Gains
Long term capital gain from sale of shares (STT paid)
Excess over ` 1,00,000 u/s 112A balance is taxable 50,000
Long term capital gain from sale of vacant site 2,00,000
Short term capital loss from sale of building (1,00,000)
Long term capital gain-after set off of short term loss against 1,50,000
long term capital gain
Income from Other sources
Gift from a friend (non relative) on 05.06.2018 60,000
Gift from maternal uncle (on 25.02.2019) `1,00,000, not taxable Nil
since maternal uncle is covered by the definition of the term’ relative’
given in explanation to section 56(2)
Gift from grand father’s younger brother on 10.02.2019. This amount
is taxable as grandfather’s younger brother is not covered by the
definition of ‘relative’. 1,00,000 1,60,000
Gross Total Income 4,86,000
Less: Deduction u/s 80C to 80U Nil
Total income 4,86,000
Set Off and Carry Forward Of Losses 379

JUNE – 2009
Question 4 (4 Marks)
Mrs. X, a resident individual, provides the following details of her income / losses for the year ended
31.03.2019:
(i) Salary received as a partner from a partnership firm ` 7,50,000.
(ii) Loss on sale of shares listed in BSE `1,00,000. Shares were held for 15 months and STT paid on sale.
(iii) Long-term capital gain on sale of land ` 5,00,000.
(iv) ` 51,000 received in cash from friends in party.
(v) ` 55,000, received towards dividend on listed equity shares of domestic companies.
(vi) Brought forward business loss of assessment year 2018-19 ` 12,50,000.
The return for assessment year 2018-19 was filed in time.
Compute gross total income of Mrs. X for the assessment year 2019-20 and ascertain the amount of loss that
can be carried forward.
Answer. Computation of Gross Total Income of Mrs. X for the Assessment Year 2019-20
Particulars `
Profits and gains of business and profession
Salary received as a partner from a partnership firm is taxable under the head
“Profits and gains of business and profession” 7,50,000
Less: brought forward business loss of assessment year 2018-19 to be (7,50,000)
set-off against business income
Nil
Capital Gains
Long term capital gain on sale of land – 5,00,000
Income from other sources
Cash gift received from friends - since the value of cash gift exceeds
`50,000, the entire sum is taxable 51,000
Dividend received from a domestic company is exempt under section 10(34) Nil 51,000
Gross Total Income 5,51,000

Note: Balance brought forward business loss of assessment year 2018-19 of `5,00,000 has to be carried
forward to the next year.

NOV – 2008
Question 4 (4 Marks)
Mr. X, a resident individual, furnishes the following particulars of his income and other details for the
previous year 2018-19:
`
(i) Income from salary 18,000
(ii) Net annual value of house property 70,000
(iii) Income from business 80,000
(iv) Income from speculative business 12,000
(v) Long term capital gain on sale of land 15,800
(vi) Loss on maintenance of race horse 9,000
(vii) Loss on gambling 8,000
Depreciation allowable under the Income-tax Act comes to `8,000 for which no treatment is given above.
The other details of unabsorbed depreciation and brought forward losses of previous year 2015-16 are: `
(i) Unabsorbed depreciation 9,000
(ii) Loss from speculative business 16,000
(iii) Short term capital loss 7,800
Compute the gross total income of Mr. X, for the Assessment year 2019-20, and the amount of loss that can
or cannot be carried forward.
Set Off and Carry Forward Of Losses 380

Answer. Computation of Gross Total Income of Mr. X for the A.Y. 2019-20
` `
(i) Income from salary 18,000
(ii) Income from House Property
Net annual value 70,000
Less: Deduction under section 24(a) (30% of `70,000) (21,000) 49,000
(iii) Income from business and profession
(a) Profit from business 80,000
Less: Current year depreciation (8,000)
72,000
Less: Unabsorbed depreciation (9,000) 63,000

(b) Income from speculative business 12,000


Less: Brought forward loss from speculative business (12,000) Nil
(Balance loss of `4,000 (i.e. `16,000 – `12,000)
can be carried forward to the next year)
(iv) Income from capital gain
Long term capital gain on sale of land 15,800
Less: Brought forward short term capital loss (7,800) 8,000
Gross total income 1,38,000
Amount of loss to be carried forward to the next year
Particulars `
Loss from speculative business (to be carried forward as per section 73) 4,000
Loss on maintenance of race horses (to be carried forward as per section 74A) 9,000
Notes:
(i) Loss on gambling can neither be set-off nor be carried forward.
(ii) It has been assumed that the brought forward losses relate to P.Y. 2015-16 or thereafter. Only then
speculative business loss can set off against income from speculative business of the current year and the
balance loss can be carried forward to A.Y. 2020-21. It may be noted that speculative business loss can be
carried forward for a maximum of four years as per section 73.

NOV – 2007
Question 5 (4 Marks)
Discuss in brief the provisions relating to set off and carry forward of losses in speculation business.
Refer answer given in the chapter

MAY – 2007
Question 4 (4 Marks)
Mr. X, engaged in various types of activities, gives the following particulars of her income for the year
ended 31.03.2019: `
(a) Profit of business of consumer and house-hold products 50,000
(b) Loss of business of readymade garments 10,000
(c) Brought forward loss of catering business which was closed in Asst. Year 2018-19 15,000
(d) Short-term loss on sale of securities and shares 15,000
(e) Profit of speculative transactions entered into during the year 12,500
(f) Loss of speculative transactions of Asst. Year 2014-15 not set off till Asst. Year 2018-19 15,000
Compute the total income of Mr. X for the A.Y. 2019-20.
Answer. Computation of total income of Mr. X for the A.Y. 2019-20
Particulars ` `
Profit of business of consumer and house-hold products 50,000
Less: Loss of business of readymade garments for the year adjusted
under section 70 (10,000)
Set Off and Carry Forward Of Losses 381

40,000
Less: Brought forward loss of catering business closed in A.Y. 2018-19
set off against business income for the year as per section 72 (15,000) 25,000
Profit of speculative transaction 12,500
Total Income 37,500
Notes: (i) Loss of speculative transaction of A.Y. 2014-15 is not allowed to be set off against the profit of
speculative transaction of the A.Y. 2019-20, since, as per the provisions of section 73, such loss can be
carried forward for set-off for a maximum period of 4 years only i.e. up to A.Y. 2018-19.
(ii) Short term capital loss of `15,000 on sale of securities and shares has to be carried forward as per section
74 since there is no income under the head Capital Gains for the A.Y. 2019-20. The loss is to be carried
forward for set off in future years against income chargeable under the head Capital Gains. Such loss can be
carried forward for a maximum period of 8 assessment years.
Provisions for Filing of Return of Income 382

PROVISIONS FOR FILING


OF RETURN OF INCOME
PARTICULARS SECTIONS
Submission of return of income 139(1)
Return of loss 139(3)
Belated return 139(4)
Revised return 139(5)
Certain information required in the return of income 139(6)
Return of Income in the case of an Assessee engaged in Business /Profession 139 (6A)
Defective return 139(9)
Permanent account number 139A
Quoting of Aadhaar Number 139AA
Scheme for submission of returns through Tax Return Preparers 139B
Return by whom to be signed 140
Self-Assessment 140A

Question 1 [V. Imp.]: Write a note on filing of return of income. Section 139(1)
Or
Write a note on filing of compulsory return of income.
Answer:
Filing of return of income/ filing of voluntary return of income
Under section 139(1), a return of income is to be filed by the following persons:
(i) Every company assessee or partnership firm irrespective of their income or loss shall be required to
file return of income e.g. ABC Ltd. has total income of `10,000,in this case, company shall be required to
file the return.
(ii) Any other person like Individual, HUF etc shall be required to file return of income if Gross total income
is exceeding exemption limit e.g. If for previous year 2018-19 gross total income of Mrs. X is `2,55,000 and
deductions allowed under section 80C to 80U are `60,000 and total income is `1,95,000 and tax liability
shall be nil but still Mrs. X has to file her return of income.
(iii) Every person who is assessable on behalf of any other person and the person on whose behalf he is
assessable has gross total income more than the income exempt from tax, in such cases also, the person is
required to file a return of income on behalf of such person. E.g. Minor son of Mr. X has gross total income
from film acting `5 lakhs. In this case, Mr. X has to file a return of income on behalf of his minor son.
(iv) If any person is resident and ordinarily resident in India and has any asset outside India including
a financial interest in any entity or has signing authority in account outside India, such person shall also
be required to file his return of income.
Due date for filing the return of income
Return is to be filed in general upto 31st July of the assessment year, however, in the following cases, the last
date shall be 30th September of the assessment year.
1. Every company assessee
Example
For the previous year 2018-19, ABC Ltd. has to file its return of income upto 30.09.2019.
2. Any other person who is required to get his accounts audited either under Income Tax Act or under any
other Act.
Example
Mr. X has his own business and his turnover for previous year 2018-19 is `102 lakhs. In this case, the last
date of filing the return of income shall be 30.09.2019, but if turnover is `97 lakhs, the last date shall be
Provisions for Filing of Return of Income 383

31.07.2019.
Similarly if a partnership firm XY has turnover of its business ` 65 lakhs for previous year 2018-19, in this
case, the last date of filing of return of income shall be 31.07.2019.
3. Working partner of a partnership firm whose accounts are required to be audited.
Question 2 [V. Imp.]: Write a note on Return of Loss Section 139(3).
Answer: Return of Loss Section 139(3)
If any person has sustained any loss under the head Business/Profession or under the head capital gains or
the loss is from owning and maintaining of race horses and such person claims that the loss is to be carried
forward, such person has to file a return of loss and such return shall be examined by the Assessing Officer
and the loss computed by the assessee shall be confirmed by the Assessing Officer by sending an intimation
under section 157 and only after that carry forward of loss shall be allowed to the assessee.
Return under section 139(3) has to be submitted within the time allowed under section 139(1).
Under section 80, if return of loss has been filed after the last date of filing of return of income, In that case
carry forward of losses is not allowed. E.g. For previous year 2018-19 ABC Ltd. has incurred business loss
of `90 lakhs. In this case, the company must file return of loss under section 139(3) maximum upto
30.09.2019, otherwise carry forward of the loss is not allowed.
The above provisions are not applicable with regard to loss under the head house property and also it is not
applicable with regard to unabsorbed depreciation.
If any return is filed under section 139(3), it will be considered to be a return under section 139(1).
Question 3 [V. Imp.]: Write a note on Belated Return of Income Section 139(4).
Answer: Belated Return of Income Section 139(4)
Every person is required to file a return of income within the time allowed under section 139(1) however
return of income can be filed even after the due date but maximum upto the end of relevant assessment
year. E.g. For previous year 2018-19 ABC Ltd. has to file its return of income upto 30.09.2019. However,
belated return is allowed under section 139(4) but maximum upto 31.03.2020
Question 4: Write a note on fee for default in furnishing return of income
Answer: Fee for default in furnishing return of income. Section 234F
where a person required to furnish a return of income under section 139, fails to do so within the time
prescribed u/s 139(1), he shall pay, by way of fee, a sum of,—
- ` 5,000, if the return is furnished on or before the 31st day of December of the assessment year;
- ` 10,000 in any other case.
Provided that if the total income of the person does not exceed `5,00,000 the fee payable under this section
shall not exceed ` 1,000.
Example: For the P.Y.18-19, Mr. X has Total Income `7,00,000 and he files his return on 10th August 2019,
in this case Penalty of `5000 is payable and if he files his return on 10th January 2020, Penalty is `10,000
but if his total income is upto `5,00,000, Penalty shall be `1000 in either case.

Question 5 [V. Imp.]: Write a note on Revised Return of Income Section 139(5).
Answer: Revised Return of Income Section 139(5)
If any person has furnished a return under section 139(1) or under section 139(4), discovers any omission or
any wrong statement, he may furnish a revised return at any time before the end of the relevant assessment
year e.g. If ABC Ltd. has filed its return of income on 30.09.2019 for previous year 2018-19 and
subsequently the company has detected any bonafide error, in this case, the company is allowed to revise its
return of income under section 139(5) but maximum upto 31.03.2020.
If the assessment on the return has already been completed, revision is not allowed after completion of
assessment e.g. For the previous year 2018-19 ABC Ltd. has filed its return of income on 30.09.2019. This
return was checked by the Assessing Officer on 01.03.2020 and the company wish to file a revised return on
15.03.2020. In this case, revised return shall not be accepted.
An assessee is allowed to revise the return of income any number of times, however, if the earlier return has
already being assessed, revised return shall not be allowed.
Revision is allowed only with regard to a return, which was filed under section 139(1) or 139(4), i.e. if the
Provisions for Filing of Return of Income 384

return has been filed under any other section, its revision is not allowed.
However, the return filed under section 139(3) is considered to be return under section 139(1), its revision is
allowed.

Question 6: What are the particulars required to be furnished with the return of income in certain
specified cases, as per section 139(6).
Answer: As per section 139(6), the following particulars are required to be submitted:
- Income exempt from tax,
- Assets of the prescribed nature, value and belonging to him,
- Bank account and credit card held by him
- Expenditure exceeding the prescribed limits incurred by him under prescribed heads and such other
outgoings as may be prescribed.

Question 7: Write a note on Particulars to be Furnished with Return of Income in the case of an
Assessee engaged in Business or Profession.
Answer: Particulars to be Furnished with Return of Income in the case of an Assessee engaged in
Business or Profession Section 139(6A)
The prescribed form of the return shall, in the case of an assessee engaged in any business or profession,
also require him to furnish –
(i) the report of any audit referred to in section 44AB.
(ii) the particulars of the location and style of the principal place where he carries on the business or
profession and all the branches thereof.
(iii) the names and addresses of his partners, if any, in such business or profession.
(iv) if he is a member of an association or body of individuals,
(a) the names of the other members of the association or the body of individuals; and
(b) the extent of the share of the assessee and the shares of all such partners or members, as the case may
be, in the profits of the business or profession.

Question 8 [V. Imp.]: Write a note on Defective Return of Income Section 139(9).
Answer: Defective Return of Income Section 139(9)
If return filed by an assessee is found to be defective, assessing officer may intimate the defect to the
assessee and give him an opportunity to rectify the defect within a period of fifteen days from the date of
such intimation or within such further period which, the Assessing Officer may, allow and if the defect is
not rectified within the said period, then the return shall be treated as an invalid return i.e. it will be
presumed that the assessee has not filed any return of income.
If the assessee rectifies the defect after the expiry of the said period of fifteen days or the further period
allowed, but before the assessment is made, the Assessing Officer may condone the delay and treat the
return as a valid return.
A Return of income shall be regarded as defective unless all the annexures, statements and columns in the
return of income relating to computation of income chargeable under each head of income, computation of
gross total income and total income have been duly filled in.
An unsigned return is not a valid return at all i.e. it will not be considered to be defective return rather it is
an invalid return.

Question 9 [V. Imp.]: Write a note on Permanent Account Number Section 139A.
Answer: Permanent Account Number Section 139A
The following persons have to apply for allotment of permanent account number.
1. Every person having total income more than the income exempt from tax and also every person who is
assessable on behalf of any other person and the person on whose behalf he is assessable has income
more than the income exempt from tax.
2. Every person who is engaged in business/profession and his turnover is likely to exceed `5 lakh during
the year.
3. Every charitable trust who is required to file a return of income under section 139(4A).
Provisions for Filing of Return of Income 385

4. Every person who has been notified by the Central Government.


5. A person who is entitled to receive any sum on which tax is deductible.
6. Being a resident, other than an individual, which enters into a financial transaction of an amount
aggregating to two lakh fifty thousand rupees or more in a financial year; or
7. Who is the managing director, director, partner, trustee, author, founder, karta, chief executive
officer, principal officer or office bearer of the person referred to in clause (v) or any person
competent to act on behalf of the person referred to in clause (v),”;
8. The Assessing Officer, having regard to the nature of the transactions as may be prescribed, may also
allot a permanent account number, to any other person (whether any tax is payable by him or not), in the
manner and in accordance with the procedure as may be prescribed.
9. Any person who is willing to take permanent account number.
The department shall allot a 10 digit alphanumeric number on a laminated card, which contains the
photograph of the assessee.

Example
AAEPG 8996E
The purpose to issue permanent account number is to have better identification of the assessee and to
facilitate faster correspondence between the department and the assessee.
Permanent account number is being used to detect concealed income and for this purpose the assessee has to
mention permanent account number in the following transaction.
1. Sale/Purchase of immovable property exceeding `10 lakhs. Properties valued by Stamp Valuation
Authority at amount exceeding `10 lakhs.
2. Sale/purchase of shares and securities exceeding `1 lakh.
3. Sale/purchase of motor vehicles.
4. Opening any account (other than Basic Savings Bank Deposit Account) in a bank (including co-operative
bank) not being a time deposit account.
5. Opening time deposit account in a Bank, Co-op Bank, Post office, Nidhi, NBFC companies for an amount
aggregating to more than `5,00,000 during the year.
6. Making a Cash payment of hotel or restaurant bill exceeding ` 50,000.
7. Deposit in cash exceeding `50,000 with a banking company in one day.
8. Payment in cash for purchase of a bank draft or a pay order or banker’s cheque for an amount exceeding
`50,000 on any one day.
9. Payment in cash in connection with travel to any foreign country or purchase of foreign currency of an
amount exceeding `50,000 at any one time.
10. Making an application to any banking company (including co-operative banks) or to any other company
or institution, for issue of a credit card/debit card.
11. Payment of an amount exceeding `50,000 to a Mutual Fund for purchase of its units.
12. Purchase or sales of shares of an unlisted company for an amount exceeding `1,00,000 per transaction.
13. Opening a DEMAT account.
14. Payment of an amount exceeding `50,000 to a company or an institution for acquiring debentures or
bonds issued by it.
15. Payment of an amount exceeding `50,000 to the Reserve Bank of India, for acquiring bonds issued by it.
16. Payment of an amount exceeding `50,000 in a year as life insurance premium to an insurer.
17. Purchase/sale of any goods or services exceeding `2,00,000 per transaction.
18. Cash Payment aggregating to more than `50,000 in a year for cash cards/prepaid instruments issued
under Payment and settlement Act.

STRUCTURE OF PAN
A typical PAN is AFZPK7190K.
First three characters i.e. “AFZ” in the above PAN are alphabetic series running from AAA to ZZZ
Fourth character of PAN i.e. “P” in the above PAN represents the status of the PAN holder. “P” stands for
Individual, “F” stands for Firm, “C” stands for Company, “H” stands for HUF, “A” stands for AOP, “T”
stands for TRUST etc.
Provisions for Filing of Return of Income 386

Fifth character i.e. “K” in the above PAN represents first character of the PAN holder’s last name/surname.
Next four characters i.e. “7190” in the above PAN are sequential number running from 0001 to 9999.
Last character i.e. “K” in the above PAN is an alphabetic check digit.
If any person do not have a Permanent Account Number, in such cases, such person has give a declaration in
Form No. 60.

Question 10 [V. Imp.]: Write a note on Quoting of Aadhaar number.


Answer: Quoting of Aadhaar number. Section 139AA
(1) Every person who is eligible to obtain Aadhaar number shall, on or after the 1st day of July, 2017, quote
Aadhaar number—
(i) in the application form for allotment of permanent account number;
(ii) in the return of income:
Provided that where the person does not possess the Aadhaar Number, the Enrolment ID of Aadhaar
application form issued to him at the time of enrolment shall be quoted in the application for permanent
account number or, as the case may be, in the return of income furnished by him.
(2) Every person who has been allotted permanent account number as on the 1st day of July, 2017, and who
is eligible to obtain Aadhaar number, shall intimate his Aadhaar number to such authority in such form and
manner as may be prescribed, on or before a date to be notified by the Central Government in the Official
Gazette:
Provided that in case of failure to intimate the Aadhaar number, the permanent account number allotted to
the person shall be deemed to be invalid and the other provisions of this Act shall apply, as if the person had
not applied for allotment of permanent account number.
(3) The provisions of this section shall not apply to such person or class or classes of persons or any State or
part of any State, as may be notified by the Central Government in this behalf, in the Official Gazette.
Question 11 [V. Imp.]: Write a note on submission of returns through Tax Return Preparers.
Answer: Scheme for submission of returns through Tax Return Preparers Section 139B
In order to help the persons having low income or tax liability, department has started scheme of Tax Return
Preparer who will file return for such persons. For this purpose department shall select and appoint TRPs.
The tax return preparer shall hold a graduation degree from a recognised Indian university or other specified
qualifications but such persons should not be a Chartered Accountant or other specified persons.
A person may approach a TRP for filing the return of income but any person who is required to get
his accounts audited shall not be allowed to file the return through the Tax Return Preparer.
Similarly any non-resident shall not be allowed to file return through Tax Return Preparer.
The department shall pay a commission of 3% of the tax paid on the income declared in the return or `1,000
whichever is less. A TRP shall be entitled for a minimum payment of `250 and if commission paid is less
than `250, he can receive the difference amount from the assessee whose return is being filed.
e.g. A TRP has deposited tax of `60,000 on the basis of return filed by it, in this case commission payable
shall be 60,000 x 3% = 1,800 but maximum `1,000. If tax paid is `20,000, commission payable shall be
20,000 x 3% = 600. If tax paid is `5,000, commission payable shall be `5,000 x 3% = 150 and TRP shall
allowed to charge `100 from the assessee.
Question 12: Write a note on verifying of Return of Income.
Answer: Verifying of return of income
Return by whom to be verified Section 140
1. In the case of an individual, the return should be verified by the individual himself but if for any reason
return cannot be verify by the individual, return can be verified by his agent and the agent should enclose
copy of power of attorney with the return.
If any individual is mentally incapacitated from attending to his affairs, return should be verified by his
guardian or any other person competent to act on his behalf.
2. In the case of a Hindu Undivided Family, by the karta, and, where the karta is absent from India or is
mentally incapacitated from attending to his affairs, by any other adult member (male or female) of such
family.
3. In the case of a company, by the managing director and if managing director is not available, return can
be signed by any director.
Provisions for Filing of Return of Income 387

In case of non-resident company, return can be signed by its agent.


If company is in liquidation, return can be signed by the official liquidator.
4. In the case of a firm, return can be signed by managing partner and if managing partner is not available,
return can be signed by any partner.
5. In the case of a limited liability partnership, return can be signed by the designated partner and if
designated partner is not available, return can be signed by any partner.
6. In the case of a local authority, return can be signed by the principal officer.
7. In the case of a political party, return can be signed by Chief Executive Officer.
8. In the case of any other association, return can be signed by the Principal Officer and if Principal Officer
is not available, by any member.
9. In the case of any other person, by that person or by some person competent to act on his behalf.

Question 13: Write a note on Self-Assessment.


Answer: Self-Assessment Section 140A
In general a person is required to pay tax in advance and balance amount should be paid in the assessment
year (after adjusting the amount of TDS) and it is called Self - Assessment tax. In case of delay, interest
shall be charged u/s 234A, 234B & 234C. For default in advance tax interest shall be charged under section
234C and for default in self-assessment tax, interest shall be charged u/s 234B. If tax is paid after the last
date of filing of return of income, interest shall be charged u/s 234A.
Tax and interest and fee must be paid before filing of return of income as per section 140A otherwise such
person shall be considered to be assessee in default and penalty shall be imposed equal to the amount not
paid by him.
Further any amount paid by the assessee shall be first adjusted towards fee and then towards interest and
afterwards towards tax.
Provisions for Filing of Return of Income 388

PRACTICE PROBLEMS
Question 1. During the previous year 2018-19, Mr. X has income under the head house property `7,00,000.
In this case, his last date of filing of return shall be ____________________.
Question 2. A partnership firm XY has turnover of his business `75,00,000 and income under the head
Business/Profession `8,00,000 for previous year 2018-19. In this case, the last date of filing of return of
income shall be ____________.
Question 3. ABC Ltd. has loss under the head Business/Profession `3,00,000 for previous year 2018-19. In
this case, the company has to file the return latest by ____________________.
Question 4. A partnership firm XY has turnover of the business `205,00,000 and income from business
`14,00,000 for the previous year 2018-19, the last date for filing of return of income shall be
______________.
Question 5. ABC Ltd. do not have any income for the previous year 2018-19. In this case, company is not
required to file any return of income. Discuss.
Question 6. ABC Ltd. has loss under the head Business/Profession `7,00,000 for the previous year 2018-19
and the company has filed the return of loss on 01.11.2019 under section 139(3). Discuss whether set off or
carried forward and set off of the loss is allowed or not.
Question 7. For the previous year 2018-19, Mr. X an assessee shall be allowed to file belated return of
income latest upto ____________________.
Question 8. For the previous year 2018-19, Mr. X has filed original return of income on 01.07.2019, he can
file revised return of income latest upto_________________.
Question 9. For the previous year 2018-19, Mr. X has filed original return of income on 01.11.2019, he can
file revised return of income latest upto ____________________.
Question 10. Mr. X has filed original return for previous year 2018-19 on 01.07.2019 and revised return on
01.11.2019 and he further wants to revise the return on 01.01.2020. Discuss whether he is allowed to do so
or not.
Ans 1. 31.07.2019
Ans 2. 31.07.2019
Ans 3. 30.09.2019
Ans 4. 30.09.2019
Ans 5. As per section 139(1), every company has to file return of income in every case.
Ans 6. The company is allowed to set off the loss during the previous year 2018-19 but its carried forward is
not allowed because return of loss has to be filed within the time allowed under section 139(1) i.e.
30.09.2019 in the above case.
Ans 7. 31.03.2020
Ans 8. 31.03.2020
Ans 9. 31.03.2020
Ans 10. A revised return can also be revised further any number of times, however, if the earlier return has
already been assessed, revised return is not allowed subsequently. In the given case, revised return can be
filed on 01.01.2020.
Provisions for Filing of Return of Income 389

EXAMINATION QUESTIONS
NOV – 2017
Question 5(b) (Marks 5)
Mr. Sachin filed return on 30th September, 2019 related to Assessment Year 2019-20. In the month of
October 2018, his tax consultant found that the interest on fixed deposit was omitted in the tax return.
(i) What is the time limit for filing a belated return?
(ii) Can Mr. Sachin file a revised return?
Justify the above with the relevant provisions under section 139. Assume that the due date for furnishing
return of income was 31st July, 2019 and the assessment was not completed till the month of October 2019.

Answer:
(i) As per section 139(4), Any person who has not furnished a return within the time allowed to him under
section 139(1) may furnish the return for any previous year at any time before the end of the relevant
assessment year or before the completion of the assessment whichever is earlier.
Therefore, in the given question, Mr. Sachin can file his belated return on or before 31st March, 2020.

(ii) As per section 139(5), If any person having furnished a return u/s 139(1) or belated return u/s 139(4),
discover any omission or any wrong statement therein, he may furnish a revised return at any time before the
end of the relevant assessment year or before completion of assessment, whichever is earlier hence, in the
given case Mr. Sachin filed belated return as per Section 139(4) and he omitted his Interest Income in the
return hence he can revised his return on or before 31st March, 2020.
MAY – 2017
Question 7 (b) (4 Marks)
By whom should the return of income be signed in the case of following persons:
(i) Political Party;
(ii) Company which is being wound up;
(iii) Hindu Undivided Family, when karta is unable to sign, and
(iv) Scientific research association.
Answer:
As per section 140, Return should be signed by the authorised person, as given below:
(i) Political Party: In the case of a political party, Return can be signed by Chief Executive Officer.
(ii) Company which is being wound up: If company is in liquidation, Return can be signed by the
Official liquidator.
(iii) Hindu Undivided Family, when karta is unable to sign: In the case of a Hindu Undivided Family,
when karta is unable to sign then by any other adult member (male or female) of such family can
verify the return of Income.
(iv) Scientific research association: Return can be signed by the Principal Officer and if Principal
Officer is not available, by any member.

NOV – 2016
Question 6(b) (2 Marks)
Specify the persons who are authorized to verify u/s 140, the return of income filed u/s 139 of the Income
Tax Act, 1961 in case of a company.
Answer: Refer answer given in the book
Question 7(a) (2 + 2= 4 Marks)
(1) Mr. Kamal filed his Return of Income for the Assessment Year 2019-20 on 30-03-2020. Can he revise
such return of income? If so Why?
Provisions for Filing of Return of Income 390

Answer: As per Section 139(5), Return filed under section 139(1) or Return filed u/s 139(4) can be revised
hence a belated return filed under section 139(4) can be revised and further return can be revised maximum
within one year from the end of relevant assessment year i.e. return can be revised upto 31.03.2020.

(2) Mr. Atal, a super senior citizen, has reported a Gross Total Income `5,60,000 and the deductions eligible
under Chapter VI-A amounting to `70,000 for the previous year 2018-19. Is he liable to file his return of
income u/s 139(1) for the Assessment year 2019-20? If so Why?
Answer: Individual shall be required to file return of income if Gross total income is exceeding exemption
limit. In the given case assessee is a super senior citizen and exemption applicable is `5,00,000 and Gross
Total Income is exceeding `5,00,000 So, Mr. Atal is required to file his Return of Income.
NOV – 2015
Question 7(a). (2 Marks)
Who are the persons authorized to verify return of income in the case of individual under section 140 of the
Income Tax Act, 1961?
Solution:
As per section 140 the return shall be verified—
In the case of an individual, the return should be verified by the individual himself but if for any reason
return cannot be verify by the individual, return can be verified by his agent and the agent should enclose
copy of power of attorney with the return.
If any individual is mentally incapacitated from attending to his affairs, return should be verified by his
guardian or any other person competent to act on his behalf.
MAY – 2015
Question 7(a)(i). (2 Marks)
Explain the term "return of loss'' under the Income-tax Act, 1961. Can any loss be carried forward even if
return of loss has not been filed as required?
Answer: Refer answer given in the Chapter

NOV – 2014
Question 7(a)
(i) “Filing of Return of Income on or before due date is necessary for carry forward of losses.” Discuss the
correctness of this statement. (4 Marks)
Refer answer given in the Chapter

MAY – 2014
Question . (2 x 2 = 4 Marks)
(A) Mr. X submits his return of income on 12.09.2019 for AY 2019-20 consisting of income under the head
house property and other sources. On 21.01.2020, he realized that he had not claimed deduction under
section 80TTA in respect of his interest income on the Savings Bank Account. He wants to revise his return
of income, since one year has not elapsed from the end of the relevant Assessment Year. Discuss.
(B) Where the Karta of an Hindu undivided family is absent from India, the return of income can be signed
by any male member of the family? Give reasoning for the statement to be true or false.
Solution:
(A) Since Mr. X has income only under the heads “Income from house property” and “Income from other
sources”, he does not fall under the category of a person whose accounts are required to be audited under the
Income-tax Act, 1961 or any other law in force. Therefore, the due date of filing return for A.Y.2019-20
under section 139(1), in his case, is 31st July, 2019. Since Mr. X had submitted his return only on
12.09.2019, the said return is a belated return under section 139(4).
As per section 139(5), a return furnished under section 139(1) or u/s 139(4) can be revised. Therefore, Mr. X
can revise the return of income filed by him under section 139(4), to claim deduction under section 80TTA.
(B) The statement is false.
Provisions for Filing of Return of Income 391

As per section 140, where the karta of a HUF is absent from India, any other adult member of the HUF, can
sign the return of income.
Thus, a male member who is not an adult cannot sign the return of income. An adult member, whether male
or female, can sign the return of income.

NOV – 2012
Question No. 7(a) (4 Marks)
Enumerate the circumstances in which an individual assessee is empowered to verify his return of income
u/s 140 by himself or otherwise by an authorised signatory.
Answer: Refer answer given in the Chapter
NOV – 2011
Question 7 (2 Marks)
State with reasons whether you agree or disagree with the following statements:
Return of income of Limited Liability Partnership (LLP) could be verified by any partner.
Answer:
Disagree. In case of limited liability partnership return is to be verified by designated partner, in case due to
any unavoidable circumstances designated partner is not available then any partner can verify.
Question 7 (4 Marks)
(i) State when a return of income can be treated as defective?
(ii) An assessing officer finds a defect in the return of income and intimated the defect vide letter dated
09.10.2018, which was received by Mr. Ram on 11.10.2018, What is the date by which Mr. Ram has to
rectify the defect, assuming that Mr. Ram has not applied for extension of time.
Answer:
(i) Refer answer given in the chapter
(ii) Date by which Mr. Ram has to rectify the mistake:-
= 09.10.2018 + 15 days = 24.10.2018
MAY – 2011
Question 7 (4 Marks)
Specify the persons who are authorized to verify under section 140, the return of income filed under section
139 of the Income-tax Act, 1961 in the case of:
(i) Political party;
(ii) Local authority;
(iii) Association of persons, and
(iv) Limited Liability Partnership (LLP)
Answer: Refer answer given in the Chapter

MAY – 2010
Question 4 (4 Marks)
What are the particulars required to be furnished with the return of income, as per section 139(6)?
Answer.
Refer Answer given in the Chapter
NOV – 2008
Question 5 (4 Marks)
Explain with brief reason whether the return of income can be revised under section 139(5) of the Income-
tax Act, 1961 in the following cases:
(i) Defective or incomplete return filed under section 139(9).
(ii) Belated return filed under section 139(4).
(iii) Return already revised once under section 139(5).
(iv) Return of loss filed under section 139(3).
Provisions for Filing of Return of Income 392

Answer.
Any person who has furnished a return under section 139(1) or under section 139(4) can file a revised return
if he discovers any omission or any wrong statement in the return filed earlier. Accordingly:-
(i) A defective or incomplete return filed under section 139(9) cannot be revised. However, the defect can be
removed.
(ii) A belated return filed under section 139(4) can be revised.
(iii) An assessee is allowed to revise the return any number of times but if earlier return has already been
assessed, further revision is not allowed.
(iv) A return of loss filed under section 139(3) is deemed to be return filed under section 139(1), and
therefore, can be revised under section 139(5).
NOV – 2007
Question 5 (4 Marks)
Enumerate eight transactions for which quoting of Permanent Account Number is mandatory.
Refer Answer given in the Chapter
MAY – 2007
Question 1 (2 Marks)
The Assessing Officer has the power, inter alia, to allot PAN to any person by whom no tax is payable.
Discuss.
Answer: True: As per section 139A, The Assessing Officer may having regard to the nature of transactions
as may be prescribed also allot a PAN to any other person, whether any tax is payable by him or not, in the
manner and in accordance with the procedure as may be prescribed.
Question 1 (2 Marks)
Where the Karta of an HUF is absent from India, the return of income can be verify by any male member of
the family.
Answer:False: Section 140 provides that where the Karta of a HUF is absent from India, the return of
income can be verify by any other adult member of the family; such member can be a male or female
member.
Question 5 (4 Marks)
Discuss briefly about the scheme to facilitate submission of return of income through Tax Return Preparers.
Refer Answer given in the Chapter
Miscellaneous Topics 393

MISCELLANEOUS TOPICS
Assessee Section 2(7)
“Assessee” means
 Any person who is liable to pay income tax or interest or penalty or any other sum under Income Tax
Act
 Any person with regard to whom any proceedings are pending under Income Tax Act for assessment
of income or loss or refund or any other proceeding
 Any person who is assessable on behalf of any other assessee i.e. deemed assessee and any proceeding
is pending with regard to such other person like assessment of income or loss or refund or any other
proceeding e.g. Minor son of Mr. X has income from talent `5,00,000, in this case Mr. X shall be
deemed to be an assessee.
MAY – 2016
Question 2(a)(ii) (4 Marks)
How is the term “Assessee” defined under the Provisions of the Income - Tax Act, 1961?
Answer: Refer answer given above

Assessment year
As per section 2(9), assessment year means a period of 12 months starting from 01st April of every year
ending with 31st March i.e. every financial year is a assessment year e.g. Financial year 2019-20 is one
assessment year.
Previous year
As per section 3, previous year means financial year preceding the assessment year e.g. If financial year
2019-20 is one assessment year, financial year 2018-19 is the previous year for such assessment year.
Income of previous year is taxable in its assessment year i.e. exact tax liability for previous year 2018-19
shall be determined in assessment year 2019-20 (however the person has to pay advance tax on estimated
basis in previous year 2018-19.)
In general previous year shall be of full year but in case of a newly setup business or profession, first
previous year will start from the date of commencement of business / profession e.g. If Mr. X started
business on 01.07.2018, first previous year shall be from 01.07.2018 to 31.03.2019.
Income Section 2(24)
Every person shall be required to pay tax on his income as per section 4 and the term income is divided into
5 different categories which are called heads of income and such incomes shall include
1. Payment by employer to employee.
2. Rent received or receivable in connection with house property
3. Payments in connection with business/profession as per section 28
4. Profit and gains on the transfer of capital asset as per section 45(1)
5. Incomes under section 56 like, dividend, interest, casual income, gift etc.
6. Any other income given under section 2(24).
Charging section of Income-Tax Section 4
Every person shall be liable to pay income tax on his income. Normal income of every person shall be
taxable at the rates given in the relevant Finance Act. Special incomes like Long term capital gains or Short
term capital gains under section 111A or casual income shall be taxable at the rates given in the Income Tax
Act. Tax shall be deducted at source as per the relevant provision also advance tax is to be paid as per the
relevant provision.
Expenditure incurred in relation to income not includible in Total Income Section 14A
If any income is exempt from income tax, expenditure incurred in connection with such income shall not be
allowed to be deducted either from same income or from some other income. If expenditure is incurred for
taxable income as well as exempt income, only expenditure relating to taxable income shall be allowed to be
deducted. E.g. if loan has been taken for purchasing shares which were sold and were exempt under section
112A, interest on such loan is not allowed to be deducted from any income.
Miscellaneous Topics 394

Question 1 [V. Imp.]: Discuss exceptions to the General Rule that the income of the Previous Year is
taxed in the Assessment Year.
Answer: Exceptions to the General Rule that the income of the Previous Year is taxed in the
Assessment Year
Generally the income of the Previous Year is taxable in the immediately succeeding year called the
assessment year. But, in the following cases, the income of the current year may be brought to tax in the
same year–being exception to the general rule that incomes earned in the previous year are taxed in its
assessment year following: -
1. Profits of non-resident from Shipping Business Section 172: If any ship owned by a non-resident
has entered in India, the ship shall not be allowed to leave India unless tax has been paid and return has been
filed
2. Assessment of persons Leaving India Section 174: If any person is leaving India with no present
intention of returning to India, the total income of such individual up to the probable date of his departure
from India shall be chargeable to tax in the previous year itself.
3. Assessment of association of persons or Body of Individuals or Artificial Juridical Person formed
for a particular event or purpose Section 174A: If any association of persons or a body of individuals
etc. has been incorporated for a particular event or purpose and it is likely to be dissolved in the same year in
which it was formed, the total income of such association or body or juridical person for the period up to the
date of its dissolution shall be chargeable to tax in that year itself.
4. Assessment of persons likely to transfer property to avoid tax Section 175: If it appears to the
Assessing Officer that any person is likely to sell, transfer or otherwise part with any of his assets with a
view to avoid payment of any liability under the provisions of this Act, the total income of such person shall
be taxable in the same previous year.
5. Discontinued Business Section 176: If any person has closed down his business/profession, such person
should inform Income Tax Department within 15 days of closing down such business/profession and the
Department may direct such a person to pay tax and file return in the previous year itself.
10(2) Share received by a member of Hindu undivided family from income of Hindu Undivided
Family
If any Hindu Undivided Family has paid income tax on the income of the family, such income shall not be
taxable in the hands of its members. E.g. Mr. X is a member of one HUF and has received `3,00,000 from
HUF as his share, it will be exempt from income tax under section 10(2).
10(10BC) Compensation received or receivable on account of any disaster
As per section 10(10BC), if any person has received any payment from government or other similar
authority as compensation for loss or damage caused by any disaster whether natural calamity or any
accident etc., such compensation shall be exempt from income tax.
NOV – 2016
Question 4(a) (2 Marks)
State with reasons whether the following receipts are taxable or not under the provisions of Income-tax Act,
1961?
Mr. Suri received a sum of `5,00,000 as compensation, from ‘Yatra Foundation’, towards the loss of
property on account of Flood Disaster at Chennai during December 2018.
Answer:
As per section 10(10BC), if any person has received any payment from government or other similar
authority as compensation for loss or damage caused by any disaster whether natural calamity or any
accident etc., such compensation shall be exempt from income tax. In the given case compensation is not
received from Government hence the compensation received is taxable.

MAY – 2016
Question 7(a)(iii) (2 Marks)
Discuss with reason, whether the following transactions are true or false, as per the provisions of Income
Tax Act, 1961:
Any amount received by an individual or his legal heir as compensation for natural disaster from the
Government, is taxable.
Miscellaneous Topics 395

Answer:
False: As per section 10(10BC), if any person has received any payment from government or other similar
authority as compensation for loss or damage caused by any disaster whether natural calamity or any
accident etc., such compensation shall be exempt from income tax.
10(26AAA) Exemption in case of Income of an Individual being Sikkimese
As per section 10(26AAA), income of an individual of Sikkim shall be exempt from income tax if such
income is from Sikkim. If he has received dividend or interest on securities from any where, it will also be
exempt from income tax. If any woman from Sikkim has married an individual who is not a Sikkimese, in
that case such woman shall not be eligible for exemption.
Question 2: Explained Treatment of Unexplained money, investments etc.
Answer: Unexplained money, investments etc. to attract tax @60% [Section 115BBE]
(i) In order to control laundering of unaccounted money by availing the benefit of basic exemption limit,
the unexplained money, investment, expenditure, etc. deemed as income under section 68 or section 69 or
section 69A or section 69B or section 69C or section 69D would be taxed at the rate of 60% plus surcharge
@25% of tax. Thus, the effective rate of tax (including surcharge @25% of tax and HEC @4% of tax and
surcharge) is 78%.
(ii) No basic exemption or allowance or expenditure shall be allowed to the assessee under any provision of
the Income-tax Act, 1961 in computing such deemed income.
(iii) Further, no set off of any loss shall be allowable against income brought to tax under sections 68 or
section 69 or section 69A or section 69B or section 69C or section 69D.
Question 3: Describe average rate of tax and maximum marginal rate under section 2(10) and 2(29C)
of the Income- tax Act, 1961.
Answer: As per section 2(10), “Average Rate of tax” means the rate arrived at by dividing the amount of
income-tax calculated on the total income, by such total income.
Section 2(29C) defines “Maximum marginal rate” to mean the rate of income-tax (including surcharge on
the income-tax, if any) applicable in relation to the highest slab of income in the case of an individual, AOP
or BOI, as the case may be, as specified in Finance Act of the relevant year.
Question 4: Explain Interest on notified securities and bonds issued to non-residents.
Answer: Interest on notified securities and bonds issued to non-residents [Section 10(4)]
As per section 10(4), in the case of an individual, any income by way of interest on moneys standing to his
credit in a Non-resident (External) Account (NRE A/c) in any bank in India, would be exempt.
Question 5: Explain Treatment of Payments received to MPs & MLAs
Answer: Payments to MPs & MLAs [Section 10(17)]
The following incomes of Members of Parliament or State Legislatures will be exempt:
(i) Daily Allowance - Daily allowance received by any Member of Parliament or of State Legislatures or
any Committee thereof.
(ii) Constituency Allowance of MPs - In the case of a Member of Parliament or of any Committee thereof,
any allowance received under Members of Parliament (Constituency Allowance) Rules, 1986; and
(iii) Constituency allowance of MLAs - Any constituency allowance received by any person by reason of
his membership of any State Legislature under any Act or rules made by that State Legislature.
Question 6: Briefly explain the purpose for which the words “PROVISO” and “EXPLANATION” are
incorporated under various sections of the Income Tax Act, 1961.
Answer: The Proviso to a section/sub-section/clause refers to sometimes additional provision, sometimes a
special condition and sometimes exceptions to the main provision, eg. Proviso to section 23(1) refers to
deduction of municipal tax but section 23(1) refers to computation of gross annual value i.e. it is an
additional provision. Similarly proviso to section 17(2) refers to medical facility but section 17(2) refers to
different facilities.
The Explanation to a section/sub-section/clause gives a clarification relating to the provision contained in
the respective section/sub-section/clause, eg. Explanation to section 23(1) covers unrealised rent i.e. an
explanation to section 23(1) which refers to gross annual value.

You might also like